本站咨询关键字

   中考语文
总复习资料大全汇编

第一讲 语音和汉字 1
第二讲 词语和句子 8
第三讲 语言运用 15
第四讲 文学常识 26
第五讲 背诵默写 33
第六讲 诗词赏析 39
第七讲 综合考查 49
第八讲 其他训练 53
第九讲 文言文阅读 56
第十讲 记叙文阅读 80
第十一讲 说明文阅读 107
第十二讲 议论文阅读 141
第十三讲 学作品阅读 175
中考语文总复习参考答案 222

第一讲 语音和汉字
【考查要点】
   语音、文字、书写都是重要的语文素养之一。课标在“识字与写字”中指出:①能熟练地使用字典、词典独立识字,会用多种检字方法。累计识常用汉字3500个,其中3000个左右会写。②在使用硬笔熟练地书写正楷字的基础上,学写规范、通行的行楷字,提高书写的速度。③临摹名家书法,体会书法的审美价值。
   【知识疏理】
   本部分涉及的知识点主要有:①给汉字注音;②根据拼音写出汉字;③多音字辨认;④汉字的音、形、义;⑤给古汉字(文言句子中指定的字)注音;⑥改正错别字;⑦书写等。
   【试题特点】
   语音和汉字题主要着眼于常用汉字的正确认读和书写。其题型多为客观型选择题,要求选出读音或书写完全正确或有误的一项,其中语音题的出现形式大多为词语中“加点的字”,也有以句子形式出现的。此外,还有要求给加点的字注音、根据拼音写汉字的题。汉字题还有改正词语或句子中写错的字。也有少数将音、形、义综合在一起设题的。
   【解题导引】
   例1.(2004·重庆北培实验区)下列词语中加点的字的注音,完全正确的一项是( )
   A.住宿(xiǔ) 衬托(chèn) 天伦之乐(lén)
   B.探究(tān) 馈赠(kuì) 生机盎然(àng)
   C.祈祷(dǎo) 携带(xié) 肃然起敬(sù)
   D.哺育(pǔ) 绽放(zhàn) 怡然自得(yí)
   此题意在考查考生对一些重要的字词读音的掌握情况,解答此类题可用两种简便方法:排除法和直接筛选法。就此而言,排除法就是发现某一项加点字注音不正确就迅速排除该项,再继续查找,留下的项就是答案。直接筛选法就是倘若能够一眼看出某项中的加点的字注音完全正确,那么答案即可确定。其他各项可不加过问。此题可用排除法。A项有两处错误,“住宿”中的“宿”应读“sù”,“天伦之乐”中的“伦”应读“lún”,这两个字都是多音字;B项中的“探”应读“tàn”;C项没有错误,为本题的正确答案。D项中的“哺”应读“bǔ”。
   解答此类题首先要认真审读题干,明确题目要求,看清是选“正确”的还是选“有误”的,或者是选出“加点字的读音与所给的读音完全相同的一项”,或者是“加点字读音完全相同的一项”。
   例2.(2004·黄冈)读下面的句子,根据拼音填汉字,给加点的字注音。
   ①不少的人对工作不负责任,niān( )轻怕重。
   ②黄与绿主宰着,无边无垠,坦荡如dǐ( )。
   ③吏二缚一人诣( )王。
   ④菊之爱,陶后鲜( )有闻。
   这是一道主观题,也是一道综合考查题,要求考生正确区分并使用同音字、形似字正
   确书写汉字,不写错字、别字,同时能给古汉字注音。①句中应写“拈”,②句中“底”和“砥”、“抵”既是同音字,又是形似字,区分难,书写时也易混淆。根据语境应填“砥”。③句中的“诣”应读“yì”。④句中的“鲜”是多音字,句中是“少”的意思,应读“xiǎn”。本题的答案是:①拈 ②砥 ③yì ④xiǎn
   例3.经过初中三年的语文学习,你的字一定写得更漂亮了吧,请你从课文中选取一句你最喜欢的话,写在下面的横线上。
   例4.书写及语音基本功考查。
   说明:本题不需要直接作答,依据你的卷面书写及语言表达基本功来评分。
   这两道题分别是湖北武汉市和恩施市题,都是书写题,也是两道创新题。例3的创新之处在于:题干首先用很亲切的语言肯定考生经过三年的语文学习,字一定写得更漂亮;然后让考生自己从课文中选喜欢的一句话写在横线上;第三是通过对这句话的书写展示自己最好的字。例4的创新之处在于:把全卷书写作为一道题。这种对书写的倡导是很有意义的。
   解答本部分试题,要注意三个方面的问题:一是要熟悉汉语拼音,能读准声母、韵母、声调和整体认读音节;能准确地拼读音节,正确书写声母、韵母和音节;能借助汉语拼音认读汉字。二是要熟记字形、了解字义,尤其是同音字和形似字;三是要按照笔顺规则用硬笔写字,注意字的间架结构。
   【常见失误】
   解答这部分试题常见的毛病是考生受方言的影响,在声母卷舌或不卷舌、韵母是鼻音还是边音上出差错。还有就是多音字的问题,考生也常常产生失误。
   【考点精练】
   1.(2004·天津)下面各组词语中加点字的读音完全正确的一项是( )
   A.附和(hé) 离间(jiàn) 称职(chèng)
   B.嗜好(shì) 辍学(zhuì) 畸形(qí)
   C.惬意(qiè) 挑衅(xìn) 腼腆(tiǎn)
   D.弓弦(xián) 玷污(zhān) 恐吓(xià)
   2.(2004·海淀)词语中加点字注音完全正确的是( )
   A.铿锵(qāng) 挪动(nuó) 不无裨益(bì)
   B.琴弦(xián) 褴褛(lǚ) 载歌载舞(zài)
   C.兑换(duì) 处理(chù) 哄堂大笑(hōng)
   D.摇曳(yè) 别墅(shù) 大事渲染(xuān)
   3.(2004·海淀)文言加点词注音和解释不正确的是( )
   A.夙夜忧叹(sù) 一狼洞其中(打洞)
   B.人不知而不愠(yùn) 才美不外见(同“现”,表现)
   C.及郡下,诣太守(yì) 小大之狱,虽不能察,必以情(案件)
   D.属引凄异(shǔ) 域民不以封疆之界(因为)
   4.(2004·遂宁)下列词语中加线字注音正确的一项是( )
   A.喑哑(yīn) 裨益(bì) 粗犷(kuǎng) 迥异(jiǒng)
   B.禁锢(gù) 玷污(diàn) 狭隘(ài) 不屑(xuè)
   C.雾霭(ǎi) 缄默(jiān) 附和(hè) 拮据(jù)
   D.畸形(jī) 造诣(yì) 渲染(xuàn) 坚韧(rèn)
   5.(2004·佛山)选出下列各项词语中加点的字注音完全正确的一项。( )
   A.绚丽(xuàn) 修葺(qì) B.酝酿(liàng ) 涉猎(shè)
   C.粗糙(zāo) 龟裂(jūn) D.轩榭(xiè ) 迸裂(bìng)
   6.(2004·湛江)下列词语中,加点的字注音完全正确的一项是( )
   A.衷心(zhōng) 衣着 (zháo) B.枯萎 (gū) 侥幸(xiāo)
   C. 凝重(níng) 应和 (hè) D.谦逊 (sùn) 婆娑(shā)
   7.(2004·广西)下列词语中加点字注音完全正确的一项是 ( )
   A.债券(juàn) 和谐(jiē) 分外(fèn) 挑拨离间(jiàn)
   B.动辄(zhé) 腼腆(tiǎn) 干涸(hé) 更新换代(gēng)
   C.谦逊 (xùn) 执拗(ào) 招聘(pìng) 破绽百出(zhàn)
   D.恐吓(xià) 檄文(xí) 纸砚(xàn) 随声附和(hè)
   8.(2004·北海)下面加点字注音正确的一项是( )
   A.酝酿(yùn)  绽放(zhàn)  忍俊不禁(jīn)
   B.蜕变(tuì)  畸形(qí)  根深蒂固(dì)
   C.修葺(qì)  窥视(guī)  杞人忧天(qǐ)
   D.猝然(cù)  镂空(lòu)  拈轻怕重(zhān)
   9.(2004·贵阳)选出加点字注音错误的一项。( )
   A.行阵(háng) 尴尬(gān) 豁免(huò)
   B.惬意(xiá) 拮据(jǖ) 穷匮(kúi)
   C.恻隐(cè) 社稷(jì) 驽钝(nú)
   D.侏儒(zhū) 打盹(dǔn) 篝火(gōu)
   10.(2004·哈尔滨)加点字注音完全正确的一项是( )
   A. 徜徉(cháng) 纤维(xiān) 称心(chèng) 苍穹(qióng)
   B. 斟酌(zhēn) 归宿(sù) 迁徙(xǐ) 婆娑(suō)
   C. 粗糙(cāo) 校对(xiào) 卓越(zhuó) 枢纽(shū)
   D. 虐待(nüè) 狼藉(jí) 玷污(zhān) 教诲(huì)
   11.(2004·永州)下列加点字的注音完全正确的一组是 ( )
   A.嶙峋(lín) 龟裂(guī) B.清洌(liàn) 殉职(xùn)
   C.黄晕(yūn) 诧异(chà) D.确凿(záo) 行辈(háng)
   12.(2004·株洲)下列词语中加点字的注音有错误的一组是( )
   A.威慑(shè) 炽热(chì) 惊心动魄(pò)
   B.渲染(xuàn) 畸形(jī) 迥然不同(jiǒng)
   C.可恶(è) 玷污(zhān) 杳无消息(yǎo)
   D.沮丧(jǔ) 屹立(yì) 同舟共济(jì)
   13.(2004·西宁)请把四组词语中加点字的读音完全正确的一项选出来。( )
   A.湍急(tuān) 归省(shěng) 随声附和(hè) 拈轻怕重(zhān)
   B.歇憩(qì) 提防(tí) 千呵万护(hē) 揆情度理(dù)
   C.畸形(jī) 龟裂(jūn) 面面相觑(qù) 姹紫嫣红(chà)
   D.追溯(sù) 供给(gěi) 风流倜傥(tǎng) 屏气凝神(píng)
   14.(2004·临沂)画线字注音完全正确的一项是( )
   A.提防(tí) 蹒跚(pán) 燃炽(chì) 挑拔离间( jiàn)
   B.蹉跎(cuō) 盘踞(jù) 湛蓝(zhàn) 拈轻怕重(niān)
   C.辍学(chuò) 确凿(záo) 憎恶(è) 肆无忌惮(dàn)
   D.罹难(lí) 造诣(zhǐ) 修葺(qì) 接踵而至(zhǒng)
   15.(2004·广安)下列加点字注音有错的一项是( )
   A.竹篾(miè) B.成绩差(chà)
   C.仄仄(zà zà)的楼梯 D.撺掇(cuān duo)起来
16.(2004·泸州)下列加点字注音正确的一项是( )
   A.斗笠(lì) 攒(zhuàn)积 B.黄晕(hùn) 忍俊不禁(jīn)
   C. 归省(shěng) 修葺(qì) D.阴晦(huì) 诘(jié)责
   17.(2004·内江)下列词语中加点字读音不完全相同的一项是( )
   A.谛听 缔造 帝王 递增 B.休憩 迄今 器材 哭泣
   C.定单 绽开 锭子 沉淀 D.义务 游弋 艺术 议会
   18.(2004·四川)下面加点字注音完全正确的一项是( )
   A.酝酿(ràng) 有条不紊(wěn) 姹(chā)紫嫣红
   B.姹(chà)异 悄(xiāo)无声息 安然无恙(yàng)
   C.峰峦(luán) 不省(shěng)人事 嗟(jiē)来之食
   D.阔绰(chuò) 坚持不懈(xiè) 相形见绌(chù)
   19.(2004·呼和浩特)读下面的句子,根据拼音填写汉字,给加点的字注音。
   ①桥在河上,位置较低,kuí( )情度理,不可能使人产生月亮从那个方向落下去了的印象。
   ②德拉总会有一天把头发悬在窗外去晾干,只是为了使那位皇后的珠宝和首饰相形见绌( )。
   ③阴mái( )弥漫的冬天是毫无生气的季节。
   ④容不得束缚,容不得羁( )绊,容不得闭塞。
   20(2004·湟中)将拼音所表示的汉字及加点字的注音分别填在文中的横线上。
   长江三峡以其险jùn 的地形、绮 丽的风光、磅礴 的气势和众多的名胜古迹闻名xiá 迩,是世界著名的旅游胜地。
   21.(2004·福州)根据拼音在田字格里工整地写出汉字,并给加点字注音。
   shǎn shuò yúnchèn
   
  
   ①星光 ②结构
  
   ③春风和煦( ) ④一唱一和( )
   22.(2004·兰州)给加点的生字注音。
   峥嵘( )岁月 相形见绌( ) 沁( )人心脾
   23.(2004·河南)给下面加点的字注音,根据拼音写汉字。
   (1)惟妙惟肖( ) (2)参天耸( )立
   (3)yǎn( )旗息鼓 (4)jī( )形发展
   24.(2004·荆州)在括号中清晰地填写常用字。
   下了雪,我扫出一块空地来,用短棒支起一个大竹匾,sǎ( )下秕谷,看鸟雀来吃时,我远远地将fù( )在棒上的shéng( )子只一拉,那鸟雀就zhào( )在竹匾下了。
   25.(2004·十堰)根据拼音用正楷写出汉字。
   chóu tián sǎ
   ①( )划 ②( )静 ③挥( )自如
   26.(2004·常州)阅读下面文字,把文中拼音所表示的汉字依次写在文下方格内。
   你的价值,决定于你在多大程度上解放你自己,开tuò你自己。你是平平淡淡,抑或轰轰烈烈,只在于你如何去选择;你是停zhì沉寂,还是奔téng呼啸,只在于你怎样去拼bó。
   (王安雄《生命之琴》)
  
  
   27. (2004·淮安)根据拼音用正楷写出相应的汉字。
   gǒu lún
   ①一丝不 ② 巧妙绝
   jiàng náo
   ③别具 心 ④ 不屈不
   28.(2004·南京)给加点字注音,根据拼音写汉字。
   ①应和( ) ②安然无恙( ) ③寂mò ④huǎng 然大悟
   29.(2004·南通)根据拼音用正楷写出相应的汉字。
   dòng( )梁 xián( )熟 簇yǒng( )
   30.(2004·宿迁)根据拼音用正楷写出相应的汉字。
   ①俯 (kàn) ②(shū) 纽
   ③闪 (shuò) ④(jī) 形
   31.(2004·无锡)根据拼音写出汉字。
   ①胆(qiè)    ②狭(ài)    ③无边无(yín)    ④造(yì)  
   32.(2004·宁德)按拼音写汉字,或给加点字注音。
   kuì( )不成军 雅俗共shǎng( ) 以身殉( )职 深恶( )痛疾
   33.(2004·宾州)给下列词语中加点的字注音,或根据注音写出汉字。
   ①不屑(  )置辩 ②杳( )无消息 ③锲( )而不舍
   ④中流dī(   )柱 ⑤sù(  )兴夜寐 ⑥气势磅bó( )
   34.(2004·北培)下列各项中,没有错别字的一项是( )
   A.烦燥 荣誉 迫不急待
   B.覆盖 喧闹 无精打采
   C.扰乱 坐位 通霄达旦
   D.稀罕 玄耀 生动曲折
   35.(2004·青岛)下列词语中有错别字的一项是(  )
   A.眼花瞭乱 肆虐     B.南辕北辙 雄浑
   C.姹紫嫣红 苍穹     D.拈轻怕重 匿名
   36.(2004·青海)下列词语的音、形和对加点字字义的解释全都正确的一项是( )
   A.怪癖(pì) 與论 肆虐(残暴)
   B.旌(jīn)旗 惋惜 迁徙(调动)
   C.剽(piāo)悍 瞬息 热忱(真实的情意)
   D.贮(zhù)藏 痉栾 希冀(希望)
   37.(2004·北京)词语书写有误的是( )
   A.因地制宜 不屑置辩 巧妙绝纶
   B.郑重其事 改邪归正 威武不屈
   C.别具匠心 世外桃源 政通人和
   D.抑扬顿挫 纵横决荡 急中生智
   38.(2004·包头)没有错别字的一项是( )
   A.标致 朦胧 斑斓 随声附合
   B.吹嘘 游弋 臃肿 获益匪浅
   C.海鸥 葱笼 孕育 重峦叠嶂
   D.缈小 宽敞 抽噎 别具匠心
   39.(2004·佛山)选出下列各项词语中没有错别字的一项。( )
   A.油光可签 贮藏 B.姹紫艳红 衰竭
   C.重峦叠障 栖息 D.无暇顾及 鄙夷
40.(2004·湛江)下列词语中,没有错别字的一项是( )
   A.学籍 不耻下问 拈轻怕重 B.安祥 五体投地 天翻地复
   C.震作 因地致宜 见异思迁 D.修葺 张冠李带 以身作则
   41.(2004·北海)下列句子中没有错别字的一项是(  )
   A.同学之间应和睦相处,任何一句挑拨离间的话都会影响团结。
   B.要想成就一番事业,积极的心态则是成功的起点。
   C.急烈的比赛就要开始了,大家既兴奋又紧张。
   D.这条商业街十分繁华,从早到晚,行人穿流不息,热闹非凡。
   42.(2004·襄樊)下列句子中没有错别字的一项是( )
   A.全体队员团结协作,终于攻刻了难关。
   B.变换无常的天气严重影响了施工进程。
   C.任长霞同志情系百姓的事迹令人感慨万千。
   D.建立“校园网”便于实行电子学藉管理。
   43.(2004·永州)下列各组词的字形全对的一组是 ( )
   A.畸形 劳禄 家喻户晓
   B.萧瑟 辩别 眼花嘹乱
   C.匀称 伟岸 不胜枚举
   D.参天 提防 声名狼籍
   44.(2004·安徽)下列句子中有错别字的一项是( )
   A.听了她的深情倾诉,老李也禁不住动了恻隐之心。
   B.既使你有出众的才华,也不能这样张狂的炫耀自己。
   C.这首诗饱含着真情实感,让那些苍白的口号诗相形见绌。
   D.快意同舒适像是一对孪生兄弟,时而相傍相依,时而南辕北辙。
   45.(2004·广安)下列有错别字的一项是( )
   A.近几年来,父亲和我都是东奔西走,家中光景是一日不如一日。
   B.这些规矩,也大概是我所不耐烦的。
   C.苏州园林裁种和修剪树木也着眼在画意。
   D.我于一八八一年生于浙江省绍兴府城里的一家姓周的家里。
   46.(2004·泸州)下列词语中有错别字的一项是( )
   A.人声鼎沸 稍纵即逝 销声匿迹
   D.俯首帖耳 拍案叫决 锐不可挡
   C.不毛之地 天经地义 与日俱增
   D.深恶痛疾 姹紫嫣红 不屑置辩
   47.(2004·自贡)下列词语中,没有错别字的一项是( )
   A.渐渐暗哑 杳无消息 抑杨顿挫
   B.傲然廷立 无精打彩 无缘无故
   C.身材魁梧 情不自禁 慷概解囊
   D.自强不息 中流砥柱 旁逸斜出
   48.(2004·嘉兴)下列词语书写全部正确的一项是( )
   A.按排 枯躁乏味 病人膏肓 B.肆虐 风靡一时 出类拔萃
   C.夜宵 坚苦卓绝 雅雀无声 D.脉搏 世外桃园 应接不瑕
   49(2004·绵阳)下列词语中,没有错别字的一项是( )
   A.安详 变换莫测 与日俱增 心有灵犀一点通
   B.派遣 浑然天成 家喻户晓 前事不忘,后事之师
   C.迁徙 目不识丁 获益匪浅 一年之际在于春
   D.惩诫 得天独厚 忧心忡忡 有福同想,有难同当
   50.(2004·广州)下列句子中加点的词语,没有错别字的一项是( )
   A.在恶势力猖獗之际,有的人苟且偷生、不辨是非,有的人为虎作伥、助纣为虐。
   B.生意场上,判断要准确,决定要坚决,绝对不能优 柔寡断,否则将会坐失良机。
   C.教书育人,培养新世纪的优秀人才,在座各位教师是责无旁代、义不容辞的。
   D.现在有少数党员干部,忘记党的宗旨,循私舞弊,贪污腐化,走上了犯罪道路。
   51.(2004·海淀)下列成语中有四个错别字,改正后将正确的写在答题卡的方格内。
   巧妙绝伦 才书学浅 猝不及防 呕心沥血 丰功伟绩 通霄达旦
   坦荡如坻 相映成趣 忘自尊大 花枝招展 世外桃园 声色俱厉
   52.(2004·广西)下列词语中共有四个错别字,请找出来,然后按顺序改正在下面的方格中。
   (1)甘拜下风 (2)相形见拙 (3)脍灸人口 (4)锐不可当 (5)油光可见
   (6)漠不关心 (7)老奸巨滑 (8)故弄玄虚 (9)再拦再厉 (10)义不容辞
   错字:
   改正:
   53.(2004·宿迁)下列词语中共有四个错别字,先在错别字下画横线,然后在方格中订正。
   身体力行 一愁莫展 相提并论 司空见惯 因地制宜
   安部当车 幕天席地 辩伪去妄 神采奕奕 离经判道
  
   54. (2004·锦州)找出下列词语中的两个错别字,将正确的字工整地写在田字格里。
     不寒而栗    理屈辞穷    嘎然而止    刻不容缓
            
   55.(2004·西宁)假如你是文字编辑,请从下列十个词语中找出五个错别字,把它依次改正过来。
   ①墨守陈规 ②以德报怨 ③辩伪去妄 ④意兴阑珊 ⑤根深蒂固
   ⑥寻人启示 ⑦默默无蚊 ⑧明察秋毫 ⑨鸡飞旦打 ⑩波光粼粼
  
   更正:
  
   56.(2004·内江)下面一段文字有3个错别字,找出来并改正,工整的写在后面的方框内。
   我没想到,在面临种族灭绝的关建时刻,斑羚群竞然能想出牺牲一半挽救另一半的办法来赢得种群的生存机会。我没想到,老斑羚们会那么从容地走象死亡。
   ① ② ③
   57.(2004·新疆)假如你是文字编辑,你能找出下列成语中的4个错别字并改正吗?
   莫名奇妙 异想天开 消声匿迹 德高望众
   司空见惯 戒骄戒燥 充耳不闻 不攻自破
   错字
   改正
   58.(2004·长沙)许多广告词借用了成语、熟语,取谐音换新义,朗朗上口,但却给我们的学习带来了许多陷阱,请识别下面广告词中的谐音字,并将其还原,写在后面的括号内。
   例:空调机——完美无夏(瑕)
   A.沐浴器——随心所浴( )
   B.咳嗽药——咳不容缓( )
   C.洗衣机——爱不湿手( )
   D.蚊 香——默默无蚊( )
   59.(2004·南宁)下面加点的字有两对同音字,请你把它们找出来。
   晨曦微露,徜徉山野搜寻前人的遗踪,迎面吹来和煦的春风,小鸟在枝头翩然起舞,鱼儿在水中快乐嬉戏,真是美不胜收,令人心旷神怡!
   ① 同 ② 同
   60.(2004·海口)认真书写,养成习惯。下列成语有错别字,请改正,将正确的成语用你最满意的字体抄写在下面的田字格中。
   步步为赢 咳不容缓


第二讲 词语和句子

   【考查要点】
   词语、成语、句子都是重要的语文素养之一。课标在“阅读”第4条强调:要“体味和推敲重要词句在语言环境中的意义和作用”。现行教材要求考生要掌握一定的词汇,要了解单义词和多义词、同义词和反义词、褒义词和贬义词等,能判断病句的类型并正确地进行修改。
   【知识疏理】
   2004年全国各地中考语文试题在词语、成语方面,主要涉及这样一些知识点:①选择词语填空;②填关联词语;③词语解释、语境词义解释;④成语填空、成语运用、成语解释等。
   句子的考查重在运用,如句子有没有语病,对病句加以修改等。
   【试题特点】
   2004年本部分中考题的题型,主要有填空题、选择题等,以选择题为主。
   【解题导引】
   例1.(2003·北京)根据句意依次填写词语最恰当的是( )
   (1)傍晚时候,上灯了,一点点黄晕的光,( )出一片安静而和平的夜。
   (2)我们上了轮船,离开栈桥,在一片( )的好似绿色大理石桌面的海上驶向远处。
   (3)汽车在望不到边际的高原上奔驰,( )你的视野的,是黄绿错综的一条大毡子。
   (4)沙丘的移动虽然慢,可是所到之处,森林全被( ),田园全被埋葬,城郭变成丘墟。
   A.烘托 寂静 进入 摧毁
   B.映衬 平静 进入 毁坏
   C.烘托 平静 扑入 摧毁
   D.映衬 寂静 扑入 毁坏
   此题考查“根据语言环境选择词语”的能力。(1)句中“一点点黄晕的光”是柔和的,只能是作为陪衬;(2)句中“好似绿色大理石桌面的海上”突出了“平”而非“寂”;(3)句依据“汽车在望不到边际的高原上奔驰”这一具体语境,用“扑入”更加生动形象;(4)句中“摧毁”比“毁坏”更能突出沙丘给人类造成危害的严重程度。经过这样认真的分析、选择,不难得出此题的答案为C。我们在解答此类题时,要审清词干要求,抓住题干中的关键性词语即“最恰当”,根据词语所处的具体语境,从词语的范围大小、语意轻重、感情色彩、搭配关系等方面进行分析,逐一辨析选项,得出正确的答案。
   例2.(2003·湛江)下列句子中,没有语病的一项是( )
   A.能否推进素质教育是保证青少年健康成长的条件之一。
   B.学校新的领导班子健全并建立了一整套班级管理制度。
   C.这次会议规定每一位发言者的发言时间最多不能超过30分钟。
   D.湛江的亚热带城市风光迷人,还有那丰富味美的海鲜也吸引着大量的游客。
   此题考查病句的辨别能力。解答时首先要明确病句的类型,它包括词语误用、成分残缺、搭配不当(主谓、动宾、中心语与修饰语等)、语序不合理、结构混乱、表意不明、重复累赘、前后矛盾、照应不周、概念不清等。然后根据这些类型逐一辨别各项。上题中的A项搭配不当,B项“健全并建立”语序不当,C项“最多不能超过”重复累赘,只有D项无语病,为本题的正确答案。
   例3.(2003·扬州)下列两个句子各有一处语病,请在原句上修改(删、添、改、调,不得改变原意)。
   (1)扬州市歌《茉莉花》具有鲜艳的地方色彩。
   (2)在抗击非典的战斗中,护士叶欣的献身精神深深地激励、感染了众多的白衣战士。
   此题为主观型题,修改时只要根据病句的类型进行对号,一一检查,就不难作出正确的修改。此题的正确答案是:(1)将“鲜艳”改为“鲜明”(“浓厚”、“浓郁”等);(2)将“激励”与“感染”对调。
   解答本部分试题,要注意三个方面的问题:一是要掌握一定数量的词语、成语并对其含义有所了解;二是要注意词语、成语所在的具体的语言环境并了解其基本的用法;三是要掌握修改病句的一些基本方法。
   【常见失误】
   解答词语试题常见的毛病是考生不太注意词语、成语所在的具体的语言环境,现代词语和文言词语不太注意对其义项的了解。这两个问题都是值得注意的。
   解答病句题往往找不准病因,因此在修改时就存在胡乱猜测的现象,该修改地方没改,不该修改的地方又乱改。
   【考点精练】
   1.(2004·北京)句中加点词解释不当的是( )
   A.为了带领乡亲告别贫穷,共同致富,老将军从不计较个人的荣辱得失。
   计较:计算比较。
   B.联合国文件用语讲究含蓄和宛转,多用中性词。例如,不说“敌人”而说“对手”。
   宛转:温和而曲折。
   C.提着理想的明灯走在人生的道路上,即使路途坎坷,也不必过多踌躇和张望。
   踌躇:得意的样子。
   D.据说“环滁皆山也”一句修改了几十次才确定下来,可见作者十分讲究锤炼语言。
   锤炼:反复推敲,加以提炼。
   2.(2004·重庆)对下列加点的词语在句中的含义,理解不正确的一项是( )
   A.有些人宁愿循规蹈矩,落个疲惫不堪,也不肯运用心智去研究和思考,以改进方法提高工作效率。(循规蹈矩:在这里是遵守规矩的意思。)
   B.许多科幻书对地球的前景都抱着一种悲观的态度,这并非杞人忧天,如果我们不注意保护环境,科幻书中所描绘的情景就会成为可怕的现实。 (杞人忧天:此处比喻不必要的忧虑。)
   C.在深海和远洋中还有许许多多尚未被我们充分利用的海洋生物,其巨大潜力是不言而喻的。(不言而喻:此处指众多海洋生物的潜力是无法用语言来形容的。)
   D.为了给失学的孩子筹建“希望小学”,他呕心沥血;新学校落成时,这位钢铁般的汉子也流泪了。(呕心沥血:此处形容费尽心思。)
   3.(2004·佛山)根据语境,选出对下列句子加点词语理解不正确的一项。( )
   A.人们在这无鱼无草的海水里,竟能自由游弋。(游来游去)
   B.据说他当初行为不正,糟蹋钱。(浪费)
   C.这藤野先生,据说是穿衣服太模糊了。(不清楚)
   D.这些日子,家中光景很是惨淡。(境况)
   4.(2004·广安)对下列词语解释有错的一项是( )
   A.“留取丹心照汗青”中的“汗青”指史册。
   B.“远近横着几个萧索的荒村”中的“萧索”指稀少。
   C.客死:死在他乡异国。
   D.落第:科举时代应试不中。
   5.(2004·资阳)结合语境解释加点词有误的一项是( )
   A.总之,一切都要为构成完美的图画而存在,决不容许有欠美伤美的败笔。
   败笔:句中指影响美观的微小缺点。
   B.我希望他们不再像我,又大家隔膜起来。
   隔膜:彼此思想感情不相通。
   C.莫扎特使我觉得左右逢源,俯拾即是,行云流水。
   左右逢源:句中指欣赏音乐的一种和谐状态。
   D.此去泉台招旧部,旌旗十万斩阎罗。
   旌旗:句中指的是军中的旗帜。
6.(2004·自贡)下列加点词的意思,解释不正确的一项是( )
   A.我的祈愿 祈愿:请求,希望。
   B.油光可鉴 可鉴:可以照明。
   C.仁慈恻隐 恻隐:对受苦难的人表示同情。
   D.恹恹欲睡 恹恹:精神不好,困倦的样子。
   7.(2004·遂宁)括号中对加线词语的解释正确是一项是( )
   A.蜜蜂是渺小的,蜜蜂又是多么高尚啊!(渺小:微不足道)
   B.后来发生了分歧:母亲要走大路,大路平顺;我的儿子要走小路,小路有意思。(分歧:因意见不一致而发生争执)
   C.就是下小雪吧,济南是受不住大雪的,那些小山太秀气。(秀气:小巧而灵便)
   D.在改革开放的大好形势下,遂宁的工农业发展如日中天。(如日中天:比喻事物正发展到十分兴盛的阶段)
   8.(2004·安徽)依次填入下列横线处的词,恰当的一项是( )
   如果不是秋风将种子吹下茎杆,那种子不是要被酷寒所冻僵吗?是秋风,把种子介绍给根须,使它找到了__生命的母体;是秋风,把种子藏进了厚实的泥土,使它有了一个__春天的温床;是秋风,把果实领进了一个温暖的家,使它__了生命的胚胎。
   A.延伸 萌生 保持  B.延续 萌动 保持
   C.延续 萌生 保存  D.延伸 萌动 保存
   9.(2004·包头)填在空缺处的词语最恰当的一项是( )
   置身于一方有书的天地,触目那些或发黄或簇新的书籍,它睿智的灵光会将我们的眼睛映照得 。打开书,走进 的思想丛林,我们便会顿觉异香弥漫, ,总能在油墨的芬芳中感悟出什么。不我们与书中那 的人和事交融在一起的时候,我们便会发觉生命曾经隐忍的种种深义。
   A.炯然有神 五彩缤纷 陶然自得 形色各异
   B.炯然有神 五彩缤纷 沁人肺腑 形色各异
   C.目光炯炯 五光十色 陶然自得 形色各异
   D.目光炯炯 五光十色 陶然自得 形形色色
   10.(2004·北京)依次填入句中横线处的词语恰当的是( )
   ①文物古迹是人类文明的 ,是祖先遗留下来的人类共同的财富。
   ②《钱塘湖春行》一诗在“早莺”“新燕”“乱花”“浅草”中蕴含着白居易的喜悦之情,
   这首诗的意境美,就是要品味诗中形象所表现出来的境界和情调。
   ③这位漫画家与别人不同,在自己的名片上印上幽默的自画像,真是 。
   ④有些人忽略了这个时代发展中的许多有利因素,讲一些 的话,如同激昂的交响乐中出现了不和谐的音符。
   A.标志 鉴赏 别出心裁 妄自菲薄 B.标致 鉴赏 别开生面 妄自尊大
   C.标志 鉴别 别出心裁 妄自尊大 D.标致 鉴别 别开生面 妄自菲薄
   11.(2004·遂宁)根据句意依次填写词语最恰当的一项是( )
   ①古代流传下来的许多农谚就包含了 的物候知识。
   ②他现在是单枪匹马,完全 在敌人控制的国土里。
   ③……那便是别人的传说,不一定可靠;要知道实际的情形,只有靠自己亲自去 。
   ④她想到织女每年还能和牛郎相会一次,而自己却被禁闭在深宫里,永远度着 的时光。
   A.丰富 陷落 观察 孤寂 B.丰厚 陷落 探察 孤苦
   C.丰富 沦落 观察 孤单 D.丰富 沦陷 观测 孤寂
   12.(2004·贵阳)请结合语境,选择恰当的词语填在横线上。(可填序号)
   备选词语:①拘谨 ②晦暗 ③滋润 ④笨拙 ⑤酸楚 ⑥终身受益
   ⑦心旷神怡 ⑧郑重其事 ⑨振翅高飞 ⑩豁然开朗
   “书籍是人类精神的食粮”。—本好书,可以 你的心田;一篇美文可以让你在失意烦恼时感到 ;即使是一则短小的寓言,也可以让你在迷惘时眼前一亮,心中 。亲爱的朋友,莫观望,莫等待,赶快 ,飞向书籍的太空尽情遨游吧!
   13.(2004·泸州)在下面语段的横线上依次填入关联词语正确的一项是( )
   有一知半解的人,并无数学知识, 根据《圣经》这一段或那—段妄肆批评或者驳斥我的著作,我    不预备答复他们,    还要轻视这样无知的见解。
   A.假使 而 不但 而且     B.如果 就 因为 所以
   C. 因为 所以 如果 那么    D.虽然 但是 不但 而且
   14.(2004·广西)填入空白处的关联词最恰当的一项是 ( )
   说五四时期的那一代有志青年为了中国的前途和命运曾在黑暗中苦苦求索是艰辛的,当代青年则是幸运的,在改革开放的实践中,我们已经找到了当代中国的马克思主义——邓小平理论,这一理论使我们创造了过去十年的辉煌,也将使我们在新世纪里继续创造辉煌。
   A.尽管 那么 因此 不但 B.如果 那么 因为 不仅
   C.如果 所以 既然 不仅 D.尽管 但是 既然 不但
   15.(2004·重庆)在下面语段横线上依次填人关联词语,最准确的一项是( )
   在一定条件下,科学知识之所以正确是因为经过了实践的检验。 条件变化了,原有的科学知识会被人们用新的实践去检验, 会被修改和发展成新的科学知识。但人们之所以要不断学习 是因为原有知识统统“过期变质”, 是因为新条件下产生的新知识能使人们的知识、思维和智慧更上一层楼。
   A.如果 从而 并非 而 B.如果 从而 不仅 而且
   C.虽然 但是 不仅 而且 D.虽然 但是 并非 而
   16.(2004·兰州)下列加点的词语运用不当的一项是( )
   A.队伍沿着山道迤逦而行。
   B.银杏树根深蒂固,抗风力很强,是扩大绿化面积、治理山河的优良树种。
   C.不想误触犯了官人,望乞恕罪,高抬贵手!
   D.他一个部曹,戴了个水晶顶子去当会办,比着那红蓝色的顶子,未免相见恨晚。
   17.(2004·佛山)选出下列句子中加点的成语使用恰当的一项。( )
   A.老百姓对脍炙人口的阜阳劣质奶粉事件感到震惊。
   B.禅城区设置路边停车场应该说是一举两得的好事。
   C.在公安机关的严厉打击下,传销集团锐不可当地被捣毁。
   D.我和老王单枪匹马参加市运动会,夺得跳远冠军。
   18.(2004·北海)找出下列句子中加点词语使用不当的一项(  )
   A.大家都能积极参加学校组织的献爱心活动,谁也不会袖手旁观。
   B.经过不断的试验,反复的推敲,关键难题一旦攻克,其他问题也就迎刃而解了。
   C.李白的“白发三千丈,缘愁似个长”把郁积在诗人心中的“愁”抒发得淋漓尽致。
   D.班会上,大家一致推选德高望重的小华担任班长。
   19.(2004·河池)下列句子中加点的词语使用不当的一项是( )
   A.花朵儿一串挨着一串,一朵接着一朵,彼此推着挤着,好不活泼热闹。
   B.莺的歌声十分美妙动听,这歌声仿佛涵养了树林的清新,描绘了恬静的生活,表达了幸福的感受。
   C.越是笨重的石块越跑不远,越是轻小的沙砾越能旅行到遥远的地方。
   D.一条条整齐的街道,一座座新建的楼房,短短三年,故乡已经面目全非了。
   20.(2004·襄樊)下列各句中加点词语使用恰当的一项是( )
   A.任何不称职的或者愚蠢得病入膏肓的人,都看不见这衣服。
   B.老师诲人不倦的嘱托,一直鞭策着我不停地前进。
   C.设计者和匠师们因地制宜,自出心裁,修建成功的园林当然各各不同。
   D.桂林的山水黄山的松,真是巧夺天工。 
   21.(2004·嘉兴)下列句中加点的成语使用正确的一项是( )
   A.这次艺术节办得栩栩如生,全校师生交口称赞。
   B.执法部门要善于虚张声势,使假冒伪劣产品无藏身之地。
   C.老师几句点评,使我茅塞顿开:“啊,这篇文章原来还有这么深刻的内涵。”
   D.我们每年都在家乡植树,现在,那里的树已是鳞次栉比,鸟儿也回来了。
   22.(2004·内江)随着社会的进步和文明的发展,人性得到了前所未有的张扬。请写出3个以“人”字开头的成语
   人 人 人
   23.2004·锦州)下面一段话中有一个词语用得不恰当,请找出并加以改正。
     生活是一个多棱镜,总是以它变幻莫测的每一面折射出生活中的每一个人。不必介意别人的流言蜚语,不必担心自我思维的偏差,要坚信自己的判断,用敏捷的目光去审视这个世界,给自己一个富有个性的回答。
     _____________改为_______________
   24.(2004·福州)写出与下面历史人物相关的成语,再另选一个历史人物,并写出相关的成语。
   ①吕蒙—— ②诸葛亮——
   ③项羽—— ④( )——
25.(2004·福州)下面是一则某市中学生篮球比赛的报道稿,文中连用5个“战胜”,行文显得单调。请根据不同的比分,用4个“战胜”的同义词语分别填入括号,替代文中加点的“战胜”,使用词准确又富有变化。
   今天进行的中学生男篮预选赛中,一中队以72比60战胜十五中队,十八中队以68比56战胜( )八中队,二中队以98比42战胜( )十一中队,三中队以78比77战胜( )七中队,二十中队以86比80战胜( )上届冠军五中队。
   26.(2004·北京)每项有两句话,意思不一致的是( )
   A. 获得是一种满足,给予是一种快乐。
   获得是一种满足,给予何尝不是一种快乐!
   B. 一时的错误不会毁掉一个性格坚强的人。
   一个性格坚强的人不会被一时的错误毁掉。
   C. 因为物种丰富、生态平衡,所以环境更美好。
   环境因物种丰富、生态平衡而更美好。
   D. 这个小区的建设规划,没有可取之处。
   这个小区的建设规划,不无可取之处。
   27.(2004·包头)没有语病的一句是( )
   A.我校深化教学改革,改善师资队伍,优化教学条件,强化管理体系,形成了正确的教学思想和办学特色。
   B.专家们呼吁,每个人都要坦率地面对心理问题,像追求躯体健康一样去追求心理健康。
   C.研究证实:孩子挑食偏食只与后天养成的习惯有关,而并非先天遗传所致。
   D.在水果中除了坚果的钙、铁、磷含量比蔬菜高外,一般都低于蔬菜含量。
   28.(2004·内江)下列语句中有语病的一项是( )
   A.内江地理位置优越,交通发达,劳动力资源丰富,是商家投资的沃土。
   B.通过“告别网吧”的主题班会,让同学受到很大的触动。
   C.《现代汉语词典》是一本在历史上有着十分重要的意义、起着引导规范汉语作用的重要词典。
   D.草长莺飞的季节,我不禁怀念起遥远的江南水乡。
   29.(2004·嘉兴)下列句子没有语病的一项是( )
   A.通过这次社会调查,使我认识到人类要与动物和谐共处。
   B.听到几位华人拾贝者在英国遇难的消息,我的心长时间久久不能平静。
   C.伊拉克局势能不能稳定下来,是联合国工作人员返回巴格达工作的基本条件。
   D.“神舟”五号载人航天飞船的成功发射和顺利返回,是载入中华民族史册的不朽盛事。
   30.(2004·温州)下面句子中没有语病的一项是( )
   A.同学们对学校的教育课程改革交换了广泛的意见。
   B.经过几个月的培训,使我掌握了使用多媒体的技术。
   C.我们在学习上应该树立起不畏艰难的勇气。
   D.房内飘出了一股龙井茶的清香。
   31.(2004·遂宁)下列句子没有语病的一句是( )
   A.自从实施素质教育以来,学生的学习方式得到了明显提高。
   B.不管天晴下雨,还是刮风飘雪,他却能按时到校。
   C.通过这次社会实践活动,使我们磨练了意志,增长了见识。
   D.人的天性犹如野生的花草,求知学习好比修剪移栽。
   32.(2004·兰州)选出没有语病的一项( )
   A.全班同学和团员在课堂上都能积极发言。
   B.一只狗在一只麻雀看来该是多么大的怪物啊!
   C.通过这次丰富多彩的活动,使我们明白了许多做人的道理。
   D.我国的人口占世界的五分之一是世界上最多的国家。
   33.(2004·襄樊)下列句子没有语病的一项是( )
   A.李校长工作很忙,常常要接待许多学生家长的来访和来信。
   B.在政府及教育界知名人士的努力下,《隆中对》终于被编入了新教材。
   C.书法家米蒂的故居是襄攀市新修建的汉江“外滩”。
   D.能否增强法律意识是青少年健康成长的保证。
   34.(2004·永州)下列句子没有语病的一句是 ( )
   A.通过学习航天英雄杨利伟的事迹,使我受到了很大的鼓舞。
   B.我们应该遵守交通规则,以免不发生事故。
   C.中国人民正在为努力建设一个小康社会。
   D.不是困难吓倒了我们,而是我们战胜了困难。
   35.(2004·株洲)下列句子中没有语病的一句是( )
   A.是否具有良好的心理素质,是考试取得好成绩的条件之一。
   B.假山的堆叠,可以说是一项艺术而不仅是技术。
   C.今天想来,她对我的爱好文学和接近文学,是有着多么有益的影响!
   D.昨天,学样发生了一件虽然不大却耐人寻味的小事。
   36.(2004·佛山)选出下列句子没有语病的一项。(C)
   A.经过大家的努力,使我们出色地完成了上级交给的任务。
   B.是否有“以农民为本”的理念,是解决“三农”问题的关键。
   C.手机作为科技进步带给人类的新工具,本身并无利弊对错。
   D.即使一直没有试验成功,但他一点儿也不气馁。
   37.(2004·肇庆)下列文段摘自一位中学生的习作,其中画横线的句子有语病,请任选四处提出修改意见。
   在学习和生活中,人们难免会遇到各种各样的挫折,当然,①挫折会因人们对待它的态度的不同而给人们带来不同的结果。有人会因为挫折而激起旺盛的斗志,②最终成就一番理想;也有人会因为一次挫折而灰心丧气,一蹶不振,③沦为一个毫无作为一事无成的庸人。勇敢的人往往能直面挫折,把它当作前进的垫脚石;④而那些长期在安逸生活中养成了懦弱性格的人则总是害怕挫折,无法承受任何挫折的打击,把挫折看成可怕的陷阱。但是,只要他们能学会在挫折中汲取力量和勇气,⑤并磨炼出坚忍不拔,就一定能够战胜挫折,创造一个美好的人生。
   38.(2004·河南)修改下的病句。
   (1)她的画简洁明快,跳动着青春的气息,一直都是同学们所喜闻乐见的。
   ___________________________________________________________________________
   (2)齐晖的这片作文,结构严谨,语言流畅,在全市作文比赛中把它评为一等奖。
   ___________________________________________________________________________
   39.(2004·常州)下面是某同学作文中的一个片断,仔细阅读后,请选择4-6处(不得超过6处),用规定的符号(不一定全使用)加以修改。
  
   (1)换用号: (2)增补号: (3)删除号: (4)调位号:
  
   人 生 是 无 法 回 头 再 来 的,然 而 做 每 件 事 都 要 细 心 瑾 慎、 小 心 翼 翼。有 时 也 许 就 是 因 为 一 个 不 留 神 的 错 误 , 造 成 无 法 挽 回 的 一 个 遗 憾。
   朋 友 :珍 重 地 走 好 每 一 步 ,不 要 对 小 事 就 吊 以 轻 心 ,希 望 你 稳 健 地 走 完 自 己 的 旅 程 。
   40.(2004·南通)下面语段中有两个病句,请把它们找出来,并加以改正。
   ①有关医学人士与心理学家认为,一些青少年长期迷恋上网,会患上一种病——“网络成瘾症”。②这种病的主要症状有:精神依赖、茶饭不思、记忆力减退和焦躁不安等。③一些家长没有意识到这是一种病,而采用简单粗暴的方式阻止孩子不上网。④为此,专家建议,我们在强化网络监管的同时,也要有效的寻找治疗疾病的方法。
   第—句,修改意见:
   第二句,修改意见:
   41.(2004·威海)下边的病句是从近年来的书报上摘录的,请加以修改。(任选三句)
   ①中国人民是赞同成立联合国的首批国家之一。(1995年9月5日《人民日报》)
   ②晋文公当上了晋国国君后,发展生产,整顿政治,训练军队,成为北方一大强国。(高中历史教科书《中国古代史》第19页)
   ③因此,为了您的人身安全,请勿用左手启动家电,以防万无一失。(1997年2月28日《今晚报》)
   ④他们坐在一张桌子的半径两端,不偏不倚面对着面。(1996年4月 5日《羊城晚报》)
   42.(2004·杭州)下面这段文字中的关联词语,有三处使用不当,请加以修改。
   工作最多的人往往最珍惜时间,这是因为他们有明确的目标,不但为了实现这个目标合 理地安排着自己的工作,而且不是在犹豫不决中浪费时间。因此,有效地利用时间、珍惜时 间只有一个方法,那就是在你的生活中确立一个奋斗目标,并且也尽心尽力地去实现这一目标。
   43.(2004·杭州)下面这段文字有三句话,各有一处语病,请加以修改。
   ①出于保护文化遗址的考虑,余杭良渚文化的大部分遗址区域均处于未发掘。②根据《文物保护法》的规定,大型基建工程施工,必须经过文物部门批准,但有些基建单位却擅自动工,导致破坏性挖掘而感到担忧。③目前,余杭区的文物保护规划已经正在制定中,以后文物保护就有了更为有力的措施。
   44.(2004·绍兴)下面两个病句,请任选一个加以修改。
   ①美军在伊拉克的“虐囚事件”,充分展示了美国政府不尊重生命的“假人权”本质。
   ②2003年10月15日,“神舟五号”载人飞船成功升空,终于成为世界上第三个能独立开展载人航天活动的国家。
   45.(2004·沈阳)下列两个句子各有一处语病,请在原句上修改.
   (1)大家看了赵本山的小品,往往忍俊不禁地笑。
   (2)同学们在考场上能否保持一颗平常心,是正常发挥水平的关键。
   46.(2004·陕西)下面一段话在字词、标点和句子三个方面存在问题,请找出并加以改正。
   豁达是一种坦荡,那心灵像一汪碧泉清澈见底,容不得半点污秽和虚伪,是黑决不会说是白,是鹿决不会说是马。豁达是一种乐观,能驱散生活中的痛苦和眼泪,不论生活怎样拆磨你、欺骗你,你永远都微笑着。豁达是一种自信,“自信人生二百年,会当击水三千里。”生活处处充满竟争和拼搏,在豁达者面前永远没有失败。豁达不是妥协退让,就是大度宽容。
   字词:
   标点:
   句子:
第三讲 语言运用
   【考查要点】
   《语文课程标准》在“前言”中指出:现代社会要求公民“具备包括阅读理解与表达交流在内的多方面的基本能力,以及运用现代技术搜集和处理信息的能力”。因此,语文课程应该“指导学生正确地理解和运用祖国语言”、“注重跨学科的学习”。“课程目标”明确指出:学生要有“较丰富的积累”,“初步具备搜集和处理信息的能力”,“能具体明确、文从字顺地表述自己的意思”,“能从文章中提取主要信息,进行缩写;能根据文章的内在联系和自己的合理想像,进行扩写、续写;能变换文章的文体和表达方式等,进行改写”。因此,我们从仿写、续写、扩写、改写、补写等角度设计了语言运用的训练题,以强化学生的积累与运用,试图以此为中考复习提供一些参考。
   【知识疏理】
   1.语言表述:①信息提取:从学习现象、时事材料、楹联品读;②信息表述:从对联中表述人物故事,用文字把数学题表述出来,用一句话概括材料的内容、观点;③读图表述:新闻、历史画面、漫画、招贴画等;④读表表述:个人所得税、网络使用量等。
   2.语言运用:①语言简明;②句子连贯;③拟写广告词;④仿写句子、语段;⑤选词造句、造段;⑥续写、改写、补写等。
   【试题特点】
   2004年本部分中考题的题型,主要有简述题、概述题、填写题、填空题、扩展题、读图题、读表题、开放题、问答题、改错题、选择题、想像题等多种题型。
   【解题导引】
   例1.(2004·黄冈)阅读下面的表格,按要求回答问题
   世界能源组成年代 石油 煤 天然气 核电 太阳能 水电 其他
   1993年 40% 27% 23% 7% 2% 1%
   2030年(预计) 35% 20%
   ①用文字概括世界能源的发展趋势。
   ②读此表后你有哪些感受?
   ①
   ②
   这是一道渗透着数学知识的图表题,这种题在考查学生的语言及概括能力的同时,还立足边缘学科的双基点,融合在试题里考查学生解决问题的综合能力。解题分三步骤:一、明确题干要求。此题要求概括“世界能源发展趋势”,而不是概括“世界能源组成特点”,如果逐一说明表格内容,就不合题干要求,所以从题干中捕捉有效信息,获得解题要领,是解答此类图表题的基本条件。二、寻求表格规律。此题通过表格来“说话”,它简明直观地把现象展示在读者面前的同时,也会把问题的本质隐藏起来。因此,我们必须采用纵比和横比的方法找出表格数据的变化特点。纵看,本表只有两组数据可供对比,即1993年和2030年世界能源中核电、太阳能的数据,两组数据上涨幅度较大;横看,1993年世界能源主要由石油、煤、天数气组成,2030年(预计)世界能源则主要由核电、太阳能组成。三、准确归纳表达。通过以上分析,我们要用恰当的词语和完整的句子把规律表达出来,做到语言准确、简洁、连贯、符合客观实际。第一问回答可概括为:预计2030年以后,世界能源将以核电和太阳能为主要能源(句中的“预计”、“核电”、“太阳能”、“主要”均为关键词)。第二问谈感受,答案多元,可拟为:①要节约能源;②使用无污染(或绿色清洁)的能源;③要保护环境;④要珍惜资源……
   解答本部分试题,要抓住两个关键:对于语言表述题,要抓住“信息”二字,从文字、图案、图画、图表诸方面获取或表述;对于语言运用题,要抓住“运用”二字,从各种用途上进行语言实践。
   例2.(2004·荆州)提取下面文段中的关键信息,将其写在横线上。
   为了“神舟”五号的发射成功,长征二号F型火箭在历时多年的研制过程中始终将可靠性、安全性放在首位。火箭上多个系统采用了“双保险”设计;同时,提高了元器件质量等级和筛选标准,提高了发动机的可靠性。由于严格的质量控制,该型火箭可靠性指标提高到了目前的0.997,成为目前国内可靠性最高的运载火箭。
   答:
   现代社会其实就是信息社会,交流信息已成为现代社会交际的基本内容,处理信息已成为学习、工作甚至娱乐须臾不可离开的任务。因此我们应学会筛选信息。其方法是:1.速读材料,整体把握;2.认真审题,明确目的;3.缩小范围,提高效率;4.把握线索,跟踪追击;5.严密分析,准确把握。此题的答案可拟为:长征二号F型火箭在历时多年的研制过程中始终将可靠性、安全性放在首位。
   例3.(2004·衡阳)今年春天,在东南亚地区流行的“禽流感”,对人类的生命健康敲响了警钟。请就此拟写两条电视公益广告语,提醒人们增强卫生、健康意识。
   答:
   解答此类题先要审准题目要求(包括字数的要求),然后巧妙化用一些广告词或诗句、名言、格言、警句来答题,一定要注意语言的简明、连贯、得体。答案可拟为:(例5)①“神舟”五号,你是中华民族的骄傲;②利天下流芳百世,振人心居功至伟(首尾嵌入“利伟”);③圆飞天之梦,振中华之威;④愿祖国航天事业如芝麻开花节节高。(例6)①讲究卫生,预防疾病;②为了你和他人的健康,请注意公共卫生;③讲究卫生等于爱护自己的生命。
   例4.(2004·江西)仔细观察下边的漫画,回答问题。
   ⑴请给漫画拟一个标题。
   ⑵请简介漫画的内容。
   ⑶请用一句话谈谈你的感想。
   解答此题先要仔细观察画面:画面的主体是“神舟”五号飞船飞上了太空,画面的右上方画有月宫的嫦娥捧着桂花迎接“飞船”,还有天宫的悟空向“杨利伟”送来“天宫蟠桃”。结合平时的所见所闻,不难看出这幅画是颂扬我国“神舟”五号飞船成功升空的壮举的。答案可拟为:⑴①神舟人,欢迎你;②“神舟”游太空;③“神舟”升天,仙人道喜;④“神舟”五号访太空;⑤众仙献宝迎“神舟”;⑥飞天成真,仙人同庆。⑵①杨利伟坐着“神舟”五号飞上天了,孙悟空端着“天宫蟠桃”、嫦娥提着装有“月宫桂花”的篮子,他们都来欢迎杨利伟;②当“神舟”五号飞船升上太空时,嫦娥姐姐向宇航员撒下鲜花,诸多神仙端来仙果庆祝,我们的宇航员杨利伟正向他们挥手致意;③“神舟”五号飞船终于上天了,各路神仙笑容满面,喜迎神舟人。⑶①如今,登天已不是难事,说不定过不了多久,我们地球人还要开发火星呢!②祖国多年的飞天梦终于圆了,我为祖国而骄傲!③没有不能实现的梦,只要我们努力,梦就不会是梦了,要发展科学!④我要学习杨利伟,将来像他一样驾驭“神舟”号,为祖国争光!
   【常见失误】
   解答本部分试题常见的毛病是对隐含在文字中、图案中和图表中的信息把握不准,对已经获取的信息表述不清;在语言运用上往往停留在一般水平上,缺少创意,也缺乏新意。
   【考点精练】
   1.(2004·河南)根据下面画线句子的特点,再仿写两个句子。
   课外阅读是提高语文水平的重要途径,可以使我们获得很多有益的启示,充实我们的生活。读《三国演义》,我们可以领略到诸葛亮舌战群儒的风采;读《钢铁是怎样炼成的》,我们能领悟到人生的真谛和生命的意义; _________________;________________。
   2.(2004·荆州)在具体的语境中仿写、续写一个“我喜欢”句。
   我心里总是充满着太多的喜欢。我喜欢倾听春天的脚步,我知道我如果不喜欢,便意味着我不珍惜鲜嫩的花碧绿的草;我喜欢欣赏大地的容颜,我知道我如果不喜欢,便意味着我不热爱高峻的山清浅的河; , 。
   3.(2004·龙岩)仿写下面的句子。
   例:拥有青春,就拥有了一份潇洒和风流;拥有青春,就拥有了一份灿烂和辉煌。
   拥有_______________________________________;拥有____ ______________
   ________________________________。
   4.(2004·湛江)请模仿下面句子的形式,从“鲜花”、“矿石”、“蜡烛”、“航船”等事物中选择两个作为陈述对象,写两组句子。也可自选其他事物来写。要突出所写对象的特征。
   例句:种子,如果害怕埋没,那它永远不能发芽。
   (1) __________________________________________________
   (2) __________________________________________________
   5.(2004·河池)仿照例句,在横线上填上适当的词语。
   例:花朵把春天的门推开了
   把夏天的门推开了; 把秋天的门推开了。
   6.(2004·荆州)自由选取一具体形象,运用“写物寓意”的手法,创造一句“名言”。
   例 船:永远在风浪中生活,但从来没有因此而停航。
   7.(2004·仙桃)按照下边语句的行文思路仿写。
   青春,是一本自传体的大书,书的作者是自己。那么,这本书怎样才能写好?我们要用智慧的头脑构思,用良好的道德立意, , ,用执着的追求出版!(湖北省仙桃市等)
   8.(2004·永州)仿照下面例句写一句话,要求与例句中的格式相似,修辞手法相同。
   例句:我愿是一朵欢乐的浪花,为大海营造一点生机。
   仿写: , 。
   9.(2004·株洲)根据提供的语境,在下面句子的横线上填上恰当的内容。
   祖国是大海,我就是大海中的一滴水;
   祖国是蓝天,我就是蓝天上的一朵云;
   祖国是 ,我就是 ;
   祖国是 ,我就是 。
   10.(2004·常州)仔细揣摩下面两个语句的内容和写法,再仿写一句。
   当一个人怀疑自己的时候,他忘记了建立自信是一种习惯;
   当一个人浑噩度日的时候,他忘记了阅读好书是一种习惯;
   答:
11.(2004·青海)从下列词语中选用四个词语写一段话。
   耐人寻味 七窍生烟 白驹过隙 屏气凝神 左右逢源 晴天霹雳
   亦师亦友 无稽之谈 绵延悠长 眼花缭乱 少年老成 南辕北辙
   答:
   12.(2004·西宁)请按照不同的要求,运用以下三个词语,分别扩展为一段话,每段不少于30个字。
   溪流 大海 生命
   描绘自然景物:
   表达你对人生价值的感悟:
   13.(2004·黄冈)请在“生活”、“幸福”、“爱心”中任选一词,并以此为中心,扩展成一段文字。要求运用比喻、排比两种修辞手法。(湖北省黄冈市)
   14.(2004·莆田)从下列词语中任选三个,写一段60左右的文字,描绘一个画面。
   绚丽 清幽 巍峨 葳蕤 参差不齐 姿态万千
   答:
   15.(2004·泉州)连词组段。请从下面选择4个词语写一段话(要求:内容集中,语意连贯,至少运用一种修辞手法)。
   跳跃 翱翔 盘旋 俯冲 风和日丽 熠熠夺目
   答:
   16.(2004·兰州)“奋斗需要立志高远,需要真才实学,需要真抓实干。”在“友谊”、“理解”、“创新”三个词语中任选一个词语代替“奋斗”写一句话。(不得照抄原句内容)
   答:
   17.(2004·荆州)把下列句子组成语意连贯的一段话。(在横线上写出表示正确语序的序号)
   ①这在江南古镇的私家花园中是创意独具的。
   ②退思园的建筑格局可以归结为西宅东园。
   ③如此巧妙的布局,
   ④西边的厅堂楼馆皆坐北朝南,从而造成横逸的形势。
   ⑤一是可使家居生活充分享受阳光,
   ⑥东园的亭桥廊榭皆错落有致地环池而列,诸般景物贴水成园、园水成趣。
   ⑦二是置身园中随处都可从不同的角度欣赏到全景。
   答:
   18.(2004·佛山)把下列句子组成语意连贯的一段文字,将序号依次写在横线上。
   ①我对一只空中的蝴蝶说,嗬,好大的浪呀。
   ②透明的空气在蝴蝶看来,像海浪一样,是浩浩荡荡的。
   ③蝴蝶像遇到知音一样,频频扇翼。
   ④于是你看到蝴蝶小姐在飞翔中起伏、躲闪。
   ⑤没有什么生物比蝴蝶更了解空气。
   答:
   19.(2004·盐城)今年5月下旬,我市举办了中国盐城首届茉莉花节暨经贸洽谈会。假如你是兴盐中学的小记者郑成,在活动期间给你一次向下列对象提问的机会,你应当问什么?
   (1)对象一 市委书记、市人大常委会主任张九汉
   (2)对象二 韩国客商、现代汽车集团会长郑梦九
   20.(2004·龙岩)伟华是位诚实而优秀的中学生,一次他到某装有监视器 (一种利用电视录像技术进行监视的设施)的大商场购物,购毕,付完款要走时,保安过来对他说:“我们怀疑你偷了本商场的东西,请跟我们到值班室,我们要对你搜身。”
   要求:请站在伟华的角度,替他严辞拒绝保安搜身的无理要求。(说2条拒绝理由)
   答:
   21.(2004·河源)有时,当父母的往往喜欢这样抱怨自家的孩子:“你看,人家的孩子……”这句话说得节省又含蓄,不过,个中滋味也只好由那“自家的孩子”慢慢地体会。如果你是“自家的孩子”,定会知道父母想要说的是                    
   假如你不是“自家的孩子”,听到别人父母说出这样的话来,他的言外之意是:   
                          。
   22.(2004·湛江)请根据下面的语境,写出张琳的转述语。要简明、连贯、得体。不要超过45字。
   李蓉是班上的语文课代表,因病住院,她的同桌好友张琳星期天到医院探望她。李蓉请张琳把自己写的作文转交给教语文的郝老师,并让张琳代自己向郝老师表示迟交作文的歉意,希望郝老师能批改好,好让张琳后天来时带给她:第二天,张琳把李蓉的作文交给郝老师时,对郝老师说: 。
   23.(2004·玉林)请根据下面情境,按要求答题。
   班里的“小书法家”小丽正在书写一幅参赛的书法作品。大功即将告成。毛手毛脚的小映突然凑上来碰到了她的手,一幅艺术作品立刻惨不忍睹。小丽气恼地瞪了小映一眼。小映赶忙道歉,见小丽还在气头上,又莞尔一笑,说:“微笑是人类最好看的表情。”
   ①小映这句话的意思是: 。
   ②假如你是小丽,你将会对小映说: 。
   24.(2004·哈尔滨)阅读下面这段文字,回答问题。
   美国莱特兄弟于1903年12月17日,驾驶动力飞机成功地遨游蓝天。人们为此举行盛大酒会,主持人要莱特兄弟发表演说,兄弟俩再三推辞,主持人执意邀请,哥哥便发表了言短意深的一句话演说:“据我所知,鸟中最会说话的是鹦鹉,而鹦鹉是永远飞不高的。”
   他这句话的意思是______________ 。
   25.(2004·十堰)上课铃声响了,某位同学还慢吞吞地往教室走,进教室之后,依然不紧不慢地走到座位上,此时老师笑着对大家说:“××同学真是一个听话的好学生,上次他在楼道里乱追乱窜,老师批评了他,你看他现在稳重多了,连打了上课铃,依旧沉得住气,都不快走两步。”教室里爆发出大声哄笑。
   读了这段文字,你认为老师的言外之意是什么?如果你是老师,你会怎么说?
   ①言外之意: 。
   ②你说的话: 。
26.(2004·陕西)请你参加下面的实践活动,完成后面的题目。
   学校征集体现办学理念、学校特色以及切合学生实际的校训(如北京师范大学校训:学高为师 身正为范)。初三(1)班同学经过认真准备,召开“校训交流主题班会”。
   ①假如你是初三(1)班的班长,请说一段简短精要的开场白。
   答:
   ②作为班组一员,请把你收集或拟写的校训写在下面,与同学交流。要求不少于两个词语,不得照抄北京师范大学校训。
   答:
   ③你若是初三(1)班的班主任,活动结束时,请你作简要总结。
   答:
   27.(2004·四川)说话要注意身份和对象,才能产生好的效果。根据下面提供的语境回答后面的问题。
   王昶同学原来学习成绩一直不错,可是今年开学以来,受他人影响迷上了电子游戏,平
   时与同学也在谈游戏,学习投入少了,上课有时也想着游戏内容。期末考试,成绩大大下降。
   这时,他才猛醒,心里很难过,在家里闷闷不乐,有时还发脾气。面对这种情况,王昶的妈
   妈和同学都准备开导他。他们怎样开导才能产生效果呢?请你替妈妈和同学分别写几句开导
   的话。
   妈妈说: 。
   同学说: 。
   28.(2004·嘉兴)海河市护城河水污染严重。有人把生活垃圾倒入河中,有人将未经处理的工业废水排入河中……为此,向阳中学将组织一次社会调查,呼吁大家保护环境。假如你是向阳中学的学生,要采访下列人员,你将如何明白得体地提问?
   (1)采访一位在护城河边晨练的老人,向他了解对这一污染现象的看法。
   答:
   (2)访问市环保局局长,向他了解环保部门对护城河污染问题采取的措施。
   答:
   29.(2004·台州)一天,妈妈发觉女儿丽丽发烧了,决定带丽丽到医院看病,可丽丽却说:“不用妈妈陪我去,我已经十六岁了, A 我自己去吧。”
   到医院后,交款处前排了长长的一队,妈妈气喘吁吁地跑来,看见女儿排在队尾,说:“这么多人,别排了,我带你去找熟人,直接拿药得了。”丽丽说:“妈妈,您不能这样, 
   B  ”。
   请在对话的A、B两处分别填上有针对性、有说服力的话。
   A:___________________________________
   B:___________________________________
   30.(2004·资阳)根据提供的情境,按要求答题。
   初三某班同学就“开卷是否有益”进行着热烈的辩论。正方主辩围绕“开卷有益”的观点进行了论辩。假若你是反方主辩,那么,你确立的观点应该是“ ”,请围绕你的观点写出两条理由。
   ⑴
   ⑵
   当你接过正方话题阐述自己的观点时,在阐述观点前应该有几句恰当的话,这几句话应是:
   31.(2004·常州)今年6月5日世界环境日的主题是“海洋存亡,匹夫有责”。假如你看见一位老人往京杭运河里倾倒污水,请根据图示,结合主题,用恰当的语气对老人进行一番劝说。
  
   32.(2004·盐城)看右面一幅漫画《呵护》,请用简洁的语言,写出一个既能概括漫画寓意又能给人以启迪的句子。
  
  
  
  
  
  
   33.(2004·盐城)下面是“中国电信(China TeleCom)”图标,仔细观察后,根据要求答题。
   (1)根据你的理解,请用简明的语言说说中国电信图标的创意。
   (2)结合这一图标,为中国电信写一广告语,不超过20个字。
  
  
   34.(2004·上虞)每一个会徽的后面都讲述着一个故事。当你翻开这本画册,拥有悠久历史、光荣现在和灿烂未来的中国北京,正在把你拥抱。在这个故事里充溢着中国北京的盛情和期盼;在这个会徽中记载着中国北京向世界做出的承诺。这就是“舞动的北京”。这就是第二十九届奥林匹克运动会会徽。
   左边是北京奥运会会徽——“中国印·舞动的北京”(印的剩余为红色),请你做下列题目:
   (1)请用简洁的语言概括画面。
   答:
   (2)请描述该会徽所表达的含义(指出其中一点即可)。
   答:
  
   35.(2004·甘肃)下面是一幅寓意深刻的漫画,想想看,它蕴涵着当今世界一个怎样的重大社会问题。
  
   答:
   36.阅读表格,按要求回答问题。
   世界能源组成年代 石油 煤 天然气 核电 太阳能 水电 其他
   1993年 40% 27% 23% 7% 2% 1%
   2030年(预计) 35% 20%
  
  
  
  
  
  
   ①用文字概括世界能源的发展趋势。
   ②读此表后你有哪些感受?
   ①
   ②
   37.(2004·西宁)喜欢唱流行歌曲吧,那么请选出两句你最喜爱的,并能给人鼓舞教育的,语言优美的歌词,把它写下来。
   (1)
   (2)
   38.(2004·永州)用一句话概括下面语段所透露出来的信息。
   据《英国医学杂志》网站报道,英格兰南部伯恩茅斯糖尿病和内分泌中心医学专家研究发现,含二氧化碳气体的饮料会使人发胖。为遏制全球日益严重的儿童肥胖问题,应严格限制他们的泡沫饮料饮用量。专家们鼓励儿童选择稀释过的果汁或者纯水为饮品,同时呼吁学校应采取措施避免儿童饮用泡沫饮料,各路明星也应停止为泡沫饮料做广告。据世界卫生组织提供的数据,在全球5岁以下的儿童中,估计有1760万名超重。(不超过16个字)
   答:
   39.(2004·株洲)将下面这段文字的内容用两句话简要地表述出来,字数在40字之内(含标点符号)。
   国家统计局经济景气监测中心对10000户居民所做专项调查的结果显示:62.7%的家庭,教育消费已成为家庭开支的重要一项,超过50%的家庭1—3年内要把钱花在教育上,而且是大额支出。然而对重金消费中的教育质量是否感觉物有所值呢?在调查中,只有25%的居民给予肯定,另有48.5%给予否定,有26.5%认为无法判断。
   答:
   40.(2004·连云港)阅读下列简讯,请给这则简讯拟一个标题。(不超过15字)
   [本报讯]5月23日,著名“战士作家”高玉宝致信“雷锋车”组,热情支持该组发出的签名支持申请美猴王为2008年北京奥运会吉祥物的倡议,并认真写出自己支持美猴王申吉的两大理由:“一、孙悟空为取真经,不怕千难万险,终于取胜,这是中国人的精神。我们希望中国运动员用孙悟空的拼搏精神,在奥运会比赛中多得金牌。二、《西游记》和孙悟空闻名中外,在中国家喻户晓。我赞同‘雷锋车’组的倡议,选孙悟空为2008年奥运会吉祥物。”
   答:
   41. (2004·宿迁)根据下面的内容拟一则新闻标题,不超过20字。
   本报北京5月16日讯:在今天举行的中国少年科学院第三届小院士表彰大会上,徐芳硕等来自全国的50位小院士受到表彰。这是团中央、全国少工委学习贯彻《中共中央国务院关于进一步加强和改进未成年人思想道德建设的若干意见》,实施“科教兴国”战略,在全国少儿中开展体验教育的重要举措,也是科技活动周的一项重要内容。
   “中国少年科学院小院士”是全国少工委、中国少年科学院为引导和鼓励少年儿童学习科学家精神,热爱科学、参与科普实践活动而设立的最高荣誉称号。小院士表彰活动作为2004年国家“科技活动周”的重点活动,在培养青少年对科学技术的兴趣和爱好的同时,重在引导他们树立科学思想、科学态度,并逐步形成科学的世界观和方法论。
   答:
42.(2004·四川)阅读下面这则新闻报道,请用一句话概括新闻的主要信息(不超过24个字,含24字)。
   本报讯(记者 凌弘)今(15)日8时30分,2004年全国暨四川省科技活动周开幕式在绵阳汉龙体育馆举行。
   科技活动周的开幕式将通过通讯卫星信号传输手段向全国转播,使北京、上海和四川绵
   阳会场丰富多彩的活动情况适时互传、实时互动。绵阳会场丰富多彩的活动将分外夺目。全
   国人民将一睹绵阳人民的风采。
   中国政府自二○○一年开始设立并组织实施全国科技活动周,每年五月的第三周被定为
   活动开展时间。活动周迄今已成功举办了三届。
   答:
   43.(2004·十堰)请给下面一则消息拟写标题,字数不超过12个。
   2004年5月24日,农夫山泉股份有限公司与丹江口市政府签订协议:农夫山泉再投资5亿元,在丹江口生产基地建设二期工程。这意味着丹江口市将成为我国饮用水最大生产基地。
   2003年8月,农夫山泉公司投资3.2亿元,在丹江口市胡家岭建设湖北生产基地,今年5月18日已有两条生产线建成投产。农夫山泉公司在丹江口市追加投资,再添4条生产线。项目建成投产后,农夫山泉丹江口生产基地年销售收入可达12亿元人民币。(湖北省十堰市)
   答:
   44.(2004·常州)为贯彻党的十六大精神,常州市政府在制定的“常州市民精神”中,倡导全体市民要“重诚信”。阅读下面几则诚信小故事,从中归纳出“重诚信”的含义。
   (1)春秋时,吴国的季子出使北方,顺道拜访了徐君。徐君很喜欢他的剑,季子虽然心中默许赠送,但因外交礼仪的需要,当时没有答应。等他返回时,徐君已死,季子就把剑挂在其坟前的树上而去。
   (2)古代有个叫尾生的人,与一位姑娘相约桥下,姑娘因事未到,正逢潮水上涨,尾生抱住桥下一石柱不肯离去。等姑娘赶到,尾生已被淹死了。
   (3)北宋词人晏殊14岁时,参加皇帝亲自主持的考试,看到题目后,对皇帝说:“这个题目10天前我就做过了,请陛下另出一题。”
   (4)去年,金坛建昌、社头、薛埠等地300余农户面对市场的高价不动心,仍把自己的蚕茧低价卖给了当初与他们签订订单的江苏晨风集团。
   (5)广东吴先生在没有付钱的情况下,仅通过电话在林海燕的投注站购买彩票,中得大奖518万。林海燕本可以占为己有,还是打电话把中奖消息告知了吴先生。吴先生开始以为她是在催要彩票钱,经多次打电话,吴先生才领走了中奖彩票。
   答:
   45.(2004·河南)用一句话概括下面文字的主要内容。(不超过20个字)
   中国教育报北京3月23日讯 记者从文化部今天举行的新闻发布会上获悉,从5月1日起,全国文化、文物系统各级博物馆、纪念馆、美术馆等公共文化设施要对未成年人集体参观实行免票;家长携带未成年子女参观的,对未成年子女免票。
   由文化部、国家文物局发出的通知强调,公共文化设施在向未成年人等社会群众免费开放的同时,要坚持把社会效益放在首位,积极开展未成年人的心理特点和教育需求,举办多种陈列和展览,为广大未成年人提供更多更好的文化服务。
   答:
   46.(2004·湟中)请用一句话概括下列新闻的主要内容(不超过10个字)。
   新华社北京6月8日电 8日晨6时23分,机身上绘着奔跑的火炬手黄色图案和雅典奥运会蓝色会徽的“宙斯”专机抵达北京首都国际机场,将雅典奥运会圣火火种送到了北京。这是奥运圣火首次飞抵中国首都北京。
   6时40分,“宙斯”号专机舱门打开,雅典奥运会“奥运大使”斯皮罗斯·兰布利迪斯手提载有雅典奥运会圣火火种的“马灯”走下飞机。北京市副市长、北京奥运会组委会常务副主席刘敬民、组委会副主席蒋效愚、秘书长顾耀铭等在停机坪迎接圣火,数百名专程前来迎接圣火的北京各届人士爆发出热烈的掌声。在隆重而简短的欢迎仪式后,雅典奥运会圣火火种被送往慕田峪长城举行展示活动。
   9日,北京将举行大规模的奥运会圣火传递活动,届时将有148名火炬手参与,这是雅典奥运会圣火全球传递活动中的第五站。6月4日正式启动的这项活动,经过了澳大利亚的悉尼、墨尔本,日本的东京和韩国的汉城后飞抵北京。
   《兰州晚报》
   主要内容:
   47.(2004·天津)根据下面提供的材料,用一句简明的话说什么是“返祖现象?”(不超过 35个字)
   什么是“返祖现象?”生物在绵长的历史进程中,因为客观条件的影响,身上的器官逐渐发生了进化或退化的现象,这种变化也一代一代遗传下去。但是,在千千万万次生育中,也可能有一次,本来已经退化了的器官却突然重新出现;仿佛某种长期被抑制了的微弱遗传因素,突然又强化起来。这时,某个生物体就出现了一部分不同于一般状态的现象,这就叫“返祖现象”。
   答: 。
   48.(2004·新疆生产建设兵团)请用一句话概括出下面一则消息的主要内容。
   今年4月19日上午,沙尘天气突袭天山北坡一带,沙尘笼罩了乌鲁木齐、石河子和昌吉等城市及周边村镇。这是近年来首府最大的沙尘天气。所有受沙尘袭击的区域,空气质量均为重度污染。由于首府沙尘可吸入颗粒的浓度已超过监测器检测上限,所以无法监测到具体浓度值。
   答:
   49.(2004·甘肃)在横线上写一句总结性的话,要能概括短文的主要含义。
   曾参是孔子的学生,品德高尚。有一次,一个和曾参同名的人杀了人,别人误以为是曾参,就对曾参的母亲说:“你儿子杀了人。”曾参的母亲不相信,说:“我儿子决不会杀人。”过了一会儿,又有人说曾参杀了人,他母亲仍然不相信。可是,当第三个人来说曾参杀了人时,他母亲就害怕地逃走了。
   这个故事告诉我们: 。
   50.(2004·青海)读下面一则材料,从中你有什么发现?写出探究结果。
   美国生物学家曾做过这样的实验:将初生的婴儿分成四组,一组听不到任何声音;二组播放录制的催眠曲;三组用节拍器仿声;四组播放母亲心跳的录音。结果表明,第四组仅用了其他三组所用时间的一半,婴儿就安安稳稳地睡着了,而第一组的婴儿在长达60%的时间内都在啼哭、骚动。
   探究的结果:
   51.(2004·荷泽)为下面的短文续写一个恰当的结尾
   十几年前,从国外进口的几百元钱一块的电子表,现在只需几元钱。细想起来,那时候外国人简直是在用一吨废铁换我们的一吨黄金。几年前,上万元钱的一部手机,现在只卖几百元,而且真正值钱的只是手机里那一块小小的芯片。真可谓知识无价。比尔·盖茨用知识创造了赚钱的神话;美国用高科技创造了战争神话。
   什么时候,我们也能实现“用一吨废铁换一吨黄金”的“美梦”? 。
   52.(2004·黑龙江)据中央电视台报道,2004年6月8日,我国观测到了“金星凌日”的天文奇观。请根据下面示意图,运用你所学习的说明方法简要说明这一现象。
   答:
   53.(2004·呼和浩特)“黑马”是生活中经常使用的词语。阅读下面句子,解释这一词语。
   ①今年国庆文化消费成为假日经济黑马。
   ②不管是从欧洲足球的实力布局,还是从历届欧锦赛的战线来看,黑马往往从东欧或者北欧球队中产生。
   黑马:
   54.(2004·甘肃)“己所不欲,勿施于人”是儒家的待人接物之道,意思是自己不愿意的,不要强加给别人。你认为应该怎样看待这句话。请写一段话阐述自己的观点,不少于100字。
   答:
   55.(2004·西宁)阅读下面的材料,按要求回答问题。
   在法国隆重纪念诺曼底登陆60周年之际,一座由中国雕塑家遥远创作的世界和平女神 像6月3日在昔日诺曼底登陆战场揭幕。
   世界和平女神像高10米,重16吨,用不锈钢制作。她那舞动绸带的双臂一前一后地舒 展着,高举的左手上一只和平鸽展翅欲飞。在微风中展开的飘带与女神的双臂连为一体,远 远看去,好似“中国”的“中”字、“和平”的“平”字,又像英文单词WORLD(世界)的第一个字母“W”和VICTORY(胜利)的第一个字母“V”。
   ①把以上材料概括成一句话新闻。(不超过20字)
   答:
   ②世界和平女神像的造型具有哪些象征意义?
   答:
   56.(2004·西宁)由青海省主办的第三届国际环湖自行车赛将于8月初拉开帷幕,省体育局拟在我市新宁广场立一宣传牌,请你为此宣传牌写一段热情洋溢、鼓舞人心的话(30字以内)。
   答:
   57.(2004·武汉)毕业在即,请你自拟一则给好朋友的赠言,激励他(她)努力奋进。
   答:
   58.(2004·无锡)富兰克林说过,很多人都曾为一个“哨子”付出过过高的代价。这“哨子”在每个人那儿具体所指各不相同。阅读下面的语段,说说两处“摔碎的茶壶”的具体所指是什么。
   从前有一个人担着茶壶去卖,不料经过山坡时,几个茶壶从筐里掉出来摔碎了。他头也不回地向前走,有人提醒他:“你的茶壶摔碎了,还不快看看!”这人回答说:“既然已经摔碎了,看又有什么用?”
   其实,人生随时都会有意想不到的事情发生,有的远远比摔碎几个茶壶严重。你能否脚不停头不回从头再来,是至关重要的。世界著名话剧演员波尔赫德活跃在世界戏剧舞台50多年,71岁时,一场意外使她破产。更糟的是,她的脚因受伤引起脉管炎,不得不截肢。当医生忐忑不安地把这消息告诉她时,她却平静地说:“既然没有其他办法,那就面对现实吧。”波尔赫德平静地面对摔碎的茶壶(A),重新站立,还活跃在舞台上7年。
   生活中,人人都渴望成功,然而成功并非唾手可得,它可能经过诸多挫折和失败才有望实现。能像那个卖茶壶的一样的人实在不多。我们所见更多的是患得患失、斤斤计较的人,他们面对已摔碎的茶壶(B),往往捶胸顿足,后悔不已,结果是白白浪费了时间不说,说不定因为分神还会摔碎更多的“茶壶”。
   答:A             B          
   59.(2004·泉州)阅读下列四则文字材料,就你感受最深的一点写一句话。
   ①我国16%以上的青少年有心理不健康问题,非常叛逆的“十五岁现象”呈上升和提前趋势。
   ②“中国关心下一代身心健康工程”于2004年5月31日在北京正式启动。
   ③一位著名学者在自述中写道:“在家乡,我在母亲的教训下住了九年,受了她极大、极深的影响……如果我学得一丝一毫好脾气,如果我学得一点点待人待物的和气,如果我能宽容人、体谅人,我都得感谢我的慈母。”
   ④社论《父母亦师》、评论《和孩子交朋友》。
   (摘自2004年6月1日《人民日报》)
   答:
   60.(2004·泰安)为你最喜欢的事物写一段描述性的文字,并说说从中受到的启示。
   事物:
   描述文字及启示:
   61.(2004·泸州)材料一:某一小学课堂。老师提问;“雪融化了是什么?”有学生回答:“春天。”老师摇头,转身在黑板上写下一“水”字,并要求同学记住这个标准答案。
   材料二:某一中学课堂。老师讲“烈士的鲜血染红了山茶花”,一学生举手发言,称鲜血不可能染红山茶花,并解释了花朵呈不同颜色的科学道理。但教师不认同。
   (1)两则材料反映了一个什么相同的问题?请用一句话概括。
   答:
   (2)请你对材料中师生的说法作出评判,并点明理由。
   答:
   62.(2004·福州)有这样一个故事:一匹骨瘦如柴的老马拉着盐车上山坡,气直喘,汗直流,竭尽全力还是拉不上去。赶车的人吆喝着,用鞭子狠狠地抽打它……这时,一个路过的相马人看见了,心疼得流下了眼泪,急忙脱下衣裳披在瘫倒在地的老马身上。老马睁开眼,看到相马人,眼睛一亮,长嘶一声而逝。
   请你展开想像,写出老马临死前想对相马人说的话。
   答:

第四讲 文学常识
    【考查要点】
   了解课本涉及的重要作家作品和文学常识;了解诗歌、散文、小说、戏剧等文学样式。
   阅读一定数量的文学名著。
   【知识疏理】
   文学常识也是中考积累与运用的必考内容。从内容上说它包括以下四个方面:1.作家的生活年代、朝代(外国作家的国别);2.重要作家的作品及出处;3.作品中的主要人物与性格;4.与课本相关的内容,包括诗文内容、写作目的。
   文学常识的复习,要以基本篇目所涉及的古今中外重要作家、作品为主,可将众多的作家、作品按古代、现当代、外国三个方面分类整理,再列出表格,将作家姓名(字号)、朝代、作品、出处、作家简介甚或作品中的主要人物、名句都网络进去,力求全面。另外复习时可与“诗文背诵默写”部分结合起来。
   【试题特点】
   《语文课程标准》中明确规定了7—9年级的学生要每学年阅读两三部名著,并列出了学生必读的中外名著目录。在总目标中也强调:“认识中华文化的丰厚博大,吸取民族文化的智慧。”因此,中外名著走进中考已经成为许多省市中考语文试题的显著特色之一。2004年中考对名著的考查在以往的基础上又迈出了一大步。
   从题型上看,文学常识题既有客观型的选择题,也有主观型的填写表述题。
   【解题导引】
   例1.(2004·常州)语文课开展“走近名著”活动,请接着甲同学的发言,也讲述一个你熟悉的名著中的故事。
   (要求:说出书名、人名和有关情节,在叙述中至少运用一个成语或名言、警句、格言。)
   同学甲:《水浒传》中,嫉恶如仇的鲁提辖听了金氏父女的哭诉,毅然出手,三拳打死了镇关西,解救了金氏父女。他真不愧是一位见义勇为的英雄。
   你的讲述: 。
   2003年中考涉及中外文学名著的试题大多是主观题,例如:《西游记》中“大闹五庄观,推倒人参果树”的是 。(2003河南省题)成语“万事俱备,只欠东风”是根据《三国演义》 (战役)中“周瑜定计火攻曹操”的故事演化而来的。(2003年宁波市题)而这一道名著阅读题在上述主观题的基础上已向开放性试题方向发展。“巧妇难为无米之炊”,要想准确做出答案必须先要细致地阅读原著,否则怎能讲出故事的情节呢?本题不设标准答案,内容正确,符合要求即可给分。但必须作品名称正确,内容叙述符合原著,而且成语、名言、警句或格言运用要恰当正确。
   例2.(2004·广东)对对联。
   上联:足智多谋,孔明巧借箭。
   下联: , 。
   对联根植于生活的土壤,同时又颇具文学韵味,再加之与文学名著连在一起,更能让学生感受到中国传统文化的深厚底蕴。解题时要注意以下几点:⑴字数须相等;⑵词性须相对(一般情况下,宽对也可);⑶搭配须得当。答案示例:①艺高胆大,悟空勇斗魔;②义盖去天,关羽私放曹;③多愁善感,黛玉悲葬花。如果不细致地阅读原著,不熟悉原著的内容,没有灵活运用语文的实践能力,很难对出下联。
   例3.(2004·无锡)⑴读一定数量的课外文学名著,是语文学习的基本要求。但读书除了积累知识外,更重要的是学会思考。初中三年,你一定在老师的指导下读了不少课外好书。下面请你做一个简要的读书札记。
   书名: 。
   作者: 。
   印象最深的人物: 。
   该人物形象给你的人生启示: 。
   这道题给我们提供了广阔的思维空间,也就是形式上更具开放性、灵活性。只要你能认真地阅读名著,并能透过作品的文字表象,对作品的形象、情感、语言加以领悟,能从作品中获得对自然、社会、人生的有益启示,答好此题就轻而易举了。
   此试题还暗示我们,读名著不能只追求浅层次的理解,要追求深层次的阅读,可采用下列方式加深对作品的理解。如:写读书笔记,即将你读书的独特感受及时记入笔记;也可以通过读书报告会、辩论会等方式与同学之间进行交流;还可尝试着对名著进行再创作等。
   【常见失误】
   文学常识题最大的失误是一些同学没读过或没认真阅读文学名著,造成不知如何答题或答题牵强附会。
   【考点精练】
   1.(2004·海淀)文学常识表述有误的是( )
   A.清代曹雪芹创作的《红楼梦》是我国一部优秀的古典长篇小说,我们学过的课文《怡红院端阳嬉闹》就节选自此书。
   B.鲁迅、茅盾、老舍、朱自清、冰心都是中国现代文学史上的著名作家。
   C.《门槛》是俄国作家屠格涅夫的作品,《我的叔叔于勒》是法国文学家莫泊桑的作品。
   D.按写作年代的先后排列下面的课文,正确的顺序是《扁鹊见蔡桓公》《三峡》《范进中举》《活板》。
   2.(2004·北京)文学常识表述有误的是( )
   A.《春》和《背影》的作者是朱自清。
   B.《马说》和《捕蛇者说》的作者分别是柳宗元、韩愈。
   C.“大漠孤烟直,长河落日圆”出自唐代诗人王维的《使至塞上》。
   D.《最后一课》的作者都德和《我的叔叔于勒》的作者莫泊桑都是法国作家。
   3.(2004·遂宁)下面中国四大古典名著中的人物与情节搭配不当的一项是( )
   A.林黛玉——葬花、焚诗 B.诸葛亮——草船借箭、借东风
   C.鲁智深——醉打蒋门神 D.孙悟空——三借芭蕉扇
   4.(2004·贵阳)选出下列文学常识判断错误的一项。( )
   A.《桃花源记》的作者是陶渊明,宋代人,他的作品还有《岳阳楼记》。
   B.《论创造》的作者是法国的罗曼·罗兰,《名人传》是他的作品。
   C.《雪》选自鲁迅的散文诗集《野草》,他的代表作品有小说《孔乙己》。
   D.《左传》相传是春秋时期左丘明所作,是根据鲁史写的编年体史书。
   5.(2004·哈尔滨)结合你的读书经历,选出下面说法正确的一项。
   在《三国演义》中有一位英雄,他曾温酒斩华雄、千里走单骑、刮骨疗毒,被后人敬仰
   并尊为中国的“武圣”,这位英雄是( )
   A. 赵云 B. 关羽 C. 张飞 D. 马超
   6.(2004·四地)下面有关文学常识的说法有误的一项是( )
   A.陆游、苏轼、李贺、辛弃疾都是我国宋代著名的爱国主义诗人或词人。
   B.表现爱国诗人杜甫忧国忧民情思的诗篇有《春望》、《茅屋为秋风所破歌》等。
   C.范仲淹在《渔家傲》中表达思乡报国之情的句子是“将军白发征夫泪”。
   D.“人生自古谁无死,留取丹心照汗青。”这一名句出自爱国诗人文天祥的《过零丁洋》。
   7.(2004·襄樊)下列作品与作家搭配有误的一项是( )
   A.《济南的冬天》——朱自清 《麦琪的礼物》——欧·亨利
   B.《古代英雄的石像》——叶圣陶 《威尼斯商人》——莎士比亚
   C.《白杨礼赞》——茅盾 《我的叔叔于勒》——莫泊桑
   D.《黎明的通知》——艾青 《皇帝的新装》——安徒生
   8.(2004·永州)下列文学常识搭配有误的一项是( )
   A.《石壕吏》——杜甫——唐代——诗歌
   B.《变色龙》——契诃夫——俄国——寓言
   C.《中国石拱桥》——茅以升——现代——说明文
   D.《故乡》——鲁迅——现代——小说
   9.(2004·株洲)下列说法不正确的一项是( )
   A.《诗经》是我国最早的一部诗歌总集,收录了从西周到春秋时期的305篇诗歌,古时也称“诗三百”。
   B.鲁智深是《水浒传》中的人物,范进是《儒林外史》中的人物。
   C.散文《从百草园到三味书屋》选自《朝花夕拾》,作者是鲁迅;小说《故乡》选自《呐喊》,作者是朱自清。
   D.童话《皇帝的新装》的作者是丹麦著名童话作家安徒生。
   10.(2004·南通)选出下列说法不正确的一项。( )
   A.《关雎》《蒹葭》《君子于役》都选自《诗经》。《诗经》是我国最早的诗歌总集,分“风”“雅”“颂”三大类。
   B.鲁迅的《从百草园到三味书屋》《社戏》、朱自清的《背影》和魏巍的《我的老师》都是脍炙人口的回忆性散文。
   C.古典文学名著《水浒传》中,智取生辰纲、醉打蒋门神、三打祝家庄等故事,生动曲折,扣人心弦。
   D.法国文学巨匠罗曼·罗兰所作的《名人传》,记述了贝多芬、米开朗琪罗、托尔斯泰等伟大人物战胜各种人生艰难困苦的精神历程。
   11.(2004·临沂)下面是中国古代四大名著中的人物与情节,其中搭配不当的一项是
   ( )
   A.鲁智深 ———— 倒拔垂杨柳
   B.孙悟空 ———— 大闹天宫
   C.诸葛亮 ———— 三顾茅庐
   D.刘姥姥 ———— 进大观园
   12.(2004·广安)下面对语文内容解说有误的一项是( )
   A.《怀疑与学问》一文主要论述如何去怀疑各种事物的真实性。
   B.《斑羚飞渡》一文讲述了一群斑羚陷入绝境时如何求生、自救的全过程。
   C.《有的人》一诗中其主要句子是“有的人活着/他已经死了;/有的人死了/他还活着。”
   D.《从百草园到三味书屋》是一篇描写童年生活的散文。
   13.(2004·泸州)下列作家作品等对应完全正确的一项是( )
   A.《皇帝的新装》——安徒生——英国
   B.《陈涉世家》——司马迁——汉朝——《史记》
   C.《岳阳楼记》——范仲淹——唐朝——《范文正公记》
   D.《曹刿论战》——孔子——春秋——《左传》
   14.(2004·自贡)下面的文学常识,有错的一项是( )
   A.《变色龙》的作者是法国作家莫泊桑。
   B.《鲁提辖拳打镇关西》节选自《水浒传》。
   C.《有的人》的作者是诗人臧克家。
   D.《汉江临眺》的作者是唐代文学家王维。
15.(2004·呼和浩特)下列各句对文学常识的表述不当的一项是( )
   A.欧阳修,字永叔,号醉翁,晚年号六一居士。北京著名文学家。他继承了韩愈“古文运动”的理论,领导了北宋诗文革新运动。
   B.莎士比亚,十九世纪英国最伟大的批判现实主义作家、戏剧家和诗人。《威尼斯商人》是他的著名喜剧。课文节选部分,描写了威尼斯商人安东尼奥和犹太高利贷者夏洛克之间在法庭上的一场生死交锋。
   C.《天上的街市》《静夜》选自《郭沫若全集》。在这两首诗中,诗人以奇妙的想像和丰富的联想,描绘出两幅奇异的世外仙境图,把我们带进了一个美妙的世界。
   D.《从百草园到三味书屋》《阿长与〈山海经〉》都选自《朝花夕拾》;《社戏》《故乡》都是小说。这四篇都是鲁迅先生的作品。
   16.(2004·天津)使用成语评价课文中的人物,不恰当的一项是( )
   A.《杨修之死》——杨修——恃才放旷
   B.《故乡》——闰土——大智若愚
   C.《葫芦僧判断葫芦案》——贾雨村——徇情枉法
   D.《范进中举》——胡屠户——趋炎附势
   17.(2004·湖州)下列文学常识搭配正确的一项是( )
   A.《骆驼祥子》——老舍——小说 B.《陈毅市长》——沙叶新——小说
   C.《海燕》 ——高尔基——中国 D.《西游记》——辛弃疾—— 小说
   18.(2004·恩施)1954年4月23日,联合国科教文组织将每年的这一天定为“世界读书日”,以向读书人致敬,向文化鞠躬。1616年4月23日,一代文豪莎士比亚和文坛巨匠塞万提斯同时辞世。请问这两位文坛巨匠各自出生在哪一个国家?你读过他们中哪一位的哪一部作品?
  
   19.(2004·十堰)我国古典文学名著《三国演义》中塑造了一个过五关、斩六将、千里走单骑的英雄形象,这个英雄是 。这部书中有关这个英雄的传奇故事还有许多,请用最简洁的语言写出一个故事的名字: 。
   20.(2004·随州)《红楼梦》中,有一个女子,她模样标致,语言爽利,心机极深细,但“机关算尽太聪明,反误了卿卿性命”,这个人是 ;还有一个女子,她寄人篱下,渴望真挚的爱情,但在森严冷漠的封建大家庭中,只能凄婉地唱出“一年三百六十日,风刀霜剑严相逼”,这个人就是 。
   21.(2004·锦州)课外阅读填空。
     祥子是老舍笔下一个被侮辱、被损害的下层劳动者形象,他本是农民,进城后以________为生。虎妞是祥子的妻子,也是车厂主_________的女儿。
   22. (2004·山西)你知道《水浒传》中鲁提辖在解救了金氏父女之后又在野猪林解救了谁吗?你还能概括一个与被解救之人有关的故事情节的名称吗?请在下面填空。
   被解救的人: 。 故事情节的名称: 。
   23.(2004·贵阳)下午放学后,李林同学来到学校图书室,准备借一本文学名著,面对眼前众多的书,李林不知怎样选择,这时,你走过去热情地向他推荐。请在下列书目中任选一本(部),从作品特点和自己的阅读感受方面来向他介绍。
   备选书目:《童年》《水浒》 《钢铁是怎样炼成的》 《格列佛游记》 《骆驼祥子》
   ①书名:
   ②作品的特点(指作品内容、主题思想、写作手法、人物形象、语言特点等,可任意选取一点作简介):
  
   ③自己的阅读感受:
  
   24.(2004·河南)《我的叔叔于勒》《变色龙》《最后一课》这三篇外国小说,为我们讲述了生动的故事,塑造了栩栩如生的人物形象,请写出给你留下印象最深的一个故事或人物,并简述理由。
  
   25.(2004·南京)名著往往是“经过时间检验和沉淀”的“磨脑子”的书。请从下列名著中任选一部,简要地写出你熟悉的一个情节,并说说对这一情节的体验和感悟。
   名著:《水浒》《西游记》《钢铁是怎样炼成的》《鲁滨孙漂流记》《童年》
   答:情节:
  
   体验和感悟:
  
   26.(2004·烟台)与下列对联、诗文有关的中国古典文学名著是:
   ①
   ②      
   ①写鬼写妖高人一等,刺贪刺虐入骨三分。
   ②滚滚长江东逝水,浪花淘尽英雄。是非成败转头空:青山依旧在,几度夕阳红。
   27.(2004·上虞)名著阅读填空。
   (1)保尔·柯察金是前苏联作家 写的长篇小说《 》中的主人公,他的事迹鼓舞了我国千千万万的读者。
   (2)根据诗句判断人物(均出自我国四大古典名著)。
   ①一头红焰发蓬松,两只眼睛亮似灯。
   不黑不青蓝靛脸,如雷如鼓老龙声。
   身披一领鹅黄氅,腰束双攒露白藤。
   项下骷髅悬九个,手持宝杖甚峥嵘。
   ②富贵不知乐业,贫穷难耐凄凉。
   可怜辜负好韶光,于国于家无望。
   天下无能第一,古今不肖无双,
   寄言纨绔与膏粱,莫效此儿形状。
   28.(2004·绍兴)请根据提供的信息,分别说出对应的人物或作品。
   示例:潇湘馆 丫环紫鹃 葬花 《红楼梦》人物:林黛玉
   ①赤兔马 单刀赴会 败走麦城 《三国演义》人物:
   ②日本仙台 医学教授 修改讲义 鲁迅作品:《 》
   ③海难 荒岛 “星期五” 英国历险小说主人公:
   29.(2004·黑龙江)华威先生、蔡芸芝先生、藤野先生、韩麦尔先生参加名人聚会,大会接待处的接待员给他们登记入册,有几处空白,请你帮助填写。
   人物 出处(篇名) 作者 人物肖像 人物性格
   华威先生 《华威先生》 张天翼
   蔡芸芝先生 《我的老师》 温柔 公正
   藤野先生 《藤野先生》 鲁迅 正直 俭朴 治学严谨没有民族偏见
   韩麦尔先生 都德 穿件绿色礼服 打着领结戴着顶绣边的小黑丝帽 有强烈的爱国主义情感
   30.(2004·昆明)根据提示完成下列填空题。
   ①文学名著中,一些极富个性特点的人物绰号常给读者留下深刻印象。《水浒传》中的“花和尚”和“智多星”分别指的是 和 。
   ②歇后语“周瑜打黄盖——一个愿打,一个愿挨。”是两厢情愿的意思。该歇后语是从《 》的故事中演绎出来的。
   31.(2004·沈阳)按要求填空。
   人最宝贵的是生命,生命每个人只有一次。人的一生应当这样度过:当回忆往事的时候,他不会因为虚度年华而悔恨,也不会因为碌碌无为而羞愧;在临死的时候,他能够说:“我的整个生命和全部精力,都已经献给了世界上最壮丽的事业----------为人类的解放而斗争。”
   这段文字出自名著《 》,其作者是 。
   32.(2004·湟中)填空。
   (1)《西游记》中“大闹五庄观,推倒人参果树”的主要人物是 。
   (2)请用〈水浒传〉中人物的官名补全下面的回目。
   风雪山神庙 陆虞侯火烧草料场
   33.(2004·青海)请猜出下列诗中涉及的人物。
   A.此地别燕丹,壮士发冲冠。昔时人已没,今日水犹寒。
   B.沅湘流不尽,屈子怨何深。日暮秋风起,萧萧枫树林。
   C.功盖三分国,名成八阵图。江流石不转,遗恨失吞吴。
   D.寄书元有雁,食雪不离羊。施尽风霜节,心悬日月光。
   A. B. C. D.
   34.(2004·荷泽)将下列人物序号填到与其有关的语段后面
   ①冰心 ②蔡芸芝 ③蒙田 ④鲁迅 ⑤周敦颐 ⑥苏轼
   A.只有乐于生的人才能真正不感到死之苦恼。 ( )
   B.油蛉在这里低唱,蟋蟀在这里弹琴。 ( )
   C.圆天盖着大海,黑水托着孤舟,远看不见山,那天边只有云头,也看不见树,那水上只有海鸥…… ( )
   D.母亲,倘若你梦中看见一只很小的白船儿,不要惊讶它无端入梦。 ( )
   E.庭下如积水空明,水中藻、荇交横,盖竹柏影也。 ( )
   F.予独爱莲之出淤泥而不染。 ( )
   35.(2004·潍坊)回顾所读名著,按要求答题。
   ①《西游记》共一百回,可以分为两大部分:第一至第十二回全书的引子,其中第八至第十二回介绍唐僧及取经的缘由,第十三回至第一百回是故事主体。请你简要概括出第一至第七回的故事(不超过20字)。
   □□□□□□□□□□□□□□□□□□□□
   ②《汤姆·索亚历险记》第31章,叙述了 和 在岩洞里奇妙、曲折、紧张的历险过程。
   36.(2004·北培)在下面的横线上填上适当的内容。
   ①《水浒传》里的梁山好汉几乎每一个都有故事,如吴用智取生辰纲等。请写出另外一个人物的一个故事: 。
   ②保尔·柯察金是前苏联作家尼古拉·奥斯特洛夫斯基的长篇小说《 》中的人物。
   37.(2004·福州)请根据课外阅读的外国名著,补全下面的名人对联。
   上联:搏命运风浪奏出一支支悲壮的乐曲(贝多芬)
   下联:炼钢铁意志 (奥斯特洛夫斯基)
   38.(2004·河南)《我的叔叔于勒》《变色龙》《最后一课》这三篇外国小说,为我们讲述了生动的故事,塑造了栩栩如生的人物形象,请写出给你留下印象最深的一个故事或人物,并简述理由。
   39.(2004·无锡)读一定数量的课外文学名著,是语文学习的基本要求。但读书除了积累知识外,更重要的是学会思考。初中三年,你一定在老师的指导下读了不少课外好书。下面请你做一个简要的读书札记。
   书名 作者 印象最深的人物 该人物形象给你的人生启示



第五讲 背诵默写
   【考查要点】
   “诵读古代诗词,有意识地在积累感悟和运用中,提高自己的欣赏品位和审美情趣。”广泛阅读各种类型的读物,课处阅读总量不少于260万字,每学年阅读两三部名著。在诵读实践中增加积累,发展语感,加深体验和感悟。
   【知识疏理】
   ⑴诗文默写:①根据课文填空;②默写诗文名句;③读图默写诗文名句;④根据提示填写诗文名句。
   ⑵课外积累:①按要求写出格言或警句;②由具体的语境联想诗文名句;③中国古代四大文学名著中的文学形象;④中外文学名著评价。
   【试题特点】
   2004年本部分中考题的题型,主要有读图填写题、填空默写题、扩展开放题、简述问答题和比较选择题等。
   【解题导引】
   例1.(2004·盐城)根据下列名句及出处,完成第(1)—(4)题。
   ①忽如一夜春风来, 。(岑参《白雪歌送武判官归京》)
   ②海内存知己, 。(王勃《送杜少府之任蜀州》)
   ③野火烧不尽, 。(白居易《赋得古原草送别》)
   ④万事俱备, 。(罗贯中《三国演义》)
   ⑤云横秦岭家何在? 。(韩愈《左迁至蓝关示侄孙湘》)
   ⑥沉舟侧畔千帆过, 。(刘禹锡《酬乐天扬州初逢席上见赠》)
   ⑦无可奈何花落去, ,小园香径独徘徊。(晏殊《浣溪沙》)
   ⑧会挽雕弓如满月,西北望, 。(苏轼《江城子·密州出猎》)
   ⑨ ,蜡炬成灰泪始干。(李商隐《无题》)
   ⑩ ,铁马冰河入梦来。(陆游《十一月四日风雨大作》)
   (1)在上面各句的空白处,填写上句或下句。
   (2)句④是根据《三国演义》(战役)中“周瑜定计火烧曹操”的故事演化而来的。句⑤出处中的“左迁”是指 。
   (3)在上面各句中,能表现诗人杀敌报国情怀的名句有 、 。(只填序号)
   (4)请你围绕一个中心意思,引用上面的某个名句写一段话,60字左右。
   上面这一中考语文名句考查题是比较有特色的。它一改以往名句考查的单纯形式的填写。它由名句演发,将文学常识、诗句理解等巧妙地结合。它不单单是考查了学生的记忆力,而且考查了学生的理解和运用的能力,并更多地融合了学生的价值观和情感体验。这道题命题者至少是从以下三个层次进行把握的。一是名句识记。第(1)题要求学生能准确写出上句或下句,所选诗句都是学生应该掌握的。二是名句理解。第(2)题考查了学生文学常识的掌握情况,同时这也告诉我们背诵诗句还要作一点探讨和分析,如诗句的由来,有关诗句的典故,也可以结合综合实践活动,开展专题诗词的研究。如《“送别诗”的研究性学习》等。第(3)题主要考查对诗句的理解。三是名句运用。第(4)题答题时要注意三点:1.所写要有中心,不可泛泛而谈,不着边际。2.引用要恰当,要根据诗句的原意、寓意准确使用。3.字数一定要在60字左右。此外要尽可能选用精美词语,运用排比、比喻、拟人等修辞手法,使文笔生动。
   例2.(2003·北京)下面一段话中四个空白处应填入的诗句分别是( )
   在盛唐的诗歌中,有一类通常被统称为“边塞诗”。这些诗作内容丰富多彩,为我们展现了一幅幅盛唐边塞画卷。这里有 ① 的雄奇景观,有“潮海阑干百丈冰,愁云惨淡万里凝”(岑参《白雪歌送武判官归京》)的壮阔雪原,有 ② 的风雪酷寒;这里有 ③ 的慷慨悲壮,有 ④ 的惆怅无奈,有“战士军前半死生,美人帐下犹歌舞”(高适《燕歌行》)的极度愤慨……
   A.“醉卧沙场君莫笑,古来征战几人回”(王翰《凉州词》)
   B.“火山五月行人少,看君马去疾如鸟”(岑参《武威送刘判官赴碛西行军》)
   C.“剑河风急雪片阔,沙口石冻马蹄脱”(岑参《轮台歌奉送封大夫出师西征》)
   D.“大漠孤烟直,长河落日圆”(王维《使至塞上》)
   E.“中军置酒饮归客,胡琴琵琶与羌笛”(岑参《白雪歌送武判官归京》)
   F.“羌笛何须怨杨柳,春风不度玉门关”(王之涣《凉州词》)
   这是一道创新题,其创新之处主要有三:第一,试题材料取自于课外,要求考生填写的诗句却在课内;第二,在短短的一段文字中,要求考生填空的诗句只有四处,给出并注明出处的诗句却有6句;第三,在要求填写诗句的地方,有比较隐晦的提示,考生解答了这道题,对“边塞诗”的印象应该是很深刻的。
   解答本部分试题,一是要弄清题干要求,二是要调动自己的积累,三是要察看具体的语境,四是要注意写字的规范。
   【常见失误】
   解答本部分试题常见的失误是考生在考场上往往不易把要求提示与诗文名句的具体内容联系起来,还有就是写字不够规范。
   【考点精练】
   1.(2004·哈尔滨)按课文原文填空。
   (1)__________________________ ,山色有无中。
   (2)斯是陋室, __________________________。
   (3)__________________________ ,小桥流水人家。
   (4)人生自古谁无死, __________________________。
   (5) __________________________,其不善者而改之。
   (6)无可奈何花落去, __________________________。
   2.(2004·包头)默写。
   ①大漠孤烟直,□□□□□。
   ②老骥伏枥,□□□□。
   ③□□□,又何妨!
   ④□□□□□□□,便引诗情到碧霄。
   ⑤峰峦如聚,□□□□,山河表里潼关路。
   ⑥□□□□,遂迷,不复得路。
   ⑦□□□□□□□□□□,则凡可以得生者何不用也?
   ⑧故□□□□,颇有奇气。
   3.(2004·河源)在下列田字格内规范书写名句。
   (1)采菊东篱下,           ,(陶渊明《饮酒》)
   (2)落红不是无情物,          。(龚自珍《已亥杂诗》)
   (3)           ,柳暗花明又一村。(陆游《游山西村》)
   (4)读书破万卷,          。(熟语)
   4.(2004·北海)背诵默写
   ① ,失道寡助。(《<孟子>二章》)
   ② ,润物细无声。(杜甫《春夜喜雨》)
   ③采菊东篱下, 。(陶渊明《饮酒》)
   ④但愿人长久, 。(苏轼《水调歌头·明月几时有》)
   ⑤长风破浪会有时, 。(李白《行路难》)
   ⑥ ,千树万树梨花开。(岑参《白雪歌送武判官归京》)
   ⑦会当凌绝顶, 。(杜甫《望岳》)
   ⑧ ,各领风骚数百年。(龚自珍《已亥杂诗》)
   5.(2004·河北)用课文原句填空。
   ①何当共剪西窗烛, 。(李商隐《夜雨寄北》)
   ②无可奈何花落去, 。(晏殊《浣溪沙》)
   ③ ,出则无敌国外患者,国恒亡。(《<孟子>二章》)
   ④ ,甲光向日金鳞开。(李贺《雁门太守行》)
   6.(2004·无锡)根据课文默写。
   ①忽如一夜春风来, 。
   ②予独爱莲之 ,濯清涟而不妖。
   ③山随平野尽, 。
   ④ ,病树前头万木春。
   ⑤野芳发而幽香, 。
   ⑥姑苏城外寒山寺, 。
   ⑦先天下之忧而忧, 。
   ⑧ ,一枝红杏出墙来。 
   ⑨稻花香里说丰年, 。
   ⑩江山代有才人出, 。
   7.王教授已退休多年,仍在勤奋著书。如果要送他一幅书法作品,用曹操的诗句“
   , ”作为内容就很合适。(福州市)
   8.我国信息产业起步晚,发展快,颇有“ , ”之势。(用岑参《白雪歌送武判官归京》中的诗句填写)(福建省莆田市)
   9.曹操《龟虽寿》一诗中,用比喻的手法表达诗人年老而壮志犹存的诗句是“________
   ___________,___________________”(广东省湛江市)
   10.在《春望》中,春天的花开鸟鸣反而使诗人杜甫生出忧国和思亲之情,即所谓“
   , ”。(广东省肇庆市)
   11.为好友送行,人们常常用王勃《送杜少府之任蜀州》一诗中的两句“ ,
   ”。(广东省肇庆市)
   12.白居易在《观刈麦》一诗中深入所写对象的内心,刻画劳动人民在特定环境下近乎变态心理的句子是:“ , 。”(青海省湟中县)
   13.岑参《白雪歌送武判官归京》最后两句,将诗人因朋友离去而产生的无限惆怅之情抒写到了极致,这两句诗是:“ , 。” (青海省湟中县)
   14.杜甫诗“吴楚东南坼,乾坤日夜浮”形象地描绘出洞庭湖的壮阔之美,孟浩然《望洞庭湖赠长丞相》中的“ , ”与杜诗有异曲同工之妙。(广西桂林)
   15.在中国文学几千年发展中,每个时代都有各自的文学经典,均有独领风骚的一代宗师,这正如赵翼《论诗》所云:“ , 。”(广西桂林)
   16.陆游《十一月四日风雨大作》中把现实与梦境自然的联系起来以抒发强烈爱国之情的句子是: , 。(福建省莆田市)
   17.白居易《钱塘湖春行》一诗中描写了“莺歌燕舞”的初春景色的诗句是
   , 。(广西壮族自治区河池市)
   18.王勃在《杜少府之任蜀州》一诗中歌颂友谊的千古名句是:       ,
        。(山东省烟台市)
   19.读到美国哲学家欧文的“志同道合的人并不需要永远呆在一起”时,我的脑海里不禁浮现出了王勃在《送杜少甫之任蜀州》中的名句 , 。(新疆生产建设兵团)
   20.白居易在《钱塘湖春行》中以小鸟的活动写出早春特色的诗句是:
   , 。(重庆市)
   21.陆游的《游山西村》中寓含哲理,广为流传的两句诗是 ,
   。(新疆生产建设兵团)
   22.《次北固山下》一诗中,表现时序变迁、新旧交替这一自然规律的诗句是:      
      ,       。(山东省烟台市)
   23.“ ? 。”崔颢登上黄鹤楼,见江雾迷漫,暮霭沉沉,对故乡的思念之情油然而生。(太原)
   24.《别董大二首(其一)》中,诗人给友人以勉励与自信的诗句是 ,
   ?(上海市)
   25.人们常常引用李商隐《无题》中的“ , ”的诗句来赞颂那些献身事业的人。(四川省)
   26.陆游在《十一月四日风雨大作》中,直接表现其爱国思想情怀的诗句是“
   , ”。(四川省)
   27.杜甫《望岳》诗中,诗人想象登上泰山之巅抒发豪情的诗句是“ ,
   。”(四川省资阳市)
   28.夕阳西下,告别荷塘。犹豫的脚步留恋的心。依然徜徉于“ ”的夏之梦中。(浙江省上虞市)
   29.王维《使至塞上》中描写边塞奇特壮美景色的句子是:“____________ __,
   _____________________。”(河南省)
   30.陆游《游山西村》中描写农村变化万千的自然风光而又有一定哲理的句子是:“_________________,___________________。” (河南省)
   31.大自然美丽的湖光山色涤荡了我们的心胸,让我们心清如水;愉悦了百鸟的性情,使它们在山林间尽情歌唱。这真是“ , 。”(填写常建《题破山寺后禅院》的诗句)(河北省)
   32.《汉江临眺》中“ , 。”两句以苍茫山色烘托水势的浩瀚,如电影镜头样展现了汉江的山光水色。(湖北省十堰市)
   33.《己亥杂诗》中表现作者仍愿报效祖国并蕴含哲理的名句是:
   。(湖北省十堰市)
   34.常建的《题破山寺后禅院》中有两句诗各含有一个成语,这两句诗是:“
   ”、“ ”。(湖北省仙桃市)
   35.友情,历来为人所珍重。王勃的《送杜少府之任蜀州》,留下真诚勉励友人的千古名句“ , 。”李商隐在《夜雨寄北》中,则想象了与友人剪烛夜谈的情景:“ , 。”(湖北省随州市)
   36.王安石的《登飞来峰》中与“会当凌绝顶,一览众山小”有异曲同工之妙的句子是:
   , 。(湖北省襄樊市)
   37.李商隐的《无题》中常被后人用来形容“鞠躬尽瘁,死而后已”精神的诗句是:
   , 。(湖北省襄樊市)
   38.在我国传统文化中,人们常以月寄情。如唐代诗人李白在《闻王昌龄左迁龙标遥有此寄》中以 , 两句,把对友人的怀念之情托付给明月。(湖北省襄樊市)
   39.《行路难》(其一)中,表现作者对理想执着追求,对未来充满希望的句子是_____________,_______________。(山东省济宁市)
   40.人们常用李商隐《无题》中的两句诗来赞美老师那种默默奉献的精神,这两句诗是:
   , 。(湖南省永州市)
   41.《游山西村》中写人在困境中会萌发出新的希望,也暗示世间事物的消长变化的诗句是: , 。(湖南省永州市)
   42.王湾在《次北固山下》一诗中描绘涨潮时水面宽阔、帆船顺风而行的句子是“
   , ”。(南京市)
   43.王维《使至塞上》中描绘沙漠雄奇壮美景色的两句诗是 ,
   。(南宁市)
   44.杜甫在《望岳》中用 , 来表现自己敢于攀登绝顶,俯视一切的雄心和气魄。(南宁市)
   45.《过零丁洋》一诗中,表现文天祥高尚节操和舍生取义的生死观,从而感召后代许多志士仁人为正义事业而英勇献身的诗句是:“ , 。” (江苏省南通市)
   46.同为送别诗,“海内存知己,天涯若比邻”表现了王勃乐观豁达的胸襟和对友人的真挚情谊,而岑参的《白雪歌送武判官归京》一诗中用“ , 。”的诗句表达了作者对友人依依不舍的惜别之情。(江苏省南通市)
   47.《次北固山下》中即景抒情又蕴含自然理趣的两句诗是_____________ ,___ ________________。(南昌)
   48.唐代边塞诗人岑参在《白雪歌送武判官归京》中以梨花喻雪的名句是
   , 。(西宁市)
   49.《酬乐天扬州初逢席上见赠》一诗中借用自然景物的变化暗示社会的发展,蕴含哲理的句子是_______________________,___________________。(山东省济宁市)
   50.杨大伟带领全厂职工进行技术改革时遇到了重重困难,他在鼓励职工们坚定信心坚持改革时引用了李白《行路难》中的两句诗。这两句诗是 , 。(山东省潍坊市)
51.德高望重的李老师退休后仍然关心青年教师的成长,正如龚自珍在《己亥杂诗》中所说的:“_____________________ ,____________________”。(南昌)
   52.宋朝人写诗往往蕴含哲理,引人思考。王安石的《登飞来峰》、朱熹的《观书有感》、陆游的《游山西村》,都体现出这一特点。请从上述作品中任选两篇,写出其中蕴含哲理、引人思考的诗句:“ , 。”“ ,
   。”(湖北省随州市)
   53.苏轼的《江城子·密州出猎》文笔豪迈奔放,有江河一泻千里的气概,词中表达作者渴望得到重用的愿望,于豪迈之外稍含不满之意的两句是:__________________(辽宁省锦州市)
   54.张养浩在《山坡羊·潼关怀古》这首曲中,对在离乱中遭受苦难的人们表示同情的句子是 , 。(山东省潍坊市)
   55.李清照《醉花阴》中形容人极度悲伤、愁苦的千古名句是 ,
   。(呼和浩特市)
   56.《天净沙·秋思》中,渲染萧条、冷落、凄凉气氛的写景的句子是: ;
   。(昆明市)
   57.晏殊的《浣溪沙》一词中直接抒发伤感情绪的句子是: ,
   。(昆明市)
   58.马致远在《天净沙·秋思》中写到:“夕阳西下,断肠人在天涯。” 而崔颢在《黄鹤楼》中也有两句诗与此意境相似,这两句是:“ , 。(重庆市)
   59.辛弃疾《破陈子·为陈同甫赋壮词以寄之》中描写战斗场面激烈,表现义军所向披靡的句子是“ , ”。
   60.同样是咏秋,刘禹锡的《秋词》通过“ ”的描述,写出了秋天明艳美丽的景致,而马致远的《天净沙·秋思》以一句“ ”写尽了秋天的肃杀之气。(湖北省随州市)
   61.“我爱水,愿以水裁衣,以云织锦,做一条弯弯的川流,在斜晖脉脉时,送千帆天际;愿以山切割,以石磋磨,做一涧高高低低的急流,在猿啼虎啸中,将一叶轻舟,载过万重山。”其中画线句子分别化用的诗句是“_____________________,_____________________”和“两岸猿声啼不住,轻舟已过万重山”。(河南省)
   62.苏轼在《水调歌头》中望着明月遥祝兄弟平安,现在人们也常常用来祝福亲友的词句是: , 。(广西壮族自治区玉林市)
   63.李清照词《醉花阴》的末尾5个字,既惜花,又自怜,既摹形,又传神。这就是
   “___________________”。(福建省龙岩市)
   64.诸葛亮的《出师表》说理和陈情得以完美的结合,在陈情部分表现他临危受命的千古名句是: , 。(长春市)
   65.生命是什么?生命就是范仲淹“ , ”的远大抱负;生命就是文天祥“ , ”的浩然正气;生命就是龚自珍“ , ”的献身精神。(福州市)
   66.《桃花源记》中写老人和孩子神情的句子是 。(新疆生产建设兵团)
   67.读了莫泊桑的《富楼拜家的星期天》,会感叹他的朋友真多,我们会不由得想起《陋室铭》中的一句话 , 。(四川省内江市)
   68.《诸葛亮集》中有这样的话:“赏不可不平,罚不可不均”,这与《出师表》中的“ , ”意思一致。(福建省莆田市)
   69.人生常常面临各种各样的选择。当“义”和“生”不能两全时,我们应当像孟子《鱼我所欲也》一文所说的那样“ ”。(广东省佛山市)
   70.张红在小组合作学习中,经常觉得其他同学的许多想法对自己很有启发,这使她对孔子所说的“           ”这句话体会更深了。(广东省河源市)
   71.孟子在《得道多助,失道寡助》中指出决定战争胜负的三要素及其关系的句子是
   , 。(广西壮族自治区)
   72.《〈论语〉十则》中阐述学习与思考的辩证关系的句子是 ,
   。(广西壮族自治区河池市)
   73.陈水扁要搞“台独”,遭到了海峡两岸人民的强烈反对。这正如《<孟子>二章》中写的那样: , (广西壮族自治区玉林市)
   74.《三峡》中用快马和疾风来写船行之快的句子是 ______________________。(哈尔滨市)
   75.《出师表》中诸葛亮在自叙本志和追述先帝知遇之恩时写下的临危受命的千古名句
   是_____________________,_____________ ______。(哈尔滨市)
   76.读书的方法有很多,可精读、细读,也可如陶源明那样“_______________”,观其大略。(河南省)
   77.《醉翁亭记》是一篇优美的游记体散文,虽题为“醉翁亭记”,但用于正面描写醉翁亭的只有六个字,这六个字是“__________________”。(辽宁省锦州市)
   78.由“四面湖山收眼底,万家忧乐在心头”这幅对联,你一定能联想到范仲淹《岳阳楼记》中的两句千古名句: , 。(湖北省黄冈市)
   79.加入世贸组织后,我们要虚心向发达国家学习,但发达国家有长处,也有短处,那么我们所持的正确态度应该是:“ 。”(用《〈论语〉十则》中的语句填写)(湖北省仙桃市)
   80.《五柳先生传》中写陶渊明独特的读书方法的句子是: ,
   。(湖南省永州市)
   81.范仲淹在《岳阳楼记》中赞美滕子京政绩的话是: , 。(江苏省淮安市)
   82.读书时,既需要字斟句酌的精读,又需要像陶渊明《五柳先生传》中主张的那样
   “ , ”。(南京市)
   83.《三峡》中“有时朝发白帝,暮到江陵,其间千二百里,虽乘奔御风不以疾也”一句突出表现了水流的湍急,《与朱元思书》一文中与它有异曲同工之妙的句子是
   。(西宁市)
   84.小明学习成绩好,组织能力强,但他刚转到这个班,不为同学所了解,因此在班干部竞选中落选了。李老师引述了《论语》十则中的两句话:“ , ”,帮他解开了心中的疙瘩。(西宁市)
   85.市委、市政府新一届领导班子响亮的提出“重振雄风、富民升位”的口号,带领全市人民齐心合力,艰苦奋斗,创设了一个安定的社会环境,各项事业蓬勃发展,老工业基地焕发了青春,真是 , 。(用《岳阳楼记》中的语句填写)(四川省内江市)
   86.去年六月,举世瞩目的三峡工程成功实现“蓄水315米”,高峡出平湖的壮观景象展现在世人面前,郦道元描述的昔日“渔者歌曰”的“ , ”的凄异景状将一去不复返了。(四川省资阳市)
   87.人生常常面临着各种各样的选择,当“义”与“生”不能两全时,我们应当像孟子所说的那样“ ”。(四川省资阳市)
   88.《捕蛇者说》中,写凶暴的小吏到乡间情形的四句话:_______________,________ ____,______ _____,________ ____。(浙江省台州市)
   89.《爱莲说》中写莲的高洁、质朴的语句是___________ ______。(浙江省台州市)
   90.《始得西山宴游记》写作者在大自然美景中的超然忘我、与自然融为一体的句子是:“
   , 。”(河北省)
   91.诸葛亮在《出师表》中指出西汉兴旺发达的原因是: , 。(江苏省淮安市)
   92.人们常用《爱莲说》中的有关语句来比喻人们不与世俗同流合污而又洁身自好的品质,这两句是“ , 。”

第六讲 诗词赏析
 
   【考查要点】
   课标和教材关于诗词欣赏的要求,主要有以下五点:①欣赏诗歌,有自己的情感体验;②领略诗歌的内涵,从中获得对自然、对社会和对人生的有益启示;③对诗歌的思想感情倾向,能联系文化背景作出自己的评价;④对诗歌中感人的情境和形象,能说出自己的体验;⑤能品味诗歌中富于表现力的语言。
   【知识疏理】
   古今诗词欣赏的知识点主要有:诗词曲的形象、诗词曲的情感倾向、诗词曲的思想内容、诗词曲的语言品味、诗词曲的名句赏析、诗词曲的阅读体验、诗词曲名句的课外积累、诗词曲中的民俗知识、诗词曲的体裁知识等。
   【试题特点】
   2004年全国各地中考诗词欣赏题的题型主要有:问答题、填写题、填空题、选择题、简述题等,以问答题为主。
   【解题导引】
   例1.(2002年甘肃省)读《送杜少府之任蜀州》,完成(1)—(3)题。
   城阙辅三秦,风烟望五津。
   与君离别意,同是宦游人。
   海内存知己,天涯若比邻。
   无为在歧路,乙女共沾巾。
   (1)诵读节奏划分有误的一项是( )
   A.城阙/辅/三秦,风烟/望/五津。
   B.与君/离/别意,同是/宦/游人。
   C.海内/存/知己,天涯/若/比邻。
   D.无为/在/歧路,儿女/共/沾巾。
   (2)赏析有误的一项是( )
   A.这是一首五言律诗,作者是被称为“初唐四杰”之一的王勃。
   B.第一句点出了送别的地点,第二句则把读者的视线引向了杜少府即将赴任之地。
   C.颔联是说,诗人要跟朋友分手,去外地做官。
   D.尾联紧承颈联而来,诗人劝慰杜少府,不要在临别之时像小儿女,一般哭鼻子、抹眼泪。
   (3)这首诗中的“海内存知己,天涯若比邻”是千古传诵的句子,请说说它好在哪里。
   此题比较全面地考查了对古诗的赏析,突出了文学鉴赏能力的重要性。第(1)题考查对诗歌的诵读。诵读是欣赏古诗的前提。只有读清节奏,方能更好地理解诗句意思。B项错在“离别”和“宦游”是一个词语不能分开,在这里它们起修饰作用。第(2)题考查对常识和内容、情感的把握。C项错在对诗句意思的曲解。诗人与朋友已在外做官了。第(3)题考查对名句妙处的品味,属开放性试题。答题时应善于对比,领悟感情基调独具一格。参考答案为:自古以来,写离别的诗作大多情感悲伤、基调低沉,而王勃的这两句诗则全无悲伤之情,写得昂扬乐观,能给朋友以安慰和鼓励,这种对待离别的积极态度值得称道。
   解答诗词欣赏题,首先是要略知诗词的基本内容,其次是要把握诗词的具体形象,第三是要品味诗词的美言妙语,第四是要了解诗词的表达技巧,第五是要揣摩作者的情感倾向。这五点没有先后之分,考生完全可以根据自己的阅读习惯,选取其中任意一点先行突破。抓住一点,再及其余。在涉及到具体试题时,还要针对题型作答。对于选择题,答题时应首先审读题干要求,看清选正还是选误,然后逐行逐字逐句地审读选项,根据自己的理解逐渐缩小正确答案的范围,直到选出符合题目要求的选项。对于主观表述题,则应准确、简洁地填写,特别是对于开放型的题,要紧扣原诗作答。
   【常见失误】
   诗词欣赏是近几年中考新增的考试内容。从考试情况来看,考生往往在这样几个方面出现失误:一是难以把握诗词的具体形象所要暗示或启迪读者的东西;二是难以领悟诗词中的比喻、象征的内涵;三是难以通过诗词具体形象的表面呈现,发掘其深层意蕴。出现这些失误的原因,不外乎以下两个:一个是诗词常识比较欠缺,尤其是古代诗词常识;另一个是接触古今诗词作品太少。
   【考点精练】
   诗词赏析
   1.(2004·海淀)对诗歌赏析有误的是( )
   登飞来峰
   王安石
   飞来山上千寻塔,闻说鸡鸣见日升。
   不畏浮云遮望眼,自缘身在最高层。
   A.这是一首七言绝句,前两句写出了飞来峰塔之高,后两句写登飞来峰塔的感想。
   B.这首诗前两句的意思是:我登上飞来峰顶寻找高高的塔,听说每天黎明鸡叫的时候,在那里可以看到日出。
   C.这首诗的后两句表面看是写自然现象,实际暗指社会现象。这里表示自己站得高,看得远,不怕阻挠。
   D.作者善于把抽象的事理寓于具体形象中,借景抒怀,表明自己不畏艰难,对前途充满信心。
   2.(2004·遂宁)下面对《望岳》一诗理解不正确的一项是( )
   望 岳
   杜甫
   岱宗夫如何,齐鲁青未了。
   造化钟神秀,阴阳割昏晓。
   荡胸生层云,决眦入归鸟。
   会当凌绝顶,一览众山小。
   A.这首诗写出了诗人登上泰山后的所见所感。
   B.这首诗将写景和抒情结合起来,情景交融。
   C.颔联前句形容泰山凝聚了所有山峦应有的“神秀”,而后句则形象的衬托出遮天蔽日、高峻峭拔的山峰。
   D.全诗意境开阔,形象鲜明,格调高昂,气势磅礴,给人以极深的印象。
   3.(2004·襄樊)对李商隐的《夜雨寄北》品析有误的一项是( )
   君问归期未有期,巴山夜雨涨秋池。何当共剪西窗烛,却话巴山夜雨时。
   A.第一句写诗人虽日日思归,却没有准确归期,无可奈何之情跃然纸上。
   B.第二句写想象景象,把已跃然纸上的愁苦交织于幻想中连绵的夜雨、涨满的秋池。
   C.第三、四句超越时空,把当前的时间推移到未来,让未来的欢聚反衬出今夜的愁苦。
   D.全诗“期”字和“巴山夜雨”重复出现,造成回环往复的意境,抒发了诗人羁旅他乡时思“君”的深情。
   4.(2004·无锡)对下列两首诗理解不正确的一项是(  )
          过零丁洋            别云间
   文天祥       夏完淳
   辛苦遭逢起一经,干戈寥落四周星。   三年羁旅客,今日又南冠。
   山河破碎风飘絮,身世浮沉雨打萍。   无限河山泪,谁言天地宽!
   惶恐滩头说惶恐,零丁洋里叹零丁。   已知泉路近,欲别故乡难。
   人生自古谁无死,留取丹心照汗青。  毅魄归来日,灵旗空际看。
   A.《过零丁洋》是一首七言律诗,四联八句,全诗押的是ing韵;《别云间》是一首五言律诗,也是四联八句,押的是an韵。
   B.《过》诗多用对偶句,如“山河破碎风飘絮,身世浮沉雨打萍”和“惶恐滩头说惶恐,零丁洋里叹零丁”,对仗极为工整。
   C.《别》诗中“三年羁旅客,今日又南冠”一联点明自己被捕,过了三年漂泊的生活,今天又成了囚徒,同时表明自己要像楚人钟仪那样忠于故国的意志。
   D.“人生自古谁无死,留取丹心照汗青”表明自己以死明志的决心,全诗透露出作者内心的恐惧和对前途的迷惘。
   5.(2004·临沂)对下面这首诗分析理解不当的一项是( )
   敕勒歌
   敕勒川,阴山下。
   天似穹庐,笼盖四野。
   苍苍,野茫茫,风吹草低见牛羊。
   A.这是一首北朝民歌。它歌唱大自然的辽阔壮丽,大草原的肥沃昌盛,反映了牧民们以天地为家的豪迈气概和对草原生活的亲切感情。
   B.“天似穹庐,笼盖四野。”写天空像巨大无比的贺顶毡帐,将整个大草原笼罩起来。比喻十分生动形象,充分显示出游牧民族的生活特色。
   C.诗中前几句写平川、大山、天空、四野,意境阔大恢宏,采用了静态勾画,形象逼真;最后一句转入动态描绘,使全诗动静结合,充满了勃勃生机。
   D.诗的最后一句堪称点睛之笔,其中的“风”字有极强的表现力。由于风气吹动使牧草低伏,人们才看见了草中洁白的羊群。
   6.(2004·温州)对下面这首古诗理解错误的一项是( )
   枫桥夜泊
   张继
   月落乌啼霜满天,江枫渔火对愁眠。
   姑苏城外寒山寺,夜半钟声到客船。
   A.这是一首记叙夜泊枫桥的春天景象和感受的诗。
   B.“江枫”指江边的枫树。
   C.“夜半钟声到客船”,说明诗人到半夜没睡着,睡不着的原因是因为漂泊在外羁旅之愁满怀。
   D.此诗景中寓情,情景交融,诗中的景物在诗人眼里都染上了浓重的感情色彩。
   7.(2004·重庆)下列诗句表现的自然季节与其他三项不同的一项是( )
   A.天街小雨润如酥,草色遥看近却无。
   B.渭城朝雨邑轻尘,客舍青青柳色新。
   C.忽如一夜春风来,千树万树梨花开。
   D.乱花渐欲迷人眼,浅草才能没马蹄。
8.(2004·昆明)品读下面这首古诗,选出理解不正确的一项( )
   长歌行
   汉乐府民歌
   青青园中葵,朝露待日晞。阳春布德泽,万物生光辉。常恐秋节至,焜黄华叶衰。百川东到海,何时复西归?少壮不努力,老大徒伤悲。
   A.“朝露待日晞”的“晞”可理解为“晒干”;“焜黄华叶衰”的“衰”可理解为“衰败”。
   B.这是一首咏物诗,诗人用托物言志的手法,通过自然景物的盛衰规律来警示我们要珍惜时光,及时努力。
   C.诗的最后两句堪称点睛之笔,它精炼醒目,与岳飞的《满江红》中的“莫等闲,白了少年头,空悲切!”有异曲同工之妙,均是千古名句。
   D.作者的目的是通过描写冬葵、百川,赞美万物的勃勃生机。
   9.(2004·资阳)品读下面这首诗,按要求作答。
   题破山寺后禅院
   常 建
   清晨入古寺,初日照高林。
   曲径通幽处,禅房花木深.
   山光悦鸟性,潭影空人心。
   万籁此都寂,但余钟磬声。
   ⑴对诗句的理解有误的一项是( )
   A.起首两句写诗人在清晨入破山兴福寺时,旭日初升,光照树林。
   B.三、四两句用“曲”“幽”“深”等字目表现禅房花木扶疏、幽深清净的环境。
   C.五、六两句书写诗人主观感受,—个“悦”字表现作者从中感受到的欢悦,“潭影空人心”,表现诗人精神上完全消除俗念,获得愉悦的内心感受。
   D.这首诗题为“破山寺后禅院”,可见“破山寺”是全诗重点所在,所以在第一、二句景致刻画后,立即转入对“破山寺”的描述。
   ⑵这首诗描写的主要内容是 ,流露出了诗人 倾向。
   10.(2004·仙桃)
   浩荡离愁白日斜,吟鞭东指即天涯。落红不是无情物,化作春泥更护花。
   (龚自珍《己亥杂诗》)
   (1)此诗第一句写诗人离别的忧伤,其中直接表现离愁深重的词语是: 。
   (2)对这首诗赏析不正确的一项是( )
   A.诗的第一、二句真实地反映了诗人当日复杂的心境:离别的愁绪和回归的喜悦相互交织。
   B.诗的第三、四句从落花到春泥展开联想,将诗人的时代使命感移情落花,从而把离愁升华为崇高的献身精神。
   C.诗中用冷色调的“白日斜”烘托离愁,用“落红”一句作为情感的转折,使整首诗从离愁中解脱出来。
   D.全诗表达的思想和陶渊明《归园田居》(种豆南山下)所表达的与官场决裂、归隐田园的遁世思想相同。
   11.(2004·甘肃)阅读下面一首词,完成(1)—(3)题。
   破陈子
   为陈同甫赋壮词以寄之
   辛弃疾
   醉里挑灯看剑,梦回吹角连营。八百里分麾下炙,五十弦翻塞外声。沙场秋点兵。马作的卢飞快,弓如霹雳弦惊。了却君王天下事,赢得生前身后名。可怜白发生!
   (1)从下列诗句中选出风格完全不同的一句,并说说理由。( )
   A.会挽雕弓如满月,西北望,射开狼。
   B.夜阑卧听风吹雨,铁马冰河入梦来。
   C.过尽千帆皆不是,斜晖脉脉水悠悠。
   D.……男儿到死心如铁,看试手,补天裂。
   理由:
   (2)对这首词的赏析,不正确的一项是( )
   A.“破阵子”是这首词的题目,表达了作者杀敌立功的决心。
   B.这首词是写给好友陈同甫的,抒发了作者的抗金壮志。
   C.“塞外声”是指塞外边境的悲壮粗犷的战歌。
   D.末句“可怜白发生”表达了作者壮志难酬的遗憾。
   (3)请写出两句描写“风”或“雪”的古典诗词名句。
   12.(2004·南通)阅读下面一首诗,然后回答问题。
   己亥杂诗
   龚自珍
   浩荡离愁白日斜,吟鞭东指即天涯。
   落红不是无情物,化作春泥更护花。
   这首诗前两句抒发了诗人不得已辞官还乡时的无限愁思,而后两句却表达了他对生命意义与价值的积极思考:
   13.(2004·西宁)阅读下面的诗与散曲,按要求填写相关文字内容。
   秋词
   刘禹锡
   自古逢秋悲寂寞,
   我言秋日胜春朝。
   晴空一鹤排云上,
   便引诗情到碧霄。
   天净沙·秋思
   马致远
   枯藤老树昏鸦,
   小桥流水人家,
   古道西风瘦马。
   夕阳西下,
   断肠人在天涯。
   上面的作品同样是写秋景,但作者表达的思想感情截然不同。刘禹锡所表达的思想感情是 ;而马致远却抒发了 。在表现手法上,《秋词》是议论兼抒情,而《天净沙·秋思》却是 。
   14.(2004·宾州)阅读下面一首诗,完成(1)—(3)题。
   春夜喜雨
   杜甫
   好雨知时节,当春乃发生。随风潜入夜,润物细无声。
   野径云俱黑,江船火独明。晓看红湿处,花重锦官城。
   (1)这首诗以“好雨”开头,请说明作者称赞春雨的原因。
   答:
   (2)这首诗的第二联、第三联分别从哪种感觉来写春雨?
   答:
   (3)你认为“随风潜入夜,润物细无声”中哪个词用得好?为什么?
   答:
   15.(2004·陕西)下面这首古诗,在内容和写法上都有特点,请选择一个方面赏析。
   过松源晨炊漆公店
   [宋]杨万里
   莫言下岭便无难,赚(骗)得行人空喜欢;
   正入万山圈子里,一山放过一山拦。
   16.(2004·泸州)品读下列古诗,按要求答题。
   江南春绝句
   杜牧
   千里莺啼绿映红,水村山郭酒旗风。
   南朝四百八十寺,多少楼台烟雨中。
   (1)为了突出江南春色,诗人选取了哪些具有江南特色的风光景物?
   答:                                
   (2)诗人笔下的江南春景,鲜明、生动、形象,请问:这种效果是通过哪些写作手法生的?
   答:                               
   (3)明朝杨慎在《升庵诗话》中谈到这首诗,认为“千里”的“千”字应改为“十”。理由是:“千里莺啼,谁人听得?千里绿映红,谁人见得?”此言遭到不少学者反驳。你认为慎的观点错在何处?
   答:                                
   17.(2004·四川)阅读下面古诗,按要求回答问题。
   春 望
   杜甫
   国破山河在,城春草木深。 感时花溅泪,恨别鸟惊心。
   烽火连三月,家书抵万金。 白头搔更短,浑欲不胜簪。
   (1)诗中“草木深”表面上写的是 ,实际上是写 。
   (2)本诗将眼前景、胸中情融为一体。通读全诗,谈谈这首诗抒发了诗人怎样的情感?
   答:
   18.(2004·桂林)
   茅屋为秋风所破歌
   杜甫
   八月秋高风怒号,卷我屋上三重茅。茅飞渡江洒江郊,高者挂罥长林梢,下者飘转沉塘坳。
   南村群童欺我老无力,忍能对面为盗贼。公然抱茅入竹去,唇焦口燥呼不得,归来倚仗自叹息。
   俄顷风定云墨色,秋天漠漠向昏黑。布衾多年冷似铁,娇儿恶卧踏里裂。床头屋漏无干处,雨脚如麻未断绝。自经丧乱少睡眠,长夜沾湿何由彻!
   安得广厦千万间,大庇天下寒士俱欢颜!风雨不动安如山。呜呼!何时眼前突兀见此屋,吾庐独破受冻死亦足!
   (1)诗的第一节是怎样描绘秋风肆虐的情景的?诗人为什么要作这样的描绘?
  
   (2)杜甫一向关心人民疾苦,他的诗素有“诗史”之称,但为什么这首诗中他却一反常态,只写自己的个人遭遇?请谈谈你的理解。
  
   19.(2004·广州)阅读下面的宋词,然后回答问题。
   浣溪沙
   晏殊
   一曲新词酒一杯,去年天气旧亭台。夕阳西下几时回?
   无可奈何花落去,似曾相识燕归来。小园香径独徘徊。
   这首词中多处用到对比的手法,请找出其中的一个例子,并简要说说这样写的好处。
  
   20.(2004·河南)阅读下面古诗,按要求回答问题。
   钱唐湖春行
   白居易
   孤山寺北贾亭西,水面初平云脚低。
   几处早莺争暖树,谁家新燕啄春泥。
   乱花渐欲迷人眼,浅草才能没马蹄。
   最爱湖东行不足,绿杨阴里白沙堤。
   ①本诗第二至六句,一句一景,在动态的描绘中写出了西湖景色的清丽明快,生机勃勃。这些景物依次是:春水初平, ,燕啄春泥,乱花迷眼, 。
   ②在前六句诗中,诗人已经把西湖春色描绘得十分美好,在结尾处却说自己“最爱”湖东的白沙堤。至于白沙堤怎么可爱,除点明“绿杨阴里”外,诗人未做任何解释。你觉得诗人这样写有什么好处?

21(2004·青海)
   已亥杂诗
   龚自珍
   浩荡离愁白日斜,吟鞭东指即天涯。
   落红不是无情物,化作春泥更护花。
   (1)诗中“吟鞭东指即天涯”中的“吟鞭”指 。
   (2)“落花不是无情物”中,作者以 比喻 。“化作春泥更护花”一句表达了作者想继续为 效力的思想感情。
   22.(2004·玉林)阅读下面古诗文,完成问题。
   饮 酒
   陶渊明
   结庐在人境,而无车马喧。问君何能尔,心远地自偏。采菊东篱下,
   悠然见南山。山气田夕佳,飞鸟相与还。此中有其意,欲辨已忘言。
   (1)下列对这首诗的理解和分析,不恰当的一项是( )
   A.诗的前四句道出了作者的体验:要拥有心灵的字根表,就应该脱离喧嚣的环境,投身到大自然中,尽情体会生活的乐趣。
   B.“山气日夕佳”中的“佳”字,写出于作者对山气缭绕、夕阳西下这土贳象的赞美,表现了诗人归隐田园的惬意情怀。
   C.“此中有真意,欲辨已忘言”的意思是:这种生活让人体会到人生的真正意义,但无法用语言表达,只能用心灵去感受。
   D.本诗描写了恬静、闲适的生活,表现了诗人与大自然相亲相融的境界。
   (2)古人往往以所喜爱的花来表明自己的志趣。你觉得诗中的“采菊”与“真意”之间有什么内在联系?
   答:
   23.比较赏析下面两首诗,然后回答问题。
   [甲]胜败兵家事不期,包羞忍耻是男儿。
   江东子弟多才俊,卷土重来未可知。(杜牧诗)
   [乙]百战疲劳壮士哀,中原一败势难回。
   江东子弟今虽在,肯为君王卷土来?(王安石诗)
   (1)杜牧、王安石两位诗人对同一历史人物评价的观点不同,请各找一句表达他们不同观点的诗。(用原诗作答)
   答:杜诗: 王诗:
   (2)①[甲]、[乙]两诗涉及的人物是谁?
   ②对他的生平事迹有详细记载的是哪部书?
   ③这部书的作者是谁?
   ④鲁迅对这部书有两句精辟的评价,请写出来。
   24.(2004·黄石)
   饮酒
   陶渊明
   结庐在人境,而无车马喧。
   问君何能尔,心远地自偏。
   采菊东篱下,悠然见南山。
   山气日夕佳,飞鸟相与还。
   此中有真意,欲辨已忘言。
   (1)下列对诗句理解有误的一项是(  )
   A.这首五言古诗通过对田园生活中自然景色的悠闲欣赏,反映出作者厌恶官场腐败、决心归隐自洁的人生追求。
   B.第一、二句直接道出作者对宁静田园生活的喜爱,对车马喧嚣的官场生活的厌倦。
   C.第三、四句自问自答,表现了作者对自由自在、超凡脱俗境界的追求。
   D.第五――八句将菊花、南山、山中晚景、归林飞鸟构成一幅大自然的美丽画面,作者完全陶醉在悠闲自得之中。
   E.“此中有真意,欲辨已忘言”两句是全诗的总结,意即大自然有人生的真义,作者想说出来却不能说出。
   (2)“采菊东篱下,悠然见南山“中的“见”改成“望”好不好?为什么?
  
   (3)“山气日夕佳,飞鸟相与还”两句写出了日近黄昏、云入山山配飞鸟入林的景象。请联系上下文,品析诗句的深层喻义。
  
   25.(2004·绵阳)阅读下面一首诗,然后回答问题。
   泊秦淮
   杜牧
   烟笼寒水月笼沙,夜泊秦淮近酒家。
   商女不知亡国恨,隔江犹唱《后庭花》。
   作者在诗的首句连用了两个“笼”字,请结合全诗简要赏析其作用。
  
   26.(2004·哈尔滨)读《白雪歌送武判官归京》一诗,回答问题。
   白雪歌送武判官归京
   北风卷地白草折,胡天八月即飞雪。忽如一夜春风来,千树万树梨花开。散入珠帘湿罗幕,狐裘不暖锦衾薄。将军角弓不得控,都护铁衣冷难着。瀚海阑干百丈冰,愁云惨淡万里凝。中军置酒饮归客,胡琴琵琶与羌笛。纷纷暮雪下辕门,风掣红旗冻不翻。轮台东门送君去,去时雪满天山路。山回路转不见君,雪上空留马行处。
   这是唐代诗人岑参的一首送别诗。诗中既着力描写了边地特有的奇异风光,又表现了诗人因朋友的离去而产生的无限惆怅。诗中画线句子是流传千古的名句,构思新颖,联想奇特,请结合诗的内容,说说这两句诗描绘了怎样的画面?
  
   27.(2004·河南)阅读下面古诗,按要求回答问题。
   左迁至蓝关示侄孙湘
   韩愈
   一封朝奏九重天,夕贬潮州路八千。
   欲为圣明除弊事,肯将衰朽惜残年。
   云横秦岭家何在?雪拥蓝关马不前。
   知汝远来应有意,好收吾骨瘴江边。
   (1)第一、二句中的“朝奏”与“夕贬”、“九重天”与“路八千”形成鲜明的对比,让我们深切地感受到诗人命运的急剧变化。其中蕴含了诗人怎样的思想感情?
  
   (2)第六句“雪拥蓝关马不前”借景抒情,并做到了眼前景与心中情的完美统一。请你对此加以分析。
  
   28.(2004·恩施)下面这首绝句通过淡淡的背景、稀疏的笔意,写出了春夜的生意盎然,读来亲切有味,境界全出。
   夜月
   刘方平
   更深月色半人家,
   北斗阑干南斗斜。
   今夜偏知春气暖,
   虫声新透绿窗纱。
   (选自《唐宋绝句选注析》)
   (1)第3句诗中专写诗人对春天的独特感受的是哪一个词?
  
   (2)第4句用“新透”写“虫声”,写出了春天来临之妙。请你把这句诗用散文的笔调展现出来。
  
   29.(2004·黄冈)阅读下面的古诗文,回答问题。
   酬乐天扬州初逢席上见赠(刘禹锡)
   巴山楚水凄凉地,二十三年弃置身。
   怀旧空吟闻笛赋,到乡翻似烂柯人。
   沉舟侧畔千帆过,病树前头万木春。
   今日听君歌一曲,暂凭杯酒长精神。
   ①这首诗感情浓郁,但前后基调不同,前四句 ,后四句 ,前后形成鲜明对比。
   ②请描述“沉舟侧畔千帆过,病树前头万木春”所展现出的画面,并揭示诗句的含义。
  
   30.(2004·荆州)阅读晏殊的《浣溪沙》,然后答题。
   一曲新词酒一杯,去年天气旧亭台。夕阳西下几时回?
   无可奈何花落去,似曾相识燕归来。小园香径独徘徊。
   ①从季节、时间的角度看,这首词写的是作者 时的生活和心情。
   ②词中“ , ”一联工整流畅,风韵天然,是深受后人赞赏的名句。
   31.(2004·宿迁)阅读下面一首唐诗,回答问题。
   绝 句
   杜甫
   江碧鸟逾白,山青花欲燃。
   今春看又过,何日是归年?
   ①诗的前两句,描绘了一幅优美怡人的风景画,画中分别描写 、
   、 、和 四种景物,突出了它们的 。
   ②这首诗作于杜甫入蜀以后,从第四句中的“归”可以看出,全诗抒发了作者怎样的感情?
   答:


第七讲 综合考查

   【考查要点】
   语段综合考查是文章整体阅读的浓缩,立足于段。主要涉及语文素养中的一些知识性内容。主要的考查点有:①根据拼音写出汉字或给汉字注音;②理解和运用词语、成语;③寻找关键词语和句子;④调整句子顺序;⑤句子归位,理解句子的深刻含义;⑥语段的基本内容;⑦语段涉及的知识扩展;⑧语段内部词语、句子的表达效果等。
   【知识疏理】
   2004年全国各地中考语文试题在语段综合考查方面,主要涉及这样一些知识点:根据拼音写字;寻找关键词;词语替换;词语、成语填空;语句归位;重点语句理解;句子修改;句子顺序调整;句子续写;根据语境写话;知识扩展;语意理解等。
   【试题特点】
   2004本部分中考题的题型,主要有填写题、问答题、改错题、选择题、想像题、复位题、扩展题等。
   【解题导引】
   例1.阅读下面一段话,按要求完成⑴~⑶题。
   ①一个窗台上有一朵花,这个屋里就有了生气;一个人送给另一个人一束花,这两个人就有了情意。让自己的生命为他人开一朵花,就是提高自己生存的质量。用自己的心为他人做圃,给他人吐一地绿阴,染一片色彩,就是给自己的人生喝彩。
   ②面对“非典”的袭击,一次无cháng( )的献血是一朵花,一个及时的电话是一朵花,一朵花是一次适时的看望,一次大度的让贤是一朵花……
   ③能为别人开花的心是善良的心,能为别人的生活绚丽付出的人是不寻常的人。
   ⑴根据拼音,把汉字写入田字格中。
   无cháng 的献血。
   ⑵结合上下文,说说你对“让自己的生命为他人开一朵花”中“花”的理解。
  
   ⑶第②段句式不够整齐,语意不够连贯,请你修改。
  
   语段综合考查主要涉及语文素养中的一些知识性内容,而且基本上在段内解决。因此解题时仍然要关注语段的基本内容,关注具体的语言环境,同时还要关注语段所牵连到的知识内容。
   【常见失误】
   解答语段综合考查试题的误区,就是没有把语段阅读看成是文章整体阅读的缩影,只见树木而不见森林。应该把每一个考查项目都放在语段的语言环境中去进行。
   【考点精练】
   1.(2004·安徽)按要求回答问题。
   ①据统计,北极的鸟类大约有120种左右,其中多为候鸟,常住的鸟类不到总数的十分之一。②作为对比,南极的鸟类只有43种,永久性的“居民”大概只有企鹅和贼鸥。③生活在北半球的鸟类,大约有六分之一要到北极繁衍后代。④据一位在北极草原观察和研究了十多年的鸟类学家说,光在阿拉斯加北极地区,就有来自世界各地的候鸟在这里居住。
   ⑤北极是全世界几乎所有候鸟的天堂。⑥在南极,飞得最远的是信天翁,但它只是绕南极作长距离的迁移。⑦这是因为,北极不仅有辽阔的草原、丰富的食物,而且还有安静而干净的环境,很少人类干扰。⑧因而,南半球的许多候鸟宁可从遥遥万里飞到北极来越冬,也不愿意到南极去送死。
   (1)第①句画线部分应该删去一个词: 。
   (2)与第④句中“居住”一词意思相近的一个成语是: 。
   (3)第二段中,与本文中心无关的一个句子是第 句。
   (4)请给本文拟一个恰当的标题。
   2.(2004·海淀)阅读下面的语段答题。
   一个人的一生,本来只能经历自己拥有的那一份痛苦与欢乐;然而人们通过阅读,却能进入不同时空的诸多他人的世界。这样,具有阅读能力的人,就可以上溯远古,下及未来,多识草木虫鱼之名,遍览奇山秀水之貌,获得丰富的知识和自身以外的经历。
   人们从《论语》中学得 ① ,从《史记》中学得 ② ,从《正气歌》中学得 ③ ;从鲁迅学得 ④ ,从马克思学得战斗的激情,从列夫·托尔斯泰学得道德追求的执着。
   因此,我们可以这样说,一个有阅读能力的人,是一个幸运的人。
   (1)请你将最恰当的选项依次填入上面选段中的空白处,把所选字母填涂在答题卡上。
   A.批判的精神 B.坚贞的气节
   C.智慧的思考 D.严肃的历史精神
   (2)联系上下文,在第一、二段文字之间补写一个承上启下的过渡句,要求用上关联词语“不仅……而且……”。
   3.(2004·龙岩)阅读下面语段,完成文后各题。
   有人说娱乐圈是个梦工厂,能把灰姑娘变公主,( )变王子。在这个新人旧将如过江之鲫的时代里,如何有效地进行包装推广,可是投资方的最高追求,( )四五百万的预算砸下去,还不见得能捧红一个新人。而无数怀明星梦的少男少女也最终发现,名目繁多的“影视表演学校”不过是一张空头支票,是一个敛钱的无底洞。从1987年初具雏形的第一个“选关赛事”到如今遍地开花的造星大赛,许多曾风光一时的明星如今都销声逆迹了。说白了,主办媒体只是在打造瞬间的极度哀艳,而这样刻意播种出来的花朵往往都在寻梦的道路上(chà)那凋谢。
   (1) 从下面两组词语中各选一个最恰当的依次填入文中括号里。
   青蛙 动用
   ①: ②:
   蜻蜓 动辄
   (2) 根据拼音写汉字及给加点字注音。
   chà ( )那 娱 ( ) 乐圈
   (3) 在下面成语的错别字下加点并更正。
   销声逆迹 更正:_____________________________
   (4) 下列句子与“可是投资方的最高追求”中“可是”一词的意义相同的一项是( )。
   A.他可是最清楚这情况的 B.可是他昨天的确说过这句话
   C.可是脸上却依然笑嘻嘻的 D.可是我从来没有去过那个地方
   (5) 用“凋谢”一词造句。
   凋谢:
   4.(2004·徐州)阅读下面一段文字,完成后面的问题。
   人的一生是不断雕琢的一生。
   雕琢自己,使生命更富有意义;雕琢自己,使人生更灿烂辉煌。青松雕琢自己,从而坚忍不拔、百折不挠,扎根岩石与青天白云游戏;小草雕琢自己, 、
   , ;大海雕琢自己,从而浩淼无边、博大深沉,敞开胸怀拥抱大山和小川;小溪雕琢自己,从而义无反顾、勇往直前,朝着远方的江河和大海奔去。
   雕琢自己并不那么容易,它需要胆识和勇气,听惯了赞誉的华丽之辞,觉得自己“完美无缺”,因而批评对我们总抱有敌意。所以雕琢自己还需要认识自己——你的优势,你的不足,你的志向,你的毅力,你的个性,你的脾气。认识了自己,才知道哪些该凿去,哪些该保留,哪些又该mí补。
   玉经雕琢才成器,剑不沉埋便倚天。雕琢的目的远胜于雕琢的技巧,用心雕琢自己,终会雕出一个yì立大地笑傲苍穹的“人”字来。
   ①根据拼音写汉字。
   mí( )补 yì( )立
   ②与“倚天”中的“倚”字读音相同的一项是( )
   A.绮丽 B.崎岖 C.椅子 D.涟漪
   ③修改画线处句子的语病,把正确的句子写在下面。
  
   ④以“小草”为对象,根据上下文仿写句子。
   小草雕琢自己,
   5.(2004·山西)阅读下面的一段文字答题。
   我们走过漫漫的一生,有时会突然发现自己的生活如此平淡,面对人生涌起的不过是淡而又淡的感觉,我们顿觉自己很平凡,为此我们惆怅,我们感叹!【甲】
   平凡是荒原,孕育着崛起,只要你肯开拓; ;
   平凡是细流,孕育着深suì,只要你肯积累。
   【乙】平凡是一场惊险搏击之后的休qì,是一次辉煌追求之后的沉思。平凡是告别了无知的炫耀之后的成熟,是终止了浅薄的狂妄之后的深沉。平凡不是人生之光的暗淡,不是生命之火的熄灭,不是超然物外的冷漠。【丙】
   白云为每一个平凡变幻多姿,微风为每一个平凡留下清爽,太阳为每一个平凡照出一个明朗的天地。【丁】
   (1)文中第三段加点的“炫”字读音是 。
   (2)根据文中的两处拼音,在下面的田字格中写出正确、规范的汉字。
  
  
   (3)“其实我们不必为平凡悲叹,因为平凡,也是一种美丽!”这句话是从文中抽出来的,它的正确位置应是( )。
   A.甲处 B.乙处 C.丙处 D.丁处
   (4)仿照第二段中的语句形式,在文中横线上补写一句话,使它与前后句衔接自然。
  
   6.(2004·太原)阅读短文答题。
   大自然给予人类的诗意是丰富而缤纷多姿的。
   烂漫的春光里有诗意,萧瑟的秋景中也能找到诗意。你可以为春日暖雨中蔓延的新绿写诗,                    。
   夏日的炎阳照耀着含苞欲放的莲花是一首诗,初冬的冷雨敲打湖面衰败的残荷也是一首诗。诗意的产生,常常是突然而又自然。你无法预知它的到来,而当它出现时,你总是深深为之陶醉。譬如在冰天雪地中行走时,蓦然见到一株灿然zhàn(   )放的梅花;譬如在乌云弥漫时,一缕耀眼的阳光突然穿过云层的缝隙照亮地面;譬如早晨从奇妙的梦中醒来,发现惊醒你的是窗外一只不知名的小鸟在唱歌……
   (1)给加点字注音,根据拼音写出汉字。
   萧瑟( ) Zhàn( )放
   (2)结合全文,说说你对“诗意”一词的理解。
  
   (3)结合语境,填出文中画横线处空缺的句子。
  
   7.(2004·临汾)阅读下面文段,完成文后题目。
   华夏的寺庙庵观何其众,张继的一首《枫桥夜泊》,竟使姑苏城外寒山寺的盛名历千载而不衰;九州的□□□□何其□,王之涣的一首《登鹳雀楼》,却使山西这个寻常楼阁与母亲河黄河一样植根于国人心中。
   在中国2000多个县份中,知名度最高的恐怕要数山西临汾的洪洞县了。洪洞所以( ),首先是因了一首在华夏大地上传唱了600多年的歌谣:“问咱老家在何处,山西洪洞大槐树。祖先故居叫什么,大槐树下老鹳窝。”洪洞所以( ),还因一位天资掩蔼的青楼女子那段凄婉哀凉的吟唱:“苏三离了洪洞县……”京剧是国粹,喜好者兴发时自会哼几句《玉堂春》,不好者偶尔打开电视机、收音机,眼睛或耳朵里说不定也会蹦出个苏三来,于是“洪洞”便深嵌在国人记忆的屏幕上。改革开放以后,中外交流频繁,好奇的洋人竟也学唱京剧,《玉堂春》遂成了他们的首选,通过那些金发碧眼的女郎丰腴的红唇,“洪洞”这个县名,便在异邦传扬流播。
   这就是文化特有的魔力。
   (1)请将下列词语依据恰当的顺序填入文中括号内(只填序号)。
   ①芳名远扬 ②名重神州
   (2)把文中画浪线的句子工工整整地抄写在下面的方格里。
   问咱老家在何处
   □□□□□□□
   □□□□□□□
   □□□□□□□
   (3)请在下面的空格内填入恰当的词语,使其与文中加点的句子构成对偶句。
   华夏的寺庙庵观何其众
   九州的□□□□何其□
   (4)揣摩文中画直线的句子,联系自己的阅读积累,再写一个结构与之基本相同,且能够证明“这就是文化特有的魔力”的句子。
  
   8.(2004·郫县)下面是某校文学社社刊征稿启事的一部分,读后按要求完成后面的题。
   本刊欢迎大家赐稿、来稿,不论是发表过作品的同学,还是刚学写作的同学,都以文章质量高低决定取舍,我刊将一视同仁。来稿体裁不限,提倡创新,以1500字左右为宜。文章写好后必须交到本刊编辑部,本刊有权对录用稿件进行修改。
   ①征高启事中,有一处用语重复、多余,应该删去。这个词语是 。
   ②征高启事中,有两句句序颠倒,影响了意思的表达,应该对调。这两句是
   。
   ③文中用语不得体的一个词语是 ,应改为 。

第八讲 其他训练
   1.(2004·兰州)下列标点运用有错的一项是( )
   A.我感到又羞恼,又冤屈!七八岁的姑娘家,谁愿意落下这么个名声?
   B.隔了几天,李嫂在楼下叫道:“猫,猫!又来吃鸟了。”
   C.别人以为是我臆想出来的——不对——我凭的是记忆。
   D.清代的一位大学问家……戴震,幼时读朱子的《大学章句》,便问《大学》是何时的书,朱子是何时的人。
   2.(2004·哈尔滨)标点符号使用不正确的一项是( )
   A. 我们信它,因为它“是”;不信它,因为它“非”。
   B.“不对,这不是将军家里的狗……”巡警深思地说,“将军家里没有这样的狗。他家的
   狗,全是大猎狗。”
   C.人,每天除了要吃进一定量的水和盐以外,还要吃淀粉、蛋白质、脂肪。
   D.他们说得很快,听不清说些什么?
   3.(2004·襄樊)下列句子标点符号使用正确的一项是( )
   A.什么是故乡?生在哪里?哪里就是故乡。
   B.气质的形成包括“实”和“虚”两个方面。
   C.保尔的一生形象地告诉了我们《钢铁是怎样炼成的》。
   D.“把这瓶家乡的水土带在行李箱里,”妈妈说:“它能保佑你一路平安。”
   4.(2004·永州)给下面句子加上标点符号,使它表示两种不同的意思。
   ①我 说 服 妈 妈 和 你 一 起 走。
   ②我 说 服 妈 妈 和 你 一 起 走。
   5.(2004·南京)下面材料中画横线的四处,标点使用错误的一处是( )
   在第33个世界环境日到来之时,国家环保总局首次推出了世界环境日中国标识。标识 采用代表海洋的大面积蓝色,由“浪花” 、 “鸟”、“手”以及“珍珠”的图形组成。“浪
   A B
   花”象征海洋资源的珍贵,提醒人们与海洋和谐相处 ; “鸟”颈的空白处象征渤海湾,
   C
   与“渤海碧海行动计划”相呼应。各种图案统一在圆形中,体现了中国传统文化中《天人合一》这一哲学思想。
   D
   6.(2004·广安)下列句子的标点符号完全正确的一项是( )
   A.战国时代的孟子有几句很好的话:富贵不能淫。贫贱不能移,威武不能屈——此之谓大丈夫。
   B.自信力的有无。状元宰相的文章是不足为据的,要自己去看地底下。
   C.这是“新约”上的句子罢,但经托尔斯泰新近引用的。
   D.我国内蒙古东部和陕西、山西北部有足够的雨量。就是西北干旱地区,地面径流和地下潜水也是很大的。有些沙荒地区,如河西走廊、柴达木、新疆北部准噶尔和新疆的南部塔里木,都是盆地,周围的高山上有大量的积雪。
   7.(2004·广安)对偶有正对和反对之分,下面属于反对的一项是( )
   A.沉舟侧畔千帆过,病树前头万木春。
   B.初月出云,长虹饮涧。
   C.横眉冷对千夫指,俯首甘为孺子牛。
   D.春蚕到死丝方尽,蜡炬成灰泪始干。
   8.(2004·泸州)下列各句所用的修辞手法判断有误的一项是( )
   A.横眉冷对千夫指,俯首甘为孺子牛。(对偶)
   B.皇帝每一天每一点钟都要换一套衣服,人们提到他的时候总是说:“皇上在更衣室
   里。”(夸张)
   C.你这样一个人玩耍,不也有点寂寞吗?(设问)
   D.春月是一只青春鸟,驮着幽幽夜色,栖落在古城的檐角。(比喻)
   9.(2004·杭州)下列句子中的标点符号使用有错误的一项是( )
   A.通过看地图上等高线的分布和疏密情况,可以知晓大致的地形:等高线越密集,这
   一带的地势越陡;等高线越稀疏,这一带的坡度越缓。
   B.谁不承认春天是美好的?诗人白居易曾这样说:“遇物尽欢欣,爱春非独我。”
   C.水是人类赖以生存的特殊资源。由于一些国家正面临水资源危机,这些国家的农业、
   工业、人民的生活、健康都受到威胁。
   D.如果以花为喻,玫瑰象征春天,石榴象征夏天,争奇斗艳的菊花象征秋天,水仙则是冬天的使者了。
   10.(2004·甘肃)给文中划线部分加标点符号,正确的一项是( )
   也许是辽河水滋润的缘故,白柳条的性格极其柔顺,修长的枝条可以作出多种图案①弯成一个圈儿,像十五的月亮②打成一个弯儿、又好像女孩好看的细眉③因为白柳条宁弯不折,所以常有人用它编织出各种精美的工艺品。
   A.①: ②; ③…… B.①—— ②, ③……
   C.①…… ②。 ③! D.①! ②? ③?
   11.(2004·天津)下面句子中加点的词语必须加上双引号的一项是( )
   A.商店和饭馆的门无精打采地敞开着,面对着上帝创造的这个世界,就跟饥饿的嘴巴一样,门口连一个乞丐也没有。
   B.抱着火,烤了一阵,他哆嗦得像风雨中的树叶。
   C.这时你涌起来的感想也许是“雄壮”,也许是伟大,诸如此类的形容词。
   D.在乌云和大海之间,海燕像黑色的闪电,在高傲地飞翔。
   12.(2004·湖州)下列标点符号的使用,错误的一项是( )
   A.鲁迅先生称赞《史记》是“史家之绝唱,无韵之《离骚》”。
   B.人总是要有点精神的,就看怎么样对待困难,是退避三舍呢,还是迎难而上?
   C.燕子去了,有再来的时候,杨柳枯了,有再青的时候,桃花谢了,有再开的时候。
   D.他去追赶一个在水波上滚动的东西——是一盒用精致纸盒装着的饼干。
   13.(2004·天津)对下面句子运用的修辞方法及其表达作用的分析不正确的一项是( )
   A.江流天地外,山色有无中。(运用对偶和夸张的修辞方法,描绘了一幅江流浩淼、山色迷蒙的优美画面。
   B.如果生命是树,尊严就是根;如果生命是火,尊严就是燃烧;如果生命是鹰,尊严就是飞翔。(运用排比和比喻修辞方法,形象而深刻地说明了尊严高于生命。)
   C.眼前的山,已把夏天的翠裙换成了深秋茶绿色的晚霞。(运用拟人和比喻修辞方法,写出了山色随季节的变化而变化。)
   D.一味地追星,一味地模仿,哪里还会有创新的意识?哪里还会有开拓的精神?(运用设问修辞方法,批评“追星族”缺乏创新意识和开拓精神。)
   14.(2004·宁安)选择这幅对联的上联或下联,用“/”划分节奏。
   绿 柳 沿 堤 皆 因 苏 子 来 时 种
   碧 桃 满 观 尽 是 刘 郎 去 后 栽
   15.(2004·绵阳)下列各项中,朗读的节奏(“/”)和重音(“·”)全都正确的一项是( )
   A.故乡的歌/是一支/清远的笛,总/在有月亮的晚上/响起
   B.开轩/面/场圃,把酒/话/桑麻
   C.故/天/将降大任/于是人也
   D.有的人/活着/他/已经/死了;有的人/死了/他/还活着
   16.(2004·沈阳)选出诵读时节奏划分不正确的一项( )
   A.二者/不可/得兼,舍生/而取义者也。
   B.以为/凡是/州之山/有异态者,皆我有也。
   C.无可奈何/花落去,似曾相识/燕归来。
   D.江流/天地/外,山色/有无/中。
   17.(2004·辽宁)选出诵读时节奏划分正确的一项。( )
   A.梳洗/罢,独/倚望/江楼。
   B.右手/秉/遗穗,左臂/悬/敝筐。
   C.无可/奈何/花落/去,似曾/相识/燕归/来。
   D.采菊/东/篱下,悠然/见南/山。
   18.(2004·锦州)诵读下列句子时节奏划分不恰当的一项是( )
   A.威天下/不以/兵革之利。
   B.不宜/妄自菲薄,引喻失义,以塞/忠谏之路也。
   C.塞下/秋来/风景异,衡阳/雁去/无留意。
   D.君子/于役,如之/何勿思!
   19.(2004·哈尔滨)下面句子朗读不恰当的一项是( )
   A.海内存知己,天涯若比邻。(朗读第二句时,应在“涯”与“若”之间有短暂的停顿,
   以显示诗句的韵律美。)
   B.但愿人长久,千里共婵娟。(朗读第一句用升调,朗读第二句用降调,从而读出诗
   人对远方亲人的思念和美好祝愿。)
   C.我还记得,放假前我默默地站在她的身边,看她收拾这样那样东西的情景。(句中
   的“默默地”要重读,以突出我对蔡老师的依恋难舍之情。)
   D.韩麦尔先生“使出全身的力量,写了两个大字:‘法兰西万岁!’”写完之后,“他呆在
   那儿,头靠着墙壁,话也不说”。(朗读时用无奈、沮丧的语气,读出韩麦尔先生面对国土沦陷的残酷现实,内心极度痛苦的情感。)
   20.(2004·四川)下面的诗句诵读节奏划分不正确的一项是( )
   A.枯藤/老树/昏鸦,小桥/流水/人家
   B.欲渡/黄河/冰/塞川,将登/太行/雪/满山。
   C.感时/花溅/泪,恨别/鸟惊/心。
   D.春蚕/到死/丝/方尽,蜡炬/成灰/泪/始干。
   21.(2004·临沂)下面句子朗读错误的一项是( )
   A.稻花香里说丰年,听取蛙声一片。
   (这两句应当用轻快的语气读,表现诗人陶醉于丰收在望的欢乐之中)
   B.毕竟西湖六月中,风光不与四时同。
   (朗读时,应该在“四”与“时“之间有短暂停顿,以体现六月西湖的特有美景)
   C.“吹面不寒杨柳风”,不错的,像母亲的手抚摸着你。
   (“母亲的手”和“抚摸”要重读,以突出春风温暖、柔和的特点)
   D.蒌蒿满地芦芽短,正是河豚欲上时。
   22.(2004·辽宁)选出对句子翻译不正确的一项。( )
   A.所识穷乏者得我与? 所认识的贫穷的人感激我吗?
   B.洋洋乎与造物者游,而不知其所穷。 洋洋啊与遗物者相处,而不知它的尽头。
   C.不以物喜,不以己悲。 我不因为外物(好坏)和自己(得失)而高兴。
   D.胸胆尚开张。 胸怀还很宽阔,胆气还很豪壮。
   23.(2004·青岛)下列句子与课文原句不一致的一项是(  )
   A.我想靠迅速抓紧时间,去留住稍纵即逝的日子;我想凭时间的有效利用,去弥补匆匆流逝的光阴。
   B.请他们准备欢迎,请所有的人准备欢迎,当雄鸡最后一次鸣叫的时候我就到来。
   C.烁起来了。它是那么大,那么亮,活像一盏悬挂在高空的明灯。
   D.而现在.乡愁是一湾浅浅的海峡,我在这头,大陆在那头。
   24.(2004·自贡)下面的句子都选自要求背诵的课文,文字与原文不一致的一项是( )
   A.抽刀断水水更流,举杯销愁愁更愁。(《宣州谢胱楼饯别校书叔云》)
   B.食肉者谋之,又何间焉?(《曹刿论战》)
   C.求天下奇闻壮观,以知天地之广大。(《上枢密韩太尉书》)
   D.蒹葭苍苍,白露为霜。(《蒹葭》)
   25.(2004·自贡)对下面句子的意思,理解有错的一项是( )
   A.夙夜忧叹[早晚忧愁叹息。]
   B.马作的卢飞快[战马像的卢马那样跑得飞快。]
   C.浩然之气[正大刚直之气。]
   D.关关睢鸠[被关着的睢鸠鸟不停地鸣叫。]
   26.(2004·泸州)指出下列文言句子翻译有误的一项( )
   A.医之好治不病以为功。(《扁鹊见蔡桓公》)
   译:医生喜欢给没病的人治病,把它作为自己的功劳。
   B.满坐寂然,无敢哗者。(《口技》)
   译:全场静悄悄的,没有人敢高声喧哗。
   C.政通人和,百废具兴。(《岳阳楼记》)
   译:政事顺利,百姓和乐,各种荒废了的事业都兴办起来了。
   D.牺牲玉帛,弗敢加也。(《曹刿论战》)
   译:损失了的玉器和丝织品,不敢以少报多。
   27.(2004·泸州)下列各句与课文完全相同的一项是( )
   A.将军百战死,壮土十年回。
   D.烟笼寒水月笼沙,夜泊秦怀进酒家。
   C.安得广厦千万家,大庇天下寒士都欢颜。
   D.受任于败军之际,奉命于危难之间。

第九讲 文言文阅读

   【考查要点】
   阅读浅易文言文,能借助注释和工具书理解基本内容。
   【知识疏理】
   1.准确理解文中字词句的含义,特别要注重课文的注解,重要的词句及一些含有深意的语句;2.字词的掌握和积累是文言文学习的关键。3.理解文章的基本内容。
   【试题特点】
   文言文阅读题从取材看以课内篇目为主,近年有向课外发展的趋势。选取的课外材料主要是比较浅显的趣味性记叙文,还有部分哲理性短文。从考查的内容看更加注重能力考查。除对字词的考查外,更多的是对材料的理解性考查,包括句意、段意、篇意以及表现手法的分析等,回答问题要理解由句到段再到篇的意义。特别是一些鼓励创新的开放性题在文言文阅读题中的出现,受到普遍欢迎。从题型看既有客观题,又有主观题。客观题多采用选择题;主观题多采用用原句回答问题、概括意思、填写等。
   【解题导引】
   例1. 岳阳楼记
   庆历四年春,滕子京谪守巴陵郡。越明年,政通人和,百废具兴。乃重修岳阳楼,增其旧制,刻唐贤令人诗赋于其上。属予作文以记之。
   予观夫巴陵胜状,在洞庭一湖。衔远山,吞长江,浩浩汤汤,横无际涯;朝晖夕阴,气象万千。此则岳阳楼之大观也。前人之述备矣。然则北通巫峡,南极潇湘,迁客骚人,多会于此,览物之情,得无异乎?
   若夫霪雨霏霏,连月不开,阴风怒号,浊浪排空;日星隐耀,山岳潜形;商旅不行,樯倾揖摧;薄暮冥冥,虎啸猿啼。登斯楼也,则有去国怀乡,忧谗畏讥,满目萧然,感极而悲者矣。
   至若春和景明,波澜不惊,上下天光,一碧万顷;沙鸥翔集,锦鳞游泳;岸芷汀兰,郁郁青青。而或长烟一空,皓月千里,浮光跃金,静影沉璧,渔歌互答,此乐何极!登斯楼也,则有心旷神怡,宠辱偕忘,把酒临风,其喜洋洋者矣。
   嗟夫!予尝求古仁人之心,或异二者之为,何哉?不以物喜,不以己悲;居庙堂之高则忧其民;处江湖之远则忧其君。是进亦忧,退亦忧。然则何时而乐耶?其必曰“先天下之忧而忧,后天下之乐而乐”呼。噫!微斯人,吾谁与归?
   时六年九月十五日。
   1.范仲淹是 (朝代)政治家、文学家,我们学过他的一首词是 。
   2.解释下列句中加点词语在文中的含义。
   ①增其旧制 ( )
   ②属予作文以记之 ( )
   ③前人之述备矣 ( )
   ④去国怀乡 ( )
   3.第三段、第四段具体表现迁客骚人的览物之情。请结合这两段内容,分别以“悲”和“喜”为首字,写一组不少于五言的对偶句。
   4.范仲淹借此文委婉地表达了对友人膝子京的劝勉,结尾作者发出“微斯人,吾谁与归”的慨叹。就全文来看,这句话有何言外之意?
   5.“政通人和,百废具兴”;“不以物喜,不以己悲”;“先天下之忧而忧,后天下之乐而乐”。这三句话因其文质兼美而脍炙人口,请选择你感受最深的一句,结合《岳阳楼记》和自己的生活谈一谈你的体会。
   上面的试题比较全面地体现了《语文课程标准》和教材对文言文阅读能力的要求,命题采用主观题型,分别从文学常识、文言实词、仿写、文章内容的理解和领悟等角度考查学生的阅读和理解运用能力。
   第1题侧重考查学生的文学常识,只要平时留心熟记,注意归类,一般都能回答:北宋(或宋朝)《渔家傲》。第2题要结合具体语境解释加点的词语。文言实词的含义是“据境择义”,要考虑文言实词的特殊用法:古今义异词(制:今作“制度”讲,古作“规模”讲,其含义为后者);“属”是通假字,通“嘱”,“嘱托、吩咐”的意思;“备”有“完备”、“准备”、“详尽”多个义项,依据语境,其义为“详尽”;“去”是个古今异义词,今义为“往、到”;古义为“离开”,该句中的“去”义为“离开”。第3题侧重考查学生的语言运用能力——仿写。读了优美的文章,自然会对文中的妙语记忆犹新,跃跃欲试。文中的对偶句所拾即是,关键是如何仿写。不仿先模仿,就是在现成对偶句的基础上加上限定的仿用词语,(如“悲阴风怒号;喜渔歌互答”。“悲景色阴晦,喜湖光晴明”。)还可以自主创作,不必苛求完美,只要大体相似即可。实在不行,就从指定的文段中抄录一组对偶句,当然,无论是仿写还是创作,其内容应与第三、四段关联,还是个对偶句即可。第4题重在分析作者在文中的情感态度。请联系作者的处境和文章的主题去分析,即可明确:作者一方面希望滕子京具有古仁人之心,志存高远;另一方面也含蓄地表达了自己愿与古仁人同道的旷达胸襟和远大抱负。第5题可任选其一即可。谈体会既要联系原文,又要结合自己的生活。如选第一句可围绕政治理想来谈,选第二句可围绕旷达胸襟来谈,选第三句可围绕远大抱负及民族责任感来谈。言之成理即可,鼓励有创见。但观点错误或偏激应酌情扣分。
   【常见失误】
   1.虽是学过的课文,但却未读懂。有的同学平时读书囫囵吞枣,不求甚解,虽然试题的阅读材料就是课堂学过的课文,但是没有真正读懂;在考试答题时,抓不住要领,或者所答非所问。泸州市试题第4小题,“促使周处认识到自身过错的原因是什么?(请用原文回答,多写漏写不得分)。”读懂原文,很好回答:“闻里人相庆”。未读懂原文的同学,其答案五花八门,在阅读文学中抄,这一句像是答案,那一句也像。有的抄了长长一段,有的把文中对周处的描写当作答案,更多的考生把“清河曰”全部内容抄上。因对原文未能把握好,答题造成失误。
   2.实词词义不落实,对一词多义不会辨别、选择。掌握一定数量的实词,是文言文阅读的基本条件。几乎各地中考文言文试题都在考查文言实词,有的特意考查一词多义。从阅卷情况来看,这类试题的得分率不高。最普遍的失误有二:一是以今天的词义去套文言词义,如“具以情告”,其中“具”有不少考生解释为“具体”;二是不会辨别一词多义,在多个词义面前不会选择。
   3.对虚词的用法不熟悉。各地试题中,考查虚词的多种用法的试题是一大难点。究其原因,一是朗读不够,缺少文言语感;二是归纳、辨析不够。也许在课堂上教师帮助总结过多次,还是不熟悉,那是因为老师总结不能代替自己的总结。要在老师指导下举一反三,自己再找例句归纳总结。
   4.对课文、句子理解不准确。失误表现在两方面:一是句子翻译不准确,对句中关键性的词语漏译、误译,对特殊句式不熟悉,不会翻译。二是对理解性的试题,答不准确。桂林市试题第4小题:“(乙)文中,从‘曹刿请见’一句可以看出曹刿有主动参与国家大事的精神,请你从(甲)文中也找一个体现子墨子这样精神的语句。”这道题并不难,但是仍有考生答不好。有的考生摘录子墨子的语言答题,似乎答对了,但不准确。
   【考点精练】
   (一)(2004·肇庆)
   见渔人,乃大惊,问所从来,具答之。便要还家,设酒杀鸡作食。村中闻有此人,咸来问讯。自云先世避秦时乱,率妻子邑人来此绝境,不复出焉,遂与外人间隔。问今是何世,乃不知有汉,无论魏晋。此人一一为具言所闻,皆叹惋。令人各复延至其家,皆出酒食。停数日,辞去。此中人语云:“不足为外人道也。”
   1.下列句中加点词意思相同的一项是:( )
   A.①率妻子邑人来此绝境 ②奇山异水,天下独绝
   B.①遂与外人间隔 ②肉食者谋之,又何间焉
   C.①此人一一为具言所闻 ②色愈恭,礼愈至,不敢出一言以复
   D.①停数日,辞去 ②鸣声上下,游人去而禽鸟乐也
   2.文中能表明桃花源中人听了渔人的话后,虽“叹惋”但仍想继续在桃花源里生活的一句话是 。
   3.用现代汉语翻译文中画横线的句子。
   问今是何世,乃不知有汉,无论魏晋。译:
   (二)(2004·西宁)
   北山愚公者,年且九十,面山而居。惩山北之塞,出入之迂也,聚室而谋曰:“吾与汝毕力平险,指通豫南,达于汉阴,可乎?”杂然相许。其妻献疑曰:“以君之力,曾不能损魁父之丘,如太行、王屋何?且焉置土石?”杂曰:“投诸渤海之尾,隐土之北。”遂率子孙荷担者三夫,叩石垦壤,箕畚运于渤海之尾。邻人京城氏之孀妻有遗男,始龇,跳往助之。寒暑易节,始一反焉。
   河曲智叟笑而止之曰:“甚矣,汝之不惠。以残年余力,曾不能毁山之一毛,其如土石何?”北山愚公长息曰:“汝心之固,固不可彻,曾不若孀妻弱子。虽我之死,有子存焉;子又生孙,孙又生子;子又有子,子又有孙,子子孙孙无穷匮也,而山不加增,何苦而不平?”河曲智叟亡以应。
   1.解释下列句子中加点词的意义。
   (1)惩山北之塞,出入之迂也。 惩:
   (2)汝心之固,固不可彻。 固:
   2.用现代汉语写出文中两处划线句子的意思,并要注意准确表现说话者的不同语气。
   吾与汝毕力平险,指通豫南,达于汉阴,可乎?
  
   子子孙孙无穷匮也,而山不加增,何苦而不平?
  
   3.文中将愚公率领子孙们决心挖山不止的壮举与智叟 的态度形成鲜明对比;并主要运用 的描写手法来表现情节内容,塑造人物形象。
   4.在你所熟知的古代寓言、神话故事中,还有哪些与《愚公移山》的意义相同?请写出故事的题目,不少于2个。
   (1)
   (2)
   5.读完这则故事后,有人认为愚公不愚,智叟不智;但也有人认为愚公确实很愚,而智叟才是有头脑的人。对此,你是如何看待的?请写出你的真实想法。要求观点明确,言之成理。
   (三)(2004·宿迁)
   五柳先生传
   陶渊明
   先生不知何许人也,亦不详其姓字。宅边有五柳树,因以为号焉。闲静少言,不慕荣利。好读书,不求甚解。每有会意,便欣然忘食。性嗜酒,家贫,不能常得。亲旧知其如此,或置酒而招之。造饮辄尽,期在必醉。既醉而退,曾不吝情去留。环堵萧然,不蔽风日。短褐穿结,箪瓢屡空,晏如也。常著文章自娱,颇示己志。忘怀得失,以此自终。
   赞曰:黔娄之妻有言:“不戚戚于贫贱,不汲汲于富贵。”极其言,兹若人之俦乎?衔觞赋诗,以乐其志,无怀氏之民欤?葛天氏之民欤?
   1.下列各组中,加点的词意思相同的两组是( )( )
   A. 先生不知何许人也
   高可二黍许(《核舟记》)
   B.好读书,不求甚解
   辙生好为文(《上枢密韩太尉书》)
   C.便欣然忘食
   主人日再食(《送东阳马生序》)
   D.因以为号焉
   以君之力, 曾不能损魁父之丘(《愚公移山》)
   E.忘怀得失,以此自终
   宠辱偕忘(《岳阳搂记》)
   2.把下列文言句子翻译成现代汉语。
   ①造饮辄尽,期在必醉。
   译文:
   ②不戚戚于贫贱,不汲汲于富贵。
   译文:
   3.文章结尾画线的句子表达了作者什么愿望?
   答:
  
   4.请结合自己的读书习惯,谈谈你是否赞成五柳先生的“好读书,不求甚解。”
   答:
   (四)(2004·玉林)
   三峡
   自三峡七百里中,两岸连山,略无阙处;重岩叠嶂,隐天蔽日:自非亭午夜分,不见曦月。
   至于夏水襄陵,沿溯阻绝,或王命急宣,有时朝发白帝,暮到江陵,其间千二百里,虽乘奔御风不以疾也。
   春冬之时,则素湍绿潭,回青倒影。绝巘多生怪柏,悬泉瀑布,飞漱其间。清荣峻茂,良多趣味。
   每至晴初霜旦,林寒涧肃,常有高猿长啸,属引凄异,空谷传响,哀转久绝。故渔者歌曰:巴东三峡巫峡长,猿鸣三声泪沾裳!
   1.下列句中加点的词语,意思相同的一项是( )
   A.虽乘奔御风不以疾也 寡人无疾 B.常有高猿长啸,属引凄异 属予作文以记之
   C.哀转久绝 忽然抚尺一下,群响毕绝  D.良多趣味 此皆良实,志虑忠纯
   2.对下列各组加点词的解释,有误的一项是( )
   A.春冬之时(的)         吾欲之南海(往,到)
   B.自非亭午夜分不见曦月(如果)  自李唐来,世人甚爱牡丹(自从)
   C.其间千二百里(其中)      其真无马邪(这里)
   D.故渔歌曰(所以)         桓侯故使人间之(特意)
   3.请把“两岸连山,略无阙处”译成现代汉语。
   答:
   4.“朝辞白帝彩云间,千里江陵一日还。两岸猿声啼不住,轻舟己过万重山。”这是诗人李白在被流放途中遇赦写下的《朝发白帝城》。请说说其中后两句诗与文中“巴东三峡巫峡长,猿鸣三声泪沾裳”之间有什么相同与不同之处。
   答:
   (五)(2004·鄂州)
   恶少年愠其诞,目间其夜归,分五六人栖道旁木上,相去各里所。候巫过,下砂石击之。巫以为真鬼也,即旋其角,且角且走。心大骇,首岑岑加重,行不知足所在。稍前,骇颇定,木间砂乱下如初。又旋而角,角不能成音,走愈急。复至前,复如初。手栗气慑,不能角,角坠;振其铃,既而铃,惟大叫以行。行闻履声及叶鸣谷响,亦皆以为鬼。号求救于人甚哀。
   1.用“/”正确标出后面句子朗读时的停顿:号求救于人甚哀。
   2.译出下面两个句子在文中的意思。
   ①相去各里所。
  
   ②首岑岑加重,行不知足所在。
  
   3.试分析越巫夜归遇鬼后的心理变化过程,结合选文说说越巫是什么样的人?
   答:

(六)(2004·安徽)
   送董邵南游河北序
   韩愈
   燕赵古称多感慨悲歌之士。董生举进士,连不得志于有司,怀抱利器,郁郁适兹土。吾知其必有合也。董生勉乎哉!
   夫以子之不遇时,苟慕义强仁者皆爱惜焉,矧燕赵之士出乎其性者哉!然吾尝闻风俗与化移易,吾恶知其今不异于古所云邪?聊以吾子之行卜之也。董生勉乎哉。
   吾因子有所感矣。为我吊望诸君之墓,而观于其市,复有昔时屠狗者乎?为我谢曰:“明天子在上,可以出而仕矣。”
   【注】望诸君:即战国时燕国名将乐毅。他忠心耿耿,战功卓著,后被人离间,受到燕王怀疑,被迫离燕至赵,赵封他为望诸君。
   1.解释下列加点词在文中的意思。
   (1)然吾尝闻风俗与化移易    易:____
   (2)明天子在上,可以出而仕矣  仕:____
   2.标示句子的诵读节奏,选择词的恰当义项。
   (1)用“/”标示下句的诵读节奏。
   苟 慕 义 强 仁 者 皆 爱 惜 焉
   (2)下面方框中文字出自《古汉语常用字字典》,解释“皆爱惜”中的“惜”,应选择第__义项。(只能序号)
   惜Xī①爱惜。《韩非子·难二》:“~草茅者耗禾穗,惠盗贼者伤良民。”(耗:减损。惠:给人好处。)②吝惜,舍不得。蔡琰《悲愤诗》:“岂敢~性命。”③痛惜,哀伤。贾谊《惜誓》:“~余年老而日衰兮。”
   3.“郁郁适兹土”的“适”解释为“往,到”。下列句中加点的词不能解释为“往,到”的一项是( )
   A、由是先王遂诣亮   B、去国怀乡,忧谗畏讥
   C、辍耕之垄上     D、造饮辄尽,期在必醉
   4.翻译下面的句子。
   聊以吾子之行卜之也。
   答:
   5.根据文意,联系“望诸君”的注释,谈谈你对韩愈委托董邵南“吊望诸君之墓”的认识。
   答:
   (七)(2004·徐州)
   二世元年七月,发闾左適戍渔阳九百人,屯大泽乡。陈胜、吴广皆次当行,为屯长。会天大雨,道不通,度已失期。失期,法皆斩,陈胜、吴广乃谋曰:“今亡亦死,举大计亦死,等死,死国可乎?”陈胜曰:“天下苦秦久矣。吾闻二世少子也,不当立,当立者乃公子扶苏。扶苏以数谏故,上使外将兵。今或闻无罪,二世杀人。百姓多闻其贤,未知其死也。项燕为楚将,数有功,爱士卒,楚人怜之。或以为死,或以为亡。今诚以吾众诈自称公子扶苏、项燕,为天下唱,宜多应者。”吴广以为然。
   1.解释下列句中加点的词语。
   ①会天大雨( ) ②度已失期( )
   ③上使外将兵( ) ④楚人怜之( )
   2.与“陈胜、吴广乃谋曰”一句中的“乃”用法相同的一项是( )
   A.当立者乃公子扶苏 B.问今是何世,乃不知有汉
   C.乃重修岳阳楼,增其旧制 D.羲之之书晚乃善
   3.用现代汉语翻译下面的两个句子。
   ①天下苦秦久矣。
   答:
   ②今诚以吾众诈自称公子扶苏、项燕,为天下唱,宜多应者。
   答:
   4.从选文中我们可以看出陈胜是一位 的人。
   (八)(2004·绍兴)
   马之千里者,一食或尽粟一石。食马者不知其能千里而食也。是马也,虽有千里之能,食不饱,力不足,才美不外见,且欲与常马等不可得,安求其能千里也?
   策之不以其道,食之不能尽其材,鸣之而不能通其意,执策而临之,曰:“天下无马!”呜呼! 其真无马耶?其真不知马也。
   (节选自韩愈《马说》)
   1.解释文中加点的词。
   或: 食: 等:
   2.用现代汉语翻译文中加横线的句子。
   安求其能千里也?
   3.根据选文,说说千里马被辱没的原因。
  
  
   (九)(2004·福州)
   马说
   世有伯乐,然后有千里马。千里马常有,而伯乐不常有。故虽有名马,祗辱于奴隶人之手,骈死于槽枥之间,不以千里称也。
   马之千里者,一食或尽粟一石。食马者不知其能千里而食也。是马也,虽有千里之能,食不饱,力不足,才美不外见,且欲与常马等不可得,安求其能千里也?
   策之不以其道,食之不能尽其材,鸣之而不能通其意,执策而临之,曰:“天下无马!”呜呼!其真无马邪?其真不知马也。
   1.用“/”标出下面句子朗读的语意停顿,每句标一处。
   ①安求其能千里也 ②食之不能尽其材
   2.解释下面加点词的意思。
   ①是马也( ) ②食马者( )
   ③策之不以其道( ) ④一食或尽粟一石( )
   3.下面句中加点词的意思或用法相同的一组是( )
   A.虽有千里之能 B.其真无马邪
   安求其能千里也 策之不以其道
   C.鸣之而不能通其意 D.祗辱于奴隶人之手
   环而攻之而不胜 学而时习之
   4.本文托物寓意,文中“千里马”喻指人才,“伯乐”喻指 ,而将愚妄浅薄的封建统治者比作“ ”。
   5.有这样一个故事:一匹骨瘦如柴的老马拉着盐车上山坡,气直喘,汗直流,竭尽全力还是拉不上去。赶车的人吆喝着,用鞭子狠狠地抽打它……这时,一个路过的相马人看见了,心疼得流下了眼泪,急忙脱下衣裳披在瘫倒在地的老马身上。老马睁开眼,看到相马人,眼睛一亮,长嘶一声而逝。
   请你展开想像,写出老马临死前想对相马人说的话。
   答:
   (十)(2004·黄冈)
   唐睢不辱使命(节选)
   秦王怫然怒,谓唐睢曰:“公亦尝闻天子之怒乎?”唐睢对曰:“臣未尝闻也。”秦王曰:“天子之怒,伏尸百万,流血千里。”唐睢曰:“大王尝闻布衣之怒乎?”秦王曰:“布衣之怒,亦免冠徒跣,以头抢地耳。”唐睢曰:“此庸夫之怒也,非士之怒也。夫专诸之刺王僚也,彗星袭月;聂政之刺韩傀也,白虹贯日;要离之刺庆忌也,仓鹰击于殿上。此三子者,皆布衣之士也,怀怒未发,休祲降于天,与臣而将四矣。若士必怒,伏尸二人,流血五步,天下缟素,今日是也。”挺剑而起。
   秦王色挠,长跪而谢之曰:“先生坐!何至于此!寡人谕矣:夫韩、魏灭亡,而安陵以五十里之地存者,徒以有先生也。”
   1.解释下列加点词。
   ①仓鹰击于殿上( ) ②长跪而谢之( )
   2.翻译下面文言语句。
   寡人谕矣:夫韩、魏灭亡,而安陵以五十里之地存者,徒以有先生也。
   译文:
   3.外交辞令往往委婉含蓄,隐藏着“潜台词”,听话者应仔细揣摩,才能灵活应付。请揣摩下列各句,说说其“潜台词”是什么。
   ①公亦尝闻天子之怒乎?
   ②与臣而将四矣。
   4.用自己的话简要回答,唐睢是怎样让秦王理屈词穷的?从中可以看出唐睢是个怎样的人?
   答:
   5.列举我国历史上两个“不辱使命”的外交人才,分别用一句话概括他们的主要事迹。
   答:
   (十一)(2004·仙桃)
   风烟俱净,天山共色。从流飘荡,任意东西。自富阳至桐庐,一百许里,奇山异水,天下独绝。
   水皆缥碧,千丈见底。游鱼细石,直视无碍。急湍甚箭,猛浪若奔。
   夹岸高山,皆生寒树。负势竞上,互相轩邈;争高直指,千百成峰。泉水激石,泠泠作响;好鸟相鸣,嘤嘤成韵。蝉则千转不穷,猿则百叫无绝。鸢飞戾天者,望峰息心;经纶世务者,窥谷忘反。横柯上蔽,在昼犹昏;疏条交映,有时见日。
   (吴均《与朱元思书》)
   1.解释下列句中加点词的意思。
   ①负势竞上( ) ②好鸟相鸣( )
   2.下列各句中的“者”与例句中“者”的意义和用法相同的一项是( )
   例句:经纶世务者,窥谷忘反。
   A.苟慕义强仁者皆爱惜焉。(《送董邵南游河北序》)
   B.是故所欲有甚于生者,所恶有甚于死者。(《鱼我所欲也》)
   C.以为凡是州之山有异态者,皆我有也。(《始得西山宴游记》)
   D.入则无法家拂士,出则无敌国外患者,国恒亡。(《生于忧患,死于安乐》)
   3.写出文中概括富春山全貌的句子并翻译。
   句子:
   翻译:
   4.文章理解。
   文章在总写中,用“从流飘荡,任意东西”表现了作者 的情态。在分写“异水”时,用“ ”一句夸张地表现了江水极深、极清的特点。在分写“奇山”时,通过泉响、鸟鸣、蝉转、猿叫,反衬了山的 的特点。之后,通过“鸢飞戾天者……窥谷忘反”的抒情短论,表现了作者
   的思想。最后,文章别具匠心地描绘了山中的又一奇景,突出了树多的特点,与前文“ ”一句相照应。
   (十二)(2004·青海)
   与朱元思书
   吴均
   风烟俱净,天山共色。从流飘荡,任意东西。自富阳至桐庐,一百许里,奇山异水,天下独绝。
   水皆缥碧,千丈见底。游鱼细石,直视无碍。急湍甚箭,猛浪若奔。
   夹岸高山,皆生寒树,负势竞上,互相轩邈;争高直指,千百成峰。泉水激石,泠泠作响;好鸟相鸣,嘤嘤成韵。蝉则千转不穷,猿则百叫无绝。鸢飞戾天者,望峰息心;经纶世务者,窥谷忘反。横柯上蔽,在昼犹昏;疏条交映,有时见日。
   1.解释加点词语在文中的意思。
   风烟俱净 负势竞上
   互相轩邈 窥谷忘反
   2.下列句中加点词含义相同的一项是( )
   A.天下独绝 B.一百许里
   猿则百叫无绝 猿则百叫无绝
   C.风烟俱净 D.在昼犹昏
   皆生寒树 有时见日
   3.文中写出的富春江的特点是 、 。
   4.把原本静止的重山叠岭之状写活的句子是: 。
   5.能暗示文中所写景色季节特征的一个字是: 。从你积累的古诗词中写一个含有该字的句子: 。
   6.下列说法不正确的一项是( )
   A.本文生动简练地描写了富阳、桐庐一带富春江上优美的景色,抒发了向往自然,厌弃尘俗的心态。
   B.开头一段是总写,叙写并赞叹了从富阳至桐庐一百许里沿江两岸的奇丽山水,诱人景色。
   C.第二段前后两个层次形成了鲜明对照,通过对照,反映了江水的动静变化,补足了富春江水的特色。
   D.第三段先描写了群山的静态美,再铺写山中的各种声音,这是以静写闹,显示春天山中热闹景象。
  
   (十三)(2004·湛江)
   公输(选段)
   公输盘为楚造云梯之械,成,将以攻宋。子墨子闻之,起于齐,行十日十夜而至于郢,见公输盘。
   公输盘曰:“夫子何命焉为?”子墨子曰:“北方有侮臣者,愿借子杀之。”公输盘不说。子墨子曰:“请献十金。”公输盘曰:“吾义固不杀人。”子墨子起,再拜,曰:“请说之。吾从北方闻子为梯,将以攻宋。宋何罪之有?荆国有余于地而不足于民,杀所不足而争所有余,不可谓智。宋无罪而攻之,不可谓仁。知而不争,不可谓忠。争而不得,不可谓强。义不杀少而杀众,不可谓知类。”
   公输盘服。子墨子曰:“然,胡不已乎?”公输盘曰“不可,吾既已言之王矣。”子墨子曰:“胡不见我于王?”公输盘曰:“诺。”
   1.解释下列语句中加点词的意义。
   (1)愿借子杀之 愿: (2)公输盘不说 说:
   (3)不可谓智 智: (4)胡不见我于王 见:
   2.下列各组语句中加点的词的意义和用法,相同的一组是( )
   A.公输盘为楚造云梯之械 吾从北方闻子为梯
   B.将以攻宋 臣以王之攻宋也,为与此同类
   C.子墨子闻之 愿借子杀之
   D.子墨子起,再拜 寓逆旅主人,日再食
   3.请将选文中“夫子何命焉为?”和“吾既已言之王矣”两句译成现代汉语。
   译文:(1)____________________________(2) ______________________________
   4.选文中,(1)墨子是从哪几个方面展开推论,使公输盘折服的?(请从文中摘录出有 关的字词回答)(2)墨子的言行反映出他的什么特点?请写出两点。
   答:(1) _________________________________________________________
   (2) _________________________________________________________
   (十四)(2004·兰州)
   自余为僇人,居是州,恒惴栗。其隙也,则施施而行,漫漫而游,日与其徒上高山,入深林,穷回溪,幽泉怪石,无远不到。到则披草而坐,倾壶而醉。醉则更相枕以卧,卧而梦。意有所极,梦亦同趣。觉而起,起而归。以为凡是州之山有异态者,皆我有也,而未始知西山之怪特。
   1.本文是柳宗元 中的第一篇。
   2.解释下列句中加点的词语。
   ①自余为僇人,僇:
   ②穷回溪,穷:
   3.用现代汉语翻译下列句子。
   其隙也,则施施而行,漫漫而游……
   答:
   4.作者为什么要写到自己“恒惴栗”?
   答:
   (十五)(2004·济南)
   今年九月二十八日,因坐法华西亭,望西山,始指异之。遂命仆过湘江,缘染溪,斫榛莽,焚茅 ,穷山之高而止。攀援而登,箕踞而遨,则凡数州之土壤,皆在衽席之下。其高下之势,岈然洼然,若垤若穴,尺寸千里,攒蹙累积,莫得遁隐。萦青缭白,外与天际,四望如一。然后知是山之特立,不与培 为类。悠悠乎与颢气俱,而莫得其涯;洋洋乎与造物者游,而不知其所穷。引觞满酌,颓然就醉,不知日之入。苍然暮色,自远而至,至无所见而犹不欲归。必凝形释,与万化冥合,然后知吾向之未始游,游于是乎始。
   (柳宗元《始得西山宴游记》)
   1.解释下列句子中加点字的意思。
   ①望西山,始指异之。
   ②然后知吾向之未始游,游于是乎始。
   2.解释下面句子中加点的字,并从学过的其他文言文中找出一个含有这个字且意义、用法相同的句子,写在下面。
   然后知是山之特立
   答:
   3.阅读选文,你有什么启发或感悟?请结合你的生活体验谈一谈自己的看法。
   答:

(十六)(2004·哈尔滨)
   若夫日出而林霏开,云归而岩穴暝,晦明变化者,山间之朝暮也。野芳发而幽香,佳木秀而繁阴,风霜高洁,水落而石出者,山间之四时也。朝而往,暮而归,四时之景不同,而乐亦无穷也。
   至于负者歌于途,行者休于树,前者呼,后者应,伛偻提携,往来而不绝者,滁人游也。临溪而渔,溪深而鱼肥,酿泉为酒,泉香而酒洌,山肴野蔌,杂然而前陈者,太守宴也。宴酣之乐,非丝非竹,射者中,弈者胜,觥筹交错,起坐而喧哗者,众宾欢也。苍颜白发,颓然乎其间者,太守醉也。
   1.解释文中加点的词语。
   时_________ 陈_________
   2.把下面句子译成现代汉语。
   野芳发而幽香,佳木秀而繁阴
   答:
   3.“乐亦无穷”在选文第1段中指何而言?
   答:
   4.从选文看,太守醉并非因酒而起,而是由于_____________和__________________
   (十七)(2004·温州)
   环滁皆山也。其西南诸峰,林壑尤美,望之蔚然而深秀者,琅琊也。山行六七里,渐闻水声潺潺而泻出于两峰之间者,酿泉也。峰回路转,有亭翼然临于泉上者,醉翁亭也。作亭者谁?山之僧智仙也。名之者谁?太守自谓也。太守与客来饮于此,饮少辄醉,而年又最高,故自号曰醉翁也。醉翁之意不在酒,在乎山水之间也。山水之乐,得之心而寓之酒也。
   若夫日出而林霏开,云归而岩穴暝,晦明变化者,山间之朝暮也。野芳发而幽香,佳木秀而繁阴,风霜高洁,水落而石出者,山间之四时也。朝而往,暮而归,四时之景不同,而乐亦无穷也。
   1.“太守自谓也”句中的“太守”指的是谁?请写出人名: 。
   2.下面各组句子中加点词的意思完全相同的一组是( )
   A. 山之僧智仙也 吾欲之南海,何如
   B.名之者谁 人有百口,口有百舌,不能名其一处
   C.朝而往,暮而归 朝晖夕阴,气象万千
   D.晦明变化者 恐托付不效,以伤先帝之明
   3.第二语段中描写傍晚之景的句子是: ;描写秋天之景的句子是:
   。
   4.用现代汉语翻译下面两个句子。
   ①有亭翼然临于泉上者。
   译文:
   ②山水之乐,得之心而寓之酒也。
   译文:
   5.“醉翁之意不在酒”现在常用来表示什么?
   答:
   (十八)(2004·海淀)
   予观夫巴陵胜状,在洞庭一湖。衔远山,吞长江,浩浩荡荡,横无际涯;朝晖夕阴,气象万千。此则岳阳楼之大观也。前人之述备矣。然则北通巫峡,南极潇湘,迁客骚人,多会于此,览物之情,得无异乎?
   若夫霪雨霏霏,连日不开,阴风怒号,浊浪排空;日星隐耀,山岳潜形;商旅不行,樯倾楫摧;薄暮冥冥,虎啸猿啼。登斯楼也,则有去国怀乡,忧谗畏讥,满目萧然,感极而悲者矣。
   至若春和景明,波澜不惊,上下天光,一碧万顷;沙鸥翔集,锦鳞游泳;岸芷汀兰,郁郁青青。而或长烟一空,皓月千里,浮光跃金,静影沉璧,渔歌互答,此乐何极!登斯楼也,则有心旷神怡,宠辱偕忘,把酒临风,其喜洋洋者矣。
   1.“去”在古汉语中有多种意思,请你为下列语句中加点的“去”选择正确的义项,将所选字母填涂在答题卡上。
   去:A.距离 B.离开 C.除掉,去掉
   ①去国怀乡 ②去粗取精,去伪存真
   ③委而去之 ④西蜀之去南海,不知几千里也
   2.用现代汉语说出下面语句的意思。
   浮光跃金,静影沉璧
   答:
   3.唐代诗人孟浩然的诗句“气蒸云梦泽,波撼岳阳城”,与选文第一段所描写的洞庭湖的景色、气势相近,都突出了洞庭湖 的特点。
   4.选文第二段描绘的画面特点是 ,第三段描绘的画面特点是
   ;如果请你朗读第二、三两段文字,应注意分别读出 和 的语气语调。
   5.范仲淹在《岳阳楼记》中写这三段文字是为了借题发挥引出议论,阐明自己“先天下之忧而忧,后天下之乐而乐”的政治抱负;与他同时代的欧阳修则在《醉翁亭记》中以“乐”字贯穿全篇,抒写情怀。请将下面两个句子补充完整,要求补写后的句子能分别体现这两篇文章的中心。(每句填7个字)
   ①范仲淹□□□□□□□。
   ②欧阳修□□□□□□□。
   (十九)(2004·烟台)
   得道多助,失道寡助
   ①天时不如地利,地利不如人和。
   ②三里之城,七里之郭,环而攻之而不胜。夫环而攻之,必有得天时者矣,然而不胜者,是天时不如地利也。
   ③城非不高也,池非不深也,兵革非不坚利也,米粟非不多也,委而去之,是地利不如人和也。
   ④故曰,域民不以封疆之界,固国不以山溪之险,威天下不以兵革之利,得道者多助,失道者寡助。寡助之至,亲戚畔之;多助之至,天下顺之。以天下之所顺,攻亲戚之所畔,故君子有不战,战必胜矣。
   1.解释下列句中加点词语。
   ①环而攻之而不胜 ( )
   ②然而不胜者,是天时不如地利也 ( )
   ③池非不深也 ( )
   ④委而去之 ( )
   2.翻译下面的句子。
   ①威天下不以兵革之利                        
   ②寡助之至,亲戚畔之                        
   3.“得道者多助”中的“道”指的是:( )
   A.正确的道理     B.统治的方法
   C.“地利”与“人和”  D.施行仁政的治国之道
   4.本文的中心论点是                    。
   5.文段④从正反两个方面论证了      的重要性,推出了
                    的结论。
   (二十)(2004·贵阳)
   故天将降大任于是人也,必先苦其心志,劳其筋骨,饿其体肤,空乏其身,行拂乱其所为,所以动心忍性,曾益其所不能。
   人恒过,然后能改;困于心,衡于虑,而后作;征于色,发于声,而后喻。入则无法家拂士,出则无故国外患者,国恒亡。
   然后知生于忧患,而死于安乐也。
   1.解释语段中加点的词。
   ①故 ②是
   2.翻译“然后知生于忧患,而死于安乐也”这个句子。
  
   3.语段主要论述了凡是能担天下大任的人才,都必须经受一定的磨炼,这样才能取得“曾益其所不能”的效果或成就。请紧扣这一论述,任意举一个恰当的事例作论据。(所举事例必须包括人名、所受的磨炼、取得的成就)
  
   4.语段说:“人恒过,然后能改”,有人对这句话持不同见解。请阅读下面链接材料后,结合自己的思考谈一点你的看法。
   答:
   (二十一)(2004·恩施)
   陈太丘与友期
   刘义庆
   陈太丘与友期行,期日中。过中不至,太丘舍去,去后乃至。元方时年七岁,门外戏。客问元方:“君在不?”答曰:“待君久不至,已去。”友便怒曰:“非人哉!与人期行,相委而去。”元方曰:“君与家君期日中,日中不至,则是无言;对子骂父,则是无礼。”友人惭,下车引之。元方入门不顾。
   (选自初中语文第五册第29课)
   1.这则故事写了陈太丘、友人和陈元方三个人的活动,重点写的是哪两个人的活动?这两个人各有什么特点?
   答:
   2.请按要求写出这则故事下述一些内容。
   答:故事的起因:
   故事的发展:
   故事的高潮:
   故事的结局:
   3.陈元方的言行有对有错,请分别指出于下:
   对:①
   ②
   错:
   4.故事中友人听了陈元方的话后感到非常惭愧,可是陈元方仍然“入门不顾”,假如是你,你会怎么做?
   答:
   (二十二)(2004·呼和浩特)
   鱼,我所欲也,熊掌,亦我所欲也;二者不可得兼,舍鱼而取熊掌者也。生,亦我所欲也,义,亦我所欲也;二者不可得兼,舍生而取义者也。生亦我所欲,所欲有甚于生者,故不为苟得也;死亦我所恶,所恶有甚于死者,故患有所不辟也。如使人之所欲莫甚于生,则凡可以得生者何不用也?使人之所恶莫甚于死者,则凡可以辟患者何不为也?由是则生而有不用也,由是则可以辟患而有不为也。是故所欲有甚于生者,所恶有甚于死者。非独贤者有是心也,人皆有之,贤者能勿丧耳。
   一箪食,一豆羹,得之则生,弗得则死。呼尔而与之,行道之人弗受;蹴尔而与之,乞人不屑也。……
   1.下列各组句子中加点词的意思或用法相同的一项是( )
   A.千里马常有而伯乐不常有 B.呼尔而与之
   由是则生而有不用也 公与之乘
   C.所欲有甚于生者 D.所欲有甚于生者,故不为苟得也
   天将降大任于是人也 太尉苟以为可教而辱教之
   2.翻译下面的句子。
   ①生,亦我所欲也,义,亦我所欲也;二者不可得兼,舍生而取义者也。
   译文:
   ②非独贤者有是心也,人皆有之,贤者能勿丧耳。
   译文:
   3.在这篇文章中,孟子提出了什么观点?这一观点是怎样提出的?
   答:
   4.古今中外有许多舍生取义的英雄人物,试举出两位,并用一句话概括其事迹。
   答:
   (二十三)(2004·新疆)
   鱼,我所欲也,熊掌,亦我所欲也;二者不可得兼,舍鱼而取熊掌者也。生,亦我所欲也,义,亦我所欲也;二者不可得兼,舍生而取义者也。生亦我所欲,所欲有甚于生者,故不为苟得也;死亦我所恶,所恶有甚于死者,故患有所不辟也。如使人之所欲莫甚于生,则凡可以得生者何不用也?使人之所恶莫甚于死者,则凡可以辟患者何不为也?由是则生而有不用也,由是则可以辟患而有不为也。是故所欲有甚于生者,所恶有甚于死者。非独贤者有是心也,人皆有之,贤者能勿丧耳。
   一箪食,一豆羹,得之则生,弗得则死。呼尔而与之,行道之人弗受;蹴尔而与之,乞人不屑也。万钟则不辩礼义而受之,万钟于我何加焉!为宫室之美,妻妾之奉,所识穷乏者得我与?乡为身死而不受,今为宫室之美为之;乡为身死而不受,今为妻妾之奉为之;乡为身死而不受,今为所识穷乏者得我而为之:是亦不可以已乎?此之谓失其本心。
   1.请解释文中加点的词语。
   ①舍: ②患: ③蹴: ④已:
   2.下列加点词语用法不同于其他三项的是( )
   A.得之则生 B.蹴尔而与之
   C.呼尔而与之 D.行道之人弗受
   3.用现代汉语翻译画线句子。
   ①二者不可得兼,舍生而取义者也。
   译文:
   ②呼尔而与之,行道之人弗受。
   译文:
   4.作者在文中赞扬和批评了怎样的人,请你各用一个成语来回答,并谈谈你的看法。
   答:赞扬了 的人;批评了 的人。
   看法: 。
  
   (二十四)(2004·青岛)
   鱼我所欲也  
   《孟子》
   鱼,我所欲也,熊掌,亦我所欲也;二者不可得兼,舍鱼而取熊掌者也。生,亦我所欲也,义,亦我所欲也;二者不可得兼,舍生而取义者也。生亦我所欲,所欲有甚于生者,故不为苟得也;死亦我所恶,所恶有甚于死者,故患有所不辟也.如使人之所欲莫甚于生,则凡可以得生者何不用也?使人之所恶莫甚于死者,则凡可以辟患者何不为也?由是则生而有不用也,由是则可以辟患而有不为也。是故所欲有甚于生者,所恶有甚于死者。非独贤者有是心也,人皆有之,贤者能勿丧耳。
     一箪食,一豆羹,得之则生,弗得则死。呼尔而与之,行道之人弗受;蹴尔而与之,乞人不屑也。万钟则不辩礼义而受之,万钟于我何加焉!为宫室之美,妻妾之奉,所识穷乏者得我与?乡为身死而不受,今为宫室之美为之;乡为身死而不受,今为妻妾之奉为之;乡为身死而不受,今为所识穷乏者得我而为之:是亦不可以已乎?此之谓失其本心。                  (此文选自《初中语文第六册》)
   1.解释句中加点词的含义
   ①所欲有甚于生者,故不为苟得也。 (苟得:________________)
   ②所恶有甚于死者,故患有所不辟也。(患:______________)(辟:______________)
   ③乡为身死而不受。        (乡:________________)
   ④是亦不可以已乎?        (已:________________)
   ⑤此之谓失其本心。        (本心:________________)
   2.用现代汉语翻译下列句子
   ①鱼,我所欲也,熊掌,亦我所欲也。
   译文:_________________________________________________________
   ②非独贤者有是心也,人皆有之,贤者能勿丧耳。
   译文:_________________________________________________________
   ③蹴尔而与之,乞人不屑也。
   译文:_________________________________________________________
   ④万钟则不辩礼义而受之。
   译文:_________________________________________________________
   3.简答题
   ①从全文看,本文先后运用了______________论证和______________论证的方法,有力地证明了当义和生不能两全时,应舍生取义这一观点。生活中有许多人将这个观点奉为行为的准则,请你根据对这个观点的认识,列举一个奉行这种行为准则的事例,加以简要阐述(要求:举例典型,叙议结合)。
   ______________________________________________________________________
   __________________________________________________________________________
   ②作为继孔子之后著名的儒家大师,孟子为我们留下了许多富有哲理性的名言。请根据自己的积累,试写出其中的两条(不得从本文中选取)。
   a.__________________________________________________________________________
   b.__________________________________________________________________________
(二十五)(2004·湟中)
   出师表(节选)
   ①亲贤臣,远小人,此先汉所以兴隆也;亲小人,远贤臣,此后汉所以倾颓也。先帝在时,每与臣论此事,未尝不叹息痛恨于桓、灵也。侍中、尚书、长史、参军,此悉贞良死节之臣,愿陛下亲之信之则汉室之隆可计日而待也。
   ②臣本布衣,躬耕于南阳,苟全性命于乱世,不求闻达于诸侯。先帝不以臣卑鄙,猥自枉屈,三顾臣于草庐之中,咨臣以当世之事,由是感激,遂许先帝以驱驰。后值倾覆,受任于败军之际,奉命于危难之间,尔来二十有一年矣。
   ③先帝知臣谨慎,故临崩寄臣以大事也。受命以来,夙夜忧叹,恐托付不效,以伤先帝之明,故五月渡泸,深入不毛。今南方已定,兵甲已足,当奖率三军,北定中原,庶竭驽钝,攘除奸凶,兴复汉室,还于旧都。此臣所以报先帝,而忠陛下之职分也。至于斟酌损益,进尽忠言,则攸之、祎、允之任也。
   1.利用工具书,解释下列加点词语在句子中的含义。(任选两小题)
   (1)躬耕于南阳 (2)先帝不以臣卑鄙
   (3)由是感激 (4)遂许先帝以驱驰
   2.用“/”给第①段划横线的句子断句(断两处)。
   愿 陛 下 亲 之 信 之 则 汉 室 之 隆 可 计 日 而 待 也
   3.第①段采用了怎样的论证方法来论述两汉的历史教训?并在选文中用波浪线划出相应的句子。
  
   4.第③段说“先帝知臣谨慎,故临崩寄臣以大事也”。“大事”在本段中具体指什么?用原文中的四字短语回答。
  
   5.诸葛亮受命以来怀着怎样的心情并采取了怎样的行动来报答先帝的?(用原文回答)。
   心情:                            
   行动:                            
   6.请你用白话文写出第②段划线句子的意思。
   原句:受任于败军之际,奉命于危难之间
  
   7.诸葛亮的建议在当时和现在有什么重要意义?
  
  
  
  
   (二十六)(2004·昆明)
   且夫人之学也,不志其大,虽多而何为?辙之来也,于山见终南、嵩、华之高,于水见黄河之大且深,于人见欧阳公,而犹以为未见太尉也。故愿得观贤人之光耀,闻一言以自壮,然后可以尽天下之大观而无憾者矣。
   辙年少,未能通习史事。向之来,非有取于斗升之禄,偶然得之,非其所乐。然幸得赐归待选,使得优游数年之间,将以益治其文,且学为政。太尉苟以为可教而辱教之,又幸矣。
   ——节选自苏辙《上枢密韩太尉书》
   1.结合句意解释下列加点的词。
   A.故愿得观贤人之光濯( ) B.闻一言以自壮( )
   C.尽天下之大观而无憾者矣( ) D.向之来,非有取于斗升之禄( )
   2.请从文中摘出表明作者见解的一句:
   3.请用现代汉语翻译下面的句子:
   太尉苟以为可教而辱教之,又幸矣。
   答:
   4.读完这段文字,请联系全文说说,苏辙的这番慷慨陈词能否打动太尉,为什么?
   答:
   (二十七)(2004·重庆)
   曹刿论战
   十年春,齐师伐我。公将战,曹刿请见。其乡人曰:“肉食者谋之,又何间焉?”刿曰:“肉食者鄙,未能远谋。”乃入见。问:“何以战?”公曰:“衣食所安,弗敢专也,必以分人。”对曰:“小惠未 ,民弗从也。”公曰:“牺牲玉帛,弗敢加也,必以信。”对曰:“小信未孚,神弗福也。”公曰:“小大之狱,虽不能察,必以情。”对曰:“忠之属也。可以一战。战则请从。”
   公与之乘。战于长勺。公将鼓之。刿曰:“未可。”齐人三鼓。刿曰:“可矣。”齐师败绩。公将驰之。刿曰:“未可。”下视其辙,登轼而望之,曰:“可矣。”遂逐齐师。
   既克,公问其故。对曰:“夫战,勇气也。一鼓作气,再而衰,三而竭。彼竭我盈,故克之。夫大国,难测也,惧有伏焉。吾视其辙乱,望其旗靡,故逐之。”
   1.下列句于中加点词含义相同的一项是( )
   A.又何间焉 B.公问其故
   立有间 并驱如故
   C.肉食者鄙 D.何以战
   顾不如蜀鄙之僧哉 域民不以封疆之界
   2.用现代汉语翻译下面的句子。
   (1)小大之狱,虽不能察,必以情。
   翻译
   (2)忠之属也。可以一战。战则请从。
   翻译:
   3.曹刿“下视其辙,登轼而望之”的原因是 。(用原文回答)
   4.读了这则短文,鲁国获胜给你启示最深的一点是什么?请联系生活实际简要谈谈你的看法。(不超过30字)
   答:
   (二十八)(2004·佛山)
   余幼时即嗜学,家贫,无从致书以观,每假借于藏书之家,手自笔录,计日以还。天大寒,砚冰坚,手指不可屈伸,弗之怠。录毕,走送之,不敢稍逾约。以是人多以书假余,余因得遍观群书。既加冠,益慕圣贤之道。又患无硕师与名人与游,尝趋百里外从乡之先达执经叩问。先达德隆望尊,门人弟子填其室,未尝稍降辞色。余立侍左右,援疑执理,俯身倾耳以请;或遇其叱咄,色愈恭,礼愈至,不敢出一言以复;俟其欣悦,则又请焉。故余虽愚,卒获有所闻。当余之从师也,负箧曳屣,行深山巨谷中,穷冬烈风,大雪深数尺,足肤皲裂而不知。至舍,四支僵劲不能动,媵人持汤沃灌,以衾拥覆,久而乃和。寓逆旅主人,日再食,无鲜肥滋味之享。同舍生皆被绮绣,戴朱缨宝饰之帽,腰白玉之环,左佩刀,右备容臭,烨然若神人;余则蕴袍蔽衣处其间,略无慕艳意。以中有足乐者,不知口体之奉不若人也。盖余之勤且艰若此。
   ……
   东阳马生君则在太学已二年,流辈甚称其贤。余朝京师,生以乡人子谒余。撰长书以为贽,辞甚畅达。与之论辩,言和而色夷。自谓少时用心于学甚劳。是可谓善学者矣。其将归见其亲也,余故道为学之难以告之。
   1.解释下列加点的词。
   ⑴媵人持汤沃灌( )
   ⑵同舍生皆被绮绣( )
   ⑶以是人多以书假余( )
   2.用现代汉语翻译下面的句子。
   以中有足乐者,不知口体之奉不若人也。
   译文: 。
   3.找出表现作者幼时守信用的句子。
   答: 。
   4.作者将自己求学时的“勤且艰”告诉马生,用意是什么?
   答: 。
   (二十九)(2004·兰州)
   亮答曰:“自董卓已来,豪杰并起,跨州连郡者不可胜数。曹操比于袁绍,则名微而众寡,然操遂能克绍,以弱为强者,非惟天时,抑亦人谋也。今操已拥百万之众,挟天子而令诸侯,此诚不可与争锋。权据有江东,已历三世,国险而民附,贤能为之用,此可以为援而不可图也。荆州北据汉、沔,利尽南海,东连吴会,西通巴、蜀,此用武之国,而其主不能守,此殆天所以资将军,将军岂有意乎?益州险塞,沃野千里,天府之土,高祖因之以成帝业。刘璋暗弱,张鲁在北,民殷国富而不知存恤。智能之士思得明君。将军既帝室之胄,信义著于四海,总揽英雄,思贤如渴,若跨有荆、益,保其岩阻,西和诸戎,南抚夷越,外结好孙权,内修政理;天下有变,则命一上将将荆州之军以向苑、洛,将军身率益州之众出于秦川,百姓孰敢不箪食壶浆以迎将军者乎?诚如是,则霸业可成,汉室可兴矣。”
   1.用“/”给下面语句标出正确的停顿。
   高祖因之以成帝业。
   2.下列“以”字意义和用法不同于其他项的是( )
   A.然操遂能克绍,以弱为强者,非惟天时,抑亦人谋也。
   B.百姓孰敢不箪食壶浆以迎将军者乎?
   C.益州险塞,沃野千里,天府之士,高祖因之以成帝业。
   D.天下有变,则命一上将将荆州之军以向宛洛。
   3.在诸葛亮的对策里,对不同的对象采取的对策不同。说说对下列对象所采取的对策分别是什么?(用原文回答)
   ①曹操:
   ②诸戎、夷越:
   (三十)(2004·郫县)
   (邹忌)于是入朝见威王,曰:“臣诚知不如徐公美。臣之妻私臣,臣之妾畏臣,臣之客欲有求于臣,皆以美于徐公。今齐地方千里,百二十城,宫妇左右莫不私王,朝廷之臣莫不畏王,四境之内莫不有求于王:由此观之,王之蔽甚矣。”
   王曰:“善。”乃下令:“群臣吏民能面刺寡人之过者,受之赏;上书谏寡人者,受中赏;能谤讥于市朝,闻寡人之耳者,受下赏。”令初下,群臣进谏,门庭若市;数月之后,时时而间进;期年之后,虽欲言,无可进者。燕、赵、韩、魏闻之,皆朝于齐。此所谓战胜于朝廷。
   1.朗读下面的文言句子,语气停顿有误的一项是( )
   A.臣/诚知/不如徐公美
   B.臣之妻/私臣,臣之妾/畏臣
   C.今齐地方/千里,百二十/城
   D.上书/谏寡人者,受/中赏
   2.下面语句中加点词的意思或用法相同的一项是( )
   A.臣诚知不如徐公美 B.世之妻私臣
   此诚危急存亡之秋也 不宜偏私,使内外异法也
   C.朝廷之臣莫不畏王 D.时时而间进
   辍耕之垄上 肉食者谋之,又何间焉
   3.解释下面句中加点的词。
   ①皆以美于徐公 以:
   ②群臣吏民能面刺寡人之过者 刺:
   ③期年之后期 年:
   ④皆朝于齐 朝:
   4.把下面的文言句子翻译成现代汉语。
   ①能谤讥于市朝,闻寡人之耳者,受下赏。
   答:
   ②此所谓战胜于朝廷。
   答:
   5.文中说“王曰:‘善。’”这里的“善”表明了齐威王什么态度?
   答:
   6.从文中看,齐威王最终能使齐国“战胜于朝廷”,达到“大治”的原因是什么?这给我们带来了什么启示?请结合你的生活体验,简要谈谈你的看法。
   答:
   (三十一)(2004·上虞)
   永州之野产异蛇,黑质而白章;触草木,尽死;以啮人,无御之者。然得而腊之以为饵,可以已大风,挛踠、瘘、疠,去死肌,杀三虫。其始,太医以王命聚之,岁赋其二,募有能捕之者,当其租入。永之人争奔走焉。
   余闻而愈悲。孔子曰:“苛政猛于虎也。”吾尝疑乎是,今以蒋氏观之,犹信。呜呼!孰知赋敛之毒有甚是蛇者乎?故为之说,以俟夫观人风者得焉。
   1.解释下面加点的字
   然得而腊之( )以俟夫观人风者得焉( )
   苛政猛于虎也( )
   2.能统领第一段内容的一个词语是 。第二段作者引用孔子“苛政猛于虎也”的目的是要说明 。
   3.下列各句中“去”字意思相同的一项是( )
   ⑴去死肌,杀三虫 ⑵登斯楼也,则有去国还乡
   ⑶西蜀之去南海,不知几千里也 ⑷挈妻子而去之走
   A ⑵⑶ B ⑴⑵ C ⑶⑷ D ⑵⑷
   4.翻译句子
   孰知赋敛之毒有甚是蛇者乎?
   答:

(三十二)(2004·大连)
   陈元方年十一时,候①袁公。袁公问曰贤家君在太丘远近称之何所履行②?元方曰:“老父在太丘,强行绥③之以德,弱者抚之以仁,恣其所安,久而益敬。”袁公曰:“孤④往者尝为邺令,正行此事。不知卿家君法孤,孤法卿父?”元方曰:“周公、孔子,异世而出,周旋动静,万里如一。周公不师⑤孔子,孔子亦不师周公。”
   注释:①候,拜访,问候。②履行,实践,做。③绥,安,安抚。④孤,封建时代王侯对自己的谦称。⑤师,学习。
   1.“法”字在古汉语字典里有如下解释:①法令,制度;②方法,做法;③效法,仿效;④标准,法则。你认为“不知卿家君法孤,孤法卿父”中的“法”的意思是: (只写序号)。
   2.文中断句正确的一项是:( )
   A.袁公问/曰贤家君在太丘/远近称之/何所履行
   B.袁公问曰/贤家君在/太丘远近/称之何所履行
   C.袁公问/曰贤家君在/太丘远近称之/何所履行
   D.袁公问曰/贤家君在太丘/远近称之/何所履行
   3.用现代汉语翻译画线的句子。
   译句:
   4.下面两题,任选一题回答。
   ①“正行此事”中的“此事”指的是什么?
   答:
   ②你认为元方所说的“周公不师孔子,孔子亦不师周公”的言外之意是什么?
   答:
   (三十三)(2004·宾州)
   (甲)临川之城东,有地隐然而高,以临于溪,曰新城。新城之上,有池洼然而方以长,曰王羲之之墨池者,荀伯子《临川记》云也。羲之常慕张芝,临池学书,池水尽黑,此为其故迹,岂信然邪?方羲之之不可强以仕,而尝极东方,出沧海,以娱其意于山水之间,岂有徜徉肆恣,而又尝自休于此邪?羲之之书晚乃善,则其所能,盖亦以精力自致者,非天成也。然后世未有能及者,岂其学不如彼邪?则学固岂可以少哉!况欲深造道德者也?
   (乙)孙莘老识文忠公,尝乘间从,以文字问之,云:无他术①,惟勤读书而多为之,自工。世人患作文字少,又懒读书,每一篇出,即求过人,如此少有至者。疵病不必待人指摘,多作自能见之。此公以其尝试者告人,故尤有味。苕溪渔隐②曰:旧说梅圣俞③日课④一诗,寒暑未尝易也,圣俞诗名满世,盖身试此说之效⑤耳。
   注释:①术:方法。 ② 苕溪渔隐:即胡仔,宋代作家。 ③梅圣俞:即梅尧臣,宋代著名诗人。 ④ 课:这里指按照规定的内容和分量学习。 ⑤效:应验。
   1.释下列加点的词在文中的意思。
   ①尝极东方 极:
   ②寒暑未尝易也 易:
   2.下面各组中句子中加点的词意思相同的一项是 ( )
   A.方羲之之不可强以仕
   以文字问之
   B.此公以其尝试者告人
   不以物喜,不以己悲
   C.羲之之书晚乃善
   当立者乃公子扶苏
   D.以临于溪,曰新城
   明有奇巧人曰王叔远
   3.请用自己的话说明墨池的位置和形态。
   答:
   4.用现代汉语解释下列文言句子。
   ①岂其学不如彼也?
   答:
   ②唯勤读书而多为之,自工。
   答:
   5.(甲)(乙)两文从不同的角度阐述怎样的观点?要想学有所成,除此之外,你认为还需要哪些条件?
   答:(甲)(乙)两文的观点:
   还需要的条件: (写出两点即可)
   (三十四)(2004·济宁)
   孙权劝学
   初,权谓吕蒙曰:“卿今当涂掌事,不可不学!”蒙辞以军中多务。权曰:“孤岂欲卿治经为博士耶!担当涉猎,见往事耳。卿言多务,孰若孤?孤常读书,自以为大有所益。”蒙乃始就学。及鲁肃过寻阳,与蒙论议,大惊曰:“卿今者才略,非复吴下阿蒙!”蒙曰:“士别三日,即更刮目相待,大兄何见事之晚乎!”肃遂拜蒙母,结友而别。
   诫子书
   夫君子之行①,静以修身,俭以养德,非澹泊②无以明志,非宁静无以致远。夫学须静也,才须学也,非学无以广才,非志无以成学。淫慢③则不能励精,险躁④则不能治性⑤。年与时驰,意与日去,遂成枯落,多不接世,悲守穷庐,将复何及!
   注释:①行:行为,操行。 ②澹泊:也写作“淡泊”,清心寡欲,不追求名利。 ③淫慢:过渡怠惰。淫,过度,慢,怠惰,不勤勉。 ④险躁:过分急躁。 ⑤治性:约束自己的性情。
   1.请用简洁的语言概括《孙权劝学》的内容。
   答:
   2.仔细阅读《诫子书》一文,说说诸葛亮写这封家信的用意是什么?
   答:
   3.解释下列加点的词语。
   ①孤岂欲卿治经为博士耶! 治经:
   ②士别三日,即更刮目相待。 刮目相待:
   ③静以修身,俭以养德。 俭以养德:
   ④淫慢则不能励精。 励精:
   4.翻译下列句子。
   ①卿言多务,孰若孤?孤常读书,自以为大有所益。
   答:
   ②夫学须静也,才须学也,非学无以广才,非志无以成学。
   答:
   5.仔细阅读《孙权劝学》一文,说说孙权是用什么方法劝吕蒙学习的?
   答:
   6.读了以上两篇短文后,你受到了怎样的启发,请就其中一个方面谈谈自己的认识。
   答:
   (三十五)(2004·温州)
   荀巨伯探病友
   荀巨伯远看友人疾,值①胡贼攻郡②,友人语巨伯曰:“吾今死矣,子可去。”巨伯曰:“远来相视③,子令吾去,败义以求生,岂荀巨伯所行耶?”贼既至,谓巨伯曰:“大军至,一④郡尽空,汝何男子,而敢独止⑤?”巨伯曰:“友人有疾,不忍委之,宁以我身代友人命。”贼相谓曰:“我辈无义⑥之人,而入有义之国。”遂班军而还。一郡并获全。
   (选自刘义庆《世说新语》)
   注释:①值:适逢。②郡:这里指城。③相视:看望你。④一:整个。⑤独止:一个人留下。⑥无义:不懂道义。
   1.解释下面加点的词。
   ①友人语巨伯曰( ) ②子可去( )
   ③宁以我身代友人命( ) ④遂班军而还( )
   2.下列译文不正确的一项是( )
   A.败义以求生。
   译文:毁坏情义而求得保全性命。
   B.岂荀巨伯所行耶?
   译文:难道是(我)荀巨伯所做的吗?
   C.贼既至,谓巨伯曰……
   译文:不久,胡人闯进家来,(友人)对荀巨伯说……
   D.友人有疾,不忍委之。
   译文:友人有病,不忍心抛下他。
   3.读了《荀巨伯探病友》一文之后,你有何感想,请用简要的语言加以回答。
   答:
   (三十六)(2004·上虞)
   献曲求诗
   北宋[苏轼]
   元丰五年十二月十九日东坡生日,置酒赤壁矶下,踞高峰,酒酣,笛声起于江上。客有郭、尤二生,颇知音,谓坡曰:“笛声有新意,非俗工也。”使人问之,则进士李委闻坡生日,作南曲目《鹤南飞》以献。呼之使前,则青巾紫裘腰笛而已。既奏新曲,又快作数弄,嘹然有穿云石之声,坐客皆引满醉倒。委袖出嘉纸一幅曰:“吾无求于公,得一绝句足矣。”坡笑而从之。
   [注释] 嘉纸:很好的纸张。嘉:美好的。
   1.解释文中加点的字
   (1)酣( ) (2)腰( )
   (3)既( ) (4)袖( )
   2.翻译
   (1)客有郭、尤二生,颇知音
   (2)坐客皆引满醉倒
   3.李委仰慕苏东坡的诗名,遂借东坡生日而作新曲吹笛于江上,并以此求诗。请用一个成语概括这件事。
   成语:
   (三十七)(2004·武汉)
   终不知车
   越①无车,有游者得车于晋楚之郊,辐②腐而轮败,輗折而辕毁,无所可用。然以其乡之未尝有也,舟载以归而夸诸③人。观者闻其夸而信之,以为车固若是,效而为之者相属④。他日,晋楚之人见而笑其拙,越人以为绐⑤己,不顾。及寇兵侵其境,越率敝车御之。车坏,大败,终不知其车也。
   注:①越:越国。②“辐(fú)”与下文的“輗(ní)”、“辕(yuán)”均是古代大车上的部件。③诸:相当于“之于”。④相属:—个接着一个。⑤绐:dài,欺骗。
   1.解释下列句子加点词语的意义。
   ①辐腐而轮败 败:
   ②以为车固若是 固:
   ③效而为之者相属 效:
   ④越人以为绐己,不顾 顾:
   2.翻译下列句子。
   ①舟载以归而夸诸人
   ②及寇兵侵其境
   3.读了这则故事后,你从越人身上吸取了哪些教训?
   答:
   (三十八)(2004·湛江)
   商汤见伊尹
   昔者汤(商朝建国君主)将往见伊尹(商朝初年的贤相),令彭氏之子御。彭氏之子半道而问曰:“君将何之?”汤曰:“将往见伊尹。”彭氏之子曰:“伊尹,天下之贱人也。若君欲见之,亦令召问焉,彼受赐矣!”汤曰:“非汝所知也。今有药于此,食之,则耳加聪,目加明,则吾必说而强食之。今夫伊尹之于我国也,譬之良医善药也,而子不欲我见伊尹,是子不欲吾善也!”因下彭氏之子,不使御。
   1.下列各句中加点的词意义的解释,不正确的一项是( )
   A.令彭氏之子御 御:驾车 B.伊尹,天下之贱人也 贱人:出身低贱的人
   C.彼受赐矣 赐:恩惠,恩德 D.因下彭氏之子 下:下令
   2,请将“君将何之?”一句译成现代汉语。
   译文:_____________________________________________________________
   3.这个故事中,作为君主的商汤有什么特点?试概括:
   答:_______________________________________________________________
   (三十九)(2004·陕西)
   昔有大长者①子,共诸商人入海采宝。此长者子善诵入海捉船方法,若入海水漩洑②洄流矶③激之处,当如是捉,如是正,如是住。语众人言:“入海方法,我悉知之。”众人闻已,深信其语。既至海中,未经几时,船师遇病,忽然便死。时长者子即便代处④。至洄洑漩流之中,唱言,当如是捉,如是正。船盘回旋转,不能前进至于宝所。举船商人没水而死。
   注释:①[长者]显贵有德的老人。②[漩洑(fú)]漩涡。③[矶]露出水面的大石头。④[处(chǔ)]处置,即驾船。
   1.解释下列句子中加点词语的含义。
   ①语众人言 语:
   ②不能前进至于宝所 所:
   ③举船商人没水而死 举:
   2.请用简洁的语言叙述故事的主要情节。
   答:
   3.读了这则故事,联系实际,你悟出了什么道理?
   答:
   (四十)(2004·陕西)
   今之学者,读古人书,多訾(zǐ指责)古人之失;与今人居,亦乐称人失。人因不能无失。然试易地以处,平心而度之,吾果无一失乎?吾能知人之失 ,而不能见吾之失;吾能指人之小失,而不能见吾之大失。吾求吾失且不暇,何暇论人哉?
   (选自钱大昕《潜研堂文集》)
   1.将文中画线句子翻译成现代汉语。
   然试易地以外,平心而度之,吾果无一失乎?
   答:
   2.请用简洁的语言概括出作者的观点。
   答:
   3.请梳理作者的论证思路,填写下面方框。
  
   → →
  
   4.你怎样看待“今之学者,读古人书,多訾古人之失”的行为?
   答:
   (四十一)(2004·昆明)
   小港渡者
   庚寅①冬,予自小港欲入蛟州城,命小奚②以木简③束书从。时西日沉山,晚烟萦树,望城二里许。因问渡者:“尚可得南门开否?”渡者熟视④小奚,应曰:“徐行之,尚开也;速进,则阖⑤。”予愠为戏。趋行及半,小奚扑⑥,束断书崩,啼未即起。理书就束,而前门已牡下矣⑦。
   予爽然⑧思渡者言近道。天下之以躁急自败,穷募无所归宿者,其犹是也夫!
   [注]①庚寅:1650年。②小奚:小书童。③木简:这里指木板。④熟视:仔细看。⑤阖:关闭。⑥扑:跌倒。⑦牡下:上锁。⑧爽然:若有所失的样子。
   1.下列句中加点词意思相同的两项是( )和( )。
   A.束断书崩 B.人不知而不愠 C.以木简束书从 D.予愠为戏
   2.渡者说:“徐行之,尚开也;速进,则阖。”他这样说的依据是什么?
   答:
   3.用自己的话或借用一个成语概括这个故事中所蕴含的道理:
   答:
   (四十二)(2004·黄石)
   曲突徒薪
   客有过主人者,见其灶直突,旁有积薪。客谓主人更为曲突,远徒其薪,不者且有火患,主人嘿然不应。俄而家果失火,邻里共救之,幸而得息。于是杀牛置酒谢其邻人,灼烂者在于上行,余各以功次坐,而不录言曲突者。人谓主人曰:“向使听客之言,不费牛酒,终亡火患;论功而请赏,‘曲突徒薪’无恩泽,焦面烂额为上客耳耶”主人乃寤而请之。(选自《汉书》)
   1.解释句中带点的词。
   A.曲突徙薪( )B.客谓主人更为曲突( )
   2.下列句中的“而”字与“出淤泥而不染”中的“而”字用法相同的一项是( )
   A.今论功而请赏 B.而不录言曲突者 C.执策而临之 D.溪深而鱼肥
   3.在横线上写出下面句中的通假字。
   A.余各以功次坐 B.终亡火患
   C.主人乃寤而请之 D.寡助之至,亲戚畔之
   4.翻译句子:向使听客之言,不费牛酒,终亡火患。
   答:
   5.你从这个故事中悟出了哪些道理?
   答:

第十讲 记叙文阅读
   【考查要点】
   在整体上,明确记叙的中心,理解记叙的要素、记叙的顺序,分析记叙的详略,理清记叙的线索,概括全文内容或概括事件内容或概括人物思想品质,评析人物性格,划分文章层次或指出文章结构模式,领会全文主旨,感受人物理解,理解多种表达方式的综合运用对表情达意的作用,从课文内容中体会出作者的态度、观点和语言所表达的思想感情。
   在局部或细节上,理解中心和材料的关系,理解并品析重要文段,品味人物语言,在具体语言环境中对词句的含义进行感受与品味,品味词语、句子、文段在文中的表达作用,辨析文中疑难之处,分析表达技巧或者手法,对文章艺术特色进行自由赏析,叙述自己的感受等等。
   【知识疏理】
   1.记叙的要素,叙述的方式。2.新闻报道的文体知识。3.记叙的顺序。4.记叙中的描写。5.记叙中的议论。6.记叙中的抒情。7.记叙的详略。8.文章结构、线索方面的知识。9.欣赏课文中优美、精辟的诗句。10.分析、欣赏记叙文中的人物形象。11.段落结构层次的划分,段落品析。12.一定的联想能力,一定的生活知识与生活见闻。
   【试题特点】
   基本上都是简答填写题,出题的角度比较丰富,如概括、填写、分析、诠释、解说、标记、描述、品析、联系、比较、探究、自由表达等等各个方面的考查角度都有所涉及。
   【解题导引】
   例1.(2003年天门)
   闻起来像妈妈一样
   ①小男孩泰迪曾有过一个虽不健全却很幸福的家,他和妈妈快乐地生活在一起。幼儿园在他的鉴定中这样写道:“泰迪是一个聪明可爱、很有前途的孩子。”一年级的时候,发生了一件不幸的事情——他的妈妈生了重病。泰迪每日里神思恍惚,变得对什么事都心不在焉。二年级时,残酷的死神终于夺走了泰迪的妈妈。随着妈妈的去世,泰迪的心仿佛也被带走了。那一年他留给老师的印象是:接受能力差,反应迟钝。泰迪全变了,浑身脏兮兮的,乱蓬蓬的头发,挑衅的目光。没有人愿意理他。
   ②三年级的时候,班里新来了一位史密斯小姐担任老师。和每个老师一样,史密斯小姐也没有将格外的关注放在泰迪身上,因为还有那么多孩子分散着她的精力。但是一件小事却使泰迪发生了巨变。
   ③那是三年级的圣诞节。按照习惯,每个孩于都送给史密斯小姐一件礼物,并且兴高采烈地围在她周围,等着她一件件打开。挤在这堆包装精美的礼物中,有一个用旧包装纸笨拙地包着的小盒子——这是泰迪的礼物。史密斯小姐打开一看,有一只旧的假宝石手镯,上面的装饰有的已经破裂。和它在一起的,是一瓶快用完的香水。让泰迪想不到的是,史密斯小姐高兴地戴上了手镯。并且把剩余的香水高兴地喷向空中。“多好啊!”她笑着。孩子们也快乐地笑着:“多好啊!多好啊!”
   ④聚会结束了,学生们离开了校园,偌大的教室里只剩下泰迪还站在那里,迟迟不肯离去。
   ⑤“还有什么事吗?”史密斯小姐问。
   ⑥“谢谢您收下了我的礼物。”泰迪小声说,“那手镯是我妈妈用过的,您戴上它非常漂亮;香水也是我妈妈用过的,现在,您闻起来就像妈妈一样。”乱发下,泰迪昔日桀骜不驯的眼睛里噙满了泪水。
   ⑦泰迪完全变了。
   ⑧几年后,离任的史密斯小姐收到了泰迪寄来的第一封短信:“亲爱的史密斯小姐,我以全班第一名的成绩毕业了。如果您能参加我的毕业典礼,我将非常感谢。爱您的泰迪”。
   ⑨史密斯小姐没有令泰迪失望。以后,每隔几年,史密斯小姐就会收到相似的短信。
   ⑩“亲爱的史密斯小姐:我以全班第二名的成绩考取了医学院,如您能参加我的入学典礼,我将不胜荣幸。爱您的泰迪。”
   “亲爱的史密斯小姐:我现在即将成为一名医生了,如您参加我的毕业典礼我将不胜感谢……”
   “亲爱的史密斯小姐:下周二将是我结婚的日子,如您能光临,我将……”
   “亲爱的史密斯小姐……”
   当年,当史密斯小姐真诚地向周围同学喷洒那半瓶香水时,她未必知道自己播下了怎样的爱与尊重的种子。今天,这一张张报喜的短信都像一枚枚爱的果实,散发着醉人的芳香。
   1.文章第①自然段,写到妈妈生重病后泰迪的神态,用了两个四字短语,是 、
   。段中侧面描写妈妈去世后泰迪变化的句子是 。
   2.第③自然段写史密斯小姐接受孩子们的圣诞礼物。试设想一下:当史密斯小姐一件件打开那些礼物时,泰迪会想些什么?
   3.第⑥自然段写泰迪,用了描写和肖像神态描写的方法。段中语句“乱发下,泰迪昔日桀骜不驯的眼睛里噙满了泪水”与第①段中“ ”形成前后照应,表现了泰迪的“变”。
   4.用“闻起来像妈妈一样”作为文章的题目,好在哪里?
   5.文章第⑧至⑩自然段用泰迪写给史密斯小姐的若干封短信来表明泰迪的进步与成功,印证第⑦自然段中的“变’。行文从第五封短信起省略了内容,请你发挥想象,试着补写第五封信。
   第1题考查学生对文段内容及表现手法的理解,第①段写泰迪的神态,用了“神思恍惚、心不在焉”两个四字短语,“那一年他留给老师的印象是:接受能力差,反应迟钝”侧面表现了妈妈去世后泰迪的变化。解答第2题需要结合全文内容、结合泰迪的生活遭遇来揣摩其心理活动,合乎情理即可。第3题可答“语言”:“乱蓬蓬的头发,挑衅的目光”。第4题“闻起来像妈妈一样”这个题目,好在将文中的关键语句化为题目,使中心事件鲜明突出;好文题本身饱含深情,能给人温馨的感觉;好在文题带有悬念,能吸引读者深入地进行阅读;好在文题形象生动,很好地体现了记叙类文体的特色……总之,只要言之成理即可。第5题是一个读写结合的开放性题目,既要全面理解材料,又要发挥自己的想象,虽然没有固定的答案,答题时要扣住“不断进步与成功”,在表达上应该得体、简明、符合原文的表达特点。
   解答记叙阅读题时,对词语的深层含义和句子含义的理解与品味,要注意研读上下文,从不同的角度进行揣摩,然后用简洁的语言进行表述。对词、句、段在文中的表达作用进行说明,或对某种表达方式如描写、议论和抒情的表达作用进行说明,要注意从全文构思的角度,从作者的表达意图及作者的思想感情的角度进行解答,需要结合文章内容进行个性化的表达,如进行联想想象、自由赏析和迁移创新时,要揣摩考题的意图,结合文章内容,调动生活积累,进行简洁明晰的表述。
   例2.换杭州题
  
  
   解答时首先,要认真地阅读选文。如,看材料是一般性的写人记事的记叙文,还是消息、通讯之类广义的记叙文。从表达目的的角度,看材料是以记事为主,还是以写人为主,或者是以绘景为主;从表达方式的角度,辨读材料中的表达方式。看材料是否明确地体现了记叙的要素,看材料大致表现出一种什么样的记叙的顺序,看材料中有没有表现其结构顺序的语言标志。
   其次,在熟悉阅读材料的基础之上,注意对“题目”的理解——它主要包括两个方面的内容:一是了解题型的类型及题目中含有的考点。二是揣摩题目的检测意图。然后在准确理解题目的基础之上,力求做到准确、简洁、有序、完整地答题。
   【常见失误】
   记叙文阅读常见的答题失误有:1.结构、层次划分不准确。2.不能分析文中的线索。3.对文中人物的性格特点或文中的情节不能进行准确的概括。4.不能概括文中人物心理变化的过程。5.不能辨识新闻报道的结构。6.文段内容概括有误。7.不能品析词语的深层含义和句子的含义。8.对词、句、文段的表达作用把握不准。9.对文章行文中的技巧,如对比、照应等不能进行判断。10.不能准确地、有层次地结合文章内容进行个性化的表达,如进行联想想像、自由赏析和迁移创新等。
   【考点精练】
   (一)(2004·武汉)
   品味牵挂
   牵挂,是一颗心对另一颗心的深深惦记,它可以联结亲情,联结友情,联结爱情。牵挂是一份真情,一缕相思,一种幸福。
   牵挂是一种生命形态,是所有人都要寻找都会珍爱的精神场所和心理磁场。鉴别感情深浅的最好方法是牵挂的长短。“孔雀东南飞”的美丽传说,“孟姜女哭长城”的千古绝唱,“梁山伯与祝英台”的悲欢离合,“思君如满月,夜夜减青辉”的妙句佳章,都描述着因牵挂到极点,终致面容渐消瘦,直至付出生命的故事,留给我们一份至真至诚的悲凉的美丽。
   走近人生,便走进了牵挂;拥有了牵挂,便拥有了感情的寄托。“慈母手中线,游子身上衣”,是充满亲情的牵挂;“少小离家老大回,乡音无改鬓毛衰”,是溢满乡情的牵挂;“但愿人长久,千里共婵娟”是徜徉恋情的牵挂;“遥知兄弟登高处,遍插茱萸少一人”,是牵挂的缺憾;“愿君多采撷,此物最相思”,是牵挂的寄托;“劝君更尽一杯酒,西出阳关无故人”,是牵挂的蕴含。
   牵挂,是一杯浓郁的感情琼浆,是一句依依惜别的殷殷祝福。父母对子女的牵挂,就像一片云,随着天空中的飞鸟四处飘荡,穿越千山万水,萦绕在子女心头。兄弟姐妹之间的牵挂,有如山间小溪,清澈透明,只要青山不老,它就会淙淙流淌不息,唱一路欢歌,激一路浪花。夫妻之间的牵挂却似以一首婉约的词,缠绵幽远,相思常使泪沾巾。还有朋友之间那份不含有血缘关系、不掺杂私心杂念的牵挂,常能给人以无穷的力量和勇气。
   牵挂,是人与人之间—种珍贵的情感。它没有虚伪的杂质,也没有功利的色彩。牵挂,是慷慨的给予与无私的奉献,是深深的祝福和默默的祈祷。牵挂,不是虚无缥缈的海市蜃楼,而是一种实实在在的真真切切的细节与作为。买一粒药丸,挤一点牙膏,是牵挂的表现;问一声“早上好”,道一声“晚安”是牵挂的表达;一张贺卡、一封家书、一个电话、一句留言……是牵挂的体现。
   牵挂,让我们在生活中品味酸甜苦辣,更为生活增添了一道亮丽的风景。人世间如果没有牵挂,就好比植物缺少阳光雨露;一个没有任何牵挂的人生,如行尸走肉,空有一条生命,难以焕发出生活的光彩。
   生活需要温馨,需要甜美,需要春风,需要夏雨……牵挂是灵魂絮语,是心灵对话。牵挂别人和被别人牵挂都是一种幸福,让我们每一个人都领会牵挂,品味牵挂,学会牵挂,让人生变得更加丰富多彩,让世界变得更加绚丽多姿。 (作者:李汉钢)
   1.作者认为牵挂的作用是什么?请用文中的一句话回答。
   答:
   2.结合文章的内容,理解“走进人生,便走进了牵挂”一句的含义。
   答:
   (二)(2004·黄冈)
   历练后的飞翔
   明飞龙
   在辽阔的亚马逊平原上,生活着一种叫雕鹰的雄鹰,它有“飞行之王”的称号。它的飞行时间之长、速度之快、动作之敏捷,堪称鹰中之最,被它发现的小动物,一般都难逃脱它的捕捉。
   但谁能想到那壮丽的飞翔后面却蕴含着滴血的悲壮?
   当一只幼鹰出生后,没享受几天舒服的日子,就要经受母亲近似残酷的训练,在母鹰的帮助下,幼鹰没多久就能独自飞翔,但这只是第一步,因为这种飞翔只比爬行好一点。幼鹰需要成百上千次的训练,否则,就不能获得母亲口中的食物。第二步,母鹰把幼鹰带到高处,或树边或悬崖上,然后把它们摔下去,有的幼鹰因胆怯而被母亲活活摔死。但母鹰不会因此而停止对它们的训练,母鹰深知:不经过这样的训练,孩子们就不能飞上高远的蓝天,即使能,也难以捕捉到食物进而被饿死。第三步则充满着残酷和恐怖,那些被母亲推下悬崖而能胜利飞翔的幼鹰将面临着最后的,也是最关键、最艰难的考验,因为它们那正在成长的翅膀会被母鹰残忍地折断大部分骨骼,然后再次从高处推下,有很多幼鹰就是在这时成为飞翔悲壮的祭品,但母鹰同样不会停止这“血淋淋”的训练,因为它眼中虽然有痛苦的泪水,但同时也在构筑着孩子们生命的蓝天。
   有的猎人动了恻隐之心,偷偷地把一些还没来得及被母鹰折断翅膀的幼鹰带回家里喂养。但后来猎人发现那被喂养长大的雕鹰至多飞到房屋那么高便要落下来。那两米多长的翅膀已成为累赘。
   原来,母鹰“残忍”地折断幼鹰翅膀中的大部分骨骼,是决定幼鹰未来能否在广袤的天空中自由翱翔的关键所在。雕鹰翅膀骨骼的再生能力很强,只要在被折断后仍能忍着剧痛不停地振翅飞翔,使翅膀不断地充血,不久便能痊愈,而痊愈后翅膀则似神话中的凤凰一样死后重生,将能长得更加强健有力。如果不这样,雕鹰也就失去了这仅有的一个机会,它也就永远与蓝天无缘。
   没有谁能帮助雕鹰飞翔,除了它自己。
   我们每个人都拥有自己辽阔而美丽的蓝天,也都拥有一双为蓝天作准备的翅膀,那就是激情、意志、勇气和希望,但我们的翅膀也同样常会被折断,也同样常会变得疲软无力,如果这样,我们能忍受剧痛拒绝怜悯,永不坠落地飞翔吗?
   1.雕鹰为什么被称为“飞行之王”?
   答:
   2.文中那“壮丽的飞翔”后面蕴含着的“滴血的悲壮”指的是:
   ① ② ③
   3.下列语句中加点词语各指什么?
   ①如果不这样,雕鹰也就失去了这仅有的一个机会,它也就永远与蓝天无缘。
   答:
   ②我们每个人都拥有自己辽阔而美丽的蓝天,也都拥有一双为蓝天作准备的翅膀。
   答:
   4.联系全文,请赏析文中划线的句子。
   ①它眼里虽然有痛苦的泪水,但同时也在构筑着孩子们生命的蓝天。
   答:
   ②没有谁能帮助雕鹰飞翔,除了它自己。
   答:
   5.阅读下面的三则材料,联系本文,写出你探究的结果。(答出三点即可)
   材料一 母虎抚养幼虎有三个过程。开始,它出去捕食回来,把最嫩的肉用爪子撕成碎片,喂给幼虎吃。后来,它捕食回来,自己把肉吃掉,剩下的骨头扔给幼虎啃。再后来,它捕食回来,自己把肉吃掉,把骨头扔掉,幼虎要吃,它就大吼一声,不让它吃。过几天,幼虎饿得实在受不了,就离开母亲,自己找食吃,且不再回来。
   材料二 孟子曰:“故天将降大任于是人也,必先苦其心志,劳其筋骨,饿其体肤,空乏其身,……”(《生于忧患,死于安乐》)
   材料三 曾有这样的一幅漫画:父亲送儿子上大学,衣着时髦的儿子空着手,与别人谈笑风生,而父亲却肩扛手提,佝偻着身子帮儿子排队报名。漫画题为“如此爱心”。
   ①
   ②
   ③
   (三)(2004·荆门)
   [甲]鹤分为几群,每群十几只不等,拧成一团,拼命向苍鹰扑去。在阳光下,鹤的双翅奋力搏击,银光闪烁。鹤群宛如暴风中飞转的云堆,迅速形成漩涡状,上下左右飞舞,光莹莹耀眼。
   苍鹰不知何时已湮没在旋转的鹤群中,踪影全无。
   战斗继续了好长一段时间。苍鹰虽然壮勇剽悍,可是寡不敌众,变得束手无策。最后只好放开费了很大力气抓到的鹤,从飞舞啼叫的鹤群中钻了出去,凌空疾翔,一瞬间,消失在雁回山那边了。
   ……
   这时鹤群一齐发出了悲痛的啼鸣。……队列最前方很快翻身飞出两只健壮的大鹤,立即降到看来已精疲力竭、缓缓下坠的伙伴身旁,引吭高鸣,盘旋周围。……
   于是,竟然发生了意料之外的事。来救的两只大鹤,从左右两侧紧紧靠近伤鹤,各伸出一只长长的翅膀,将伤鹤(夹、架)起来了。然后各自奋力扇动单翅,静静地把负伤的伙伴送回高空的队列。
   在这段时间里,其他鹤高声啼叫,盘旋高空,一齐等待伤鹤返回。待伤鹤归队后,才似乎放下心来,又排成圆形队,向高空飞去。(节选自《鹤群翔空》)
   [乙] 随着镰刀头羊的那声吼叫,整个斑翔群迅速分成两拨,老年斑羚为一拨,年轻斑羚为一拨。……镰刀头羊本来站在年轻斑羚那拨里,眼光在两拨斑羚间转了几个来回,悲怆地轻咩了一声,迈着沉重的步伐走到老年斑羚那一拨去了。
   ……
   山涧上空,和那道彩虹平行,又架起了一座桥,那是一座用死亡做桥墩架设起来的桥。没有拥挤,没有争夺,秩序井然,快速飞渡。我十分注意盯着那群注定要送死的老斑羚①心想②或许有个别滑头的老斑羚会从注定死亡的那拨偷偷溜到新生的那拨去③但让我(感动、震惊)的是④从头至尾没有一只老斑羚调换位置。
   它们(心甘情愿、义无返顾)用生命为下一代搭起一条生存的道路。
   绝大部分老斑羚都用高超的跳跃技艺,帮助年轻斑羚平安地飞渡到对岸的山峰。只有一头衰老的母斑羚,在和一只小斑羚空中(衔接、对接)时,大概力不从心,没能让小斑羚踩上自己的背,一老一小一起坠进深渊。
   我没想到,在面临种群灭绝的关键时刻,斑羚群竟然能想出牺牲一半挽救另一半的办法来赢得种群的生存机会。我没想到,老斑羚们会那么从容地走向死亡。(节选自《斑羚飞渡》)
   1.给加点的字注音
   深渊 剽悍 湮没 引吭高鸣
   2.根据原文选择括号内恰当的词语,划上“√”。
  
   3.给选文乙①②③④处加标点。
  
   4.选文甲和选文乙表达的主旨各有侧重。甲文侧重表现 的精神;乙文侧重表现 的精神。
   5.选文甲和选文乙中作者饱含激情地讴歌了鹤群和斑羚,但在表达感情时,写法上有所不同,选文甲通过 寄寓感情,选文乙运用 表达感情。(2分)
   6.选文乙写彩虹之桥属于 ,写老斑羚用肉体作桥墩搭建的桥属于
   ,两者结合,对突出斑羚飞渡时的形态之美起到了 作用。
   7.品析选文甲中划横线的句子,具体说说该句是怎样写得生动形象、具有美感的。
   答:
   8.阅读选文乙第⑴段,揣摩镰刀头羊的心理活动,说说当时它在想些什么?
   答:
   9.爱是一种天性,动物有,人类更有。请根据材料填写。
   课文材料 人物 表达情感
   我还很记得,是她告假回家以后的四五天,她穿着新的蓝布衫回来了,一见面,就将一包书送给我,高兴地说道:“哥儿,有画儿的‘三哼经’,我给你买来了!” 阿长和迅哥儿
   “你父女两个将去做盘缠,一面收拾行李。俺明日清早发付你两个起身,看那个店主人敢留你!”金老并女儿拜谢去了。 侠义之情
   过铁道时,他先将橘子散放在地上,自己慢慢爬下,再抱起橘子走。到这边时,我赶紧去搀他。 “我”和父亲
   杰姆还没有看到送给他的美丽礼物呢!她热切地把它托在自己的掌心上递给他。这无知无觉的贵重金属似乎闪闪地反映着她的快活和热诚的神情。 杰姆和德拉


 (四)(2004·荆州)
   蝈蝈
   窗台上挂起一只拳头大小的竹笼子。一只翠绿色的蝈蝈在笼子里不安地爬动着,两根又细又长的触须不时从竹笼的小圆孔里伸出来,可怜巴巴地摇晃几下,仿佛在呼唤、祈求着什么。
   “怪了,它怎么不肯叫呢?买的时候还叫得起劲。真怪了……一位白发老人凑近蝈蝈笼子看了半天,嘴里在自言自语。
   老人的孙子和孙女,两个不满8岁的孩子,也趴在窗台上看新鲜。
   “它不肯叫,准是怕生。”小女孩说。
   “把它关在笼子里,它生气呢!”
   小男孩说着,伸出小手去摘蝈蝈笼子。
   “小囡家,别瞎说!”老人把笼子挂到小孙子摘不到的地方,然后又说:“别着急.它一定会叫的!”
   整整一天,蝈蝈无声无息。两个孩子也差点把它忘了。
   第二天,老人从菜篮里拿出一只鲜红的尖头红辣椒,撕成细丝塞进小竹笼里说:“吃了辣椒,它就会叫的。”他很自信。两个孩子又来了兴趣,在窗台上看蝈蝈怎样慢慢把一丝丝红辣椒吃进肚子里去。
   整个白天,蝈蝈还是没有吱声,只是不再在小笼子里爬上爬下。夜深人静的时候,蝈蝈突然叫起来,那叫声又清脆又响亮,把屋里所有的人都叫醒了。
   “听见么,它叫了,多好听!”老人很有点得意。
   两个孩子睡眠蒙眬,可还是高兴得手舞足蹈,把床板蹬得咚咚直响。
   蝈蝈一叫就再也没有停下来,从早到晚,不知疲倦地叫,叫……它不停地用那清脆洪亮的声音向这一家人宣告它的存在。很快他们就习以为常了。蝈蝈的叫声仿佛成了这个家庭的一部分。
   蝈蝈的叫声毕竟太响了一点。在一个闷热得难以入睡的夜里,屋子里终于发出了怨言:
   “烦死了,真拿它没办法。”说话的是孩子的父亲。
   “爸爸,蝈蝈为什么不停地叫呢?”
   男孩问了一句,可大人们谁也不回答。于是两个孩子自问自答了。
   “它大概也热得睡不着,所以叫。”
   “不!它是在哭呢!关在笼子里多难受,它在哭呢!”
   大人们只是静静地听着两个孩子的议论,只有白发老人,用只有自己能听见的声音叹息了一声……
   早晨醒来时,听不见蝈蝈的叫声了。两个孩子趴在窗台上一看,小笼子还挂在那儿,可里面的蝈蝈不见了……
   “它咬破了笼子,逃走了。”老人看着窗外,自言自语地说。
   1.速读全文,将自己的理解填写在括号中。
   本文的主体部分,从蝈蝈的( )写到( )再写到( ),以此来展开故事情节。
   2.浏览全文,在横线上填写自己的理解。
   这篇文章以 为顺序,以 为线索来叙说故事、展开情节、表现人物。
   3.品读文中第一段,说说你抓住了哪些词语来理解内容、感受情感。
  
   4.研读文章内容,写出作者借人物之口说出的最有情感表现力的两句话。
   ①
   ②
   5.赏析文章的结尾,你认为蝈蝈是“逃走”的吗?说说理由。
  
   6.体味这个故事,用简洁的语言点示作者的写作意图。
  
   (五)(2004·襄樊)
   想讨一本书
   一个寒冷的冬日,我驱车去接方先生和他刚相识的申女士。共进午餐后,申女士接受了方先生的邀请,来到了方先生的住处——位于龙华的一幢豪华别墅。
   下车后,我们看见门柱上斜靠着一个乞丐样的人,他身上裹一件脏兮兮的棉衣,似有所求地望着我们。
   心情很好的方先生赶紧走上去问他:“你是不是想吃点什么?”
   想不到乞丐回答:“这会儿太阳很好,我吃饱喝足了,只想在您这儿晒会儿太阳。还想……”
   “还想什么?”方先生从口袋里掏出100元钱,晃动在乞丐眼前,调侃追问道,“别难为情,尽管说吧,我会满足你的。”他料定一个乞丐的要求不会特别难以应付。
   “我想……”乞丐支吾着,最后鼓起勇气说,“您千万别笑话我。您可以想像我的日子,饭是每天都能吃上的,只可惜好长时间没读书了,总想讨一本书看看,可是一直难于启齿。您能不能让我进您书屋里,随意挑一本书呢?”
   方先生一下子愣住了。一方面,他惊奇于这个乞丐非同寻常的奢求;另一方面,他羞愧自己满足不了这个乞丐这一简单的要求;更重要的是:让申女士目睹了自己的窘迫。
   乞丐也看透了方先生的尴尬,急忙说:“天底下没有书的人很多。只是,我没想到这家房屋的主人也会没有。不好意思,打扰您了。”说罢,抬脚欲走。那溢于言表的鄙薄与不屑令在场的人都很难堪。
   一直在一旁不动声色的申女士急忙走上去,将随身携带的一本文搞读物递给乞丐,和颜悦色地说:“也许,你会喜欢这本书,不妨读读吧。”
   乞丐接过,连声道谢,然后席地而坐,旁若无人地读了起来。
   送走申女士后,方先生满腹心事。不久后的一天,方先生突然决定:“阿伟,送我去书店。”
   在书店经理室,方先生将一张2万元的支票拍在办公桌上,对接待他的小姐说:“愿意与我做成一笔大生意吗?”
   小姐说:“当然愿意。不过,我们这儿没有什么大生意,只有书,您任意选吧!”
   方先生将支票扔到小姐面前,说:“我才懒得选!下午两点,我来取货。”
   小姐惊奇地盯着眼前这个财大气粗的人,不知如何应付。当方先生准备掉头而去时,她才醒过神来追问道:“你想要一些什么书呢?”
   “只要是书,只要有名气,都要!”方先生头也不回地说。
   下午两点,我们驱车来到书店。那位小姐迎了出来,把支票还给方先生,并说:“很抱歉,我们经理不想接受这笔生意。”
   方先生再一次愣住了。他咆哮道:“你们的经理呢?让他出来见我。”
   “不必了。”小姐笑笑,“我们经理看过支票,就知道您是谁了。她要我一定转告您:本书店没有一本专作摆设的废书。每一本书都是有灵魂的生命,最终总会归属于那些真正爱书的人。”
   1.本文以□为线索,通过对□□和□□两个情节的描写,深刻表达了“一个人,物质的匮乏并不可怕,可怕的是心灵的荒芜,理想的泯灭”的主题。
   2.本文的情节描写十分出色,从文章后一情节中的 等动作描写可看出方先生是一个 的人;从 等语言描写可看出方先生还是一个 的人。
   3.本文多处运用了对比手法,请找出一组并分析其作用。
   答:
   4.你怎样理解申女士“本书店没有一本专作摆设的废书。每一本书都是有灵魂的生命,最终总会归属于那些真正爱书的人”这句话?
   答:
   5.请结合自己的读书实践,谈谈你对“每一本书都是有灵魂的生命”的理解。
   答:
   6.请写两句你平时积累的有关“书”的格言警句。
   答:
   (六)(2004·十堰)
   春天
   看看日历,知道已是春天,可走在户外,觉得风还是冬天的风,冰凉刺骨。太阳依旧病恹恹的样子,起伏的山,一片片的林子,全是灰蒙蒙的颜色,铅笔画似的,哪里有一丝春天的踪迹?有一天,我却意外地从学生的课桌上,发现了第一抹春痕。
   学生在写作业,我在静悄悄地教室里巡视,蓦然看见,一个课桌的缝隙里,有一撮小草芽,用细细的白线娇娇地扎着。草芽针一样细,顶端嫩绿,往下是鹅黄,根部则嫩白。我站在那里端详了许久,心中一时有些感动。我相信,这是天地间的第一抹春色。在春寒料峭的二月里,在灰黄苍茫的天地间,发现这一抹淡到极致的春色,需要怎样的耐心和细心呀!也许只有灵秀的孩子们才能感觉得到。当孩子们采集到它时,一定十分快活,乃至大声地欢呼过。我捏起那小撮纤细的草芽看了看,又插进桌面的裂缝里,坐在位子上的男子,这第一抹春色的主人,仰脸望着我,笑了。
   这以后,稍一留心,便天天可以从学生的课桌上,感受到春意的萌动和蔓延了。桌缝里,有一二截刚刚泛青或萌出芽苞的小树枝,三五朵小野花——那么小,白的似米粒,黄的、红的,像蜡笔上削下的碎屑。想这些鲜艳的粉末,该是二月的风荡来的春天的彩尘,细心的孩子发现了,便用小手指将它们拈起来,染在了他们的课桌上。终于有一天,我看见学生的课桌上,插了一枝迎春,枝条上繁密的金色小花,如一串耀目的阳光。教室里,被映上了一层淡淡的暖意。
   ……
   有一天,我迎着学生的歌声走进教室,看见我放着教科书的课桌上,也插上几朵野花。我的课桌最破,桌面上满是裂缝,循着纵横的缝隙,长满了青草、绿叶、小花。那课桌,仿佛是从春天剪下的方方正正的一块芳草地。我打开教科书,书页里也夹了几朵指甲大小的紫色小花。我笑了,学生们也喜形于色。我没有说什么,便开始讲课。其实不必说什么,那一笑,已使师生的心沟通了,大家共守默契。这一节课,上得格外好,学生始终情绪高昂。下课后,我拿起一枝开着淡紫色花朵的葛条,嗅了嗅,对学生说:“真是春天了,连咱们的课桌也都开花了!”学生大笑,欢呼起来。这时候,一个调皮的男孩,指着一个女孩子说:“老师,她也开花了!”我一看,可不,她的小辫子上,簪了一枝粉红的野花。学生们又是一阵击掌大笑。
   在这开花的课桌间踱步,听着学生们那爽朗的笑声,我觉得,这教室该是春天的源头了。春天是从孩子们身上产生,先染了他们的课桌,然后漫出窗子,染了山川。和孩子们在一起,就是和春天在一起,我想起了一位诗人的话:
   孩子
   是春天的另一种姿势。
   1.结合语境,解释下面词和句子的意思。
   ①娇娇:
   ②太阳依旧病恹恹的样子:
   2.文中说“发现了第一抹春痕”,“发现了第一抹春痕”指的是: 。
   3.将“这些鲜艳的粉末……细心的孩子发现了,便用小手指将它们拈起来,染在了他们的课桌上。”中的“染”换成“插”好不好,为什么?
   答:
   4.文章开头部分的环境描写有什么作用?
   答:
   5.如何理解“这教室该是春天的源头了”这句话?
   答:
   (七)(2004·广东)
   糖罐的秘密
   李晓琴
   ①上高中时,学校坐落在清江边上的一个小村子里。宁静的村落三面临水,四季风景如画,如同古人笔下的世外桃源。但也极其A(piānpì)闭塞,周围疏疏落落全是民居,连买
   一根针也非要上十里外的小镇不可。
   ②这可苦了我们这群高三的可怜虫们。读书实在太耗心智了,以至于整天唯一的感觉就是饿,连睡梦中都满是各种各样令人B(chuíxián)的好吃的东西。不知是谁带了一罐糖来,是那种黄亮如金、细软如沙的黄砂糖。
   ③于是,寝室里便流行罐装的黄砂糖。十二个糖罐,恰似我们十二个女孩子,亲亲热热地排成一排。临睡前,美滋滋地喝上一杯热腾腾的糖水,月儿便甜甜地照进梦乡。
   ④唯独秦霜是不大喝糖水的。因此她那个别致的青瓷陶罐里的糖,比起我们的总是又多又满。每晚,我们一边啜着糖水,一边叽叽喳喳地品头论足,或嘀嘀咕咕地发着牢骚,或嘻嘻哈哈地相互取笑时,秦霜总是在灯下读着她那本似乎永远也读不完的小说:问她为什么不喝,她说:“坏牙齿呢!”
   ⑤后来有人跟我咬耳朵,说秦霜的糖罐根本就只是做做样子罢了。她自幼父母双亡,跟着年迈的外婆一起过活,学费都交不齐,哪还有闲钱买糖吃?她那一罐糖,吃了再没得添的,又怕人瞧不起,就胡说什么坏牙齿的鬼话!我听了之后觉得心头一紧,有说不出的悲凉。一次下课间操,口渴了,我匆匆忙忙回寝室找水喝-经过寝室门前的花坛时,不经意地向寝室的窗户一瞥,却见秦霜正狼吞虎咽地吃什么东西,不由一惊。细细看,竟是在吃糖呢!她挨次从每个糖罐里舀上一大勺,大口大口地往嘴里塞。
   ⑦我看得目瞪口呆。可不知怎地,慢慢地,所有的惊讶、愤怒、鄙夷渐渐散去,两行温热的泪却无声无息地淌下来,滴落在那暗香袭人的花丛中。我悄悄地离开了那扇窗户,贼一样地潜回教室。
   ⑧晚饭后,待一寝室人走得一个不剩,我一跃而起,飞快地闩上门,拉上窗帘,一把抱起我的糖罐,先给另外的几个逐一补上一大勺糖,然后,将剩下的通通倒进那个青瓷糖罐:又从箱子里抽出一袋糖,倒入自己的空罐儿。胆战心惊地忙完这一切,我狂跳不止的心才慢慢平静下来。
   ⑨前不久,我收到了一封寄自深圳的来信:信是这样写的——
   晓琴:
   你一定还记得那个糖罐儿吧,那是我外婆的嫁妆,据说还是宫廷里的东西。现
   在,居然有人愿出五万元买它呢!我舍不得出手,因为,你倒进去的糖,远远不止
   值这个数儿。
   那个偷糖吃的女孩儿,她其实觉察到了花丛中的那双眼睛——那双世界上最纯
   最美的眼睛。因为它的注视,那个差点成为偷儿的女孩,在后来充满苦难的岁月里,
   却再也不曾妄动过一回。
   ⑩不用说,这封信是我多年的挚友——已任深圳一家电脑公司执行总经理的秦霜寄来的。
   (选自(现代交际)2003年第11期)
   1.清根据拼音写出文中应填入的词语。
   A.piānpì ( ) B.chuíxián ( )
   2.根据语境,第④段划线句“秦霜总是在灯下读着她那本似乎永远也读不完的小说。”
   中两个需要重读的词浯是 、 。
   3.第⑦段写道:“我悄悄地离开了那扇窗户,贼一样地潜回教室。”请发挥想象,把此刻“我”的心理活动描写出来。
   我想:
   4.结合上下文,理解第⑨段中划线句“因为,你倒进去的糖,远远不止值这个数儿。”
   的含义。
   答:
   5.本文以 为线索展开情节,先后写了:喝糖、 、 、 、
   等四件事。
   6.请结合本文谈谈自己在做人方面获得的启示。
   答: 。
  
   (八)(2004·湛江)
   牵手
   ①女儿很小的时候,带她出门,总是伸一根食指让她紧紧牵住。那时的女儿多小啊,脑袋刚刚齐到我的大腿,走路深一脚浅一脚趔趔趄趄,小小的胖手攥住我的食指,不要命地抓着,那真是甩也甩不脱,割也割不掉。不知道那只小手哪来那么大的劲道,我的一根食指对她来说还是庞然大物呢。
   ②稍大的时候领她上街,牵手已经不够了,牵手之后还要用她的胳膊勾住我的小臂,结结实实的,一步不落,仿佛生怕稍不留意我就会从她的身旁逃之夭夭。我觉得小臂被她勾拉得像要脱臼,甩动不灵也妨碍走路。我会冷不防地用劲,从她胳膊里怀抱里抽出自己的手。她“嗷”地一声扑上来,仍然是不屈不挠地抓住,而且比刚才更加用劲。后来我就怕带她上街,或者喝令她去牵爸爸的手。她牵爸爸的手也是一样全心全意,她爸爸让她牵着也会一脸陶醉和幸福。
   ③女儿现在已经十三四岁了。十三四岁的女儿人高马大,我们俩并排走路,我不穿高跟鞋比她矮一个头尖。人高马大的女儿出门依旧牵着我的手,但不是攥住我一根食指了,是把我的食指中指无名小指捏成一排,而后囊括进她的掌心。我说不行,你太大了,你看街上有没有这么大的女孩子还牵妈妈手的?她“嗯”一声说,我想牵。我试图甩脱她,一次两次,三次四次,但她固执地不让我的手滑开。她的手劲多大啊!我的指骨在她掌心里酥麻酥麻,只要她再加一把劲我就会叫唤出声。可我内心也喜欢这种指骨酥麻的感觉。
   ④我真不知道女儿牵我的手要牵到什么时候,今生今世我们的手还会不会分开。我嘴里说着:不要,不要。我心里默念着:牵吧,牵吧。我的孩子!妈妈牵女儿的手天经地义;女儿牵妈妈的手地久天长。女儿把她的手交到我手里,她就把她的一切都交给我了:在她满十八岁之前,她无忧无虑,因为忧虑有妈妈担着,荆棘由妈妈去砍,给她一把毒药她会不眨眼地当糖吞下,领她走上悬崖她会一步不落紧紧跟着。谁让我是妈妈呢?她的手不交给妈妈还能够交给谁呢?妈妈生来就是要牵着孩子走路的。
   ⑤可是妈妈老了之后,你还能这样紧紧牵住妈妈的手吗,我的孩子?跟现在你把一切交给妈妈一样,那时候妈妈也该把一切都交给你了。从前你交给妈妈的是花朵儿一样的身体,诗一般梦一样的年华;以后妈妈却要回赠你一段枯萎皱缩的躯干,一个琐碎不堪的灵魂。到那时候,你会牵我的这只手吗,我的孩子,我的女儿?
   1.本文题目“牵手”有哪些方面的含义,请结合全文简要说说你的理解。
   __________________________________________________________________________
   2.文中有三次女儿牵着妈妈手的细节描写,请简要概括写出来并说说这几处细节描写突出了什么(或“有什么作用”)。
   答:____________________________________________________________________
   3.请结合上下文,简要说说你对下面两个句子的理解。
   (1)妈妈牵女儿的手天经地义,女儿牵妈妈的手地久天长。
   答:_____________________________________________________________
   (2)给她一把毒药她会不眨眼地当糖吞下,领她走上悬崖她会一步不落紧紧跟着。
   答:______________________________________________________________
   4.试概括说说文章最后两段表达了作者怎样的感情。
   答:_____________________________________________________________
   5.请写出文中与“我真不知道女儿牵我的手要牵到什么时候,今生今世我们的手还会不会分开”这句话相照应的一个句子。
   答:________________________________________________________________

 (九)(2004·佛山)
   遍地黄金
   ①人蜷缩在湿漉漉的雨季,整个儿像一颗受潮的糖,沮丧而又无奈。
   ②前日忽然放晴,心,也就跟着晴了。从南窗望去,那边楼缝的坡地上一如往年,绽出一抹黄色。还刚刚惊蛰呢,油菜花这么快就开了?中午儿子放学回家,进门就说,路边的菜花约好了似的,一夜工夫全黄了!下午乘车去市郊,果见满畈满坡一片片明艳的黄,一种生命的喜悦油然灌注全身。
   ③几乎已成条件反射,人一见到油菜花,忧郁的心境就豁亮了。油菜花,我从小年年见,但着意关注它,是在病重以后。四年前腿骨发生病变,每周需去武汉两次,接受希望渺茫的康复治疗。也是雨季,天色像心境一样阴晦,人默坐车上,打不起丁点精神,对人生已失去信心,甚至对生命也没有更多的留恋。就是这个时候,对车窗外成片成片的油菜花,我有了不同于以往任何时候的感觉,那种蓬蓬勃勃的美,使我的心情变得轻松许多。它那浓郁的生气无形中感染了我,唤醒了我生命中沉睡的信念,生活是美的,生命是可留恋的呀!从此,每年油菜花开时节,虽是行走艰难,也要拄着拐杖到郊外看看,我才不怕得什么花粉症呢。
   ④要论好看,任何一种单朵的花都会比单朵的油菜花好看得多,单朵的油菜花细小单调而不起眼,它之所以让我有了生命的感动,是因为这些花是以集团军的面貌出现的,显现出一种浩浩荡荡的生机,一种攻城略地的气势,一种汪洋恣肆的活力。油菜花才真正是太阳之光,是光和热的象征。
   ⑤多少个响晴天,我恋恋地望着窗外,对妻子说,陪我去看看那些油菜花吧。去了,在地头一站就是好半天。所有的油菜花,不论高的矮的,壮的瘦的,一例顽强地展示着自己那一份生命的本色。它们的黄,既不是初春的柳芽那种嫩黄,也不是菊花、葵花的那种老黄,那是一种青春的黄,黄得明净,黄得酣畅,黄得秀朴,黄得平净如水,黄得恬柔如笑。我偷偷给它取了个名字:女儿黄!灿烂的黄花下面,是绿得发暗的秆和叶,青葱的生命高擎着纯情的黄,托展着一个欣欣向荣的季节的美丽扉页。黄花满眼,清芳扑鼻,看花人沉醉了,恍觉自己也变成了一株充满青春活力的植物。
   ⑥油菜花似乎不怕雨打风吹,它前谢后继地开着,整体花期比一般的花都长些。菜花开了,再阴晦的日子也不难耐了,那爽心的亮色风雨收不去,望一眼,心田就会洒满阳光,它们其实就是活的香的阳光!更多的时候,我于户内凝望菜花,在病榻上斜靠着,看书倦了,总喜欢透过窗子看对面楼缝里的那片油菜花,那差不多成了一种生命的需要。油菜花谢了,没有画家会为我在窗外画一片女儿黄,但我不会再阴郁,花事一过,就是青阳朗朗的夏季了,阳光里流淌着无尽的菜花黄。我本质上仍是个农夫,春天于我,是希望的季节,更是收获的季节,我收获遍地黄金,那是一年乃至一生受用不尽的黄金!
   (选自2004年3月长江文艺出版社《中国精短美文100篇》,有删改)
   1.油菜花“生命的本色”是什么?
   答:
   2.第④⑤段中从哪些方面描写了油菜花?请概括回答。
   答:
   3.第⑤段写油菜花的黄,为什么还要写柳芽的黄、菊花的黄、葵花的黄?
   答: 。
   4概括说明油菜花对“我”生命态度产生的重要影响。
   答:
   5.文章主要内容是写油菜花,却以“遍地黄金”为题,说说你的理解。
   答:
  
   (十)(2004·广西)
   奇迹
   那年,文参加高考,以六分之差落榜了。
   那晚,文把窗外的月光望了一夜。
   吃早饭时,文对爹说:“爹,我想复读一年。”
   听了这话,爹白了他一眼,说:“算啦!回家帮我吧!”
   文知道爹的脾气,爹决定了的,九头牛也拉不回,错了也不改。何况这次,爹没错。
   那年,家里真是陷入了困境:奶奶病故了;刚埋了奶奶,娘又病了;家里贷款买的那头奶牛也不明不白地死了;村里还要庄基排队。两个弟弟还在上小学。家里就一个劳力了。
   这天晚上,文听了一夜的蛐蛐叫。
   第二天天刚亮,文就去找爷爷。文知道爹是个大孝子,爷爷的话,爹句句听。爷爷很疼文,立马去找爹。爹听了爷爷的话,气呼呼地说:“叫他复读,你能挣回钱来?”一句话问得爷爷不吱声儿了。
   文去找爹的好朋友赵伯。赵伯和爹在堂屋里谈了半天,叹着气摇着头走了。
   文咬咬牙,骑上他那辆破旧的自行车,去县城中学找当教师的姑父。这是最后一线希望了。
   半下午,文用自行车把姑父捎回家,姑父的脸晒得通红,文浑身出汗了,像刚从河里爬出来一样。
   姑父一口水也顾不上喝,就坐在正扎笤帚的爹身边:“叫娃复读吧!”
   爹硬硬地说:“家里没钱!”
   姑父说:“我供他!”
   爹白了他一眼,说:“你供他一年,他考不上,还要读,你供吗?”
   姑父坚决地说:“供!”
   爹连珠炮似的说:“还考不上你供吗?考上了你供吗?”
   气得姑父胸脯一起一伏地走了。
   文真想冲上去,给爹两脚,可他不敢。文也是个孝子呀。
   文默默地走到家后面的窖背上,靠着一棵柏树,无力地软了下去。文的脑幕上闪现出一双双专注听课的脸,文的耳畔回响着脆生生的上课铃声。
   这时侯 不知哪儿的广播里传来“敢问路在何方?路在脚下”的歌声。歌声里,文流下了伤心的泪:我的脚下哪有路?我的脚下明明是崖呀!
   不知过了多久,文听到谁在叫他。这声音多么熟悉而亲切呵!文把头倔强地扭过去。爹走上来,和蔼地说:“娃,咱回!”文硬硬地答:我不!”爹问:“你要咋?”文大声说:“你不叫我复读,我就从这里跳下去掉死算啦!”
   文想用这话激怒爹,让爹狠狠地打他一顿,这样,他心里也许好受些。
   谁知,听了他的话,爹的眼睛却湿了,他一下子把文拉起来,拉进他的怀里,激动地说:“爹答应你!”
   听了这话,文不敢相信自己的耳朵,把眼睛瞪得老大。
   爹说:“你找爷爷,找你赵伯,找你姑父,你把爹给感动啦!冲着这股牛劲,还怕考不上大学?爹就是把腰挣弯,也要供娃复读!”
   “爹!”文大叫一声,把爹抱住,热泪如泉涌。
   文复习了一年,考上了北京大学。
   只要有1%的希望,就用99%的努力去争取,这时,事情往往会出现转机,甚至产生奇迹! 不是么?
   1.文中揭示产生“奇迹”的前提条件的一句话是:
   答:
   2.文中画线句交代“爹的脾气”,在文中有何作用?
   答:
   3.文中两次写“文”一夜未眠,其表达作用是:
   答:
   4.文中依次写“文”、“爷爷”、“赵伯”、“姑父”求爹让“文”复读,其作用是:
   5.文章大篇幅地写“文”争取复读的过程,而对复读的经过和结果一笔带过。为什么这样处理?
   答:
   6.从故事中我们可以感受到“文”是一个什么样的人?试用一句话进行表述。
   答:
   (十一)(2004·资阳)
   家园如梦
   夜很深,也很静。浅浅的月光流进了我的村予,挤进了那扇用皮纸蒙住的三字窗。风轻轻地梳理着窗外还略耳薄的树枝,嗓音很低,却让我听得清楚那来自远方的呼唤.
   庭院里的那口古井,清楚地倒映着我曾经在井旁的柳树上猴跃的童年。辘轳上那长满黑斑的麻绠,依然牢牢地吊着我的心事,绷得像调紧的弦。
   “月光光,亮堂堂,背书包,进学堂……”井边学会的童谣鲜活如初,只是教我童谣的母亲,却已独卧寒山。母亲的声音已成记忆,然而母亲的血必将灌溉我的一生。
   流浪的脚步离开家园,只把乡愁饲养在井中,任何一丝不经意的涟漪,都有可能荡得我遍体伤痕。
   屋后的荒坡上,零零散散地落户了一些三月莓树,它们在贫瘠中送走一个个春夏秋冬,又迎来一个个春夏秋冬。
   母亲为我摘莓子时被刺破的手指,滴着血,凝成一团不褪的火红,永远燃烧在我记忆的深处。那些吃三月莓当饭的甜甜的日子,是母亲用手一分一分地扳来的。今年的三月,我想母亲还会在另外的世界里为我采摘三月莓。只是母亲已移居黄泉,即使我将膝盖埋进坟土,也无法缩短母子间的距离。
   等到三月莓红透的时候,我该回趟老家,去荒坡上采摘一包三月莓,捧撒在母亲的坟头。母亲曾经为我寻找三月莓的目光,擦亮一串串累累的爱。
   屋右的古枫树——鸟的天堂。孩提时,父亲总是架着长长的梯子,猫着腰一回又一回地爬上树去为我取鸟,样子很吃力,可父亲的脸上却从不滚落丝毫吃力的神情。
   如今,鸟渐渐地少了,只剩下乱七八糟的鸟巢搁在树桠间,可年迈的父亲却像童年的我一样,在鸟归季节里一遍遍地数着鸟巢。又是鸟儿孵殖的季节,隐约中,我感觉父亲佝偻着身子站在古枫前学舌一般地重复着“一、二、三、四……”那深深陷进了眼窝的眸子,专一地注视着通往山外的羊肠路。
   屋左蜿蜒蛇行的山路依旧在为我走出大山的举动作注脚,那浅浅的一行不知打上了我多少若隐若现的脚印。从山村走进城市,实际上是走进一种诱惑,甚至是一种折磨。
   山路的源头是生活,山路的尽处还是生活。生活就是生生死死,造化平衡世界,谁能适应这个世界,谁就是赢家。做个赢家,赢家有能力随遇而安。无论生活把自己推到哪个位置,都要用一颗平常心去面对,轻松靠自己给予,快乐只属于创造快乐的人。
   怀念家园,更怀念家园里的某些人。我茹苦一生而今永隔幽冥的母亲,愿您有您的天堂;我艰难活命又思儿念女的父亲,愿您有您的寄托!
   在家门前那堵不倒的竹篱笆上,我将自己攀援成一株不忘的牵牛,紫色的喇叭始终朝向敞开着的家门,芬芳屋里的每一道墙缝。
   家园如一件厚厚的袄,等待着每一个伶仃的流浪者去穿;家园如一双不破的鞋,永远套在流浪者缺暖的脚上: 家园如一柄永新的伞,一直搭在流浪者风雨兼程的肩膀上;家园如一块啃不完的饼,让流浪者一次又一次去补充能量;家园如一根拉不断的线,末端总系着一个流浪者的大风筝。
   1.文章表达的主题是什么?
   表达的主题是
   2.结合全文,理解文中加波浪线的A、B两个句子的含义。
   A句
   B句
   3.根据提示,在句后米字格中补写—个句子,表现游子对母亲的炽热感情。
  
   谁言寸草心,
   4.仿下边例句,以“校园”二字开头写一句话。
   例:家园如—块啃不完的饼,让流浪者一次又一次地补充能量;家园如一根拉不断的线,末端总系着—个流浪者的大风筝。
   校园
  
  
   5.从文章中摘录一个你认为有哲理的句子,然后说说你对这个句子的理解。
   摘录的句子是
  
   该句理解为
  
  
   (十二)(2004·湟中)
   母亲病了,住在医院里,我们兄弟姐妹轮流去守护。轮到我守护母亲那天,护士进来换床单,叫母亲起来。母亲病得不轻,下床很吃力。我赶紧说:“妈,你别动,我来抱你。”
   我 ,没想到母亲轻轻的,我用力过猛,差点朝后摔倒。
   护士在后面托了我一把,责怪说:“你使那么大劲干什么?”我说:“我没想到我妈这么轻。”护士问:“你以为你妈有多重?”我说:“我以为我妈有100多斤。”护士笑了,说:“你妈这么矮小,别说病成这样,就是 甲  的时候,我猜她也到不了90斤。”母亲说:“这位姑娘有眼力,我这一生,最重的时候只有89斤。”
   母亲竟然这么轻,我心里很难过。护士取笑我说:“亏你和你妈生活了几十年,眼力这么差。”我说:“如果你跟我妈生活几十年,你也会看不准的。”护士问:“为什么?”我说:“在我记忆中,母亲总是手里拉着我,背上背着妹妹,肩上再挑100多斤的担子翻山越岭。这样年复一年,直到我们长大。我们长大以后,可以干活了,但每逢有重担,母亲总是叫我们放下,让她来挑。我一直以为母亲力大无穷,没想到她是用80多斤的身体,去承受那么多重担。”
   我望着母亲瘦小的脸,愧疚地说:“妈,我对不住你啊!”
   护士也动情地说:“大妈,你真了不起。”
   母亲笑一笑说:“提那些事干什么,哪个母亲不是这样过来的?”
   护士把旧床单拿走,铺上新床单,又很小心地把边边角角拉平,然后回头吩咐我:“把大妈放上去吧,轻一点。”
   我 乙 ,说:“妈,你把我从小到大抱大,我还没有好好抱过你一回呢。让我抱你入睡吧。”母亲说:“快把我放下,别让人笑话。”护士说:“大妈,你就让他抱一回吧。”母亲这才没有做声。
   我坐在床沿上,把母亲抱在怀里,就像小时候母亲无数次抱我那样。
   母亲终于闭上眼睛。我以为母亲睡着了,准备把她放到床上去,可是,我看见有两行泪水,从母亲的眼里流了出来……
   1.(1)请为文中甲、乙两处选择恰当的词语。
   甲:A、年轻气盛 B、年轻力壮 C、年轻有为 ( )
   乙:A、心领神会 B、灵机一动 C、突发奇想 ( )
   (2)这篇文章中运用最多的描写方法是:
   2.联系上下文,根据你的理解和想像,在文中划横线的地方补写几句描写“我”抱母亲动作的句子。请把补写的内容写在下面的横线上。
  
   ___________________________________________________________________
   3.揣摩下列句中加点词的含义,说说表现人物怎样的心理状态?
   护士取笑我说:“亏你和你妈生活了几十年,眼力这么差。”
   ___________________________________________________________________
   母亲竟然这么轻,我心里很难过。
   ___________________________________________________________________
   4.在文末“我看见有两行泪水,从母亲的眼里流了出来……”母亲为什么会流泪?
   ___________________________________________________________________
   5.请用简洁的语言为本文拟一个标题(不超过8个字)。
   ___________________________________________________________________
   6.你了解你的母亲吗?读完上文,请谈谈你的感受。
   ___________________________________________________________________
  
   (十三)(2004·潍坊)
   苦石竹,香石竹
   她小的时候,家境不好,和所有清贫家庭的中年妇女一样,她的母亲被窘迫的生活压得展不开眉头。家里气氛虽然平静,但有些压抑。
   她也如许多平平常常的女孩一样,上学,放学,做家务,照看弟妹,生活没有一丝波澜,她以为自己的一生就是这样了。虽然只有14岁的年纪,但是她想,她长大了,也只能过这样的生活,只不过上学换成上班,照顾丈夫和孩子。对于前程,她真有些不敢想。
   但是生活还是有了小小的改变。班里新来了一位年轻的女班主任,非常活泼。阳春温暖的风吹过时,她告诉学生,在这个春天,有一个节日叫“母亲节”。
   她望着班主任年轻的脸,竟有些目瞪口呆的感觉。她听说过妇女节、劳动节、儿童节,怎么还有母亲节?她一直以为,母亲是女人天生该做的事,一点都不值得庆贺和祝福,而且,母亲的职业类似苦役,没有出头的日子,想到母亲紧皱的眉头,她更确信了这一点。
   但是班主任说,在地球上的另一个国家,每到这一天,都有很多男孩儿女孩儿送花给他们的母亲,表达对母亲的爱:“爱是一定要说出来的,即使是家人,也要让他们知道你对他们的亲情。”
   听到这句话,班里静了有半分钟的时间,即使他们正是爱表现的年龄,但是恐怕没有人对父母说过一句充满感激的话,更别谈送一朵花了。老师端端正正地在黑板上写了三个字:“康乃馨”。这是一种多么优雅的花名啊,她悄悄地记在了心里。过后的很长时间,她都在想着这花,康乃馨,这到底是一种怎样的花,竟会作为一种最圣洁的花献给母亲。
   这个想法是在一天深夜里冒出来的,她半夜醒来,这个想法就明明亮亮地冒了出来,像一片初生的青草,娇娇嫩嫩,又微微酥麻,不停地在她心中温柔地擦来擦去。她开始睡不着,她奇怪自己这个大胆的想法,在他们这个家,即使是一小块肥皂都要精打细算地用,怎么可能花钱去买一束花,一束花店里的花呢?天亮的时候,她决定要送一大束康乃馨给母亲,灿灿烂烂的一大束,会把春天带到他们家,让母亲的脸上充满灿烂的笑。
   她开始为这个梦想努力。她不吃早饭,把早点钱省了下来;她也不再买新练习本了,自己用毛边纸拼凑了一本;她还利用空余时间织一些小孩儿的毛衣,她的手巧,以前她都把织毛衣的钱交给家里,现在她不交了,而是很小心地用手绢包起来,放在一个隐秘的地方。有一次她偷偷包起来的时候,转过身看见母亲在看她,她的脸飞快地红了,想说什么,终于什么也没说。母亲也没问,只是背过身,轻轻地叹了口气,她的心像针刺了一样难受。
   终于攒够了20元钱,路过花店的时候,她大胆地走了进去。“请问,康乃馨在哪里?”连她自己都听得出自己的声音小得出奇,她的脸已经涨得通红了。
   “康乃馨就在那边。”老板友善地看着她。原来,这是她认识很久的花,她5岁之前,是在外婆家长大的,外婆种了很多这样的花,重重叠叠的花瓣那样温顺、朴素、美丽,它的名字叫石竹。
   “原来康乃馨就是石竹。”她不禁失望地叹了口气。老板奇怪地望着她,“对,但它是有香味的,叫香石竹”。
   香石竹?石竹是没有香味的,她记得外婆每过一周,都要剪下一大把石竹,大把大把深红粉红的石竹,映得夏日的黄昏特别璀璨。她总是奇怪这种花,花瓣总是重重叠叠的,一点都不像别的花朵那样单纯天真。
   “石竹苦,石竹心思多,石竹护孩子。”外婆慈祥地说。她再找不到外婆家的石竹了,外婆已经亡故好多年了。她记得,外婆在的时候,母亲还年轻,脸上也还带着笑,因为外婆帮母亲照顾孩子。家事没有如此重,最主要的,恐怕是在外婆面前,母亲还觉得自己是个饱受母爱温情的孩子。
   石竹心思多,石竹护孩子,所以这种花是献给母亲的花。她买了一大把康乃馨。那一晚,香石竹在她家绽放得无比艳丽,母亲的脸上也盛开了消逝已久的笑容。那一晚,她忽然觉得,原来世间还是充满美丽的,像她苦苦寻觅的康乃馨,原来就是石竹花啊,它一直在她的记忆中,只不过换了一个名字。不过她还是找到它了。
   1.文中划线的“对于前程,她真有些不敢想”一句中“她”为什么“不敢想”?
   答:
   2.当听到班主任老师谈到“母亲节”时说“地球上的另一个国家,每到这一天,都有很多男孩儿女孩儿送花给他们的母亲”,班里静了有半分钟的时间,班里为什么会静?请说说你的理解。
   答:
   3.本文运用了多种修辞方法,请你从文中找出一处,指出所使用的修辞方法,说明其作用。
   答:
   4.下面对文章的理解正确的一项是( )
   A.“她”在听班主任说起“母亲节”之前,一直以为母亲的职业类似苦役,没有出头的日子,是女人天生该做的事。所以听班主任说有母亲节时感到吃惊。
   B.当“她”听到班主任老师讲外国的许多男孩儿女孩儿每年母亲节时送花给他们的母亲,表达对母亲的爱时,她深受触动,立刻决定送一束康乃馨给自己的母亲。
   C.“她”为了给母亲买一束“康乃馨”而把织毛衣的钱藏起来,当被母亲发现时,在母亲的叹息声里,她的心像针刺了一样难受,但她没有解释,她知道母亲不会理解她。
   D.当“她”知道“康乃馨”就是“石竹”时,失望地叹了口气,因为“她”早已知道这种花,知道这是一种常见的平凡的花。但“她”还是买了,因为“她”觉得定下的事不能变。
  
   (十四)(2004·临汾)
   青木瓜之味
   ⑴大约是2000年初春的一个星期天下午,我去邮局发信。就在快到邮局的时候,一个年轻的女子和我擦肩而过。忽然,她停住脚步,回头看了我一眼。那眼神很亲切,也有些意外的惊奇,仿佛认出一个熟人而与之意外相逢。那眼神闹得我以为真的碰见了什么认识的人,便也禁不住停住脚步,看了她一眼:年龄不大,也就二十出头,模样清爽,中等身材,瘦瘦的。看她的装扮,初春时节还穿着一件(臃、拥、痈)肿的棉衣,就猜得出是一个外地人,大概是打工妹。我仔细地想了想,从来没有见过这么个人,她肯定是认错了人。于是,我暗笑自己的自作多情,向邮局走去。
   ⑵我走了没几步,她从后面跑了过来,跑到我的面前,这让我很吃惊,不知碰见了什么人。只听见她用南方人那种软绵绵的声音仔细而小心翼翼地问我:“你是不是肖复兴老师?”我越发惊讶,她居然叫出了我的名字,木讷地站在那里,近乎机械地点了点头。她一下子显得很兴奋,接着说:“刚才你迎面向我走来,我看着你就像。我读中学时就看过你写的书,你和书上的照片很像。真没有想到怎么这么巧,今天在这里遇见了你!”
   ⑶原来是一位读者,大概她这番热情的话,很能够满足我的虚荣心,尤其是听她说她喜欢我写的一些东西,特别是说她读中学的时候读我写的东西对她有帮助,一直忘不了……我就像小学生爱听表扬似的,立刻有些发晕,找不着北了,站在街头和她聊了起来,一任身边车水马龙,喧嚣不已。
   ⑷从她那话语中,我渐渐地听明白了,她从小在南方农村长大,中学毕业,没有考上大学,家里生活困难,就跟着乡亲来到北京打工,住的地方离我家不算太远,要走半个小时左右,今天星期天休息,她是刚刚到邮局给家里寄钱,并发了一封平安家信。虽是萍水相逢,只是些家常话,却让我感到她像是在掏心窝子,一下子竟有些感动,没有想到只是写了一些平常的东西,能够让心拉近,距离缩短,心里想也应该说是如何没什么用处的文学的一点特殊功能吧。于是,我进一步犯晕,沿着斜坡继续顺溜地下滑,不知对她的热情如何回报似的,竟然指着马路对面我家住的楼对她说:“我家就住在那里,你有空,欢迎你到我家做客。”说着把地址写给了她。她高兴地说:“太好了,我一定去。”
   ⑸回到家后,我就把这件事当作喜帖子,向家人讲了,不想立刻遭到全家一盆冷水浇头,纷纷说我,“你以为你遇到了知己呢?别是个骗子吧?”“可不是,现在骗子可多着呢,你可别忘了狐狸说几句赞扬的话,是为了骗乌鸦嘴里的肉。”“什么?你还把咱家的地址告诉了人家?你傻不傻呀?你就等着人家上门找到你头上来骗你吧!”“要真是找上门来,骗几个钱倒没什么,可别出别的事……”
   ⑹一下子,说得我发(蒙、懵、朦)。一再回忆街头和那个年轻女子的相遇和交谈,不像是个狐狸似的骗子呀,再说,她肯定是读过我写的书,要不也说不出书名,并且能对照着书上的照片认出我来呀。但家里的人说得也没有错,谁也不会把骗子两字写在脑门上,高明的骗子越来越多,防不胜防。这么一想,心里连连后悔,而且不禁有些发虚。一连好几天,都有些提心吊胆。
   ⑺好在一连好多天过去了,都平安无事。时间一长,这件事情渐渐淡忘了。
   ⑻将近一年过去了,春节过后的一天,我们全家从天津孩子的姥姥家过完年回家,刚上电梯,开电梯的老太太对我说:“你先等我一会儿,前两天有个年轻女子来找你,你没在家,把带来的东西放在我那儿了。”不一会儿,就拿来一包用废报纸包着的东西。回家打开一看,是两个青青的木瓜。木瓜的旁边有一张小纸条,上面写着几行小字:“肖老师:
   。”落款是“你的一个读者”。
   ⑼全家都愣在那里,谁都说不出一句话来。
   ⑽这件事虽已过去了四年,但我怎么也忘不了这个年轻而真诚的女子,忘不了这件事情,忘不了这两个木瓜。总记得切开木瓜时的样子,别看皮那样青,里面却是红红的,格外鲜艳,特别是那独有的清香味道,在房间里飘荡着,好多天没有散去。
   1.请从括号内选择正确的字填在句中横线上。
   ①看她的装扮,初春时节还穿着一件 (臃、拥、痈)肿的棉衣。
   ②一下子,说得我发 (蒙、懵、朦)。
   2.“我”在与年轻女子交往的过程中,感情发生了多次变化。请你用简洁的语言概括出“我”在不同时间、地点的感情变化。
   ①在邮局前与年轻女子交谈时:
   ②回家受到家人埋怨后:
   ③收到木瓜后:
   3.请你在文章第⑴段前加几句话,另写一个开头,使新的开头与结尾相互照应。
   答:
   4.文中描述“我”和年轻女子相遇,分别用了“意外相逢”和“萍水相逢”两个词语,第⑴段中的“意外相逢”不能改为“萍水相逢”,因为:
  
   5.文中写年轻女子除运用了语言描写的方法外,还运用了外貌(肖像)描写的方法。阅读第⑴段,说说主要写了她外貌的哪些方面(至少说出两个方面)?由此可以看出,她给“我”留下了怎样的第一印象?
   ①写了外貌的这几个方面:
   ②给“我”留下的第一印象:
   6.文章第⑸段写“我”向家人讲了“我”与年轻女子相遇并留下地址的情况后,受到家人的埋怨,你认为家人的话有无道理?说一说你的看法。
   答:
   7.文章的题目与结尾都提到了青木瓜之“味”,你认为青木瓜有哪些“味”?谈谈你的理解。
   答:
   8.文中第⑻段横线上空缺的是年轻女子所写纸条的内容。假如你是文中的年轻女子,请将纸条内容补写在下面。要求合乎情理,60字左右。
   答:
  
   9.读了本文之后,你认为哪个地方写的最有特色?试就此作一点简要的评价。
   答:

(十五)(2004·南通)
   人格——最高的学位
   白岩松
   ⑴很多年以前,有一位学大提琴的年轻人向20世纪最伟大的大提琴家卡萨尔斯讨教:我怎样才能成为一名优秀的大提琴家?
   ⑵卡萨尔斯面对雄心勃勃的年轻人,意味深长地回答:先成为优秀而大写的人,然后成为一名优秀而大写的音乐人,最后成为一名优秀的大提琴家。
   ⑶听到这个故事的时候,我还年少,老人回答时所透露的含义我还理解不多,然而随着采访中接触的人越来越多,这个回答在我脑海中越印越深。
   ⑷在采访北大教授季羡林的时候,我听到一个关于他的真实的故事。有一年秋天,北大新学期开始了,一个外地来的学子背着大包小包走进了校园,实在太累了,就把包放在路边。这时正好一位老人走来,年轻学子就拜托老人替自己看一下包,而自己则轻装去办理手续。老人爽快地答应了。近一个小时过去了,学子归来,老人还在尽职尽责地看守。谢过老人,两人分别。
   ⑸几天后是北大的开学典礼,这位年轻的学子惊讶地发现,主席台上就坐的北大副校长季羡林正是那一天替自己看行李的老人。
   ⑹我不知道这位学子当时是怎样的一种心情,但我听到这个故事之后却强烈地感觉到:人格才是最高的学位。
   ⑺这之后,我又在医院里采访了世纪老人冰心。我问先生,您现在最关心的是什么?
   ⑻老人的回答简单而感人:是年老病人的状况。
   ⑼当时的冰心已接近自己人生的终点,而这位在80年前的“五四”运动爆发那一天开始走上文学创作之路的老人,心中对芸芸众生的关爱之情,历经近80年的岁月而仍未老。这又该是怎样的一种传统!
   ⑽冰心的身躯并不强壮,即使年轻时也少有飒爽英姿的模样,然而她这一生却是用自己当笔,拿岁月当稿纸,写成的一篇关于爱是一种力量的文章。她在离去之后,给我们留下了伟大的背影。
   ⑾世纪老人在陆续地离去,他们留下的爱国心和高深的学问却一直在我们心中不老。但在今天,我还想加上一条,这些世纪老人所独具的人格魅力是不是也该作为一种传统由我们向后延续?
   ⑿前几天我在北大听到一个故事,清新而感人。
   ⒀一批刚走进校园的年轻人,相约去看季羡林先生。走到门口,却开始犹豫,他们怕冒失地打扰了先生。最后决定,每人用竹子在季老家门口的土地上留下问候的话语,然后才满意地离去。
   ⒁这该是怎样美丽的一幅画面!离季老家不远,是北大的伯雅塔在未名湖中留下的投影,而在季老家门口的问候语中是不是也有先生的人格魅力在学子心中留下的投影呢?只是生活中,这样的人格投影在我们心中还是太少。
   ⒂于是,我也更加理解了卡萨尔斯回答中所具有的深意。怎样才能成为一个优秀的主持人呢?心中有个声音在回答:先成为一个优秀的人,然后成为一个优秀的新闻人,最后自然地成为一个优秀的节目主持人。
   ⒃我知道,这条路很长,但我将执著的前行。
   [注]白岩松:中央电视台著名节目主持人。
   1.这篇文章叙写了季羡林和冰心两位老人的各一件事,请分别用一句话加以概括。(每句话不超过15个字)
   季羡林先生
   冰心老人 
   2.第⑽段中,作者为什么说,冰心在离去之后给我们留下了“伟大的背影”?(用自己的语言简要概括)
                                       
   3.⒁段中,写“北大的伯雅塔在未名湖中留下的投影”是为了衬托                               
                        
   4.文章结尾说“这条路很长,但我将执著的前行”,请结合第⒂段回答:这条很长的路指的是怎样的路?(用原文语句回答)
                                       
   5.阅读课文《幽径悲剧》中季羡林先生的心灵独白,回答问题。
   世界上像我这样没有出息的人,大概是不多的。古藤的哭泣声恐怕只有我一个能听到。在浩茫无际的大千世界上,在林林总总的植物中,燕园的这一棵古藤,实在渺小得不能再渺小了。你倘若问一个燕园中人,决不会有任何人注意到这一棵古藤的存在的,决不会有任何人关心它的死亡的,决不会有任何人为之伤心的。偏偏出了我这样一个人,偏偏让我住到这个地方,偏偏让我走这一条幽径,偏偏又发生了这样一个小小的悲剧;所有这一些偶然性都集中在一起,压到了我的身上。我自己的性格制造的这一个十字架,只有我自己来背了。奈何,奈何!
   但是,我愿意把这个十字架背下去,永远永远地背下去。
   季先生关注的是一棵“渺小得不能再渺小”的古藤,并为之而背上了“十字架”。联系选文中有关季羡林先生的事例,你认为他具有怎样的人格魅力?
   6.世纪老人冰心曾谆谆告诫:“青年人,珍重的描写罢,时间正翻着书页,请你着笔!”结合选文回答:你打算怎样在自己的人生书页上“着笔”?
                                            
                                         
  
   (十六)(2004·北京)
   伤 害
   巴金
   ⑴一个初冬的午后,在泸县城里,一条被燃烧弹毁了的街旁,我看见一个黑脸小乞丐寂寞地立在面食担子前,用羡慕的眼光,望着两个肥胖孩子正在得意地把可口的食物往嘴里送。
   ⑵我穿着秋大衣,刚在船上吃饱饭,闲适地散步到街上来。
   ⑶但是他,这个六七岁的孩子,赤着脚,露着腿,身上只披着一块破布,紧紧包住他那瘦骨的一身黑皮在破布的洞孔下发亮。他的眼睛无光,两颊深陷,嘴唇干瘪得可怕,两只干瘦得像鸡爪的手无力地捧着一个破碗,压在胸前。
   ⑷他没有温暖,没有饱足。他不讲话,也不笑。黑瘦的脸上涂着寂寞的颜色。
   ⑸我不愿多看他,便匆匆走过他的身旁。但是我又回转来,因为我也不愿意就这样地离开他。
   ⑹这样地一来一往,我在他的身边走过四五次。他不抬头看我一眼,好像他对这类事情并不感到惊奇。我注意地看他,才知道他的眼光始终停留在面食担子上。但甚至这眼光也还是无力的。
   ⑺我站在他面前,不说什么,递了一张角票给他。
   ⑻他也默默地接过角票,把眼光从担子上掉开。他茫然地看看我,没有一点表情,仍然不开口。于是他埋下眼睛,移动一下身子,又把脸掉向面担。两个胖小孩还在那里吃“连肝肉”、“心肺”一类的东西,口里“嘘嘘”作声。
   ⑼我想揩去他脸上的寂寞的颜色,便向他问两句话。他没有理我。他甚至不掉过头来看我。
   ⑽我想,也许他没有听见我的话;也许我的话使他不高兴。我问的是:你有没有家?有没有亲人?
   ⑾我不再对他说话,我默默地离开了他。我转弯时还回头去看那个面担,黑脸小乞丐立在担子前,畏怯地望着卖面人,右手伸到嘴边,一根手指头衔在口里。两个肥胖小孩却站到旁边一个卖糖食的摊子前面去了。
   ⑿七天后我再到泸县城里,又经过那条街。仍然是前次看见的那样的街景。面食担子仍然放在原处。两个肥小孩还是同样得意地在吃东西。黑脸小乞丐仿佛也就站在一星期前立过的那个地方,用了同样羡慕的眼光望着他们。一切都没有改变。我似乎并没有在别处耽搁了一个星期。
   ⒀我走到黑脸小孩面前,又默默地递了一张角票到他的手里。他也默默地接着,而且也茫然地看我一眼,没有表情,也没有动作。以后他仍旧把脸掉向面担。
   ⒁我们两个都重复地做着前次的动作。我甚至没有忘记问他:你有没有一个家?有没有一个亲人?
   ⒂这次他仍旧不回答我,不过他却仰起头看了我一两分钟。我也埋下眼睛去看他的黑脸。茫然的表情消失了。他圆圆地睁着那对血红的眼睛,泪水像线一样地从两只眼角流下来。他把嘴一动,没有发出声音,就掉转身子,用劲地一跑。
   ⒃我在后面唤他,要他站住。他不听我的话。我应该叫他的名字,可是我不知道他有什么样的姓名。我站在面担前,希望能够看见他回来。然而他的瘦小身子像一股风似地飘走了,并没有一点踪迹。
   ⒄我等了一会儿,又走到旁边那个在废墟上建造起来的临时广场上,跟着一些本地人听一个老烟客讲明太祖创业的故事。那个老烟客指手划脚地讲得津津有味。众人都笑,我却不作声,我的心并不在这里。
   ⒅过了半点多钟,这附近还不见那个黑脸小孩的影子。我便到城里各处走了一转,后来再经过这个地方,我想,他应该回来了,但是我仍旧看不到他。那两个肥胖小孩还在面担前吃东西。
   ⒆我感到疲倦了。我不知道黑脸小孩住在什么地方,或者他是否就有住处。我不知道他什么时候可以再到这里来。看见阳光离开了街市,我觉得疲倦增加了,我想回到船上去休息。
   ⒇最后我终于拖着疲倦的身子离开了泸县。那一段路是不容易走的,我的心很沉重。我想到那个黑脸小孩和他的突然跑开知道自己犯了过失了。
   (21)我为什么两次拿那问话去折磨他呢?这原是明显的事实:要是他有家,有亲人,他还会带着冻和饿寂寞地立在街旁么?他还会像一棵枯草,一只病犬那样,木然地、无力地捱着日子么?
   (22)他也许不知道家和亲人的意义。但是他自己和那两个胖小孩间的差别,他应该了解吧。从这差别上他也许可以明白家和亲人的意义。那么,我大大地伤害了他,这也是很明显的事实了。
   (23)今天,八个月以后的今天,我还记得那个黑脸小孩的面貌和他两个眼角的泪水。他一定早忘记了我。但是我始终忘不掉他。我想请求他那小小的心灵宽恕我。然而我这些话能够达到他的耳边么?他会有机会看到我的文章么?
   (24)我不知不觉间在那个时候犯了不可补偿的过失了。
   1941年8月1日
   (选自《巴金选集·散文随笔选》)
   1.作者用“但是”衔接第⑵⑶段,有什么效果?
   答:
   2.第⑸段中的“这样”指什么?为什么“我”不愿意就“这样地离开他”?
   答:
   3.第⑼段中的“揩去”照应了文中的哪个词语?“我想揩去他脸上的寂寞的颜色”的意思是什么?
   答:
   4.从下面一段描写中,你体会出“我”第二次问话后黑脸小孩怎样的复杂心情?
   (黑脸小孩)茫然的表情消失了。他圆圆地睁着那对血红的眼睛,泪水像线一样地从两只眼角流下来。他把嘴一动,没有发出声音,就掉转身子,用劲地一跑。
   答:
   5.第⒄段“我的心并不在这里”与第⒇段“我的心很沉重”,在表现“我”的心情上有什么相同之处和不同之处?
   答:
   6.“乞丐”是指靠向别人要钱要饭生活的人。从第⒀段起,“我”就不再称那个“六七岁的孩子”是“黑脸小乞丐”,而称他“黑脸小孩”了。你怎样理解这一变化?
   答:
   7.第(21)—(24)段是“我”对自己过失的反思,从中你获得了什么感悟?(不少于80字)
   答:
   (十七)(2004·太原)
   特蕾莎修女
   ①她只是一位满面皱纹、瘦弱文静的修女。
   ②一九九七年九月,当她去世时,印度政府为她举行国葬,全国哀悼两天。成千上万的人冒着倾盆大雨走上街头,为她的离去流下哀伤的眼泪。她就是被誉为“活圣人”的特蕾莎修女。
   ③十八岁的时候,特蕾莎修女从她的家乡——马其顿一个偏僻的乡村来到遥远的印度,来到被称为“噩梦之城”的加尔各答。在这里,她看到有病的人无人照看,孤独的男人和女人躺在街头等死,成百上千失去父母的儿童四处游逛……她感到了工作的召唤,于是脱下修女服,开始了护理和救助穷人的工作,并一直延续了半个世纪。
   ④特蕾莎修女清醒的认识到,居高临下的给予,接受者会有被施舍的屈辱感觉,这对于一个人的尊严是极有害的,它可能导出苦涩和敌意,而不是和谐与和平。在街头,这个瘦小的修女亲手握住快要横死的穷人的手,给他们临终前最后一丝温暖;在医院,这个受着病痛折磨的修女亲吻艾滋病患者的脸庞,为他们筹集医疗资金;她给柬埔寨内战中被炸掉双腿的难民送去轮椅,也送去生活的希望;她细心的从难民溃烂的伤口中捡出蛆虫,帮助他们减轻痛苦……
   ⑤在特蕾莎修女的感召下,数以万计的人参与了她的“慈善传教士”活动,数以千万计的人从这个修道会的社会福利和救援工作中受益。
   ⑥一九七九年,特蕾莎修女被授予诺贝尔和平奖,颁奖辞说:“她的事业的一个特征就是对单个人的尊重……最孤独的人、最悲惨的人、濒临死亡的人,都从她的手中接受到了不含施舍意味的同情,接受到了建立在对人的尊敬之上的同情。”面对巨大的荣誉,特蕾莎修女的答辞是:“今天,我来接受这项奖金,是代表世界上的穷人、病人和孤独的人。这个奖是对贫穷世界的承认。”她把奖金全部用于救助那些穷人和受苦难的人,并向诺贝尔委员会提出了一个小小的要求——取消例行的授奖宴会,因为那太浪费了。几句朴实的话语,一身寻常的打扮,但特蕾莎修女身上的光芒却照耀着整个颁奖礼堂。委员会接受了这一请求,并且将省下来的七千一百美元赠与她领导的仁爱修会。
   ⑦特蕾莎修女自己也是一个穷人,她的生活朴实无华;但同时她又是世界上最富有的人,因为她拥有爱、给予爱、收获爱。她曾动情的说:“我们所做的不过是汪洋中的一滴水,但若欠缺了那一滴水,这汪洋总是少了一滴水。”特蕾莎修女说这些话的时候,就好像母亲给孩子讲故事,没有花招、没有卖弄,有的只是一颗直白坦率的心灵。她微笑着说:“让我们记住一点:没有人不需要关爱,我们要总是以微笑相见,尤其是在微笑起来很困难的时候,更需要微笑。”
   ⑧是的,我们的生活中太缺少这样的微笑了。那些脸上肌肉只会作机械运动的美女模特们,当她们面对特蕾莎修女的微笑的时候,才会发现什么是真正的美。美与爱联系在一起,美是爱的一部分。不懂得爱的人,永远跟美无缘。
   〔注〕修女:天主教或东正教中出家修道的女子。
   1.阅读文章,要把握人物特点。用自己的话说说特蕾莎修女是一个怎样的人。
   答:
   2.阅读文章,要分析典型材料。从文章第②、⑥两段中选取一个典型事例,具体说说它对表现人物所起的作用。
   答:
   3.阅读文章,要品味关键语句。联系课文《麦琪的礼物》或课外阅读,谈谈你对结尾画线句的理解。
   答:
   4.诺贝尔委员会授予特蕾莎修女的颁奖辞语言精练、饱含情感,极具感染力。2004年2月20日,十大感动中国2003年年度人物评选揭晓,入选人物是杨利伟、钟南山、陈忠和、尾山宏、梁雨润、巴金、高耀洁、达吾提·阿西木、成龙和衡阳武警消防官兵。请你选择其中之一(尾山宏除外),运用个性化的语言为他设计一段精彩的颁奖辞,100字左右。(6分)
   人物:
   颁奖辞:
   【帮帮你】若设计有困难,可参照评选委员会授予尾山宏的颁奖辞:“一位70岁的日本老人,承受着巨大的压力,用自己大半生的时间对日本政府侵华战争的罪行进行着不懈的追问。在他身上,人们看到了跨越国家和民族的正义力量,这力量启示着人们,在捍卫正义的道路上,人们可以超越一切界限,而惟一不能失去的就是正义响在心中的声音。”
  
   (十八)(2004·宾州)
   燕 子
   席慕容
   初中的时候,学会了那一首《送别》的歌,常常唱:
   长亭外,古道边,芳草碧连天……
   有一个下午,父亲忽然叫住我,要我从头再唱一遍。很少被父亲这样注意过的我,心里觉得很兴奋,赶快再从头来好好地唱一次:
   长亭外,古道边……
   刚开了头,就被父亲打断了,他问我:“怎么是长亭外?怎么不是长城外呢?我一直以为是长城外啊!”
   我把音乐课本拿出来,想要向父亲证明他的错误。可是父亲并不要看,他只是很懊丧地对我说:
   “好可惜!我一直以为是长城外,以为写的是我们老家,所以第一次听这首歌时就特别地感动,并且一直没有忘记,想不到竟然这么多年是听错了,好可惜!”
   父亲一连说了两个好可惜,然后就走开了,留我一个人站在空空的屋子里,不知道如何是好。
   前几年刚搬到石门乡间的时候,我还怀着凯儿,听医生的嘱咐,一个人常常在田野间散步。那个时候,山上还种满了相思树,苍苍翠翠的,走在里面,可以听到各式各样的小鸟的鸣声。田里面也总是绿意盎然,好多小鸟也会很大胆地从我身边飞掠而过。
   我就是那个时候看到那一只孤单的小鸟的,在田边的电线杆上,在细细的电线上,它安静地站在那里,黑色的羽毛,像剪刀一样的双尾。
   “燕子!”我心中像触电一样地呆住了。
   可不是吗?这不就是燕子吗?这不就是我从来没有见过的燕子吗?这不就是书里说的,外婆歌里唱的那一只燕子吗?
   在南国的温热的阳光里,我心中开始一遍又一遍地唱起外婆爱唱的那一首歌来了:
   燕子啊!燕子啊!你是我温柔可爱的小小燕子啊……
   在以后的好几年里,我都会常常看到这种相同的小鸟,有的时候,我是牵着慈儿,有的时候,我是抱着凯儿,每一次,我都会兴奋地指给孩子看:
   “快看!宝贝,快看!那就是燕子,那就是妈妈最喜欢的小小燕子啊!”
   怀中的凯儿正咿呀学语,香香软软的唇间也随着我说出一些不成腔调的儿语。天好蓝,风好柔,我抱着我的孩子,站在南国的阡陌上,注视着那一只黑色的安静的飞鸟,心中充满了一种朦胧的欢喜和一种朦胧的悲伤。
   一直到了去年的夏天,因为内政部的邀请,我和几位画家朋友一起,到南部国家公园去写生,在一本报道垦丁附近天然资源的书里,我看到了我的燕子。图片上的它有着一样的黑色羽毛,一样的剪状的双尾,然而,在图片下的注释和说明里,却写着它的名字是“乌秋”。
   在那个时候,我的周围有着好多的朋友,我却在忽然之间觉得非常的孤单。在我的朋友里,有好多位在这方面很有研究心得的专家,我只要提出我的问题,一定可以马上得到解答,可是,我在那个时候惟一的反应,却只是把那本书静静地合上,然后静静地走了出去。
   在那一刹那,我忽然体会出来多年前的那一个下午,父亲失望的心情了。其实,不必向别人提出问题,我自己心里也已经明白了自己的错误。但是,我想,虽然有的时候,在人生的道路上,我们是应该面对所有的真相,可是,有的时候,我们实在也可以保有一些小小的美丽的错误,与人无害,与世无争,却能带给我们非常深沉的安慰的那一种错误。
   我实在是舍不得我心中那一只小小的燕子啊!
   [注]席慕容,台湾知名画家、散文家和诗人。祖籍内蒙古,1943年生于重庆,1954年迁居台湾,1963年毕业于台湾师范大学艺术系。初中语文教材选了她的《乡愁》一诗。
   1.作者在文中记叙了两件事,请你各用一句话进行概括。
   ①
   ②
   2.本文以“燕子”为题,但第一件事与燕子没有关系。作者这样写的原因是什么?
   答:
   3.当“父亲”发现了自己的错误后“一连说了两个好可惜”,你认为“父亲”“可惜”什么?
   答:
   4.文中划线的一段文字连用了四个问句,你认为这样写的作用是什么?
   答:
   5.作者通过这两件事悟出了哪些道理?
   答:
   6.联系作者生平,说说作者为什么“舍不得”“心中那一只小小的燕子”?
   答:
   7.文中说:“天好蓝,风好柔,我抱着我的孩子,站在南国的阡陌上,注视着那一只黑色的安静的飞鸟,心中充满了一种朦胧的欢喜和一种朦胧的悲伤。”请你仔细体会作者的思想感情,以作者的口吻写一段话,表达那种“朦胧的欢喜”和“朦胧的悲伤”。
   答:
  
   (十九)(2004·南京)
   父亲的斧头
   一把斧头完成的最后一道工序是淬火①。
   父亲的习惯是把一把刚刚淬过火的崭新斧头钳起来,将斧头对准砧子后的那尖角,在那上面用力啃一啃,看这把斧头的钢口如何,它能否吃得动这铁。
   正因为这样,那只砧子的尖角斧痕累累,刚刚削过的新痕泛着银白。而那把父亲才试过后用力抛在地上的斧头还很烫手,新斧头发着蓝光。
   这时候,父亲瞅一眼躺在前面的斧头,一只脚踩在砧墩上,端起那只水烟锅,咕嘟咕嘟抽起烟来。而此时,我就能歇歇手,赶快离开打铁铺,跑到大门外边去。我始终想远离这丁当作响的日子,跑到外面的世界闯荡。那时候我像一把刚刚打造好的斧头,准备磨快刃子,等待机会,狠狠砍生活两斧头。
   一次,放了暑假,父亲要我给他搭下手,打造一批镰刀。满山遍野的庄稼都黄了,都在等待镰刀来收割。人们需要镰刀,庄稼更需要镰刀,金黄的麦子都张了口,几乎要叫出声来。父亲心里很着急。我不在乎这些,我想我的事。
   我对父亲说,我不想打镰刀,我想去采药。我想像着采到了一大麻袋药。那时候我们那里的秦艽②正在卖着好价钱,我想自己挣回自己的学费。我觉着打镰刀挺费事的。
   父亲并没有反对我去采药。他说,去吧,去干你爱干的事。
   其实,我不知道什么是我爱干的事。比如说父亲,打一把斧头,打一张镰刀,然后抽一锅水烟,睡时喝二两烧酒。这些他都肯定爱干,而且每样都干得从容不迫。我呢?截止那一个秋天,还没有干成一件事。我总是喜欢想入非非。
   我打定了主意去采药。我在离家二十里的山上转悠了三天就没有耐心了。别人总在低头 工作,而我却怎么也找不到药,那些长在灌木中的药材总是与我擦身而过。
   二十里外我似乎听到父亲锻打镰刀的声音。我想,那些刚刚打好的镰刀正被它的主人磨得锋利无比,一张张镰刀正伸向成熟的麦子。
   父亲打完了镰刀,紧接着又开始打造斧头。父亲的斧头总是供不应求。
   我垂头丧气地站在父亲面前,父亲一声不吭,他钳起一把刚淬过火的斧头,在砧子上狠狠啃了两下。
   这时候,我确实该为我自己羞愧了。我不能眼看着自己这把刚出炉的斧头就这样白白地锈掉,然后当废铁处理掉。我总得好好用上两下子,砍出两道新印子。父亲打造了大半辈子钢口很硬的斧头,不能败在我这把斧头上。
   [注释:①淬cuì火:把金属工件加热到一定温度,然后浸入冷却剂(油、水等)急速冷却,以增加硬度。 ②秦艽“jiāo草本植物,可入药。
   1.标题“父亲的斧头”在文中有哪两层含义?
   答:
   2.文中的“父亲”给人印象很深。结合全文,说说他是一个怎样的人?
   答:
   3.文中的语言富有表现力,试从下列两句中任选一句,结合上下文作简要品析。
   ①金黄的麦子都张了口,几乎要叫出声来。
   答:
   ②父亲一声不吭,他钳起一把刚淬过火的斧头,在砧子上狠狠啃了两下。
   答:
   4.文章结尾说“我确实该为我自己羞愧了”,“我”为什么羞愧?”我”今后会怎么做?
   答:
  
   (二十)(2004·常州)
   用 爱 倾 听
   方冠晴
   那段日子,我被楼上楼下的住户折磨得快疯掉了。
   我家住二楼。住我楼下的,是一对下岗夫妇。为了生活,这对夫妇买了一辆破旧的三轮摩托车,每天出去载客,深更半夜才回来。那辆摩托车破旧得像严重的哮喘病人,“突突突”的响声像哮喘病人的咳嗽,不但巨大,而且让人揪心般的难受。每晚,我躺在床上,刚有一点睡意的时候,那辆摩托车就拼命“咳嗽”着回来了,声音攀上楼来,钻进窗内,搅得我睡意全消。
   我楼上的那家住户,不知怎么心血来潮,给女儿买了一支箫。每天天刚麻麻亮,他就逼着女儿练习。那声音呜呜咽咽,毫不连贯,毫无乐感,听在耳里,像鬼哭狼嚎。
   我每晚被楼下摩托车的“咳嗽”搅得没有睡意,早晨又早早地被楼上的箫声“哭”醒,弄得我精神不振,心情烦躁。我想,应该好好与楼上楼下的住户谈一谈。但临到他们的门口,我又犹豫了,谈什么呢?让他们不要再发出噪音?可楼下的那个住户,破摩托车就是他们的饭碗;楼上的那个住户,箫声就是家长对孩子的希望。难道我要他们放弃饭碗放弃希望?我不忍开口,他们也不会答应。
   几经考虑,我决定搬家,搬到一个清静的地方去居住,那样有利于我的写作,也有利于我的健康。我找到一位朋友,诉说了我的苦衷,叫他帮我物色好的住所。朋友笑眯眯的听着,然后问我:“你觉得我居住的环境怎么样?”我说:“就是觉得你这里清静,所以叫你帮我找住的地方。”朋友得意的点点头,说:“好吧,你先在我家里坐一个小时,感受一下。”
   一个小时后,隔壁的阳台上传来一种含糊不清的类似于说话的声音,像原始部落的人用特殊的声音在喊叫,声音刺耳而使人不明所以,让人听了格外不舒服。
   我问朋友这是什么声音。朋友说:“一个9岁的男孩,在学说话。你仔细听听,他说的是什么?”我侧耳倾听,那男孩无疑在重复一句话,但我怎么听都听不明白他在说什么。我猜测说:“他好像在说,羊刚扑倒在地。”朋友哈哈大笑,说:“你错了。他是说阳光普照大地。”说着话,他拉开了通往阳台的门,以使那孩子的声音更大一些。这时我听到,有一位妇女,在不断地纠正那个男孩。妇女说的,正是“阳光普照大地”。但无论妇女怎么纠正,那男孩说的,仍是“羊刚仆倒在地”。
   朋友见我一脸诧异,便解释说:“这孩子是个弃儿,一出生就又聋又哑,所以他的生身父母抛弃了他。是我的邻居将他捡了回来,不但抚养他,而且到处求医问药为他治疗。从他4岁开始,我的邻居就开始锲而不舍地坚持每天教他说话,到他5岁的时候,有一天,他居然开口叫妈妈了,虽然声音那么模糊,但我们都听清了。我的邻居当时就激动得哭了,我们在场的许多人都热泪盈眶。我的邻居含辛茹苦这么多年,终于让这孩子开口说话了,这怎么不让人激动?你听这孩子的声音,很刺耳,很不舒服,那是因为你是用耳朵在听。而我们听这孩子的声音,很动听,很欣慰,那是因为,我们是用爱在听。这世间的许多声音,有动听的,有刺耳的,有美妙的,有聒噪的,这些声音全部入耳,可以让你觉得是一种享受,也可以让你觉得是一种折磨。但如果用爱去听,这世界上,就只有一种声音,那就是,美妙与和谐,让人觉得欣喜和欣慰。”
   我打消了搬家的念头。奇怪的是,再听楼下摩托车的轰鸣,我没觉得刺耳,而是觉得欣慰,这对下岗夫妇今天又有生意了,又有收入了,我为他们感到高兴。而再听楼上的箫声,我也能听到小女孩的进步。
   上帝给了我们耳朵,是让我们能听得到世间所有纷杂的声音;人类给了自己爱心,是让我们将所有纷杂的声音,转换成美妙动听的音乐。想享受美妙的音乐,就要学会用爱倾听。
   (摘自《做人与处世》)
   1.阅读全文,说出“我”倾听到的“美妙音乐”的具体内容?怎样才能做到用爱心去倾听?
   答:
   2.联系上下文,说出下面句子中加点词语的含义或作用。
   (1)我找到一位朋友,诉说了我的苦衷,叫他帮我物色好的住所。
   含义:
   (2)声音攀上楼来,钻进窗内,搅得我睡意全消。
   作用:
   3.如果楼下传来的是搓麻将的聒噪声,楼上传来的是打骂孩子的刺耳声,该怎样正确理解文中画线的句子?
   答:
   4.“我”楼上的那家住户为什么在“每天天刚麻麻亮”时就逼女儿练习吹箫?请就生活中的类似现象谈谈你的看法。
   答:
   5.研读上文中朋友的邻居教小男孩说话的有关内容和下面两则材料,你认为小男孩最终能不能学会与人正常交谈,为什么?
   材料一:海伦·凯勒原先失明,“整天处于黑暗世界之中,感到很痛苦,对任何事物都不感兴趣,缺少强烈的爱”。莎利文老师将手语字母按入凯勒掌心,教她各种物件的名称,将她手指放在自己喉头,听其振动说话。在莎利文老师持之以恒的爱心教育下,海伦终于成为一位作家和教育家。
   (选自初中语文课本第一册)
   材料二:丹是一位失聪孩子,据著名耳科专家杰夫瑞医生诊断,他恐怕是属于永远都无法恢复的失聪者。但富有爱心的露茜修女并没有放弃,坚持让他去听睡莲开放的声音。有一天,当太阳升起的时候,丹终于听到了睡莲花瓣开时的“叭”、“叭”声。 (选自《知音.海外版》)

第十一讲 说明文阅读

   【考查要点】
   在整体上,理解或概括全文的内容,准确抓住全文的中心句,把握说明的对象的特征,辨别与判定说明的顺序,准确地看出全文的结构特点,快速判断文中的说明方法,正确筛选和提炼文中的信息,领会作品中所体现的科学精神和科学思想方法,联系文章与生活实践谈自己的感受、发现或设想,等等。
   在局部或细节上,准确抓住文段的中心句,给不同结构的段落划分层次,要言不烦地对文段的内容、事物的特点进行概括或诠释,品析说明的语言,说明词或句子的含义、词或句子的表达作用及语言的表达特点,根据文章内容对陌生的科技内容进行阐释,对答题的理由进行解说,对文中的有关内容进行创造性的文字“再表达”,阅读图形,辨识表格,解说表格,或者根据文章某些内容设计表格,或画出图形,等等。
   【知识疏理】
   1.说明文中的举例子、列数字、打比方、分类别、作诠释、作比较等说明方法。2.说明文语段中的中心句与文句。3.说明文的文章结构特点与文段结构特点;总说与说。4.说明中限制语的运用,确数和约数。5.说明文语段的层次划分角度。6.空间顺序、时间顺序、逻辑顺序。7.平实与生动的语言。8.说明中的描摹。9.科学小品的特点。10.图表的设计。11.联想与想像。12.一定的生活知识与生活见闻。
   【试题特点】
   主要题型为填写题、简答题、简述题、概括题、图表题,简答题设题最多,比例最大。次要题型为仿写题、判断题、选择题。
   【解题导引】
   例1.(2003年盐城)
   人体内有一种淋巴细胞,它们是人的“生命卫士”,担负着消灭外来入侵病毒的重任,医学上称之为“免疫细胞”。人类二十世纪,这些“生命卫士”遇到了一种强悍凶险的对手——艾滋病毒。这种病毒十分狡诈,一旦侵入人体细胞组织,它们就会用“融合”的手段进入免疫细胞内部。原先的“生命卫士”被迅速异化为病毒感染细胞,并会“走火入魔”发疯似地复制,病人每天大约会产生1000万个变异的病毒颗粒。由于它的高速的变异和复制,病人即使采用抗病毒药物进行剿杀,这些被病毒感染的细胞也马上会针对性地再度变异,使抗病毒药物在一段时间后失去攻击能力,也就是说病人很快会出现“耐药”反应。当人体“生命卫士”的组织系统被瓦解后,病人就会并发多种疾病,直至死亡,所以艾滋病也被称为人体免疫系统缺陷综合症。
   八年前,科学家针对耐药性,变单一抗病毒药物治疗为多种抗病毒药物的配伍联合用药,创造了“鸡尾酒”疗法,虽然有较好抑制病毒的作用,但最终还是不能逃避“耐药”的黑洞。
   最近美国和瑞士的科学家共同研制的一种新型的抗艾滋病病毒药物,有可能使之成为首个能逃避“耐药”黑洞的新型抗艾滋病毒药物。该药的神奇之处在于它能有效阻断艾滋病毒融人免疫细胞内部,就像给人体的“生命卫士”穿上了坚固的防弹衣,使病毒无法进入免疫细胞内部进行“策反”,并无从复制和变异,只能处于“孤立”状态,最终在免疫细胞和抗病毒药物面前束手待毙。这一新型药物已得到美国药品监督管理局的批准,将用于临床。
   1.第1段开头为什么把人体内的一种淋巴细胞称之为人的“生命卫士”?
   2.根据第1段内容,为下图的3个空白处填写恰当的词语。
   3.第3段“只能处于‘孤立’状态”中“孤立”的意思是什么?(依据文中语句回答)
   4.第3段中加点的“有可能”能不能去掉?为什么?
   5.请用简洁的语言概括这篇文章的大意。
   上面用于命题的材料是一篇科技说明文。第1题考查对句子的理解能力,要求对陌生的内容进行阐释,解答时要反复浏览相关内容,找到与题目有关的辅助材料作为答题的依据;第2题考查对段落的理解,采用了图表题,要能通过阅读抓住关键词语作答;第3题考查对句子中关键词语的理解,并要求依据文中语句回答,要细读词语前后的文字;第4题是体会说明文语言的准确性,要求对答题的理由进行解说,此时的解说一要利用平时的积累,二要利用文中的材料,语言表达要准确、精练、层次分明;第5题考查对语言材料的整体理解,要精心地从文章中撷取带实质性的有关内容,精心组合语言,写出简洁、通顺而又合乎题目要求的答案。此题的参考答案为:1.因为这种细胞能消灭外来入侵的病毒。(或:因为这种细胞能使人免疫。)2.免疫细胞(内部)病毒感染细胞组织系统3.指艾滋病毒无法进入免疫细胞内部,也无从复制和变异。4.不能。“有可能”表示情况并不确定,去掉后就变成肯定了。(或:“有可能”体现了说明文语言的严密性,准确地说明了事物的状态,去掉后就不准确了。)5.新型抗艾滋病毒药物给免疫细胞穿上了“防弹衣”。(或:说明了艾滋病病毒侵入人体,致人死亡的过程,以及新型抗艾滋病毒药物的神奇功效。或:新型抗艾滋病毒药物将置艾滋病毒于死地。或:新型抗艾滋病病毒药物能有效阻断艾滋病毒融入免疫细胞内部。)
   例2.(2004·荆门)
   ①人类能在地球上生活多久?这既涉及地球为人类的生存和发展所提供的资源,也涉及地球的外在环境究竟能在多少年内维持不变。
   ②太阳是决定地球外在环境最重要的因素。根据近代天文学家的理论,太阳将持续而稳定地向地球提供光和热,地球绕太阳旋转的平均半径,将长期维持不变,至多只有极小的摆动,这一过程将至少还持续40亿年。过了40亿年后,太阳将逐渐膨胀而演化为红巨星,最后将地球完全吞吃到它的“肚子”里。
   ③太阳所蕴含的能量实在是太巨大了,它为地球持续提供长达400万年的光和热是没有问题的。因为在400万年的时间里,所消耗的能量还不到太阳总量的1%!所以,研究人类在地球上持续生存和发展的问题,至少要以人类能在地球上持续生存400万年为目标!
   ④但是人类面临的真正的威胁,却是来自人类自身。近400年来科学、技术以及工业、农业的发展,就远远超出自有人类历史以来的400万年间的成就。与此同时,近400年来人类所消耗的地球上的资源大大超过了在400万年间人类所消耗的地球资源总量!如果按照现在的消耗不断增长的趋势发展下去,试问400年后乃至400万年的地球将是什么样的面貌?
   ⑤地球上的资源可分为两类:一类是可再生资源,另一类是不可再生资源。如果人类可用消耗可再生资源的办法替代一些不可再生资源,那么这种替代在数量上毕竟是有限度的。毕竟,人类的生存和发展的问题,归根结底取决于地球上的资源能在多少年内按照某些资源的消耗标准维持人类的正常生活。
   ⑥其实,400万年只是一个保守的说法,太阳的光和热,完全可能持续更长一些时间,即使太阳系内出现某些反常事件,如小行星撞击地球,但也不太可能在400万年内发生,而且人们完全能发射有超强破坏力的导弹,使小行星改变航道。地球上的居民,至少在相当长的一个时期内,是大可不必吃咸萝卜操淡心的!
   ⑦所以,人类真正值得忧虑的,是人,是人能否控制人类自身!(《人类能在地球上生活多久》)
   1.阅读全文回答问题:人类在地球上生活时间的长短,取决于什么?
   2.文章第②段中“这一过程”指代的内容是什么,请用波浪线在原文中划出。
   3.文章第③段划横线的句子中能量消耗者应是_________________,这句话强调了________________________。
   4.请用一句话简要概括第③段中划线部分的内容,20字以内。
  
   5.文章第⑤段关联词语使用有误,请改后将正确的填入下面的括号里。
   如果( )……那么( )……毕竟( )……
   6.文章第⑥段中“吃咸萝卜操淡心”一语是个______语,它形象地说明了________
   _______________________________,请写出一个与“吃咸萝卜操淡心”意思相同的成语 。
   7.阅读文章第⑦段,指出人类不能控制自身的含义并举例加以说明。
   含义:_________________________________________________________________
   例子:_________________________________________________________________
   8.仔细推敲文章结构,如果将文中的第⑥段调换位置,你认为应放在哪一段前为宜?
   9.请你以“地球与人”为主题策划一则有创意的公益广告。要求有明确的主题词和具体的画面描写,40字左右。
   上面的第1题考查整体阅读能力,不妨从首段中找出总起句作答:①“地球为人类的生存和发展所提供的资源”;②“地球的外在环境究竟能在多少年内维持不变”。当然,善于自己概括更好,即可答作:①地球资源的多少;②地球外在环境的稳定与否。第2题看看是否明确复指的内容,当划出“太阳将持续而稳定地向地球提供光和热,地球绕太阳旋转的平均半径,将长期维持不变,至多只有极小的摆动”。第3题的判定需承前蒙后,酌指“太阳”,因为“太阳所蕴含的能量实在是太巨大了”。第4题务必考虑到限制的条件,答案并不过死,或曰:人类文明的发展是以地球资源的消耗为代价的,或曰:人类文明的发展与地球资源的关系,关键在于积极地提取主要信息,思维聚合要快。第5题考到关联词语,凭语感可知道前一个复句应为转折关系,由此得到下一句的结论,连贯看来,需改为“虽然……但是……所以……”才较为合适。第6题直击熟语(俗语),熟语自有其妙处,“吃咸萝卜操淡心”如同“杞人忧天”一样,纯属“庸人自扰”,于是很艺术地告诉人们,在相当长的时期内,人类都无须担忧地球的外在环境会对地球产生威胁。第7题是要发掘句子的含义,推敲而言,这个句子说的是人类在消耗地球资源方面不能做到自我约束(无节制、肆意破坏、掠夺性开采……),举例来说,人类无限度地开采石油,无限度地采伐树木等等,就更加使人折服了。第8题关乎文章的结构,商榷的结果还是放在第⑷段前适宜,这也与语义语境关系甚密。第9题借题发挥,由选文的题材生发开,提出创意广告,足以实实在在考核出考生的素质。此题的主观性特强,但无论怎样也不能游离于主题之外。所谓主题,得含两方面的内容,一是要有生动的画面,譬如:黄河已经成了悬河,长江之底已干裂见缝,旁边站立一位饱经风霜的老人,老人脸上挂着一颗硕大的泪滴(特写)……二是要有鲜明的主题词,譬如:如果说地球上还有一滴水,那就是人类自己的眼泪!整个作品务求具有撼人心魄的艺术力量,发人深省,产生必要的社会效应。
   (参考答案:1.地球资源的多少和地球外在环境的稳定与否(或“地球为人类的生存和发展所提供的资源和地球的外在环境究竟能在多少年内维持不变”) 2.指代的内容是“太阳将持续而稳定地向地球提供光和热,地球绕太阳旋转的平均半径,将长期维持不变,至多只有极小的摆动” 3.太阳 太阳所蕴含的能量实在是太巨大了。 4.人类文明的发展是以地球资源的消耗为代价的 或:人类文明的发展与地球资源消耗的关系 5.虽然……但是……所以 6.熟语(或俗语) 在相当长的时期内,人类都无须担忧地球的外在环境会对地球产生威胁。 杞人忧天(或“庸人自扰”等) 7.含义:人类在消耗地球资源方面不能做到自我约束。或:人类无节制地消耗地球资源(肆意破坏或掠夺性开采) 8.第⑷段 9.例:画面中出现黄河、长江的景象,黄河早成了悬河,长江之底已干裂成缝,旁边立一饱经沧桑的老人,老人脸上挂着一硕大的泪滴(特写)。主题词:如果说地球上还有一滴水,那就是人类自己的眼泪!)
   解答说明文时:1.先读阅读题,了解所考查的内容,判断其难易。2.阅读选文,反复读两遍,了解文章内容、理清说明顺序,初析文章结构,辨识说明方法,理解事物特点。3.读文并答题,可“顺流而下”,也可“先易后难”。先在心中默答一遍,然后再答在试卷上。
   【常见失误】
   说明文常见的答题误区有:不能准确地对词语的限制、修饰作用进行评析,不能掌握根据语言材料给事实下定义的方法,不能熟练地运用表格等方式表现或整理文中的重要信息,不能对事物的特点进行准确的概括,在阅读图形和辨识表格上有困难,解说答题的理由有困难等等。要针对说明文阅读中的这些难点进行有的放矢的复习。
   【考点精练】
   (一)(2004·海淀)
   成功登陆火星的“勇气”号探测器
   ⑴2004年1月4日,带着人类探索太空奥秘的千年梦想,“勇气”号火星探测器经过半年多的飞行,成功登陆火星。这对于了解火星及其演化,了解地球自身的过去和未来,都有十分重要的意义。
   ⑵火星是地球外侧的比邻星,距离地球十分遥远,探测器登陆火星远比登陆月球艰难得多。月球表面几乎没有大气,制动火箭可以引导登月舱准确地进行软着陆;而火星大气的密度虽然远远低于地球大气的密度,但仍可使登陆舱在着陆的数秒钟内烧毁。同时,火星风也能把登陆舱吹离目的地。因此,航天专家们给火星起了个绰号叫“死亡行星”,意指探测火星非常艰难,稍有疏忽就会前功尽弃。
   ⑶美国科学家总结了四十多年来国内外火星探测的成功经验,并汲取了多次登陆失败的教训,采用了多种技术措施相结合的方案,终于使“勇气”号成功登陆。
   ⑷“勇气”号是迄今人类遣往其他行星的第一个可以移动的、自动化的大型实验室。其实,它就是一台车式机器人,长1.6米,宽2.3米,高1.5米,重174公斤,靠太阳能电池板获得能源。它有自己的大脑、颈、头、眼睛和手臂。
   ⑸“勇敢”号的“大脑”是一台每秒能执行约2000万条指令的计算机。不过,与人类不同的是,它的大脑并不在头部,而在腹部。所谓“颈”和“头”,是指“勇气”号上部伸出的一个桅杆式结构,上面有一对“眼睛”——其实是两台可拍摄火星表面彩色照片的全景照相机。(甲)两台相机的高度与人眼高度差不多,使“勇气”号能以类似人的视角环视四周,这儿瞧瞧,那儿看看,寻找有价值的探测目标。(乙)当发现值得探测的岩石等目标时,“勇气”号会像地质学家一样,走上前去对岩石敲敲打打,搜集数据,然后向下一个目标前进,因此科学家们亲切地称其为“野外地质学家”。
   ⑹“勇气”号配备了野外科学考察所需的“全套武装”,其中包括全息摄像机和显微镜成像仪。“勇气”号上有多台高清晰度摄像机,可以拍摄高分辨率的火星地表和天空影像。而显微镜成像仪就像地质学家手中的放大镜一样,能以几百微米的超近距离对火星岩石纹理进行审视。探测器还配备了一个相当于地质学家常用的小锤子式工具,能对岩石内部进行采样研究。另外还有一些先进仪器,可用来对岩石构造做进一步分析。
   ⑺“勇气”号火星探测器拥有一套独特的通讯系统。借助这套系统,“勇气”号可以直接与地球控制中心交流信息。
   ⑻火星上是否存在生命,一直是人们非常关心的问题。“勇气”号也将围绕此问题进行有益的探索。它肩负着两大使命,一是用先进的生化手段探测火星上是否存在生命,二是探测火星上是否有水。
   ⑼“勇气”号成功登陆火星,为人类进一步探索银河系打开了一扇希望之门:不过,这条探索之路仍是漫长而艰难的。尽管在这条道路上随时都可能遭受挫折,但是人类绝不会停止探索宇宙奥秘的脚步。目前,航天专家们正计划在发射无人探测器的基础上,选派航天员登陆火星,对这颗神秘而美丽的红色星球做进一步的科学探索与研究。
   1.请你根据文章提供的相关信息,用一句话概括介绍“勇气”号探测器。(答案不超过42个字)
   答:
  
   2.文中对“勇气”号的结构、装备及其功能进行说明的是第 段到第 段。
   3.既然火星被称为“死亡行星”,登陆非常困难,为什么人类还要坚持对火星的科学探索?请根据文章内容简要写出四点理由。
   答:
  
   4.形象、生动是这篇科普短文的语言特点之一,请你以第⑸段中画线的(甲)(乙)两个句子的一个为例,进行具体分析。
   答:
  
   5.“勇气”号这一名称是从参加火星探测器命名比赛的一万份美国小学生作文中挑选出来的,选中文章的作者是一名9岁的小女孩。人们为什么把为火星探测器命名这样神圣的使命交给孩子们呢?为什么“勇气”号这一命名能够中选呢?谈谈你对这两个问题的理解。
   答:
  
   (二)(2004·大连)
   不可忽视的土壤污染
   ⑴土壤、水、阳光和空气是大自然赋予人类和其他生物生存的四大要素。一般的污染马上就能引起大家的关注,而土壤污染引起的不良后果要在几年、几十年甚至上百年后才能显现出来。所以,人们常常忽视对土壤污染的整治。
   ⑵近年来,由于人口的急剧增长和工业的迅猛发展,固体废物的倾倒和堆放量日益增多,有害废水不断向土壤中渗透,大气中的有害气体和漂浮的尘土也不断随雨水降落在土壤中,导致了土壤污染。
   ⑶土壤污染所造成的危害有以下几个方面:第一,土壤污染导致严重的直接经济损失,对此目前尚缺乏系统的调查资料。仅以土壤重金属污染为例,我国每年因重金属污染而减产的粮食就有1000多万吨,被污染的粮食每年也多达1200万吨,合计经济损失至少200亿元。第二,土壤污染导致食物品质不断下降。我国大多数城市近郊土壤都受到了不同程度的污染,许多地方的粮食、蔬菜、水果等食物中的镉、铬、砷、铅等重金属含量超标和接近临界值。第三, 。土壤污染会使污染物在植物体中积累,并通过食物链富集到人体和动物体中,危害人畜健康,引发多种疾病。另外,受到污染的土壤表土在风力和水力的作用下,进入到大气和水体中,导致大气污染和水体污染。
   ⑷为了控制和消除土壤污染,科学家们提出了许多解决办法:首先要控制和消除污染源,加强对工业“三废”的治理。其次是采用生物(例如蚯蚓等)降解净化土壤。还可以通过增施有机肥、换土和深翻等手段,治理土壤污染。
   ⑸近年来,世界各国的环保专家和生物学家都提出了让植物来净化土壤的新方案。他们培养出各种转基因植物,让它们吸收土壤中的有害物质,然后集中起来焚烧处理,这样,经过植物吸收后的重金属还可以提炼出来,变废为宝,不但净化了土壤,还获得了贵重的重金属。与传统的化学、物理等除污手段相比,植物除污具有投资和维护成本低、操作简便、不造成二次污染、经济效益明显等优点。
   1.给文中加点的字注音。
   渗 透
   2.本文的说明内容除了土壤污染不被重视、土壤污染的原因和危害之外,还有
   。
   3.第⑶段中加点的词“至少”在句子中的作用是: 。
   4.填充下面方框,完成植物净化土壤的过程。(每空6个字)
   → → →
  
   5.下面两题,任选一题回答。[如两题都答,按第①题计分]
   ①请在下边给第⑶段空白处补写一句话。
   答:
   ②第⑷段中列举的治理土壤污染三种办法的顺序为什么不能颠倒?
   答:
   6.为了提高人们对土壤及环境的保护意识,请你设计一条公益广告。(含标点20字以内)
  
  
   (三)(2004·包头)
   黄河安澜的今天和明天
   ⑴继上个世纪八九十年代以后,黄河屡屡发生的断流让人揪心,曾经那样奔腾咆哮、一泻千里的黄河,连维系自身流淌的水量都成了问题。
   ⑵正当黄河多年“汛期无汛”,防汛就要进入历史教科书之际,2003年秋季,持续两个月的降雨,造成黄河下游9处漫滩,十几万人受灾,让人们又一次领教了黄河的桀骜不驯。
   ⑶今天的黄河究竟河情如何?怎样应对?这些问题成为人们再次关注的热点。
   ⑷水少沙多是黄河的老问题。黄河多年平均天然径流量为580亿立方米,仅相当于长江的1/17,而含沙量世界第一。这一特性造成了黄河下游河床不断抬升,大堤不断加高,形成地上悬河。
   ⑸近一二十年来,随着两岸工农业及社会经济各项事业的发展,各方面对水资源的需求不断增大,黄河早已不堪重负。由于水量不断减少,黄河基本的生态用水得不到保障,河水在大堤内只形成了窄窄的大道,于是在过去“地上悬河”的基础上,现在局部又出现了大堤内河槽高于滩地的情况,形成“二级悬河”,近几年,这一情况变得越发严重。
   ⑹黄河的泥沙淤积和河床的逐年抬高始终是问题的症结所在。然而,现在黄河又面临着“生产堤”的新问题。
   ⑺曾经是“三年两决口”的黄河,在新中国建立后的50多年里,实现了黄河大堤的安然无恙。正是由于这样的成就,使得全社会防洪这根“弦”有所松懈,几十年内,在河南、山东两省黄河大堤与河道间的滩区,现已居住着180万人,有耕地375万亩。滩区群众为了保护人口和耕地的安全,修建了“生产堤”,而“生产堤”一旦决溢,在“二级悬河”情况下,高水位的洪水横流就会直冲大堤,对大堤的安全造成严重影响。
   ⑻按照专家的观点,“造成今年黄河下游水灾的水量根本称不上洪水”,只能称为“小水”,可为什么小水也能酿大灾?这其中,“生产堤”的存在是重要原因。“生产堤”经过长时间雨水冲泡以后,极易决溢,因此,经历了2003年黄河秋汛之后,“生产堤”是留存还是消除,再一次成为各方议论焦点。从黄河的长治久安来说,黄河应该利用难得的汛期洪水,加强对河道的冲刷,带走泥沙;从眼前来说,保住滩区群众生命财产安全也是大事。
   ⑼2003年黄河秋汛,雨量大,持续时间长。针对这一情况,黄河防汛总指挥部决心充分运用工程手段,进行一次综合目标调控的尝试。
   ⑽为了降低洪峰流量,首次进行了三门峡、小浪底、陆浑和故县等干支流水库的联合调度,使花园口多次6000多个流量的洪峰被削为2500个,“四库联动”使数亿立方米的洪水成为名副其实的“小水”,有效地缓解了下游防洪压力。同时,这又是一次水沙的联合调度。即便是场“小水”,通过科学调度水沙关系,让水带沙的作用发挥到最大,也能起到对中下游河道的冲刷和减淤作用。经过此次黄河秋汛,黄河下游河道普遍刷深30~40厘米,过流能力增大了100~400个流量,这对黄河来说,是一个喜讯。对实现黄河河床20年不抬升的规划也是一个极大鼓舞。
   ⑾在2003年抗洪过程中,局部救灾和整体防洪的矛盾仍很突出。在这次抗洪中,2001年投入使用的小浪底水利枢纽发挥了决定性作用,但按照水利专家的的意见,此次小浪底工程已是非常规调度使用:一方面,洪水持续不断发生,下游险情不断加重,要求小浪底水库多拦洪;另一方面,小浪底水库是新建土石坝,不能一次性蓄洪过快,否则坝体稳定将受到冲击;同时,高水位运用会加快水库淤积速度,缩短水库使用寿命。令人担忧的是,水库小水大用,今后真来了大水怎么办?专家认为只有将滩区群众全部撤出才是根本解决问题的途径。
   ⑿2003年黄河秋汛的水库调度可谓“精细”,上亿立方米库容的水库,水位调度精确到了厘米。水文测报数据表明,秋汛期间,黄河防总对花园口下泄流量的控制误差不超过10%。这一切,与黄委会正在建设的“数字防汛”紧密相连。在黄河防总会商室,一条数字黄河展现在墙上,原型黄河上已经和正在发生的汛情、险情、灾情、河势工情以及上堤抢险等情况,只要轻点鼠标就会一一呈现。虽然“数字防汛”系统在2003年黄河秋汛中发挥了重要作用,但“数字防汛”仅仅是“数字黄河”工程的第一步,今后要加快建设“数字黄河”工程,这才是推进黄河治理开发与管理现代化的必由之路。
   1.第⑶段的两个问句在文中起的作用是
   2.右侧是“悬河”示意图,读⑷~⑺段后,将下列名称填写在相应的括号内(只填字母)。
   A.黄河大堤 B.二级悬河 C.生产堤
   3.概括说明黄河今天的河情(最好不超过22个字)。
  
   4.第⑽段运用列数字的说明方法是为了说明什么?
   答:
   5.第⑾段“非常规调度使用”的意思是
   6.为了祖国母亲河的长治久安,在明天的治理中,该注意些什么?读⑾⑿作答。
   答:
  
   (四)(2004·河北)
   走近森林
   张志毅
   ①森林是以树木为主体的许许多多生物组成的生物群落。森林里蕴藏着丰富的资源,森林资源按自然属性可划分为生物资源和非生物资源两大类。其中生物资源又可分为植物 资源、动物资源和微生物资源三类。植物资源包括林木资源(乔木、灌木和竹子)和非林木资源(藻类、地衣、苔藓、蕨类和其他种子植物等);动物资源主要包括哺乳动物、爬行动物、森林昆虫、鸟类和鱼类等;微生物资源主要包括各种菌类、支原体、衣原体等。非生物资源主要是指支撑森林生物的林地土壤、水分等资源。可见森林是地球上一个丰富多彩的大资源库,而且其主要部分即森林生物资源部分,是可以持续利用的可再生资源。
   ②森林资源是人类赖以生存的基础资源。同时,森林还具有维护地球生命、改善人类生存环境的生态价值。
   ③森林可以给人类源源不断地提供木材产品和林副产品。木材产品主要包括原木、锯材、纸浆材、人造板材等;林副产品主要包括森林植物的叶、花果、茎、树皮、树脂、树胶、树液等,还包括经济林以及森林动物与微生物提供的各种产品等。为人类提供这些产品等。为人类提供这些产品无疑是森林的重要功能。
   ④森林是生态平衡的主要调节器。森林可以把太阳能转化为化学能,使无机物转变为有机物,是生物和非生物之间物质和能量交换的重要(niǔ) 带,对保持生态系统的整体功能起着中枢和杠杆作用。可以说没有森林就没有生态平衡。
   ⑤木森林能够有效地涵养水源和防止水土流失。森林凭借庞大的林冠、深厚的枯枝落叶层和发达的根系,能够起到良好的蓄水保土和减轻地表侵蚀的作用。据测算,5万亩森林的蓄水量约有一百万立方米。在森林被破坏或无森林的地区,水土流失严重,许多河道和水利设施不断受到泥沙淤积,经常造成水灾。
   ⑥森林能够有效地保护生物多样性。目前地球上的500万~3000万物生物中,有一半以上是在森林中(qī) 息繁衍。
   ⑦森林能够有效地缓解“温室效应”。由于近代人类大量使用石化燃料,使大气中二氧化碳、甲烷等温室气体浓度不断升高,引起地球上的“温室效应”。每公顷森林平均每年可吸收20~40吨二氧化碳,同时放出15 ~ 30吨氧气。
   ⑧                。据有关专家研究,一公顷森林平均每年能吸引700多千克的二氧化碳,可减轻工业酸雨的危害;城市中路旁的林带可以阻挡沙尘,滞尘率高达70%~90%。同时林带还有降低噪声的作用,噪声经过30米宽的林带可降低6~8分贝。
   ⑨森林能产生直接的经济效益,同时又有巨大的生态环境效应,这是任何其他东西都无法代替。
   1..根据注音写出汉字。
   ①(ni ǔ) 带 ②(qī) 息
   2.文章是从哪两个方面对森林的价值进行说明的?
   答:    
   3.根据对第①段的理解,在下面的横线上填写相应的内容。
   生物资源:植物资源、①      、微生物资源
   森林资源
   ②        :林地土壤、水分等 
   4.文章第⑤段中画线句子列举数字进行说明有什么作用?
   答:                                 
   5.第⑦段画线句子中的“缓解”改成“消除”可以吗?为什么?
   答:                                                                           
   6.文章第⑧段画线处应填写的表明这段文字说明中心的句子是什么?请写在下面。
   答:                                     
   7.据报道,目前我国土地沙化正以平均每年2460平方公里的速度扩展,相当于一个中等县的土地面积。防治土地沙化势在必行。请你结合森林的作用,为治理土地沙化拟一条标语。要求语言简洁,有感染力。
   答:

 (五)(2004·太原)
   奇妙的对称世界
   许多大哲学家、大思想家和大科学家都相信,宇宙是被一种无比完美的对称规律支配的。
   整个动物世界,最明显的特点是躯干部分两侧的对称性。以一匹马为例,通过鼻子到两腿中间可以作一条中轴线,在其两侧有完全对称的器官:眼睛、耳朵、鼻孔、腿……动物为什么会演变出这种对称性呢?大家知道,任何动物在其所处的环境中,左和右两面的情况是基本相同的,它们为了更好的适应环境,需要在两面都能同样的看、听、嗅、触摸,于是就形成了这种对称性。
   艺术家则利用对称创造出美。铜器、漆器、雕刻、壁画、织锦、刺绣……其中的图案,一概少不了对称。在被称为“立体的画”、“凝固的音乐”的建筑艺术中,也留下了对称的足迹。我国的建筑,从古代的宫殿到近代的一般住房,绝大部分是对称的,故宫是其中的典范。从天安门到端门、午门形成了一条中轴线,各种各样的建筑都围绕这条中轴线铺开。三大殿更是依据对称的原则而建,整体形成了一种端庄凝重、气势恢宏的美,传达着王权的威严。 为什么对称就美呢?万花筒里杂乱无序的碎玻璃片并不美,奥妙就在于三片反光镜构成了三重反射对称,使得杂乱无序的彩色碎玻璃片经过镜面反射后,形成对称的美丽图案。可见,对称的美在于:在杂乱中形成规律,在无序中引入秩序。
   对称的现象引发了科学家对未知领域的探索。我们知道原子内有原子核,核外有电子。电子质量很微小,带有负电;原子核内还有质子,质子的质量要比电子大得多,却带正电。这种情况是不大相称的。那么,会不会存在着带正电荷的“电子”,带负电荷的“质子”呢?1932年,人们果然发现了带正电荷的电子——反电子。后来又发现了反质子、反中子。总之,粒子和反粒子的对称,已经是千真万确的事实了。
   那么,会不会存在反氧、反氢、反水,以至于反行星、反宇宙呢?科学家正是运用对称原理,在进一步提出和探索这些问题呢。
   1.文章从哪三个方面说明“宇宙是被一种无比完美的对称规律支配的”?
   ①
   ②
   ③
   2.文中画线处所举万花筒的例子的作用是什么?
   答:
   3.对称可以创造美;同样,不对称也可以创造美。自由选用下面资料,结合文章内容和自己的理解,说说这两种美的差异。至少使用两种说明方法。
   资料①:苏州园林可绝不讲究对称,好像故意避免似的。东边有了一个亭子或者一道回廊,西边决不会来一个同样的亭子或者一道同样的回廊。……池沼或河道的边沿很少砌齐整的石岸,总是高低屈曲任其自然。 (选自《苏州园林》)
   资料②:右图是我国古代艺术瑰宝——马踏飞燕。飞奔的骏马只有一只蹄踏在飞燕上,四蹄姿态各不相同,打破了对称。铜马足下的飞燕用来衬托良马之神速。
   答:
  
  
  
   4.对偶就体现了语言的对称美。请你使用对偶的方法,补全《西游记》第七回的回目。
   第七回回目: 五行山下定心猿
   【帮帮你】“五行山下定心猿”指孙悟空被如来佛祖压在五行山下,“心猿”即孙悟空。第七回前半部分内容精选:真个光阴迅速,不觉七七四十九日,老君的火候俱全。这一
   日,开炉取丹。那大圣双手捂着眼,正自揉搓流涕,只听得炉头声响。猛睁睛看见光明,他就忍不住,将身一纵,跳出丹炉,唿喇的一声,蹬倒八卦炉,往外就走。
  
   
   (六)(2004·南京)
   ①你见过一个能炒一盘菜的巨型青椒吗?你尝过长1米、重15 00克的特大黄瓜吗?你听说过一亩地能产一吨优质稻谷吗?这些农作物并非神话而是事实,它们的种子都有一个共同的特点,即经过太空之旅后而变得神奇无比。到太空中去培育更多更好的良种成为人类的新希望。空间生物学的发展为人类的这一希望奠定了基础。
   ②在太空,物体的重量只有地面的百万分之一,被称为微重力状态,生物极易发生遗传变异,对细胞、蛋白质、微生物的培养十分有利。太空还剥去了大气层的“外衣”,太阳光辐射比地面强得多,宇宙高能重粒子极其丰富,将使种子、微生物等细胞中的遗传密码发生有益突变,产生新物种。另外太空环境近似真空,没有污染。专家们认为,看不见、摸不着、甚至感觉不到的这些环境条件,为发展新材料、新物种、新医药等提供了理想的实验场所和生产基地。在太空环境里,生物的变异和进化要比地面快成千上万倍。
   ③率先提出空间植物和微生物变异理论的美、俄科学家,多年来却与航天育种这一领域失之交臂,捷足先登的是“太空俱乐部”的后来者——中国。
   ④我国返回式卫星和宇宙飞船的发射成功,为空间育种和生物学科研提供了契机。早在 1986年,王大珩教授就提出运用航天搭载进行生物科学研究的建议。1987年我国首次利用卫星进行了植物种子和微生物材料的搭载试验,它拉开了我国航天育种研究的序幕。
   ⑤十多年来,我国先后在9颗卫星和两艘宇宙飞船上搭载了500多种植物、微生物等,培育出一批生物新品种、新品系、新种源。我国的空间育种成果赢得了世界的赞誉,美、俄、保加利亚等国科学家们纷纷表示了要求合作的愿望。
   ⑥1996年10月,经“杂交水稻之父”袁隆平院士精选处理的杂交水稻种子被放入返回式卫星,在太空中“修炼”了15天后返回地球。这些种子在海南繁育基地种植了44987株。经实验分析发现,一些水稻发生了高频率的变异,个别植株的性状分离变异率高达12%,比用射线辐射变异率高100倍,比自然变异率高数万倍。如今这批航天水稻已经进入第六代,形成了新品种,单季亩产达到800公斤。
   ⑦据不完全统计,我国空间育种的总种植面积已超过100万亩。
   1.本文的说明对象是 。
   2.第②段加点的“这些环境条件”具体指哪些?
   答:①
   ②
   ③
   3.请用简明的语言概括④一⑦段的主要内容。(不超过15字)
   答:
   4.文章开头加点的部分如果去掉,表达效果有什么不同?
   答:
  
   (七)(2004·连云港)
   谁染枫林醉
   苏更林
   (1)唐代诗人杜牧的《山行》以形象、明快、洗练的语言,勾勒出层林尽染、枫叶流丹的画面,并吟诵出“霜叶红于二月花”的千古绝唱。
   (2)为什么晚秋的枫叶会变得满山锦绣、遍地彩霞呢?宋人杨万里有诗云:“小枫一夜偷天酒,却倩孤松掩醉容。”说是枫叶在一夜间偷饮了“天酒”而“醉”成了红色。这虽然为枫林红叶增添了几分神秘和浪漫,但并没有道出枫叶变红的确切道理。
   (3)到底“谁染枫林醉”呢?现代科学认为,植物叶片的颜色取决于其中所含的色素物质,如叶绿素、叶黄素、花青素、胡萝卜素等等。这些色素物质往往会随着植物成熟期的不同和环境条件的改变而发生变化,从而呈现出各自具有鲜明特征的颜色。叶片中含有较多叶绿素时就会呈现绿色,含有较多叶黄素时就会呈现黄色,而含有较多的花青素时则会呈现紫、蓝、红等不同的颜色。这是因为花青素是一种水溶性植物色素,独特的化学结构使其颜色多变。花青素的基本结构母核是花色基元。由于其中的氧原子是4价的,所以它和它的衍生物具有碱的性质,即能够与酸性物质作用生成盐;而花青素又是花色基元的羟基取代衍生物,因而,它又具有酸的性质,即能与碱性物质作用生成盐。花青素的这种结构特点决定了它具有随介质pH值改变而改变结构并从而改变颜色的可能性。试验证明,花青素在酸性介质中呈现其表征的红色。而枫树的叶片细胞液呈现酸性,春夏季节由于枫叶中一般只含有叶绿素而不含有花青素,所以此时的枫林还是郁郁葱葱的绿色;可到了深秋季节,由于气温骤降,光照减少,使得叶片内光合作用制造的淀粉不能完全适合叶片,这样就造成叶细胞内糖分的积累,从而促进了花青素的形成。由于枫叶细胞液的酸性环境,花青素呈现红色也就是顺理成章的事了。
   (4)“霜叶红于二月花”体现了“枫林红叶”外在美和内在美的统一。如果你一旦置身于枫林之中,一定会为祖国的大好河山所陶醉,并为“霜叶”抗严寒傲霜雪的生机和力量而感动。
   (选自《语文报》,有删改)
   1.第(2)节中,杨万里的诗并没有“道出枫叶变红的确切道理”,作者为什么还要引用这两句诗?从文中可以看出枫叶变红的主要原因是什么?
   答:
   2.花青素是一种水溶性植物色素,其结构特点是什么?
   答:
   3.枫叶中的花青素是如何形成的?(不超过50字)
   答:
   4.大自然中的植物呈现出万紫千红的景象的主要原因是什么?
   答:
  
   (八)(2004·宿迁)
   人能与鸟比翼齐飞吗
   ①人类自古以来就向往能像鸟一样在蓝天上翱翔。据《后汉书》记载,我国在1900多年前就有人用大鸟的羽毛制造成翅膀试验过飞行。直到17世纪,欧洲还有人试图这样模仿鸟类飞行,但都未能成功。这是为什么呢?
   ②经过长期研究,人类终于弄明白:人的肌肉能量与鸟类比较,相对来说要小得多,因此靠双臂来扇动翅膀,是无法克服自身重量进行有效飞行的。
   ③扑翼飞行的失败,并不能阻挡人类的探索。1783年,人们利用轻于空气的飞行器——气球,第一次升上了天空。随着生产技术和科学的发展,人们又在重于空气的飞行器方面探索,开始也进行过扑翼机的研究,但是由于鸟类飞行时翅膀的运动十分复杂,限于当时的生产技术水平,还不能提供轻而坚固的材料和高效率的动力,这种尝试仍未成功。后来,还是鸟类的翱翔(不需扇动翅膀)和风筝的飞行给人以启示,人们终于采用固定机翼的形式,于1903年发明了世界上第一架有动力的载人飞机,实现了飞行的理想。
   ④从那以后,航空事业有了飞速的发展。今天,千姿百态的飞机飞向蓝天,为国防和国民经济服务着,乘坐飞机已成为极为普通的事了。不过,人们感到乘坐飞机并不能享受在蓝天自由飞翔的乐趣。于是,许多航空爱好者绞尽脑汁进行扑翼飞行的研究。不久前,科学家们制成一种“机械鸟”,它以一台小电动机为动力,扇动两只3米长的涤纶薄膜翅膀,便可轻盈地飞向天空。这种扑翼机可用于体育运动,也可用于军事侦察和通讯联络,引起各国航空界的极大关注。我们相信,在科学技术高度发展的今天,人类一定能实现像鸟一样在广阔的蓝天中自由翱翔的美好愿望!
   1.填空。
   ①第②节中运用了 说明方法;
   ②本文是按 顺序说明的。
   2.细读第④节,根据文章所提供的信息简要回答。
   ①“机械鸟”的飞行方式是:
   ②今天航空事业飞速发展的根本原因是:
   3.揣摩下列句中加点词语的作用。
   于是,许多航空爱好者绞尽脑汁进行扑翼飞行的研究。
  
   ②这种扑翼机可用于体育运动,也可用于军事侦察和通讯联络,引起各国航空界的极大关注。
  
   4.请你谈谈读了本文后得到的启示。
   答:
                                          
  
   (九)(2004·杭州)
   细菌和病毒都是可以致病的微生物,但它们的特征区别很大。细菌虽然小,要在光学显微镜下才能看得见,但它除了拥有生命的基本单位核酸之外,还有一大套赖以生存的配套设施。包括作为居住“公馆”的细胞壁,储存营养物质的“仓库”,以及进行新陈代谢的“化工车间”。依靠这些,细菌能够摄取外界的物质并加工成需要的能量。
   而病毒就更小了,也可怜得多,且不奢谈“库房”和“车间”,就连作为保护外壳的“茅草房”也没有。实际上它只有一个分子大小,用电子显微镜才能看得见。整个家当也只是一条表示生命的核酸而已。如果拿人来做比喻,细菌最起码也有条裤子,有只讨饭碗,有根打狗棒。所以细茵虽然必须在人体内部的良好环境中才能繁殖,但处在恶劣环境中仍能生存较长的一段时间。而病毒则像个刚出生的婴儿,除了它的生命和一张吃奶的嘴外便一无所有,毫无独立生存的能力。因而病毒只能寄生在人或动物的细胞内部,靠“窃取”细胞里的现成营养才能生存。一旦被排出体外,病毒就活不了几小时。
   大部分抗生素对细菌起作用,是因为抗生素可以抑制细胞繁殖,干扰它们形成新的遗传结构或者细胞壁。而正因为病毒只能寄生在别人的细胞内,自己不能完成这些生化反应,所以抗生素对病毒全无作用。
   病毒的生存能力既然这么弱,为什么还会那样猖獗呢?例如埃博拉的感染力极强,病死率可达80%。实际上大多数病毒远没有那么可怕,有的也不会使人得病。冠状病毒本来致病的能力并不强,问题在于“变种”上。变异的病毒和原来的不同了,它可能是无害的,但也可能变成“杀伤力”更大的病毒。然而我们不必担心它会因反复变异而使杀伤力次第增大。因为就杀伤力而言,变异就像赌博,总是有输有赢,不断地赢下去的几率是微乎其微的。至少有史以来还没有过这样的记录。如果有过,可能现在就没有我们的存在了。
   必须指出的是杀伤力的大小不仅取决于病毒一方,更重要的是人群的免疫力。变种冠状病毒之所以为患甚大,很重要的一个因素是人类还没有接触过这样变种后的新病毒。当人群有足够多的人产生了抗体之后,这种病毒对人类的威胁也就小得多了。
   1.文章多处运用比喻说明的方法。下列比喻分别具有什么作用?
   ①(病毒)且不奢谈“库房”和“车间”,就连作为保护外壳的“茅草房”也没有。
   答:
   ②变异就像赌博,总是有输有赢,不断地赢下去的几率是微乎其微的。
   答:
   2.文章从哪几方面把病毒和细菌作比较?请分点概括回答。
   答:
   3.根据病毒的特征,给病毒下一个定义。(不超过35个字)
   答:
   4.本文说明的中心是什么?请简要回答。
   答:
  
   (十)生物入侵者
   梅涛
   ①当你在路边草地或自家庭院里发现一两只从未见过的甲虫时,你肯定不会感到惊讶。但在生物学家和生态学家们看来,这或许不是件寻常小事。专家们把它们形象地比喻为“生物入侵者”——它们不仅会破坏某个地区原有的生态系统,而且还可能给人类社会造成难以估量的经济损失。
   ②在人类早期社会,陆路和航海技术尚不发达,自然界中的生态平衡并没有受到太大破坏。在自然条件下,一颗蒲公英的种子可能随风飘荡几十千米后才会落地,如果各种条件适合。它会在那里生根、发芽、成长;山问溪水中的鱼虾可能随着水流游到大江大河中安家落户……凡此种种,都是在没有人为干预的条件下缓慢进行的,时间和空间跨度都非常有限,因此不会造成生态系统的严重失衡。
   ③如果一个物种在新的生存环境中不受同类的食物竞争以及天敌伤害等诸多因素制约。它很可能会无节制地繁衍。1988年,几只原本生活在欧洲大陆的斑贝(一种类似河蚌的软体动物被一艘货船带到北美大陆。当时,这些混杂在仓底货物中的“偷渡者”并没有引起当地人的注意。它们被随便丢弃在五大湖附近的水域中。然而令人始料不及的是,这里竟成了斑贝的“天堂”。由于没有天敌的制约,斑贝的数量便急剧增加,五大湖内的疏水管道几乎全被它们“占领”了。到目前为止.人们为了清理和更换管道已耗资数十亿美元。来自亚洲的天牛和南美的红蚂蚁是另外两种困扰美国人的“入侵者”,前者疯狂破坏芝加哥和纽约的树木,后者则专门叮咬人畜,传播疾病。
   ④“生物入侵者”在给人类造成难以估量的经济损失的同时,也对被入侵地的其他物种以及物种的多样性构成极大威胁。二战期间,棕树蛇随一艘军用货船落户关岛。这种栖息在树上的爬行动物专门捕食鸟类,偷袭鸟巢,吞食鸟蛋。从二战至今,关岛本地的11种鸟类中已有9种被棕树蛇赶尽杀绝,仅存的两种鸟类的数量也在与日俱减,随时有绝种的危险。一些生物学家在乘坐由关岛飞往夏威夷的飞机上曾先后6次看到棕树蛇的身影。他们警告说,夏威夷岛上没有任何可以扼制棕树蛇繁衍的天敌,一旦棕树蛇在夏威夷安家落户,该岛的鸟类将在勃难逃。
   ⑤许多生物学家和生态学家将“生物入侵者”的增多归咎于日益繁荣的国际贸易,事实上许多“生物入侵者”正是搭乘跨国贸易的“便车”达到“偷渡”目的的。以目前全球新鲜水果和蔬菜贸易为例,许多昆虫和昆虫的卵附着在这些货物上,其中包括危害性极大的害虫。如地中海果蝇等。尽管各国海关动植物检疫中心对这些害虫严加防范,但由于进出口货物数量极大,很难保证没有漏网之“虫”……
   ⑥一些生物学家们指出,一旦某种“生物入侵者”在新的环境中站稳脚跟并大规模繁衍,其数量将很难控制。即使在科学技术高度发达的今天,面对那些适应能力和繁殖能力极强的动植物。人们仍将束手无策。
   ⑦目前,世界上许多国家已开始认识到这一问题的严重性,并采取了相应措施。
   (有删改)
   1.结合语境,解释文中加点的词语。
   (1)归咎:
   (2)束手无策:
   2.这是一篇科学小品文。品读后,请说说在人类文明的早期,为何不会造成生态系统的严重失衡?文中通过介绍“生物入侵”的现象,告诉人们“生物入侵”的不良后果。请写出这些不良后果。再根据文章内容,结合自己的理解,给“生物入侵者”下个定义。最后。请举例说明这篇文章的语言特点。
   答:
   3.阅读下面两则材料,请联系上文,并就生物物种迁移问题,谈谈你的看法。
   材料(1):一些专家主张人类不该干预生物物种的迁移过程,因为一个物种在新的环境中必然遵循物竞天择的法则,“生物入侵者”能够生存下来的虽然是强者,但它也会受到各种因素的制约。自然界的平衡终究会实现。
   材料(2):2003年,我国外来有害生物入侵问题日益突出,紫茎泽兰、豚草等在辽宁、云南等地蔓延,累计造成1.5亿亩森林受害。这些地方正在全面围剿外来入侵生物。
   答:____________________________________ ________________________
   ________________
  
   (十一)(2004·福州)
   “空气长寿素”——负氧离子
   每当我们来到原野,漫步海边或走进森林的时候,总感到那里的空气特别的新鲜,浑身充满了轻松的感觉。这是什么原因呢?原来,这些地方的空气中含有较多的“长寿素”——负氧离子。
   负氧离子是一种带电的微粒。早在一百年前,科学家们就发明了空气带电现象。空气之所以带电,就是因为其中含有负氧离子。那么,负氧离子又是怎样形成的呢?我们知道,大气中时常发生高压放电现象——闪电,就在闪电发生的瞬间,其周围的空气即被电离,从而产生了大量的负氧离子。
   研究表明,树木、花卉释放出的芳香挥发物质具有增加负氧离子的功能。此外,喷泉本身就是一个负氧离子发生器,甚至海里的浪花、喷花的淋浴头也都会产生负氧离子。这就是原野、海边、森林里的负氧离子格外多的原因。
   负氧离子在空气中不会无限增多,也不会长期停留,而是不断产生、不断消亡。据测定,北京北海公园每立方厘米的空气中含有负氧离子800多个,而在行人拥挤的王府井大街仅200个左右。负氧离子惧怕灰尘、细菌、病毒,一旦碰到这些小的颗粒,就会发生电荷的传递,致使灰尘和细菌的颗粒变大,当空气托不住它们时便降至地面。这就是负氧离子多的地方有毒颗粒少、空气格外清新的原因。
   负氧离子对人的健康非常有益。它不仅能使空气清新,还可以灭菌、除尘、治病。当空气中负氧离子浓度很高时,能抑制多种病菌的繁殖,降低血压,消除疲劳,促进肌体的新陈代谢,调节和促进人体的生长发育。正因如此,人们才称负氧离子为“空气长寿素”。
   据最新出版的美国《生活》杂志报道,世界上三个有名的“长寿村”——巴基斯坦东部的芬扎、厄瓜多尔南部的毕路卡其巴以及前苏联高加索的一个村庄,仅百岁以上的寿星就有数百人。这三个长寿村皆是群山环抱、绿树成荫,负氧离子浓度很高。
   懂得了负氧离子的有关知识,我们就要在工作、学习之余经常到郊外、森林、海边去放松放松,这对身体健康是非常有益的。
   (节选自《知识文库》2004.5)
   1.本文从 、消亡、 三个方面介绍负氧离子的知识。
   2.本文举三个“长寿村”的例子,具体说明了 。
   3.要增加我们校园空气中负氧离子的含量,可采取哪些措施?
   答:
  
   (十二)(2004·莆田)
   鸟的建筑
   苇 岸
   ①鸟巢是鸟的建筑。和我们盖房子不同,鸟筑巢不是为定居。鸟只在繁殖期筑巢。
   ②鸟类学家依据鸟巢的位置和性质,把鸟巢分为地面巢、水面巢、 、 和
   等几种类型。
   ③地面巢大多简单、随便,往往仅在地面凹处略敷草物即告完工。这种巢,主要由雉、雁、鸭、鹤等笨拙的大型鸟类所为。出乎我们意料的是,像云雀、百灵、歌鸲、画眉这些名字美丽,叫声动听的玲珑小鸟,也在地面营巢。当然,它们的巢编织得都很精致。这是些疏于林木,常年出没在旷野,为土地处处留下歌声的可爱精灵。在水面营巢的鸟屈指可数,能够列举的如涉禽中善游的骨顶鸡与董鸡。它们借助水生植物搭造的可随水面升降的盘状浮巢,风险最小。洞穴巢包括崖壁洞穴和树干洞穴两种,前者的主人有翠鸟和沙燕;后者居多,如椋鸟、山雀、斑鸠、八哥、鹪鹩及肮脏的戴胜等,都是天然树洞或啄木鸟弃巢的受益者。如果顺着这个行列数下去,我们还会惊讶地发现鸳鸯。应该指明的是,营树洞巢的鸟,只有攀禽中的啄木鸟,真正具备开凿本领。啄木鸟还有一种英雄秉性,即它从不使用自己往年的旧洞。利用我们的屋宇营巢的鸟(不提麻雀),主要为燕科成员,原因在于它们的泥巢无法在露天筑造。最后说到的编织巢,就是指我们观念上认定的,代表“鸟巢”这一词语本义的,由鸟类中广大的鸣禽在树上(个别在草丛或灌木基部)精心营建的巢。这是那群勤奋的鸟类艺术家呕心沥血的作品,也是我们这篇短文想要谈论的核心。
   ④除涉禽中的鹭,游禽中的鹈鹕和猛禽中部分鹰隼(这是些在树上筑粗陋大巢的鸟)外,编织巢几乎全部为雀形目鸟类所造。它们长于鸣啭,巧于营巢,故根据分类上的说法,我们前面又称它们为鸣禽。雀形目是新鸟亚纲中种数最多的一目,其庞大数量占现代鸟类总体一半以上。
   ⑤编织巢的形态,可说多姿多彩。我们易于见到的杯状巢、碗状巢、盘状巢及瓶状巢,是其中主要的几种。营哪种巢型,与鸟的科属有关。但我愿意相信,它取决于鸟类个体的偏爱与审美因素。因此,这里无规律可循。
   ⑥杯状巢是多数营巢鸟喜爱的一种巢型,像我们熟悉的伯劳、卷尾、柳莺、寿带等夏候鸟,都营此型巢。太平鸟、灰山椒鸟、乌鸫及北红尾鸲等,营碗状巢。树鹨和灰喜鹊的巢很浅,呈盘状。攀雀和棕扇尾莺的巢收口,巢体似瓶。文鸟、黄眉柳莺和“告春鸟”短翅树莺,能够营造顶部具盖,侧面开门的球状巢。更为精巧和高超的,是黄鹂和绣眼鸟的吊篮式悬巢。南方有一种富于传奇色彩的小鸟,会将芭蕉或其他大型树叶卷合,然后在叶缘穿孔,贯以丝线,缝成袋状巢。这种天才的小鸟,鸟类学家就叫它“缝叶莺”。
   ⑦真正和我们的生活密不可分,在我们的视域内最为显著的鸟巢,实际是喜鹊粗糙的球状巢。这种“仰鸣则晴,俯鸣则雨,人闻其声则喜”(《禽经》)的民间吉祥鸟,同淳朴的麻雀一道,终年祥和地围绕着我们。特别是在空旷的冬天,它们的巢很像一座座村庄,醒目地坐落在原野高大的树上(每巢都有一定巢距的巢区。个别也有一树双巢现象。在北京的沙河附近,我曾见过一树四巢)。每次看到这些高耸的星罗棋布的“家”,我都很动情,我觉得这是一种世间温暖与平安的象征,是这个季节比雪与太阳升落更优美的景色。
   ⑧在神造的东西日渐减少、人造的东西日渐增添的今天,在蔑视一切的经济的巨大步伐下,鸟巢与土地、植被、大气、水,有着同一莫测的命运。在过去短暂的一二十年间,每个关注自然和熟知乡村的人,都已亲身感受或目睹了它们前所未有的沧海桑田性的变迁。
   1.根据第③段内容,在下面为第②段空白处填上适当的“巢名”(每处3个字)。
   鸟类学家依据鸟巢的位置和性质,把鸟巢分为地面巢、水面巢、 、 、 和 等几种类型。
   2.本文说明的重点,亦即本文谈论的核心是 。
   3.简要说说第③段第二句加点的“主要”一词在文中的作用。
   答:
   4.根据第④段的内容,解释什么是“鸣禽”。
   答:鸣禽是
   5.你如何理解第⑦段末尾划线句子的含义?
   答:   
   6.作者在文末说:“每个关注自然和熟知乡村的人,都已亲身感受或目睹了它们前所未有的沧海桑田性的变迁。”你对此又有什么感想?
   答:    
  
   (十三)(2004·龙岩)
   上海能造摩天大楼吗
   有人说,上海滩是个滩,水多、地低、土软,对高大建筑物不胜负担。例如,座落于市中心的上海展览馆主楼,人们每天经过它身边并不觉察到它有什么变化,可它从建成到现在却静悄悄地下沉了1.60米,相当于一个人的高度;刚建成不久的上海胸科医院外科大楼,在它下降最迅速时甚至以每天5毫米的速度沉落。由于它们都是均匀下降,所以并没有给宠大的建筑物和人们的安全带来威胁。但是,也有一些建筑物在建成后不久就出现了倾斜性的下降,发出了危险性的信号。
   其实,上海滩下并不是清一色的软土层。如果把滩下的土层绘制成一幅竖向剖面图,你就会发现在厚达300米松散的“千层糕”里却是群山起伏:表层为褐黄色亚粘土和灰色粉砂夹亚砂土;往下为三个不同厚度的淤泥质软土,如桩打在这里,地面上高层建筑物就容易往下沉降;以下为暗绿色粘性硬土层,历来是上海高层建筑的桩基持力层;再往下则是以粉细砂为主的承压含水层和基岩,摩天大楼的桩基打到这里,当然是稳如泰山的,不过要花费很大的经济代价。
   在上海滩建造摩天大楼,除了必须伸长腿、打深桩外,能否有较经济的办法呢?上海地质工作者发现在“千层糕”的土层中,有零星的山包崛起,浅的离地表仅70余米,如把桩打在山包之上,摩天大楼就能站稳脚跟,昂首雄视世界。
   (说明:大写字母表示土层;小写字母表示山包。以下答案均用图中字母填写。)
  
  
   1. 如果某建筑公司要在上海滩建
   一幢摩天大楼,那么桩基应选择打在哪个地方最科学合理又经济?
   应打在:___________处。
   2.“历来是上海高层建筑的桩基
   持力层”的位置在_____________处。
   3.地面高层建筑的桩基不宜打在
   哪些土层:_________________。
  
   (十四)(2004·黄冈)
   人类能移居火星么?
   早在一个多世纪以前的科幻小说里,人类就开始在火星上散步了,然而直到今天这还只是个设想。但许多人都坚信,问题不是人类能不能登上火星,而是在什么时候通过什么方式到达那里。至今,仍然没有人确知怎样才能最方便最省力地到达火星。有人估算,人类登上火星需要600亿美元的资金,甚至更多。不过,火星协会主席罗伯特·祖布林认为这个数字应该是300到400亿美元,相当于阿波罗登月计划的花费。祖布林出版了一本名为《移民火星》的书,书中勾勒了人类定居火星的宏伟蓝图。
   为什么人类对火星始终充满向往?因为就目前的观测情况看,火星是最有可能成为人类第二家园的地方。尽管火星上的温度是零下200℃,但在所有太阳系的行星中,火星的环境与地球最相像的。火星的直径大约是地球的一半,那里还有相当于地球上三分之一的重力和百分之一的大气浓度。最主要的是,探测器已经发现了在火星冰冻的极点和地表下面有水存在。每两年,火星会靠近地球一次,今年夏天,火星来到了距离地球5576万公里的地方,这是6万年来它距离地球最近的一次。
   祖布林说:“人们很想知道地球之外是否有生命存在,火星也许是解开这个问题的关键。在火星上,即便是只能找到原始状态的微生物,那也意味着类似人类的生命不只存在于地球上。”
   行星协会主席路易斯·弗雷德曼说,人类想要回答的基本问题是,人从哪里来?要到哪里去?“火星最吸引我们的地方在于它很可能帮助我们了解生命的缘起和归宿。”“并且,火星是我们目前知道的人类有机会移民的唯一地方,人类将以此证明自己不会永远呆在地球上;如果我们(A)能往火星上移民,那么地球可能(B)是我们唯一的家园,我们将不得不再次审视人类和地球的关系。”
   1.阅读第二自然段,指出火星最有可能成为人类第二家园的四个条件
   ① ②
   ③ ④
   2.祖布林和弗雷德曼对探测火星的意义各有什么看法?简要回答。
   祖布林:
   弗雷德曼:
   3.由于编者的疏忽,文中划线句子中掉了一个“不”字。有人认为这个“不”字应放在(A)处,也有人认为应放在(B)处。你认为应该放在哪里?为什么?
   答:
   4.假如有一天火星人来到地球做客,请以导游的身份向客人介绍你知晓的一处旅游景点及特征。
   答:

 (十五)(2004·荆州)
   有趣的种子旅行
   ①一株植物少则有几十颗种子,多则有几十万颗种子。如果它们都只停留在原地,挤成一堆,很难想像能怎么生活下去;如果发生自然灾害,如干旱、洪涝、火灾等等,集中在一块儿就更有灭绝的危险。因此,种子必须以各种方式进行迁移。这也是长期自然选择的结果。
   ②自然界中植物多种多样,种子旅行的方式也各有千秋。
   ③像大豆、绿豆、油菜、芝麻等植物的种子,有一种“弹射”的本领,果实成熟后包藏种子的果荚会突然扭曲、炸裂,将种子弹射到几十厘米甚至几米之外。根据现在掌握的资料,植物界中的弹射“冠军”,可能是一种生长在美洲的木犀草,射程可达14米之远。
   ④在非洲北部和欧洲南部,有一种有趣的植物叫喷瓜。它的果实成熟时,那些包藏种子的浆汁,就像气球中的空气那样,对果皮产生强大的压力,只要轻轻一碰,果实的粘液和种子就一起喷发出来,可射6米之远,所以当地人称它为“铁炮瓜”。
   ⑤但是这些种子的旅行路程,在整个自然界中只是低水平的。很多植物的种子,能以各种特有的办法,旅行到几千米、几十千米之远,有的甚至还能进行“环球漫游”呢!
   ⑥19世纪中叶,航行在大西洋上的一艘双桅帆船,遇到一场大风暴,船在风浪中颠簸飘流,好不容易才来到一个平静的海湾。水手们上岸,挖掘沙石来压舱,意外地发现一个黑色的沥青圆球,但拿在手上却感到很轻。打开一看,原来是一个椰子壳,里面藏着一张羊皮纸,上面写着古文。原来,这是当年哥伦布写的一封信。当时的航海者没有无线电通讯设备,常常利用椰子壳等飘浮物来传递信息。
   ⑦椰子的果实外壳坚硬,不透水,中间是蓬松的纤维层,充满了空气,很轻,很适宜在海水中随波飘流,作长途旅行。一旦被海浪冲到沙滩上,就可以发芽、生长。由于椰子以这种方式传播后代,所以我们见到的椰子树,除了人工栽培的之外,几乎都长在海滩边上。
   ⑧在水中旅行的种子还有很多。比如荷花,它的果实莲蓬像一艘小船,轻浮在水面上,四处飘流,把种子带到远方。待莲蓬腐烂了,种子也就沉到了水底。第二年春天,一栋新的荷花又成长起来了。
   ⑨还有睡莲,它的海绵质的浆果结构十分奇特,浆果里面装有好多种子,每粒种子外面包有一个充满空气的囊,就像套着个救生圈,在长途飘流中,囊中的空气慢慢消失,种子就沉到了水底。
   1.你感觉到了吗——文中起“总说”作用的一个句子是(在下面横线上写出其序号):
   答:
   2.你发现了吗——表示说明的内容将更进一层的一个自然段是(在横线上写出其序号):
   答:
   3.你提炼了吗——抓关键词语,可用四个字概括出种子这样两种有趣的旅行方式:
   答:
   4.你揣摩了吗——从全文的内容看,“种子旅行”的含义是:
   答:
   5.你品析了吗——第⑥自然段在文中所起的作用是:
   答:
   6.你联想了吗——请介绍种子旅行的另外一种有趣的自然方式:
   答:
  
  
   (十六)(2004·十堰)
   一个医生嘴巴尖,天天出诊到林间;
   敲敲听听多仔细,要动手术把头点。
   上面这个谜语猜一种动物,谜底是啄木鸟。可是起初我不但猜不中,而且人家揭了谜底后,我还不懂为什么把啄木鸟誉为“森林医生”。后来我随一支森林科学考察队去林区调查研究,通过一位生物学家对啄木鸟的介绍,和我自己对它的观察,我才真正认识到,“森林医生”这个称号,啄木鸟是当之无愧的。
   啄木鸟每天都起得很早,起身后第一件事就是给树木“检查身体”。它用嘴将一棵棵树从根到梢笃笃地轻轻敲打一通,听听有没有空声。经验告诉它,敲打时如发出空声,树干里面一定有虫,就立刻“动手术”。它的嘴长而直,末端尖锐,不但能啄破树皮和腐败的树干,而且能啄开坚硬的木质部;它的舌骨很发达,包围着头骨,舌骨起着特殊的弹簧作用,能使细长的舌条伸缩自如;舌面富含黏液,能把树中昆虫的幼虫和虫卵粘住;舌端生有向后倒长的小钩,能钩取树中较大的昆虫。它的鼻孔上覆盖着保护性的刚毛,可以防止碎土屑飞入鼻孔。值得一提的是,它的操作路线也很科学,工作时总是围绕着树干螺旋式地向上攀登,这样既可避免重复劳动,又不会有所遗漏。啄木鸟就凭着这套完备的“医疗器材”和独特的“医术”,专门捕食危害树木的昆虫,保护树木健康地成长。
   由于长期过着树栖生活,啄木鸟的翼变得既短又钝,不适于快飞和远翔,只能在林间通道或林中旷地作短距离滑翔,但是它的足趾却特别强劲有力,趾的排列也不像一般鸟类那样三趾向前,一趾向后,而变成二趾向前,二趾向后,而且具有锐利的钩爪,因此即使攀缘在笔直的树干上也不会滑落下来。此外,它的尾羽强韧坚直,羽轴特别粗硬,有弹性,可以当做支柱顶住身体,与两条岔开的腿形成一个“三足鼎立”之势,增加了悬立时的稳固性。
   1.本文为什么将啄木鸟誉为“森林医生”?
  
   2.文章开头引用谜语有什么作用?
  
   3.文中说“它(啄木鸟)的操作路线也很科学”。请回答:啄木鸟工作时的操作路线是怎样的?为什么说这种操作路线是科学的?
   答:①第一问: 。
   ②第二问: 。
   4.文中划横线句子运用了哪种说明方法?有什么作用?
   ①说明方法:
   ②作用: 。
   5.从节选的文字看,还不足以说明“‘森林医生’这个称号,啄木鸟是当之无愧的”,如果让你续写下文,你认为还应写进哪方面的内容?
   答:
  
   (十七)(2004·广东)
   ①从医学角度分析,噪音往往能引发身体的疲劳和不适,对人的心理也造成一定的伤害。
   ②60分贝以下的噪音一般不会引起人的厌烦。假如在超过70分贝噪音的环境中,想要更好地休息就得服用镇静剂或安眠药。当然,75分贝的噪音算中等级别,它能影响人的思维和情绪。在我们生活的环境中,常有一些尖利的噪音,它们虽然比较短暂,但其激烈程度可以严重干扰人们的生活。例如:摩托车在行驶中的噪音可以达到100—110分贝,其强度超过电锯锯木发出的声音;摇滚乐队的某些演奏可能产生140分贝的噪音,它大大超过喷气式飞机降落到距地面100米时产生的轰响。科学实验表明,超过115分贝的噪音能引起人的严重的烦躁和不安,这种情况相当于癫痫病发作时的一系列大脑变化;面对160分贝的噪音,
   动物有可能死亡。
   ③虽然我们生活在喧闹声中,但不是人人都能察觉。人的耳朵有一种“掩蔽”功能,能
   自动清除环境噪音,而把那些我们感兴趣的声音突现出来。因此,我们能听见站在人群中或
   公共场所的人对我们的讲话。耳朵还能将讲话人的声音分离出来,同时在一定程度上忽略环
   境噪音。但这只是大脑的一种“应对”技巧,噪音已经丝毫不漏地进入我们的听管和神经
   系统,其后果是,强烈和长时间的噪音会破坏声感受细胞上的纤毛。这些纤毛就是我们的
   “传声器”,随着它们的逐渐被破坏,我们甚至可能失去听觉。长此以往,耳鸣和心悸将接
   踵而来,并且身体的平衡系统也会因此而遭到破坏。从国际上的一些统计数据可以发现,生
   活在喧闹地区的一部分居民的听力正在变得越来越差。在美国64-75岁的人群中有1/4的人失去听力;在意大利这个比例达到了18%,是40年前的两倍。据一些国家征招新兵的人介绍,现在有听力缺陷的年轻人越来越多。
   (选自(科学世界)2004年第4期)
   1.本文主要说明了 (限10个字以内)。
   2.作者是按照什么顺序来安排第②段内容的?这样安排有什么好处?
   答:
   3.第③段划线句子表明人们的听力 ,运用的说明方法有 、
   、 、 。
   4.下列句子中,删去加点词语后对原句意思表达影响最小的一项是 ( )。
   A.常有一些尖利的噪音,它们虽然比较短暂,但其激烈程度可以严重干扰人们的生活。
   B.耳朵还能将讲话人的声音分离出来,同时在一定程度上忽略环境噪音。
   C.摇滚乐队的某些演奏可能产生140分贝的噪音。
   D.面对160分贝的噪音,动物有可能死亡。
  
  
   (十八)(2004·湛江)
   ①在我国大陆最南端,湛江雷州半岛徐闻县西海岸,有一个闻名全国,被誉为“水族大观园”的珊瑚自然保护区,区内的珊瑚礁绵延20多公里,面积达2000公顷。据专家研究,珊瑚礁已有近万年的发育史。
   ②徐闻西海岸一带岸礁众多,海水清澈,水温、盐度稳定,非常适合珊瑚虫的繁殖生长:多年来当地一直十分注重生态的保护,保护区内的珊瑚种类繁多,达五十多个品种。其中软体珊瑚有千年佛、白汽泡、鸡冠花,硬体珊瑚有角星、海脑、扇形,等等。
   ③当艳阳高照,风平浪静的日子,乘着小船在珊瑚丛上面缓缓滑行,犹如在一片美丽的丛林中漫游,那密密麻麻的鹿角、牛角、羊角几乎探出水面,触手可及。还有散落在“丛林”中的“翡翠”、“玛瑙”,形态各异;时稳时现的“鲜花”,橙黄蓝白红,煞是可爱,美不胜收。不时还可见五彩缤纷的鱼儿与游船一起穿梭漫游,构成一幅奇异的海底风景画。
   ④珊瑚礁群向来被海洋学家称为海上“热带雨林”。它占海洋面积不到0.2%,却栖息了1/4的海洋生物,供养着9万多种鱼类。我国的海南岛沿岸本来有中国最大的珊瑚礁群,历史在十万年以上,但因过度开发,仅10多年有的地方已成了海底废墟。而徐闻的珊瑚礁群作为目前祖国大陆保存最完好的珊瑚自然生长区,正吸引着越来越多的旅游观光者。为了永远给子孙后代保留这一片天赐的生态自然景观,当地政府已制订了以保护为前提,结合生态和环境旅游的利用计划,要让这一颗明珠永远闪亮在祖国大陆南端的西海岸边。
   1.文中为什么称珊瑚礁群(区)为“水族大观园”和“热带雨林”?
   答:_________________________________________________________
   2.文章第①②段分别从哪些方面介绍了徐闻珊瑚自然保护区?
   答:第①段:_________________________________________________________
   第②段:_________________________________________________________
   3.请从写作手法或语言运用特点的角度对第③段的表达效果(特点)作简要分析:
   答:_________________________________________________________
   4.文章末段写海南岛珊瑚礁群受损的事例是为了说明什么?你从中得到什么启示:
   答:(1) _________________________________________________________
   (2) _________________________________________________________
  
  
   (十九)(2004·北海)
   ①资源短缺的表现之一,是可耕土地资源不足,粮食生产的增长赶不上人口的增长。于是,许多人纷纷发出警告:地球将无法养活超过100亿的人口。然而,一些乐观的人士反对这种危言耸听的说法。他们认为,虽然陆地上可耕地的开发已近极限,但地球还有广阔的海洋可供开发,大海完全有可能成为人类未来的粮仓。
   ②当然,海洋所能提供给我们的并不是传统意义上的粮食——大米、小麦和玉米等,而是广义的粮食——其他的能够满足人类营养需要的食物。一些海洋学家指出:仅仅是位于近海水域自然生长的海藻,每年的生长量就已相当于目前世界小麦年产量的15倍。如果把这些藻类加工成食品,就可以为人类提供足够的蛋白质。其实,把藻类叙谈食品,我们并不陌生。仅以我国沿海来说,人们比较熟悉的可食用藻类就有:褐藻类的海带、裙带菜、羊栖菜、马尾藻;红藻类的紫菜、鹧鸪菜、石花菜;绿藻类的石莼、浒苔等。它们在人工的精心养殖下,产量正在不断翻新。其中仅海带一处,目前年产量就比早先的野生状态提高了2000多倍,可见增产潜力是多么巨大!
   ③除海藻类,海洋中还有丰富的肉眼看不见的浮游生物。有人作过计算,在不破坏生态平衡的前提下,若能把它们捕捞出来,加工成食品,足可满足300亿人的需要。
   ④至于海洋中众多的鱼虾,则更是人们熟悉的食物。尽管近海的鱼虾捕捞已近极限,但我们还可以开辟远洋渔场,发展深海渔业。例如南极的鳞虾,每年产量可高达50亿吨,我们只要捕获其中的1亿——1.5亿吨,就比当今全世界一年的捕鱼量多出1倍以上。何况,在深海和远洋中还有许许多多尚未被我们充分开发利用的海洋生物,其巨大潜力是不言而喻的。
   ⑤综上所述,说大海是人类未来的粮仓,一点儿也不夸张。
   1.海洋给我们提供的粮食和传统意义上的粮食的区别是:              
                                       。
   2.第②段中画线句子运用了哪些说明方法?
   答:          、          、             。
   3.文章第②段中加点的一句说:“它们在人工的精心养殖下,产量正在不断翻新”,请
   在文中找出与“翻新”一词对应的例子:                      
                                     
   4.第③段中画线的“在不破坏生态平衡的前提下”一句是否可以删掉,为什么?
     答:                                    
                                         
   5.请你为上文安一个恰当的题目(不要超过10个字)。
     答:                                    
  
  
  
   (二十)(2004·内江)
   克隆技术造福人类
   ⑴克隆技术会给人类带来极大的好处。例如,英国PPL公司已培育出羊奶中含有治疗肺气肿的α—I抗胰蛋白酶的母羊。这种羊奶的售价是6000美元一升。一只母羊就好比一座制药厂。用什么办法能最有效、最方便地使这种羊扩大繁殖呢?最好的办法就是“克隆”。同样,荷兰PHP公司培育出能分泌乳铁蛋白的牛,以色列LAS公司育成了能生产血清白蛋白的羊。这些高附加值的牲畜如何有效地繁殖?答案当然还是“克隆”。
   ⑵母马配公驴可以得到杂种优势特别强的动物——骡,然而骡不能繁殖后代,那么,优良的骡如何扩大繁殖?最好的办法也是“克隆”。我国的大熊猫是国宝,但自然交配成功率低,因此已濒临绝种。如何挽救这类珍稀动物?“克隆”为人类提供了切实可行的途径。
   ⑶除此之外,克隆动物对于研究癌生物学、研究免疫学、研究人的寿命等都有不可低估的作用。
   ⑷不可否认,“克隆绵羊”的问世也引起了许多人对“克隆人”的兴趣。例如,有人在考虑,是否可用自己的细胞克隆成一个胚胎,在其成形前就冰冻起来。在将来的某一天,自身的某个器官出了问题时,就可从胚胎中取出这个器官进行培养,然后替换自己病变的器官。这也就是用克隆法为人类自身提供“配件”。
   ⑸有关“克隆人”的讨论提醒人们,科技进步是一首悲喜交集的进行曲。科技越发展,对社会的渗透越广泛深入,就越有可能引起许多有关的伦理、道德和法律等问题。我想用诺贝尔奖获得者、著名分子生物学家J.D.沃森的话来结束本文:“可以期待,许多生物学家,特别是那些从事无性繁殖研究的科学家,将会严肃地考虑它的含意,并展开科学讨论,用以教育世界人民。”
   1.克隆技术会给人类带来哪些好处?
   答:
   2.第⑴段运用了哪些说明方法,列出两种说明方法并举例加以说明。
   答:
   3.第⑸段运用的主要表达方式是 ,这段中支撑“科技进步是一首悲喜交集的进行曲”的句子是
   ?
  
   (二十一)(2004·泸州)
   风是空气流动的表现,是我们地球上最常见的自然现象。然而,你听说过太阳也会刮风吗?
   太阳风是一种来自太阳的物质流。这种物质虽然与地球上的空气不同,不是由气体的分 子组成,而是由更简单的比原子还小一个层次的基本粒子——质子和电子等组成,但它们流 动时所产生的效应与空气流动十分相似,所以称它为太阳风。当然,太阳风的密度与地球上 的风的密度相比,是非常非常稀薄而微不足道的,一般情况下,在地球附近的行星际空间 中,每立方厘米有几个到几十个粒子。而地球上风的密度则为每立方厘米有2687亿亿个分 子。太阳风虽然十分稀薄,但它刮起来的猛烈劲,却远远胜过地球上的风。在地球上,12级台风的风速是每秒32米以上,而太阳风的风速,在地球附近却经常保持在每秒350~450千米,是地球风速的上万倍,最猛烈时可达每秒800千米以上。
   太阳风虽然猛烈,却不会吹袭到地球上来。这是因为地球有着自己的保护伞——地球磁 场。地磁场把太阳风阻挡在地球之外。然而百密一疏,仍然会有少数漏网分子闯进来,尽管 它们仅是一小撮;但还是会给地球带来一系列破坏。它会干扰地球的磁场,使地球磁场的强 度发生明显的变动;它还会影响地球的高层大气,破坏地球电离层的结构,使其丧失反射无 线电波的能力,造成我们的无线电通信中断;它还会影响大气臭氧层的化学变化,并逐层往 下传递,直到地球表面,使地球的气候发生反常的变化,甚至还会进一步影响到地壳,引起 火山爆发和地震。例如,1959年7月15日,人们观测到太阳突然喷发出一股巨大的火焰 (它就是太阳风的风源)。几天后,7月21日,也就是这股猛烈的太阳风吹袭到地球近空时, 竟使地球的自转速度突然减慢了0.85毫秒,而这一天全球也发生多起地震;与此同时,地 磁场也发生被称为“磁暴”的激烈扰动,环球通信突然中断,使一些靠指南针和无线电导航 的飞机、船只一下子变成了“瞎子”和“聋子”……
   太阳风对地球的影响,只是乘虚而入的漏网分子所为。由此可见,在无所阻拦的行星际 空间,太阳风的威力有多大了。
   1.本文的说明对象是                        。
   2.简要回答:为什么把来自太阳的物质流称为太阳风?
   答:                                     
   3.太阳风如果来到地球,会造成哪些破坏?(至少三方面)
   答:①                                  。
      ②                                  。
     ③                                  。
   4.文中加点的“一般情况下”能否删去?为什么?
   答:                                  
   5.本文第二段主要运用的两种说明方法是       、        。
  
  
   (二十二)(2004·资阳)
   ⑴目前世界上第一台光脑已由欧共体研制成功,这是一台全光数字计算机,其运算速度比电脑快一千倍。
   ⑵光脑和电脑的工作原理基本一样,所不同的是光子代替了电子;光互连代替了电子导线互连;光开关、光三极管、光存储器、反馈装置和集成光路等部件,代替了电脑中的电子硬件;用光运算代替了电运算;用非冯·诺伊曼结构代替了冯·诺伊曼结构;从而使光脑中的功能为电脑所望尘莫及。
   ⑶光脑具有超并行性。目前最新的并行处理电子计算机常具有NⅪ的并行性,其结构和运行极为复杂,速度和精度也相当低。光脑则不同,它具有NXI的并行性,具有并进通信和并行处理能力强,可用简单的运算去处理大阵列的特点。
   ⑷光脑可在接近室温条件下具有超运算速度。电子的传播速度每秒只能达到593公里,而光子的速度每秒30万公里,因此利用光在光缆中互连通信要比利用电子在互连的导线中通信减少大量的时间,提高了运算速度。同时超高速电脑的计算器件只能在极低的温度下工作,而光脑则可以在接近室温下进行超高运算也在于光的频带远大于无线电波和微波,具有极大的信息存储量,存储量可达108位。
   ⑸光脑抗干扰能力强。光脑靠光子传播信息,光电子没有带电荷,对其他邻近的光子和电子毫无影响。所以光信号不仅不相互干扰,而且可以与电子控制信号交叉,此外光脑容错性能好,具有与人脑类似的容错性,系统中某一元件损坏或出差错时,并不影响到最后结果。
   1.这段文字说明的对象是
   2.简略回答光脑同电脑相比具有哪些优势。
   答:
   3.这则短文主要运用了哪些说明方法?
   答:
   4.第4自然段从哪两个方面介绍了光脑在接近室温条件下具有超运算速度的?(2分)
   一方面从光脑的
   另一方面从光脑的
   5.“光脑和电脑的工作原理基本—样,所不同的是光子代替了电子”一句中,“基本”用得准确是因为
  
   6.根据你平时的积累,说出两个高科技产品的名称。
   答:

 (二十三)(2004·桂林)
   水之经典
   ⑴世上丽水秀水晶莹之水清澈之水恢弘之水浩翰之水,多的是,在我看来,最富性格最值得一看的是这两处:都江堰和九寨沟。
   ⑵看都江堰的水,看的是强悍奔腾的水如何层层叠叠化为生命的涓涓细流。飞奔如兽、桀鹜不驯的江水,经过都江堰,立刻将仰天长啸变为 ,将浪涛如山变为 ,将凶猛如火变为 ……出宝瓶口流入内江,立刻呈现一派水光潋滟的情景,让人叹为观止,看到水的柔劲、可塑和万难不屈,常流不懈的生命活动。那是一种将绚烂归于平淡,将刚劲寓于柔顺,将一时融于永恒的生命。
   ⑶都江堰看水,看的是水如何从天上流入人间,如何从神话流入现实,如何将自己化为一种哺育人类、灌溉庄园的生命。
   ⑷都江堰的水,是一种入世的现实的水。
   ⑸看九寨沟的水,看的是宁静的恬淡的水,如何凝聚成生命的湖泊。镜海、长海、珍珠滩……每一个湖泊都是那样清澈透明、纤尘不染。孔雀的蓝色,蓝得让人心醉,让人如同看到教堂洗礼用的圣洁露水,如同听到教堂管风琴演奏的圣母颂,而不敢有丝毫杂念俗念,懂得并真正地看到人世间居然有纯洁美好真诚和透澈的净,就在这远避尘嚣而静静地存在。
   ⑹那水几乎一动不动,任外面的世界如何纷繁变幻,将污染、噪音连同人心泛起的种种污浊的泡沫一起抛向天空和大地。它独自坚持着自己的贞操,不动丝毫涟漪,不染丝毫尘俗,将水底的虬枝沉木、水藻水锦,将天上的薄云丽日、山岚清风,将身旁的雪峰幽谷、古树老藤……一一映在自己的怀中,映得那么明净,如同脱胎换骨一般,玉洁冰清,重新塑造了自己一番。尘世沾惹的市俗庸俗、风骚矫情、浪声虚名、欲火利海……起码不敢在这里抖擞,而被这水洗却大半。
   ⑺九寨沟看水,看的是水如何从人间流向天上,如何从现实流向神话,如何将自己化为一种启迪人类、净化心灵的艺术。
   ⑻九寨沟的水,是一种出世的艺术的水。
   ⑼日本黑田孝高在《水五则》中的第一则说:“自己活动,并能推动别人的,是水。”第四则说:“以自己的清洁,洗净他人的污浊,有容情纳浊的宽大度量的,是水。”前则,可以送给都江堰的水;后则,可以送给九寨沟的水。
   1.第⑴段在全文中的作用是 。
   2.第⑴段画线句按常理应断句,你认为应该加入什么标点符号?作者为什么不加标点?
   答:
   3.给第⑵段文中空白处依次选择恰当的词语,正确的一项是( )
   A.喃喃细吟 珍珠四溢 柔情万缕
   B.柔情万缕 珍珠四溢 喃喃细吟
   C.珍珠四溢 柔情万缕 喃喃细吟
   4.为什么说“都江堰的水,是一种入世的现实的水”?请用自己的语言概括回答。
   答:
   5.与第⑼段中“以自己的清洁,洗净他人的污浊,有容情纳浊的宽大度量的,是水”一句相照应的是第 段的内容。
   6.你如何理解“水之经典”中“经典”一词在文中的含义?
   答:
   7.文中所写的都江堰之水和九寨沟之水各有性格,你更喜欢哪一处的水?为什么?
   答:
  
   (二十四)(2004·桂林)
   生命与气候
   ⑴你能想到吗?千百万年以来,地球气候是由地球的两大生命王国——动物界与植物界的斗争所决定的。动物吸进氧气而呼出二氧化碳,植物主要吸进二氧化碳而放出氧气。二氧化碳为温室气体,因此可以立即得出结论:动物能使地球升温,而植物则使地球降温。如果哪一方占上风,那么地球就会面临温室效应或又一次严寒期。
   ⑵5亿多年前的“寒武纪大爆炸”时期,到处都是将二氧化碳释放到空气中的节肢动物,它们使空气中的二氧化碳含量高达今天的20倍,所以当时的气候极其温暖。
   ⑶但是,植物进行了反击。化石显示,4.5亿年前,含有木质素的植物首次出现,木质素使细胞坚硬,这样植物就得以长大,地球上出现了第一批树木。随着全球的植物大量生长,毫无约束的光合作用从空气中吸取着二氧化碳,使二氧化碳含量大大减少,而不能吸收木质素的、饥饿的节肢动物对此无能为力——地球进入了严寒期。
   ⑷动物又通过进一步进化进行了回击,主角是白蚂蚁和恐龙,它们学会了如何食用木质素,如何繁荣昌盛。它们粗重的呼吸及对植物的破坏使空气中的二氧化碳高达今天的3倍,地球大部分地区酷热而潮湿,又恢复了温室效应。
   ⑸然而植物并没有被消灭。在6500万年前,恐龙因为小行星的碰撞或其他灾难而彻底消失后,植物抓住了这一机会再次反击,突破点是第一批草地的出现。青草本身不会保存很多二氧化碳,但能形成软土,软土可以保存大量的二氧化碳,从而使空气中的二氧化碳含量降低。事实上,草地生态系统含有的二氧化碳比森林生态系统还多。
   ⑹过去约4000万年来,大量草地布满全球,取代了许多年以前的森林地带。在草地的影响下,地球逐渐降温,终于在200万年前进入冰川期。
   ⑺大约1万年前,由于人类活动的显著增强,使地球空气的二氧化碳含量再度升高,尤其是近百年来,地球的温室效应愈来愈明显。
   ⑻也许,地球上的整个生物圈就像我们所熟知的生物个体调节其体内环境那样,不停地调节着地球的大气环境,使气候能适应地球上生物生存进化的需要。换句话说,地球的生物圈是在主动地调节环境,而不是在被动地适应环境。果真如此,数十亿年的气候变迁不仅仅是太阳或地球的原因,生物为了自己的利益也深深地参与其中了。
   (选自《科技日报》,有改动)
   1.本文说明的主要内容是: 。
   2.本文主要采用的说明顺序是 。
   3.你认为文章使用了哪些说明方法?请写出两种,并各举一例加以说明。
   答:
   4.第⑻中加点的词语“也许”能否删除?为什么?
   答:
   5.本文中,植物吸收或保存二氧化碳有哪些方式?请分列出来。
   答:
   6.作者说:“地球的生物圈是在主动地调节环境,而不是在被动地适应环境”,你认为正确吗?请联系生活实际加以说明。
   答:
  
   (二十五)(2004·河南)
   宇宙飞船的“避火衣”
   ⑴我国“神舟”五号载人宇宙飞船在2003年10月的顺利发射升空与安全返回,是我国载人航天的历史性突破。它从天外“下凡”——返回地球,过程复杂,场面惊心动魄。
   ⑵当宇宙飞船以第一宇宙速度(7.9公里/秒)进入大气层时,飞船的外壁与空气剧烈摩擦,产生极大量的热,在距离地面60公里左右的稠密大气层区域,其表层温度可以达到1000~3000摄氏度,整个机身变成一个炽热耀眼的火球(就像我们常见到的火流星一样)。进入距离地面80~40公里区域时,由于周围空气在高温下电离,宇航员与地面的无线电通讯会暂时中断,进入“黑障区”,就连雷达也无法发现它的踪迹……
   ⑶飞船返回舱中有宇航员和珍贵的仪器资料,必须保证它顺利通过大气层安全返回地面,必须为它研究制造一件“避火衣”。
   ⑷由于宇宙飞船只使用一次,科学家为它(A)精心设计了一件奇妙的“避火衣”,它(B)是用“瞬时耐高温材料”制成的。这(C)是一种由特殊纤维材料或多孔颗粒加上有机物组成的低导热复合材料,这一层材料的厚度经过精心计算设计,在宇宙飞船不同部位的厚度是不同的。
   ⑸当飞船返回舱由太空“下凡”通过大气层,与空气剧烈摩擦产生大量的热量时,这件“避火衣”就会“引火烧身”,自己先燃烧起来,其中大量的有机物会发生化学分解和汽化,带走极大量的热量。更奇妙的是,“避火衣”在燃烧自己的同时,还形成一层厚厚的多孔炭化层,紧紧地附着在返回舱的外壁,这一炭化层具有极好的隔热效能,在它严密的包裹下,能有效地防止外界热量传入舱内,使返回舱中的温度保持在35摄氏度以下,保证了宇航员的安全。
   ⑹这种自我牺牲的一次性“避火衣”设计十分巧妙,结构相对比较简单,重量就比较轻,成本自然也就比较低,具有双重奇妙功能,使宇宙飞船顺利“下凡”,为宇航事业做出了卓越的贡献。
   1.请根据文意,概括宇宙飞船“下凡”时的“惊心动魄”的场面。
   答:
   2.第⑵段中有两处使用了括号,这两处括号中的内容在表达上分别有什么作用?
   第一处:
   第二处:
   3.第⑷段中三个加点的词在文中分别指代什么内容?
   A.它: B.它: C.这:
   4.文中能体现“避火衣”“自我牺牲”特点的词语是 。
   5.从文中看,宇宙飞船的“避火衣”具有的“双重奇妙功能”是什么?
   ①
   ②
  
  
   (二十六)(2004·沈阳)
   (1)在浮冰成群的南大洋栖息着南极鳕鱼、松浮鱼、蛇齿鱼、南极多斑鱼等100多种鱼类。这些鱼鱼体小,一般体长只有25厘米,生长速度缓慢,多数系底栖性鱼类。
   (2)人的血液在-0·56度就会冻结,一般鱼类也大致相同。但是,冬天生活在南大洋中的鱼类却能经受-2度左右的低温而安然无恙。海水的冰点约为-1·85度,为什么南极鱼的血液在这一温度下却不会冻结呢?原来,在这些鱼的血液中存在着一种抗冻糖蛋白。有趣的是,一到夏天,随着海水温度的上升,这些鱼的脑垂体将会自己调节,使血液中不再产生抗冻糖蛋白。
   (3)70年代,美国加利福尼亚大学费尔教授从南极的一种鳕鱼的血液中,分离出8种分子量不同的抗冻糖蛋白,发现它们不同程度地具有使水难以冻结的本领。科学家经过实验表明,如果将被冻糖蛋白从南极鱼的血液中除去,那么它的血液与一般血液没有多大区别,也会在同样的温度下结冰。
   (4)抗冻糖蛋白是一种相对分子质量非常大的物质,即使溶解的质量稍多些,摩尔浓度也是非常低的,而血液基本上仍旧保持着相同的张力。那么,为什么抗冻糖蛋白的浓度如此之小,却能使血液的凝固点发生这样大幅度的下降呢?显然,这一事实用传统的“溶液的凝固点降低理论”是无法解释的,费尔教授认为,这是因为抗冻糖蛋白分子能挤入互相靠近的两个细胞的间隙中,从而阻止了冰晶体的生长。即抗冻糖蛋白分子向冰晶体方向移动,使凝固点大幅度降低。
   1.本文说明的内容是 。
   2.本文第(2)段划线句采用了 、 的说明方法,它的作用是什么?
   答:
   3.本文第(3)段加点的字“经过实验”不能去掉,为什么?
   答:
   4.指出本文的说明顺序: ,并用波浪线在原文划出说明对象特征的句子。
   5.水的冰点是0度,在第(2)段中提到“海水的冰点约为-1·85度”,为什么?根据你掌握的知识,结合选文找出能解释这一现象的依据。
   答:
  
   (二十七)(2004·湟中)
   “能吞能吐”的森林
   ①森林涵养水源,保持水土,防止水旱灾害的作用非常大。据专家测算,一片10万亩面积的森林,相当于一个200万立方米的水库,这正如农谚所说的:“山上多栽树,等于修水库。雨多它能吞,雨少它能吐。”森林因这种特殊的“吞吐”功能而被科学家称之为“吞水吐雨器”。
   ②说起森林的功劳,那还多得很。它除了为人类提供木材及许多种生产生活的原料之外,在维护生态环境方面也是功劳卓著,它用另一种“能吞能吐”的特殊功能孕育了人类。因为地球在形成之初,大气中的二氧化碳含量很高,氧气很少,气温也高,生物是难以生存的。大约在4亿年以前,海里的先进植物登陆,陆地才产生了森林。森林慢慢将大气中的二氧化碳吸收,同时吐出新鲜氧气,调节气温,这才具备了人类生存的条件,地球上才最终有了人类。所以科学家又称森林是“吞碳吐氧机”。
   ③森林,是地球生态系统的主体,是大自然的总调度室,是地球的绿色之肺。森林维护地球生态环境的这种“能吞能吐”的特殊功能是其他任何物体都不能取代的。因此,我们必须高度重视植树造林,并且保护好森林。目前,值得我们每个人关注的是地球的绿色之肺在日益萎缩。近200年间,地球上的森林已有1/3以上被采伐和毁掉。而另一方面,由于地球上的燃烧物增多,二氧化碳的排放量在急剧增加。此消彼长,使得地球生态环境急剧恶化,主要表现为全球气候变暖。全球气候变暖对人类的生产和生活有着巨大的影响,甚至威胁人类生存。因为全球气候变暖,水分蒸发加快,改变了气流的循环,使气候变化加剧,从而引发热浪、飓风、暴雨、洪涝及干旱。
   ④为了使地球的这个“能吞能吐”的绿色之肺恢复健壮,以改善生态环境,抑制全球变暖,减少水旱等自然灾害,我们应该大力植树造林,使每一座荒山都绿起来。
   1.阅读全文,简洁地写出森林的两大功劳。
   (1)吞水吐雨器:
   (2)吞碳吐氧机:
   2.阅读第③段,概括说明“地球的绿色之肺在日益萎缩”的两个原因。
   (1)_________________________________________________________________
   (2)________________________________________________________________
   3.将“森林”说成“地球的绿色之肺”,这是使用了 的说明方法,其作用是_________________________________________ _____
   4.第③段画线句中的“近200年间”强调了___________ ___ __,“1/3以上”强调了_____________________________________。
   5.森林遭到破坏后,地球上的生态环境急剧恶化。除了文中介绍的全球性气候变暖外,还有哪些方面的恶化?请举出两例。
   ①
   ②
   6.过度砍伐、无节制使用是森林资源遭到破坏的重要原因。2001年武汉市政府在全体市民中发起了“禁止使用一次性木筷”的活动,得到了广大市民的积极拥护和支持。现请你根据平时的观察和思考,提一条保护森林资源的建议或拟一条含警示性的标语。
   答:
  
   (二十八)(2004·潍坊)
   英国科学家最近指出,根据目前已有的建筑技术,人们完全能够建造出高抗震性的房屋,减少地震带来的人员伤亡和物资损失。现在问题的关键是,除地区和建筑本身有所差异外,抗震建筑的成本一般来说就相当可观。
   英国牛津大学工程科学院的布莱克伯勒认为,建筑物抗震能力的大小,主要由4个因素决定:一是建筑物下的土质及地面的移动量有多少能够转移到建筑物上来;二是支柱的数量;三是支柱的牢固性;四是采用何种方式减少横摆度。他解释说,如果建筑物下面是坚硬的石质结构,则十分有利于抗震,但目前大部分建筑物都建在泥土上,泥沙和填土在遇到地震压力时,常常会液化,从而使建筑物从根基部倒塌。布莱克伯勒认为,在地基处堆放一些起排流作用的大石堆,就可有效防止泥土流失。另外,在建筑物下安放巨型弹簧,也可使建筑物与地面隔离,不受地面移动的影响。但是,他认为,如果地面移动太远,巨型弹簧就不再适合,而且有些附加设备,例如供残疾人使用的软椅坡道等,就会将建筑物与地面重新连接起来,消弱弹簧的功效。对于防止横摆,布莱克伯勒说,一些日本建筑的做法值得借鉴。他们通常在屋顶放置大型重物,利用水压活塞使重物运动,且运动方向与地面横移方向相反,抵消了横移的影响,从而把建筑物的横摆度减到最小。英国科学家认为,将巨型弹簧、牢固的支柱、水压活塞及屋顶重物等方法相结合,肯定能抵抗地震灾害,只是因其成本高昂现在无人敢于问津。
   1.要使建筑物抗震能力强,应该考虑哪些主要方面?
   答:
   2.英国科学家最近指出,根据目前已有的建筑技术,人们完全能够建造出高抗震性能的房屋的根据是什么?
   答:
   3.根据文章的介绍,你认为目前建造抗强震的房屋要解决的最主要矛盾是什么?
   答:
   4.要使建筑物防震,除文章介绍的方法外,还可有其它许多方法,请发挥你的想像,为建筑抗震房屋想一想办法。
   答:
  
   (二十九)(2004·新疆)
   ⑴罗布泊,这是带有几分神秘色彩的地名。它是塔克拉玛干大沙漠边缘的一个戈壁滩。这里没有一棵草,没有一条溪,夏季气温高达摄氏70度。罗布泊天空中不见一只鸟,没有任何飞禽敢穿越此地。自二十世纪初,瑞典探险家斯文闯入这里,罗布泊才为世人所知。但是,近十几年来,此地却不时传出因缺水而有人遇难的消息。1980年6月17日,中科院新疆分院副院长彭加木穿越干涸的罗布泊盆地进行科学考察时,外出找水,不幸遇难;时隔十多年后,著名探险家余纯顺探险至此地,也因高温缺水而亡。
   ⑵但是,罗布泊原来并不是一片沙漠。据史书记载,在四世纪时,罗布泊水面超过20万平方千米。到了十世纪初,还有1000平方千米的水域。瑞典探险家斯文在二十世纪三十年代进罗布泊时,还乘小舟。他在回国后写的那部著名的《亚洲腹地探险八年》中写道:“罗布泊像座仙湖,水面像镜子一样……”
   ⑶那么,罗布泊的“泊”怎么不见了?
   ⑷问题出在塔里木河。塔里木河全长1321千米,是中国第一世界第二的内陆河。到了本世纪六七十年代,塔里木河两岸人口剧增,出现了四个“盲目”:盲目开垦耕地灌水,盲目修建水库截水,盲目掘堤引水,盲目建泵站抽水。四个“盲目”使塔里木河的大部分水被吸去,使塔里木河的长度由六十年代的1321千米萎缩到不足1000千米,320千米的河道干涸了。河岸两边的五万亩耕地受到威胁;罗布泊成了一个死湖。此后,这里的草本植物全部枯死,防沙卫士胡杨林成片死亡,沙漠不断向湖中心推进。20多年前,曾是“一望草湖,村舍不断……,水鸟群飞”的台特马湖,成了新的荒漠,而罗布泊也与塔克拉玛干大沙漠连成一体。活人走进去,就很难走出来。
   ⑸塔里木河的干涸,导致了罗布泊的消失。到头来,罗布泊也无情地报复了人类。斯文当时看到的水面像镜子一样的“仙湖”已不复存在,人们见到的是不时而来的滚滚沙漠风暴,一片昏天黑地,飞沙走石,沙石的撞击声如急雨降临,沙流像一条黄龙一样在地面上滚翻,地上会突然冒出许多飞动的白气流,顷刻间,平地堆起了一座沙丘,彭加木很可能就葬身于沙丘之中。这是多么可怕的一幕。
   ⑹生态平衡,真是一个奇妙的东西。它存在着,人们不觉得什么;但是,你破坏了它,只是一点点,也许依然不觉得什么,大自然还是宽容的;但是,如果破坏程度到了天地为之不容的地步,大自然就要变脸,主要翻脸不认人,对你不客气。它咆哮起来,山河变色,天地呜咽,飞沙走石,寸草不生,洪波接天,赤地千里,它将要从人们手里夺回被侵吞的一切。到那时,急功近利者就悔之晚矣!但是,欠账总是要还的,要连本带利的还。当然,迟还不如早还,早还损失可以少得多。
   1.根据文段⑴,用精炼的语言概括出罗布泊环境的特点。
   答:
   2.文段⑵的中心句是 。为了说明这个中心句,作者运用了 和 两方面的材料。
   3.罗布泊变成死湖后,其周围生态环境发生了怎样的变化?
   答:
   4.请用简要的文字概括作者写这篇短文的目的。
   答:
   5.请你为本文拟一个恰当的标题。
   答:
   6.从生态平衡角度,谈谈塔里木河、胡杨林、罗布泊、沙漠之间的关系。
   答:
   7.近年来,为了改善塔里木河流域的生态环境,国家采取了哪些具体措施?你还有什么更好的建议吗?
   答:
  
   (三十)(2004·青岛)
   走近纳米技术
   [阅读提示]
   人类对物质世界的认识不断发生革命性的飞跃。从最初的分子、原子到电子以及质子、中子等更小的微粒的发现,人类正不断揭开微观物质世界的神秘面纱。随着这一认识的不断深入,纳米技术应运而生。纳米技术也叫超微科技,是21世纪的一个全新的概念,它研究和制造极其微小的物体(十亿分之一米),广泛用于工业、农业、军事等各个领域。
   ⑴1959年,著名物理学家、诺贝尔奖获得者理查德·费曼预言,人类可以用小的机器制作更小的机器,最后将变成根据人类意愿,逐个地排列原子,制造产品,这是关于纳米技术最早的梦想。
   ⑵近年来,一些国家纷纷制定相关战略或计划,投入巨资抢占纳米技术战略高地。日本设立纳米材料研究中心,把纳米技术列入新5年科技基本计划的研究重点;德国专门建立纳米技术研究网;美国将纳米计划视为下一次工业革命的核心,美国政府部门将纳米科技基础研究方面的投资从1997年的1.16亿美元增加到2001年的4.97亿美元。
   ⑶纳米技术制造的产品固然小得令人眼花缭乱,可是有什么用途呢?老实说,由于这个技术发展的时间还很短,可以说还处于打基础的阶段,目前应用并不广泛,但是,就是这样,其应用前景已经显现在人们面前,仅2001年,纳米技术产品的销售额就高达500亿美元,而德国萨尔布鲁肯市的一个专门研究纳米化学的研究所,就有90多家来自全世界的公司前来签订合同和合作协议。下面我们只择其要者略举几项。
   ⑷首先,当然还是计算机信息存储芯片,它越小而存储量越大,计算机就会在体积缩小的同时,增大性能。今年年初,克林顿总统在宣布为美国纳米技术研究拔款50亿美元的讲话中赞叹地说,“请大家想像,整个国会图书馆的图书都能存储在一个糖块儿大小的芯片中”,这该是多么令人惊奇。
   ⑸其次,使用纳米技术可以选定原子来构成分子,这样就可以制造新物质,而这样的应用多得简直无法叙说。仅就涂料来说,将使用纳米技术制造出来的硬度极强的涂料涂在刀具上,机械工人就不会因刀具不锋利而苦恼了;将抗磨的涂料镀在玻璃和眼镜片上,玻璃和镜片再也不会有划痕;将抗热又抗压的涂料镀在建筑物的玻璃幕墙上,不仅抗压,而且可以不让阳光透过,甚至可以防火;将不产生摩擦的涂料镀在输液和排液的医疗用管道内壁,护士就不会为总在管道中有遗留物而发愁了。
   ⑹至于新材料的应用,就连想像力极强的人也无法全部想到,这里只举关于能嗅到气味的材料的几个应用例子就可见一斑。将这种物质放入冰箱中就能检测里面食物的新鲜程度;将这种物质放入牙刷中,医生可以根据它检测出来的口腔气味来判断人的健康程度和生病的迹象;将这种物质放入电缆中,在电缆即将要燃烧但还没有燃烧的时刻,它就可以报警,这是非常重要的。几天前报道的美国一家航空公司在4年前的失事原因,就是因为电缆起火而引起发动机起火的,莫斯科电视塔的大火也怀疑是由于电缆起火造成的。如果使用纳米技术产品,一切就平安无事了。
   ⑺一个新技术的发展,必然要导致其他技术的发展,因此,难怪从事纳米技术的科学家骄傲地宣称:“我们正在引起一场新的技术革命。”
   1.解释文中加点的词语。
   “至于新材料的应用,就连想像力极强的人也无法全部想到,这里只举关于能嗅到气味的材料的几个应用例子就可见一斑。”
   可见一斑:
   2.通读全文,试说说本文围绕纳米技术主要说明了什么内容。
   答:
   3.通过本文的阅读,你对纳米技术肯定有了一定的了解。请联系课文内容,结合“阅读提示”,根据自己的理解,给“纳米技术”下一个定义。
   答:
   4.从本文看,人类运用纳米技术取得了哪几个方面的成果?
   答:
   5.本文在说明纳米技术的优越性时,运用了大量生产和生活事例。请你说说这样写的好处。
   答:
   6.纳米技术广泛的应用前景就连相像力极强的人也无法全部想到。请结合对本文内容的理解,联系生活实际,发挥联想和想像,设计一个运用纳米技术的科技小制作。(要求:说出小制作的名称、特点和用途)
   名称:
   特点:
   用途:
  
  
   (三十一)(2004·四省区)
   一把钥匙开一把锁
   一把钥匙开一把锁,这是人人皆知的生活常识。当你拿错钥匙开不开锁的时候,就甭提有多着急了。但是钥匙和锁配不上,再着急也没用。
   在人体内部所进行的各种生物化学反应,也像一把把锁一样,需要有一把把相应的钥匙来打开。不然的话,反应就无法进行,生命活动也就停止了。那么,到哪里去找钥匙呢?这一把把“钥匙”就是酶!也就是说,一种酶一般只对一种生物化学反应起作用。科学上把酶的这种“一把钥匙开一把锁”的特性叫做专一性。例如淀粉酶只对淀粉起作用,使淀粉很快地分解成麦芽糖;脂肪酶只对脂肪起作用,使脂肪分解成为甘油和脂肪酸。蛋白质有很多种,相应的蛋白酶也就有很多种。例如,人胃肠中的蛋白酶能加速食物中的蛋白质分解成为氨基酸,而寄生在肠道中的蛔虫也含蛋白质,胃肠里的蛋白酶对蛔虫就丝毫不起作用,这就是锁和钥匙不配对。如果让蛔虫遇到从木瓜树果实中提炼出来的木瓜蛋白酶,蛔虫体内的蛋白质会被它分解得支离破碎,这就是锁和钥匙配对了。
   说来也有趣,每一种酶的表面都有自己特殊的形状,这些形状与它们所催化的物质的形状恰好咬合,就像锁和钥匙配对一样,参加化学变化的物质和酶一经配好对,钥匙插入锁孔,一扭动,“咔嚓”一声,生命之锁就打开了!
   人体中有多少种“锁”,需要配多少种“钥匙”,细胞核中蛋白质制造厂的总工程师早已胸有成竹。当需要某种钥匙的时候,总工程师就下达密码指令,细胞质就按指令生产。
   其实,酶在反应中只是催促反应的快速进行,自身并不消耗掉。这样是不是每种酶中生产一次就够用了呢?不是的,就像钥匙有时会丢失或折断一样,酶也会受到破坏。不过,这不要紧,蛋白质制造厂会随时生产出新的酶来补充的。
   我们完全可以这么说:没有酶,就没有生命。
   1.本文所阐释的主要事理是( )
   A.生命活动离不开酶
   B.一种酶一般只对一种生物化学反应起作用
   C.酶是生物化学反应中的“钥匙”
   D.蛋白质制造厂能随时生产新的酶
   2.本文主要运用了打比方和举例子的说明方法。请任选一种,并从文中找出一个相应的例子,具体说明运用这种方法的作用。
   例子:
   作用:
   3.下面句中的加点词不能去掉,结合说明文语言的特点,作具体分析。
   一种酶一般只对一种生物化学反应起作用。
   答:
   4.根据文中提供的信息,完成有关酶的科学知识卡片。
  
  
  
  
  
   5.人体汗液分泌出的蛋白质,浸渍到衣服上很难清洗,因此市场上出现了加酶洗衣粉。请综合文中知识,用简洁准确的语言解释加酶洗衣粉的洗衣原理。
   答:
  
  
   (三十二)(2004·郫县)
   电子书的普及出现转机
   不久的将来,学生们不用背着沉甸甸的书包去上学了,取而代之的是人手一本小巧的“电子书”。这本电子书里包括了上课所需要的所有教材,同学们还可以把自己喜欢的课外读物也存储在这本电子书中,只要轻轻一按,就能显示出当天应该学习的章节,省却了一页页翻书的繁琐。
   这可不是不切实际的空想,国外已经有多家厂商开发并推出了这种电子书。日本松下公司已于2004年2月份开始供应一种名为“西格玛书”的电子书。该产品使用对开式液晶屏幕,可以像书一样折叠,屏幕规格为7.2英寸(1英寸为2.5厘米),最大分辨率为1024×768像素,在白色背景中显示蓝色文字。书籍内容储存在小型存贮卡“SD”中,支持面向可擦写光盘的版权保护技术“CPRM(可刻录介质内容保护)”。该产品打开后的尺寸为29厘米×20厘米,大概相当于“A4”尺寸,厚度1.3厘米,质量约550克(不含电池)。
   相比之下,日本索尼公司2004年4月24日开始供货的一款超薄型电子书,虽然只有分辨率为800×600像素的6英寸单面显示器,但其外形要小巧许多,它宽126毫米,高190毫米,厚13毫米,质量仅190克(不含电池),足可单手持机阅读,内置约10MB内存,可存储大约20册数据资料(一册250页的小说约500KB)。相对于西格玛书单纯的读书功能,索尼公司推出的这款电子书内置了4本电子词典,附带声音播放和检索功能,可用于朗读文章和学习外语。
   也许你会觉得拥有这些功能的电子书已经很完善了,但那些不断在开发新产品的科技人员可不是这样想的。日本东芝公司计划在2004年秋季投产采用低温多晶硅TFT液晶面板的彩色电子书籍SD—BOOK,该电子图书可以像普通书籍一样打开使用,左右各配备一个7.7英寸640×960像素的彩色液晶面板,大小相当于B5纸。由于采用了彩色显示装置,阅读时可以观看彩色图片,还可以观看带插图的小说以及彩色漫画等。投产所面临的问题之一是如何减轻产品质量。目前样机的质量为1.4千克,目标是减轻到1千克以下;减小质量的措施包括芯片的集成、采用更轻的材料等。
   日本业内人士指出,如果电子书籍得到普及,目前的出版市场将会扩大5%,还能够节省大笔的纸浆和人工成本,消费者每一分钱买到的都是有价值的信息。但是,电子书籍是否真的有如此实力还是个未知数。有关厂商的计划是:将电子书的主要用户锁定为商务人士,以此为突破口,然后向家庭用户普及。
   据有关媒体报道,松下公司计划于2004年9月实验性向北京大家附属中小学提供西格玛书这种手持阅读器。如果这一计划成功,我国教育产业电子信息化的进程无疑又迈出了重要的一步。
   也许在若干年后,许多年轻人家中根本就没有传统的书,也不需要买报纸,因为所有的信息都可以即时传送到电子书中阅读。到时,纸上谈兵的时代就一去不复返了。
   1.根据本文提供的信息,下面说法不正确的一项是( )
   A.日本松下公司推出的“西格玛书”是一种像书一样可以折叠并使用对开式液晶屏幕的电子书。
   B.日本索尼公司生产的超薄型电子书从其外形看,比“西格玛书”电子书要小巧许多。
   C.电子书籍目前有了较大普及,它的出现使大笔的纸浆和人工成本得到节省。
   D.国外厂商正在不断地开发电子书,电子书的功能将会不断完善,给我们学习生活带来更大的方便。
   2.日本松下公司推出的“西格玛书”电子书和日本索尼公司推出的“超薄型电子书”各有其特点和优势。从这两种电子书的质量和功能角度看,哪一种优势更明显?请作简要说明。
   答:
   3.与日本松下的“西格玛书”电子书和索尼公司的超薄型电子书相比较,日本东芝公司即将投产的彩色电子书籍SD——BOOK,在功能的完善上体现在哪里?目前要解决的问题是什么?
   答:
   4.文中说“如果这一计划成功,我国教育产业电子信息化的进程无疑又迈出了重要的一步”这里的“又”能否删去?为什么?
   答:
   5.文章最后说:“也许若干年后,许多年轻人家中根本就没有传统的书,也不需要买报纸。”对此,有人认为电子书的普及带来了阅读的方便,也有人认为电子书的普及带来传统书的消失并不是一件好事。你是如何认为的?请简要谈谈你的看法。
   答:
  
   (三十三)(2004·海口)
   人类何时飞出太阳系
   人类的航天时代,如果从1957年苏联发射第一颗人造卫星算起,已有40多年的历史。这期间,人类挣脱了地球的引力,飞出了地球村,登上了月球。可人类总嫌飞天的步伐太慢,总以焦急的心情盼望着——如何才能飞出太阳系,进入宇航时代。20世纪70年代人类放飞的太空天使“先驱者”和“旅行者”探测器,到目前已飞出了二三十年,跋涉60亿千米,已飞越了最远的冥王星轨道,但它是否已飞出了太阳系呢?
   其实,太阳系的边界十分广阔,如果以太阳风影响所及的范围来计算,半径可达135亿千米~180亿千米;如果以太阳的引力所及的范围来计算,半径可达15万亿千米,约1.5光年。由此可见,上述的飞行探测器离太阳系的边界还差得很远很远。从宇宙的尺度来说,太阳系只是沧海一粟,跨越一条小河沟容易,要跨越大洋,就必须具备足够的条件。就目前人类对宇航的了解认识和所具有的航天技术,对宇航来讲是远远不够的,还远远不能适应飞出太阳系的要求。
   所以“先驱者”与“旅行者”探测器只能算是冲出地球大气层的“航天”,还远远谈不上冲出太阳系的“宇航”。
   对于飞出太阳系的宇宙航行来说,眼下还只是一张白纸,任凭人们去设计、去幻想。目前人们的设想之一是必须开发质量小、作用时间长和高能的空间动力能源。如电能火箭、激光火箭、核能火箭以及微波动力飞船、激光动力飞船、反物质推进星际飞船等等。必须大大提高宇宙飞船的速度,以每秒几百千米、几千千米、几万千米甚至以接近光速的速度飞行,这样,人们在数十年的有生之年,才有可能飞出太阳系,去别的恒星系、银河系或更遥远的星系进行星际航行。
   何时能飞出太阳系,取决于我们地球人类的智慧和科学技术的发展水平。努力吧,也许在几十年、上百年后,人类向往星际航行的梦想就会成为现实。
   1.本文是按什么顺序来说明的?请从文中摘录出能体现这一顺序的词语。
   答:
   2.联系全文内容,说说第2段中加点的“沧海一粟”一词有什么表达效果。
   答:
   3.请简要概括出“先驱者”与“旅行者”没有飞出太阳系的原因。
   答:
   4.人类要飞出太阳系实现宇宙航行,必须解决的两大主要问题是什么?你运用什么方法得出这一答案的?
   答:
   5.下列两小题,难度各异,分值不同,请任选一题作答。
   ①第3段在全文结构上起什么作用?
   答:
   ②如果有一天人类飞出了太阳系,实现了星际航行的梦想,能够充分利用太空资源,人类的生活将会发生怎样的变化?请发挥你的想像,描述一个人类未来生活美景的片断。(不超过80个字)
   答:

 第十二讲 议论文阅读

  
   【考查要点】
   在整体上,考查的重点是对文章的整体感知、理解和领悟能力,如分析、概括文章的思想内容,对全文中心论点的辨析、判断、提取、归纳,对全文论证结构的梳理、划分,对全文论证方法的辨识、理解,对全文论据类型的认识、分析,理解文中的表现手法如语言特色、比喻论证、正反论证,结合文章内容谈自己的看法或感想等等。
   在局部或细节上,概括文段的大意,分析文段的思想内容,分析文段的层次结构,快速辨识文段的中心句、关键句,理解文段的论证方法或表达手法,提取与概括文中的论点,品读段与段之间的关系,体味、理解关键词语与重点句子的含义及表达作用,等等。
   【知识疏理】
   1.论点、论据、论证、驳论方面的知识。2.全文中心论点的判断。3.全文结构与议论的层次性分析,用提纲概括全文每层次论述的大意。4.全文分论点的表现形式与层次性,编写段落提纲。5.全文段落层次的划分,段的中心句的理解,段意的概括。6.论证的要点和层次。7.全文中论证方法的使用。8.正面论证与反面论证。9.驳论点的写法与驳论据的写法。10.句子的哲理含义的理解,长句的理解,段中的句序。11.句式与表达思想感情的作用。12.设问和反问的作用。13.句中感情色彩鲜明的词语的含义与作用。
   【试题特点】
   题型以填写简答题为主,少有选择题。填写简答题的出题角度比较丰富,填写、概括、答问、解释、阐述、自由表达等为主要出题角度。大多数选文都设计有开放式、迁移式的考题。
   【解题导引】
   例1.(2004·徐州)
   理性的阅读
   世间许多事情都是经过不断地积累经验才会上升到一定的层次,读书也不例外。对于读书,人们说得最多的话题不外乎多读、勤读。多读、勤读固然能够学到一些知识,但是读书倘若能够“深”入书中,即使读的书数量不是很多,也能尝到读书的滋味,瓴悟到更多的道理。这就需要克服随意性,多一些理性。
   随着社会的发展,书籍的数量不断增多,在读书上人们只有进行一番选择,才能找到引起自己阅读兴趣又有阅读价值的书。而不加选择的阅读,至少说明在读书上还不成熟。一位学者把读书的艺术概括为不读的艺术,理由是:“书为无限,生命有限;以有限应无限,只能采用此策。”所谓“不读”,实际上就是要有选择地阅读,多读经典著作,少读甚至不读流行作品,更不用说那些粗制滥造的文字垃圾。从某种意义上说,读书的水平也反映在书籍的选择上,犹如在琳琅满目的货架上,就看谁有眼力能拿到货真价实的东西。而选择的对象,恰恰也能标明一个人的情趣、欣赏水平乃至人生追求。因此,读什么书会像一面镜子,将人们的精神境界映照得一清二楚。
   同是一本书,有的人读后没有留下什么印象,时间久了,脑海中只剩下一片空白;而有的人读后,不仅能记住书中的内容,甚至能背诵出精彩的语句,更能谈出自己对这本书的看法。读书的这种差异,就在于读书者是用眼读还是用心去读。孔子说:“学而不思则罔,思而不学则殆。”用“心”阅读,正在于读书的同时也伴随着不断的思考。
   记得杨绛先生曾把读书比作串门。串门总要有进有出,读书也如此。认真地读一本书,就会情不自禁地进入到一种环境之中;情感随着书中情节的变化而起伏,思想沿着作者的思路向前发展。读书不仅应该能够读进去,重要的是能跳出书外,静观默想,分析对比,理清哪些是对的,哪些是错的。这样的阅读会使人从欣赏水平到修养都得到提高。不读“死”书,意为不去读那些毫无生气的书;不死读书,还在于不可尽信书,而要有主见。理性的阅读会引导人们在知识的台阶上攀得更高。
   (选自《青年科学》2004年第1期,略有改动)
   1.读了这篇文章,你认为作者阐述的主要观点是什么?
   2.文章围绕主要观点从三个方面进行了具体阐述,它们分别阐述了什么?
   ①
   ②
   ③
   3.文中说“杨绛先生曾把读书比作串门”,“串门总要有进有出”。这里的“进”和“出”分别指什么?
   “进”是指:
   “出”是指:
   4.就本文提出的读书经验,请你选择其中的一点结合自己的实际谈谈看法。赞同与否都要说出自己的理由。
   第1题的考查目的是概括、提取全文的主要观点,这就要求考生总览全文,进行感受与提炼,这是设题的第一个层次。第2题的考查目的是分析、概括全文论述的主要内容,这就要求考生对选文进行比较细致的阅读,对选文内容进行概括,这是设题的第二个层次。第3题的考查目的是在具体的语境中对文中词句的含义进行品味,这关涉到精段的阅读理解,这就要求考生对文中最后一段进行更加细致的品读,这是设题的第三个层次。第4题的考查目的是让考生结合自己的读书实践对文中的观点进行谈论,这是有机的恰到好处的迁移而不是硬贴上去的问题,每一位考生都有话可说。这是设题的第四个层次。
   (参考答案:1.阅读要克服随意性,多一些理性。(要理性的阅读)(理性的阅读会引导人们在知识的台阶上攀得更高) 2.要有选择地阅读、读书的同时也伴随着不断的思考 (要用心去读书)、读书要读进去,跳出来。 3.“情不自禁地进入到一种环境之中;情感随着书中情节的变化而起伏,思想沿着作者的思路向前发展”、“跳出书外,静观默想,分析对比,理清哪些是对的,哪些是错的。” 4.略)
   解答议论文阅读题可从以下几个方面入手。
   第一,在阅读文章或段落时先要有“自测”的意识。这种意义和阅读糅和在一起,在阅读之中瞬间的辨识和判断。其基本内容有:1.文或段是立论还是驳论。2.全文或全段的大意是什么。3.论点及其出现的位置。4.中心句与支撑句的关系表现在哪里。5.文章或段落的论证方式是以事例为主还是以事理为主,或是二者兼而有之。6.有没有比喻论证的方法,有没有正反论证的方法,有没有引用,有没有设例。7.结构层次是总分、总分总,还是分总或者并列。8.文中的语言表现出什么特色。带着上述问题阅读文段,有两好处:一是有目的地读文,能够读得深刻些;二是进行了思考,心中有了一些底,答题的过程就会顺畅一些。
   第二,在答题的过程中要注意操作的步骤。1.要了解题型。题型用于议论文阅读,主要有简答题、填写题、选择题。表示题型的主要标志是题干,我们要通过阅读题干知道题型的类型,以便正确使用答题的方法。2.要认真审读答题要求,特别是简答填写题的答题要求。3.对自由表达的题目要进行周密的思考,写出的答案要注意有清晰的层次,使别人容易看出答题的要点。
   【常见失误】
   议论文常见的答题误区有:对文或段的思想内容的分析、概括不准确,对文或段的一些表达手法的理解如语言特色、比喻论证、正反论证等不能进行简明准确的阐释,不能准确地对文句进行品析,解释不清词句的含义,不了解词、句、段的表达作用和表达效果,难以结合文章内容谈自己的看法或感想,讲不出答题的理由等等。
   【考点精练】
   (一)(2004·北京)
   要成为你自己
   周国平
   童年和少年是充满理想的美好时期。如果我问你们,你们将来想成为怎样的人,你们一定会给我许多漂亮的回答。譬如说,想成为拿破仑那样的伟人,爱因斯坦那样的大科学家,曹学芹那样的文豪,等等。这些回答都不坏,不过,我认为比这一切都更重要的是,首先要成为你自己。
   姑且假定你特别崇拜拿破仑,成为像他那样的盖世英雄是你最大的愿望。好吧,我问你:就让你成为拿破仑,生成在他那个时代,有他那些经历,你愿意吗?你很可能会激动得喊起来:太愿意啦!我再问你:让你从身体到灵魂整个儿都变成他,你也愿意吗?这下你或许有些犹豫了,会这么想:整个儿变成了他,不就是没有我自己了吗?对了,我的朋友,正是这样。那么,你不愿意了?当然喽,因为这意味着世界上曾经有过拿破仑,这个事实没有改变,唯一的变化是你压根儿不存在了。
   由此可见,对于每一个人来说,最宝贵的还是他自己。无论他多么羡慕别的什么人,如果让他彻头彻尾成为这个别人而不再是自己,谁都不肯了。
   也许你会反驳我:你说的真是废话,每个人都已经是他自己了,怎么会彻头彻尾成为别人呢?不错,我只是在假设一种情形,这种情形不可能完全按照我所说的方式发生。不过,在实际生活中,类似情形却常常在以稍微不同的方式发生着。世上有许多人,你可以说他是随便什么东西,一种职业、一种身份、一个角色,或别的什么,惟独不是他自己。如果一个人总是按照别人的意见生活,没有自己的独立思考,总是为外在的事务忙碌,没有自己的内心生活,那么,说他不是他自己就一点也没有冤枉他。因为确确实实,从他的头脑到他的心灵,你在其中已经找不到丝毫真正属于他自己的东西了,他只是别人的一个影子或事务的一架机器罢了。可见,真正成为自己可不是一件容易的事。
   ①那么,怎样才能成为自己呢?②这是真正的难题,我承认我给不出一个答案。③我还相信,不存在一个适用于一切人的答案。④我只能说,最重要的是每个人都要真切地意识到他的“自我”的宝贵,有了这个觉悟,他就会自己去寻找属于他的答案。⑤在茫茫宇宙间,每个人都只有一次生存的机会,都是一个独一无二、不可重复的存在。⑥正像卢梭所说的,上帝把你造出来后,就把那个属于你的特定的模子打碎了。⑦名声、财产、知识等等都是身外之物,人人都可求而得之,但你对人生的独特感受是没有人能够替代的。⑧你死之后,没有人能够代替你再活一次。⑨如果你真正意识到了这一点,你就会明白,活在世上,最重要的就是活出你自己的特色和滋味来。
   你的人生是否有意义,衡量的标准不是外在的成功,而是你对积极人生的独特领悟和坚守。坚持这一标准,你的自我才能闪放出个性的光华。
   (选自《周国平文选》,略有删改)
   1.第3段中“彻头彻尾”指的是什么?
   答:
   2.文章第2、3段针对 的人进行议论,提出了“对于每一个人来说,最宝贵的还是他自己”的观点;第4段针对生活中自以为成了自己的人进行分析,阐明了
   的道理。
   3.第4段中,作者把生活中的某一种人比作“事务的一架机器”,这种人的具体表现是
   。(用原文回答)
   4.对第5段内容理解有误的一项是( )
   A.要成为你自己,首先要真正意识到“自我”的宝贵。
   B.作者借卢梭的话证明第⑤句所表述的观点。
   C.第⑨句中的“这一点”指的是“名声、财产、知识等等都是身外之物,人人都可求而得之”。
   D.“怎样才能成为自己”,这是一个难题,作者认为不存在一个适用于一切人的答案。
   5.文章最后一句是“坚持这一标准,你的自我才能闪放出个性的光华”。其中,“这一标准”是什么?你怎么理解文章最后这句话?
   答:
   (二)(2004·上海)
   蒙娜丽莎微笑揭秘
   ⑴近五百年来,人们对文艺复兴时期艺术大师达·芬奇的名画《蒙娜丽莎》怀着极大的兴趣,尤其是她那神秘的微笑,引起了种种猜测和议论.
   ⑵蒙娜丽莎微笑的神秘之处在于:当你注视她的笑容时,先是看到微笑,既而似乎隐去,然后微笑重新出现,过一会儿再次神秘地隐去……
   ⑶哈佛大学脑神经科学家玛格丽特博士,是一位研究视觉系统的权威学者,特别对人的眼睛和大脑如何处理视觉形象的光照度和对比度感兴趣。她认为蒙娜丽莎微笑之所以时隐时现,是由于视觉系统的本性所致。她在一本名为《美术故事》的书中读到这样一段:”我知道你已看这幅画不下一百次,但是再看,就这么看……”她照着做了——近看远看,左看右看,上看下看……她感觉《蒙娜丽莎》具有一种闪烁的特性,但还是不明白到底是为什么。一天,她在骑自行车回家的途中,忽有所悟:微笑的忽隐忽现是由于我眼睛注视的部位不同所造成的。玛格丽特大喜过望:蒙娜丽莎微笑之迹终于解开了!
   ⑷玛格丽特解释说,人眼的视网膜具有两个不同的区域:中心的小凹区对阴影不敏感,而善于分辨彩色和细节:环绕小凹区的外围区对彩色和细节不敏感却善于辨别运动和阴影.人们在欣赏蒙娜丽莎时专注于她的美目,视网膜的外围区恰好落在她的嘴部和面颊部。由于外围区善于辨别阴影,将蒙娜丽莎嘴角和颧骨部位的曲线突出了,从而显示出笑容。人们发现蒙娜丽莎在微笑,很自然地将视线移到她的嘴部,在看她的嘴时,会发现笑容消失了
   ⑸为了证实这个论断,玛格丽特用电脑将蒙娜丽莎的面部进行了图像处理。面部阴影完全消除时,笑容也随之消失了。面部阴影逐渐加强时,笑容就出现了.蒙娜丽莎微笑的时隐时现,原来是人们的视线在画面上游移所致。
   ⑹至此,有人会说:“让蒙娜丽莎微笑的神秘美流传百世不是更好吗?为什么非得要去揭秘呢?科学家真是无事忙。就算你对了,将旷世奇美归结成眼球的转动、视线的游移、脑电波的闪烁,还有什么电脑图像处理等等,以这些世俗之物来亵渎艺术大师,岂不大煞风景?达·芬奇在天之灵闻之,当跌足叹日:焚琴煮鹤,莫此为甚!”
   ⑺但也有人会说:“ ”
   ⑻孰是孰非?惟有去问达·芬奇本人了。
   相关链接:
  
  
  
  
  
  
  
   1.阅读正文和相关链接,蒙娜丽莎的微笑被称为“神秘的微笑“是因为:
   (1) (2)
   2.细读第4段,联系上下文,完成下表。
   视网膜区域名称 功能 论断
  
  
   3.第6段中加点的“此”指的是 ,“焚琴煮鹤”是指拿琴当柴烧,在文中比喻 。
   4.从玛格丽特揭秘过程中可以看出,要探究一个事物,需要具备一定的素质。填写出其中三点。(1); (2); (3)
   5.从文中推断,第7段的内容显然应与第6段相对。请你用一段话来驳斥对方(即第6段)的观点,阐明自己的见解。
   答:
   (三)(2004·重庆)
   聆听历史是一种伟大的才智
   钱乘旦
   中华民族自古以来就不封闭,中华民族本身就是在长期的兼容并取、包收舍纳中形成的。中华民族在几千年时间里融合了许多古代先民,这个过程就是古代先民相互开放、相互吸收的过程。由于开放,我们的民族取长补短,创造了伟大的文明成就;由于融合,民族的精神保存了活力,历经磨难而终久不息。古代民族的兼容并取,最终铸造了中华民族,由此奠定了东方最灿烂的古代文明。文明生存的条件之一,就是对外开放的环境与心态,环境的封闭看起来是安逸与宁静,但同时也意味着丢失前进的机会。心态的封闭则是自己闭起眼睛来不看竞争的现实,在竞争的环境中碰得头破血流。这样的心态,在世界进入“近代”以来就让中国人吃足了苦头,而当我们克服这种心态后,我们就又起步了。中国近一两百年的历史,就是这样一部“心态史”。所以,一个开放的环境与一个开放的心态,对国家的发展,至关重要。
   那么,开放的心态意味着什么?意味着:抬起眼睛来看别人,既看别人的过去,也看到人的现在。改革开放以来,我们越来越明白,不看别人的现在,那就是夜郎自大,自取落后。但看别人的过去之重要性,却是多数人所忽视的。
   “一切历史都是当代史。”看别人的过去实际上就是在看别人的现在。别人现在成功,他为什么成功——成功的根源在过去;别人现在失措,他为什么失措一一失措的原因也在过去。世界上没有无本之木,现在这棵树,是长在过去这个“本”上。有谁能够相信:一个国家的兴盛,会没有社会、经济、政治、文化等诸因素作为其条件,而这些条件是在一瞬之间一蹴而就的,并不是出自于其历史经历的积累,或其历史创造性的不断涌现?又有谁能够相信:一个国家的挫折,不由其社会、经济、政治、文化等等诸方面的问题所造成,而是一日之间成三尺之寒,并不由其主观、客观各种原因积淀而成为历史的顽疾?不懂历史,就不懂现在;看不懂历史,就看不懂现在。我们如今已经有了开放的心态,懂得要张大眼睛来看别人;但不看别人的过去就等于不看别人的现在,看不懂别人的过去也就看不懂别人的现在。
   历史不仅是一种知识,而且是一种智慧,外国历史不仅是关于外国的历史知识,而且是世界其他民族积累的智慧的结晶。常有人问:学习历史有什么用?我的回答是:历史给人以聪明。为什么这样说?因为历史记录的是前人的经历,提供的是前人经历中的经验和教训。人常说:吃一堑,长一智。经验提供的是成功的智慧,教训使人有可能成功。我们总希望自己成功,不失败。历史的经验与教训,就是成功和不失败的一面镜子。实际上,任何人都站在历史的镜子前,而看不看镜子、能够在镜子中看见什么,这就是智慧的表现。历史虽然无言,但它却会说话;能够听见历史说话已经很了不起了,而能够听懂历史在说什么,这就是伟大的智慧。听懂历史说话是一种巨大的才智,但是想听历史、懂历史,首先就必须学历史、读历史,在历史的知识中聆听历史的教诲。
   当然,要真正让每一个人懂得历史的重要性,不是一时半会儿就能做到的。我们要长期坚持历史教育,恢复我们民族悠久的历史感,而这种历史感,正是我们优秀的文化传统之一。但是对这种传统,我们需要做一点补充,这就是:“不仅要学习中国历史,还要学习世界历史;不仅要有深远的历史眼光,而且要有宽广的世界眼光。”我们民族已经走向世界了,我们看过去,不仅看我们自己,也看全世界;而这就意味着:我们展望未来,不仅展望自己,也展望全人类。
   1.作者写这篇文章的目的在于提倡学习历史。纵观全文,作者是从哪两个方面来论述为什么要学习历史的?
   答:
   2.文中“环境的封闭”和“心态的封闭”各指什么?
   答:“环境的封闭”是指:
   “心态的封闭”是指:
   3.“一切历史都是当代史”这句话具有丰富的内涵,请你谈谈这句话在文中的含义。
   答:
   4.作者为什么说“聆听历史是一种伟大的才智?”请联系全文分析,用自己的话概括作答。
   答:
   5.世纪是一个多元化的、高速发展的新型社会,竞争更为激烈,开放的环境和心态无论是对国家还是个人都是至关重要的。联系这篇文章,谈谈你现在会怎样用开放的心态去看待周围的人或事。请联系现实生活中的实例谈谈你的看法。
   答:
  
   (四)(2004·辽宁)
   ①曾经欣赏过台湾漫画家几米一幅题为《有效期限》的漫画,画的中心是一片浅绿的水,上部有一些叶片粗大、开满了紫花的藤儿,中间偏下是两块石头,大石头上坐着一大一小两个人,小石头上蹲着一只好奇的小青蛙,左下角一只小纸船正悄然无声地驶来。旁边有诗云:
   一艘小纸船,
   悠悠地飘过来,
   吸饱水分,
   渐渐沉没。
   世界上所有的美好,
   都有有效期限。
   ②美好的事物永远是有有效期限的。我们喜欢梅花,是因为它独自绽放在寒冷的冬天;我们喜欢菊花,是因为它只是微笑在金色的秋日; 。假若世界上的花朵没有有效期限,我们对花的那份期待就会大打折扣。世界上所有的美都有有效期限,所以我们应该学会珍惜。
   ③珍惜亲情的“有效期限”。父母可以陪伴你的上半生,却无法呵护你的下半生;儿女能够陪伴你的下半生,却不可能参与你的上半生……你无法在所有的时空里称心如意拥有你想要的全部天伦之乐,就像一只鸟无法在每一个季节都拥有自己优美的歌喉。于是,有了孔子“父母在,不远游”的教诲;有了“香九龄,能灌席”的孝行;有了“子欲养而亲不待”的慨叹……亲情的“有效期限”贯穿了我们生命的全过程,充塞着我们心灵的每一个角落。
   ④珍惜梦想的“有效期限”。每个人都拥有自己的梦想,有了梦想却不为之付出真诚努力,梦想便会成为空想。怀抱强国梦想的屈原,用血泪凝结成千古传诵的《离骚》;济世救人为目标的李时珍,踏千山尝百草著成鸿篇巨著《本草纲目》;视科学研究为生的居里夫人,历经十二年的努力,提炼出世界上第一克镭……梦想的“有效期限”激发人们的进取精神。
   ⑤珍惜生命的“有效期限”。庄子说,“人生天地间,若白驹过隙”。因此,司马光用警枕提醒自己珍惜时间;雨果剃去了头发和胡须来拒绝浪费生命;鲁迅把别人喝咖啡时间用在不间断的工作上……生命向我们昭示这样的真理:越是害怕时间消失的人,他们脚步走得越远,生命的半径越大;越是觉得时间过剩的人,他们的世界越小。
   ⑥世界上所有的美好,都有有效期限,这是大自然不可移易的规律,所以我们必须学会珍惜。
   (选自《黄河黄土黄种人》,作者游宇明,有改动) │
   1.结合上下文语境,仿照第②段画线语句,在下面写出一个句子。
   答:
   2.选文的中心论点是什么?
   答:
  
   3.选文第①段引用几米小诗的作用是什么?
   答:
   4.诗中的小纸船象征着什么?
   答:
   5.选文第③段运用了哪些论证方法?
   答:
   6.根据你的积累,再写出两个珍惜时间的名言、警句,并注明作者或出处。
   答:
   7.选文中有许多值得品味的地方,把你最喜欢的句子写在下面,联系生活实际,谈谈你的感受。
   答:
   (五)(2004·河北)
   学会谅解
   ①谅解是人类的美德,是一种高尚 的品质。有人这样形容:谅解是一股和煦的春风,能消融凝结在人们心中的坚冰;谅解是 ,能 ;谅解是  ,能      。这生动地道 出了谅解在人际交往中的重要作用。因而一位伟人意味深长地说:同志之间的谅解、支持与友谊比什么都重要!
   ②确实,谅解非常重要。先哲们早就认识到这一点。圣人孔子曾说:“己所不欲,勿施于人。”意思是说,无论做什么事,都要推己及人,将心比心,以自己的感受去体会别人的感受,以自己的处境去推想别人的处境。这种以己推人的思想就包含了理解他人、谅解他人的深刻含义。唐代韩愈在《原毁》中说:“古之君子,其责己也重以周,其待人也轻以约。”强调做人要严于律已,宽以待人,同样体现了人际交往中的谅解精神。
   ③谅解在我国传统的伦理道德观念中,一直占有重要的位置,是为人处事的重要原则。
   ④唐太宗李世民谅解、重用魏征的故事被传为佳话。在李建成与李世民的皇位之争中,魏征为李建成出谋划策,多次使李世民陷入困境。玄武门之变后,魏征成了李世民的阶下囚。这时,李世民完全可以治他的罪,杀他的头。但是,李世民十分欣赏魏征的才干和人品,于是便谅解并重用了他,让他做到了宰相,帮助自己成就了历史上著名的“贞观之治”。这里虽然有李世民作为一代明君尊贤爱才的一面,但我们不能不认识到,这也与他宽宏大度、关于谅解他人分不开。再往前翻一翻历史的画卷,齐桓公谅解并重用曾险些射死自己的管仲,从而成就了霸业;蔺相如谅解多次羞辱自己的廉颇,留下“将相和”的美谈……相反,那些小肚鸡肠、斤斤计较之人,又有哪一个能作出一番事业呢?
   ⑤古人尚能如此,在社会生活日趋复杂的今天,人与人之间更需要谅解。那么,怎样才能学会谅解呢?
   ⑥谅解,需要沟通。客观事物纷繁复杂,个人的思想认识常常带有一定的局限性和片面性,人与人之间难免会产生误解和冲突。因此,当自己与他人产生矛盾或误会时,要主动与对方交谈,认真倾听对方的诉说。这样,才能沟通彼此的思想,从而消除误会和隔阂。
   ⑦谅解,需要设身处地地为别人着想。积极地换位思考,更多地站在对方的角度考虑问题,我们就更容易谅解别人。
   ⑧谅解,需要忘却。忘却,是谅解的良方。克制性的谅解,不是真正的谅解,它不能祛除感情伤口上的脓水,医治感情上的创伤。只有那种不记、不究的谅解,才是真正的谅解。
   ⑨我们提倡谅解,但是,谅解也不是无原则的一味迁就、退让。对于那些有损民族、国家和集体得益的行为,还需要勇敢地站出来,与之作坚决的斗争。
   ⑩让我们学人谅解吧!这样,人与人之间的关系才会更加和谐,人们的生活才会更加美好。
   1.文章是从哪两个方面对“谅解”进行论述的?
   答:                                                                               
   2.文章第④段在论证上有什么特点?这样论证的作用是什么?
   答:                                                                                                                   
   3.文章第⑤段在结构上起什么作用?
   答:                                 
   4.如果将第⑨段删去,对文章的论证会产生什么影响?
   答:                                   
   5.仿照文章第①段中的画线部分,将后面的句子补充完整并写在下面。
   谅解是 ,能 ;谅解是  ,能       。

 (十七)(2004·广东)
   ①从医学角度分析,噪音往往能引发身体的疲劳和不适,对人的心理也造成一定的伤害。
   ②60分贝以下的噪音一般不会引起人的厌烦。假如在超过70分贝噪音的环境中,想要更好地休息就得服用镇静剂或安眠药。当然,75分贝的噪音算中等级别,它能影响人的思维和情绪。在我们生活的环境中,常有一些尖利的噪音,它们虽然比较短暂,但其激烈程度可以严重干扰人们的生活。例如:摩托车在行驶中的噪音可以达到100—110分贝,其强度超过电锯锯木发出的声音;摇滚乐队的某些演奏可能产生140分贝的噪音,它大大超过喷气式飞机降落到距地面100米时产生的轰响。科学实验表明,超过115分贝的噪音能引起人的严重的烦躁和不安,这种情况相当于癫痫病发作时的一系列大脑变化;面对160分贝的噪音,
   动物有可能死亡。
   ③虽然我们生活在喧闹声中,但不是人人都能察觉。人的耳朵有一种“掩蔽”功能,能
   自动清除环境噪音,而把那些我们感兴趣的声音突现出来。因此,我们能听见站在人群中或
   公共场所的人对我们的讲话。耳朵还能将讲话人的声音分离出来,同时在一定程度上忽略环
   境噪音。但这只是大脑的一种“应对”技巧,噪音已经丝毫不漏地进入我们的听管和神经
   系统,其后果是,强烈和长时间的噪音会破坏声感受细胞上的纤毛。这些纤毛就是我们的
   “传声器”,随着它们的逐渐被破坏,我们甚至可能失去听觉。长此以往,耳鸣和心悸将接
   踵而来,并且身体的平衡系统也会因此而遭到破坏。从国际上的一些统计数据可以发现,生
   活在喧闹地区的一部分居民的听力正在变得越来越差。在美国64-75岁的人群中有1/4的人失去听力;在意大利这个比例达到了18%,是40年前的两倍。据一些国家征招新兵的人介绍,现在有听力缺陷的年轻人越来越多。
   (选自(科学世界)2004年第4期)
   1.本文主要说明了 (限10个字以内)。
   2.作者是按照什么顺序来安排第②段内容的?这样安排有什么好处?
   答:
   3.第③段划线句子表明人们的听力 ,运用的说明方法有 、
   、 、 。
   4.下列句子中,删去加点词语后对原句意思表达影响最小的一项是 ( )。
   A.常有一些尖利的噪音,它们虽然比较短暂,但其激烈程度可以严重干扰人们的生活。
   B.耳朵还能将讲话人的声音分离出来,同时在一定程度上忽略环境噪音。
   C.摇滚乐队的某些演奏可能产生140分贝的噪音。
   D.面对160分贝的噪音,动物有可能死亡。
  
  
   (十八)(2004·湛江)
   ①在我国大陆最南端,湛江雷州半岛徐闻县西海岸,有一个闻名全国,被誉为“水族大观园”的珊瑚自然保护区,区内的珊瑚礁绵延20多公里,面积达2000公顷。据专家研究,珊瑚礁已有近万年的发育史。
   ②徐闻西海岸一带岸礁众多,海水清澈,水温、盐度稳定,非常适合珊瑚虫的繁殖生长:多年来当地一直十分注重生态的保护,保护区内的珊瑚种类繁多,达五十多个品种。其中软体珊瑚有千年佛、白汽泡、鸡冠花,硬体珊瑚有角星、海脑、扇形,等等。
   ③当艳阳高照,风平浪静的日子,乘着小船在珊瑚丛上面缓缓滑行,犹如在一片美丽的丛林中漫游,那密密麻麻的鹿角、牛角、羊角几乎探出水面,触手可及。还有散落在“丛林”中的“翡翠”、“玛瑙”,形态各异;时稳时现的“鲜花”,橙黄蓝白红,煞是可爱,美不胜收。不时还可见五彩缤纷的鱼儿与游船一起穿梭漫游,构成一幅奇异的海底风景画。
   ④珊瑚礁群向来被海洋学家称为海上“热带雨林”。它占海洋面积不到0.2%,却栖息了1/4的海洋生物,供养着9万多种鱼类。我国的海南岛沿岸本来有中国最大的珊瑚礁群,历史在十万年以上,但因过度开发,仅10多年有的地方已成了海底废墟。而徐闻的珊瑚礁群作为目前祖国大陆保存最完好的珊瑚自然生长区,正吸引着越来越多的旅游观光者。为了永远给子孙后代保留这一片天赐的生态自然景观,当地政府已制订了以保护为前提,结合生态和环境旅游的利用计划,要让这一颗明珠永远闪亮在祖国大陆南端的西海岸边。
   1.文中为什么称珊瑚礁群(区)为“水族大观园”和“热带雨林”?
   答:_________________________________________________________
   2.文章第①②段分别从哪些方面介绍了徐闻珊瑚自然保护区?
   答:第①段:_________________________________________________________
   第②段:_________________________________________________________
   3.请从写作手法或语言运用特点的角度对第③段的表达效果(特点)作简要分析:
   答:_________________________________________________________
   4.文章末段写海南岛珊瑚礁群受损的事例是为了说明什么?你从中得到什么启示:
   答:(1) _________________________________________________________
   (2) _________________________________________________________
  
  
   (十九)(2004·北海)
   ①资源短缺的表现之一,是可耕土地资源不足,粮食生产的增长赶不上人口的增长。于是,许多人纷纷发出警告:地球将无法养活超过100亿的人口。然而,一些乐观的人士反对这种危言耸听的说法。他们认为,虽然陆地上可耕地的开发已近极限,但地球还有广阔的海洋可供开发,大海完全有可能成为人类未来的粮仓。
   ②当然,海洋所能提供给我们的并不是传统意义上的粮食——大米、小麦和玉米等,而是广义的粮食——其他的能够满足人类营养需要的食物。一些海洋学家指出:仅仅是位于近海水域自然生长的海藻,每年的生长量就已相当于目前世界小麦年产量的15倍。如果把这些藻类加工成食品,就可以为人类提供足够的蛋白质。其实,把藻类叙谈食品,我们并不陌生。仅以我国沿海来说,人们比较熟悉的可食用藻类就有:褐藻类的海带、裙带菜、羊栖菜、马尾藻;红藻类的紫菜、鹧鸪菜、石花菜;绿藻类的石莼、浒苔等。它们在人工的精心养殖下,产量正在不断翻新。其中仅海带一处,目前年产量就比早先的野生状态提高了2000多倍,可见增产潜力是多么巨大!
   ③除海藻类,海洋中还有丰富的肉眼看不见的浮游生物。有人作过计算,在不破坏生态平衡的前提下,若能把它们捕捞出来,加工成食品,足可满足300亿人的需要。
   ④至于海洋中众多的鱼虾,则更是人们熟悉的食物。尽管近海的鱼虾捕捞已近极限,但我们还可以开辟远洋渔场,发展深海渔业。例如南极的鳞虾,每年产量可高达50亿吨,我们只要捕获其中的1亿——1.5亿吨,就比当今全世界一年的捕鱼量多出1倍以上。何况,在深海和远洋中还有许许多多尚未被我们充分开发利用的海洋生物,其巨大潜力是不言而喻的。
   ⑤综上所述,说大海是人类未来的粮仓,一点儿也不夸张。
   1.海洋给我们提供的粮食和传统意义上的粮食的区别是:              
                                       。
   2.第②段中画线句子运用了哪些说明方法?
   答:          、          、             。
   3.文章第②段中加点的一句说:“它们在人工的精心养殖下,产量正在不断翻新”,请
   在文中找出与“翻新”一词对应的例子:                      
                                     
   4.第③段中画线的“在不破坏生态平衡的前提下”一句是否可以删掉,为什么?
     答:                                    
                                         
   5.请你为上文安一个恰当的题目(不要超过10个字)。
     答:                                    
  
  
  
   (二十)(2004·内江)
   克隆技术造福人类
   ⑴克隆技术会给人类带来极大的好处。例如,英国PPL公司已培育出羊奶中含有治疗肺气肿的α—I抗胰蛋白酶的母羊。这种羊奶的售价是6000美元一升。一只母羊就好比一座制药厂。用什么办法能最有效、最方便地使这种羊扩大繁殖呢?最好的办法就是“克隆”。同样,荷兰PHP公司培育出能分泌乳铁蛋白的牛,以色列LAS公司育成了能生产血清白蛋白的羊。这些高附加值的牲畜如何有效地繁殖?答案当然还是“克隆”。
   ⑵母马配公驴可以得到杂种优势特别强的动物——骡,然而骡不能繁殖后代,那么,优良的骡如何扩大繁殖?最好的办法也是“克隆”。我国的大熊猫是国宝,但自然交配成功率低,因此已濒临绝种。如何挽救这类珍稀动物?“克隆”为人类提供了切实可行的途径。
   ⑶除此之外,克隆动物对于研究癌生物学、研究免疫学、研究人的寿命等都有不可低估的作用。
   ⑷不可否认,“克隆绵羊”的问世也引起了许多人对“克隆人”的兴趣。例如,有人在考虑,是否可用自己的细胞克隆成一个胚胎,在其成形前就冰冻起来。在将来的某一天,自身的某个器官出了问题时,就可从胚胎中取出这个器官进行培养,然后替换自己病变的器官。这也就是用克隆法为人类自身提供“配件”。
   ⑸有关“克隆人”的讨论提醒人们,科技进步是一首悲喜交集的进行曲。科技越发展,对社会的渗透越广泛深入,就越有可能引起许多有关的伦理、道德和法律等问题。我想用诺贝尔奖获得者、著名分子生物学家J.D.沃森的话来结束本文:“可以期待,许多生物学家,特别是那些从事无性繁殖研究的科学家,将会严肃地考虑它的含意,并展开科学讨论,用以教育世界人民。”
   1.克隆技术会给人类带来哪些好处?
   答:
   2.第⑴段运用了哪些说明方法,列出两种说明方法并举例加以说明。
   答:
   3.第⑸段运用的主要表达方式是 ,这段中支撑“科技进步是一首悲喜交集的进行曲”的句子是
   ?
  
   (二十一)(2004·泸州)
   风是空气流动的表现,是我们地球上最常见的自然现象。然而,你听说过太阳也会刮风吗?
   太阳风是一种来自太阳的物质流。这种物质虽然与地球上的空气不同,不是由气体的分 子组成,而是由更简单的比原子还小一个层次的基本粒子——质子和电子等组成,但它们流 动时所产生的效应与空气流动十分相似,所以称它为太阳风。当然,太阳风的密度与地球上 的风的密度相比,是非常非常稀薄而微不足道的,一般情况下,在地球附近的行星际空间 中,每立方厘米有几个到几十个粒子。而地球上风的密度则为每立方厘米有2687亿亿个分 子。太阳风虽然十分稀薄,但它刮起来的猛烈劲,却远远胜过地球上的风。在地球上,12级台风的风速是每秒32米以上,而太阳风的风速,在地球附近却经常保持在每秒350~450千米,是地球风速的上万倍,最猛烈时可达每秒800千米以上。
   太阳风虽然猛烈,却不会吹袭到地球上来。这是因为地球有着自己的保护伞——地球磁 场。地磁场把太阳风阻挡在地球之外。然而百密一疏,仍然会有少数漏网分子闯进来,尽管 它们仅是一小撮;但还是会给地球带来一系列破坏。它会干扰地球的磁场,使地球磁场的强 度发生明显的变动;它还会影响地球的高层大气,破坏地球电离层的结构,使其丧失反射无 线电波的能力,造成我们的无线电通信中断;它还会影响大气臭氧层的化学变化,并逐层往 下传递,直到地球表面,使地球的气候发生反常的变化,甚至还会进一步影响到地壳,引起 火山爆发和地震。例如,1959年7月15日,人们观测到太阳突然喷发出一股巨大的火焰 (它就是太阳风的风源)。几天后,7月21日,也就是这股猛烈的太阳风吹袭到地球近空时, 竟使地球的自转速度突然减慢了0.85毫秒,而这一天全球也发生多起地震;与此同时,地 磁场也发生被称为“磁暴”的激烈扰动,环球通信突然中断,使一些靠指南针和无线电导航 的飞机、船只一下子变成了“瞎子”和“聋子”……
   太阳风对地球的影响,只是乘虚而入的漏网分子所为。由此可见,在无所阻拦的行星际 空间,太阳风的威力有多大了。
   1.本文的说明对象是                        。
   2.简要回答:为什么把来自太阳的物质流称为太阳风?
   答:                                     
   3.太阳风如果来到地球,会造成哪些破坏?(至少三方面)
   答:①                                  。
      ②                                  。
     ③                                  。
   4.文中加点的“一般情况下”能否删去?为什么?
   答:                                  
   5.本文第二段主要运用的两种说明方法是       、        。
  
  
   (二十二)(2004·资阳)
   ⑴目前世界上第一台光脑已由欧共体研制成功,这是一台全光数字计算机,其运算速度比电脑快一千倍。
   ⑵光脑和电脑的工作原理基本一样,所不同的是光子代替了电子;光互连代替了电子导线互连;光开关、光三极管、光存储器、反馈装置和集成光路等部件,代替了电脑中的电子硬件;用光运算代替了电运算;用非冯·诺伊曼结构代替了冯·诺伊曼结构;从而使光脑中的功能为电脑所望尘莫及。
   ⑶光脑具有超并行性。目前最新的并行处理电子计算机常具有NⅪ的并行性,其结构和运行极为复杂,速度和精度也相当低。光脑则不同,它具有NXI的并行性,具有并进通信和并行处理能力强,可用简单的运算去处理大阵列的特点。
   ⑷光脑可在接近室温条件下具有超运算速度。电子的传播速度每秒只能达到593公里,而光子的速度每秒30万公里,因此利用光在光缆中互连通信要比利用电子在互连的导线中通信减少大量的时间,提高了运算速度。同时超高速电脑的计算器件只能在极低的温度下工作,而光脑则可以在接近室温下进行超高运算也在于光的频带远大于无线电波和微波,具有极大的信息存储量,存储量可达108位。
   ⑸光脑抗干扰能力强。光脑靠光子传播信息,光电子没有带电荷,对其他邻近的光子和电子毫无影响。所以光信号不仅不相互干扰,而且可以与电子控制信号交叉,此外光脑容错性能好,具有与人脑类似的容错性,系统中某一元件损坏或出差错时,并不影响到最后结果。
   1.这段文字说明的对象是
   2.简略回答光脑同电脑相比具有哪些优势。
   答:
   3.这则短文主要运用了哪些说明方法?
   答:
   4.第4自然段从哪两个方面介绍了光脑在接近室温条件下具有超运算速度的?(2分)
   一方面从光脑的
   另一方面从光脑的
   5.“光脑和电脑的工作原理基本—样,所不同的是光子代替了电子”一句中,“基本”用得准确是因为
  
   6.根据你平时的积累,说出两个高科技产品的名称。
   答:
  
   (二十三)(2004·桂林)
   水之经典
   ⑴世上丽水秀水晶莹之水清澈之水恢弘之水浩翰之水,多的是,在我看来,最富性格最值得一看的是这两处:都江堰和九寨沟。
   ⑵看都江堰的水,看的是强悍奔腾的水如何层层叠叠化为生命的涓涓细流。飞奔如兽、桀鹜不驯的江水,经过都江堰,立刻将仰天长啸变为 ,将浪涛如山变为 ,将凶猛如火变为 ……出宝瓶口流入内江,立刻呈现一派水光潋滟的情景,让人叹为观止,看到水的柔劲、可塑和万难不屈,常流不懈的生命活动。那是一种将绚烂归于平淡,将刚劲寓于柔顺,将一时融于永恒的生命。
   ⑶都江堰看水,看的是水如何从天上流入人间,如何从神话流入现实,如何将自己化为一种哺育人类、灌溉庄园的生命。
   ⑷都江堰的水,是一种入世的现实的水。
   ⑸看九寨沟的水,看的是宁静的恬淡的水,如何凝聚成生命的湖泊。镜海、长海、珍珠滩……每一个湖泊都是那样清澈透明、纤尘不染。孔雀的蓝色,蓝得让人心醉,让人如同看到教堂洗礼用的圣洁露水,如同听到教堂管风琴演奏的圣母颂,而不敢有丝毫杂念俗念,懂得并真正地看到人世间居然有纯洁美好真诚和透澈的净,就在这远避尘嚣而静静地存在。
   ⑹那水几乎一动不动,任外面的世界如何纷繁变幻,将污染、噪音连同人心泛起的种种污浊的泡沫一起抛向天空和大地。它独自坚持着自己的贞操,不动丝毫涟漪,不染丝毫尘俗,将水底的虬枝沉木、水藻水锦,将天上的薄云丽日、山岚清风,将身旁的雪峰幽谷、古树老藤……一一映在自己的怀中,映得那么明净,如同脱胎换骨一般,玉洁冰清,重新塑造了自己一番。尘世沾惹的市俗庸俗、风骚矫情、浪声虚名、欲火利海……起码不敢在这里抖擞,而被这水洗却大半。
   ⑺九寨沟看水,看的是水如何从人间流向天上,如何从现实流向神话,如何将自己化为一种启迪人类、净化心灵的艺术。
   ⑻九寨沟的水,是一种出世的艺术的水。
   ⑼日本黑田孝高在《水五则》中的第一则说:“自己活动,并能推动别人的,是水。”第四则说:“以自己的清洁,洗净他人的污浊,有容情纳浊的宽大度量的,是水。”前则,可以送给都江堰的水;后则,可以送给九寨沟的水。
   1.第⑴段在全文中的作用是 。
   2.第⑴段画线句按常理应断句,你认为应该加入什么标点符号?作者为什么不加标点?
   答:
   3.给第⑵段文中空白处依次选择恰当的词语,正确的一项是( )
   A.喃喃细吟 珍珠四溢 柔情万缕
   B.柔情万缕 珍珠四溢 喃喃细吟
   C.珍珠四溢 柔情万缕 喃喃细吟
   4.为什么说“都江堰的水,是一种入世的现实的水”?请用自己的语言概括回答。
   答:
   5.与第⑼段中“以自己的清洁,洗净他人的污浊,有容情纳浊的宽大度量的,是水”一句相照应的是第 段的内容。
   6.你如何理解“水之经典”中“经典”一词在文中的含义?
   答:
   7.文中所写的都江堰之水和九寨沟之水各有性格,你更喜欢哪一处的水?为什么?
   答:
  
   (二十四)(2004·桂林)
   生命与气候
   ⑴你能想到吗?千百万年以来,地球气候是由地球的两大生命王国——动物界与植物界的斗争所决定的。动物吸进氧气而呼出二氧化碳,植物主要吸进二氧化碳而放出氧气。二氧化碳为温室气体,因此可以立即得出结论:动物能使地球升温,而植物则使地球降温。如果哪一方占上风,那么地球就会面临温室效应或又一次严寒期。
   ⑵5亿多年前的“寒武纪大爆炸”时期,到处都是将二氧化碳释放到空气中的节肢动物,它们使空气中的二氧化碳含量高达今天的20倍,所以当时的气候极其温暖。
   ⑶但是,植物进行了反击。化石显示,4.5亿年前,含有木质素的植物首次出现,木质素使细胞坚硬,这样植物就得以长大,地球上出现了第一批树木。随着全球的植物大量生长,毫无约束的光合作用从空气中吸取着二氧化碳,使二氧化碳含量大大减少,而不能吸收木质素的、饥饿的节肢动物对此无能为力——地球进入了严寒期。
   ⑷动物又通过进一步进化进行了回击,主角是白蚂蚁和恐龙,它们学会了如何食用木质素,如何繁荣昌盛。它们粗重的呼吸及对植物的破坏使空气中的二氧化碳高达今天的3倍,地球大部分地区酷热而潮湿,又恢复了温室效应。
   ⑸然而植物并没有被消灭。在6500万年前,恐龙因为小行星的碰撞或其他灾难而彻底消失后,植物抓住了这一机会再次反击,突破点是第一批草地的出现。青草本身不会保存很多二氧化碳,但能形成软土,软土可以保存大量的二氧化碳,从而使空气中的二氧化碳含量降低。事实上,草地生态系统含有的二氧化碳比森林生态系统还多。
   ⑹过去约4000万年来,大量草地布满全球,取代了许多年以前的森林地带。在草地的影响下,地球逐渐降温,终于在200万年前进入冰川期。
   ⑺大约1万年前,由于人类活动的显著增强,使地球空气的二氧化碳含量再度升高,尤其是近百年来,地球的温室效应愈来愈明显。
   ⑻也许,地球上的整个生物圈就像我们所熟知的生物个体调节其体内环境那样,不停地调节着地球的大气环境,使气候能适应地球上生物生存进化的需要。换句话说,地球的生物圈是在主动地调节环境,而不是在被动地适应环境。果真如此,数十亿年的气候变迁不仅仅是太阳或地球的原因,生物为了自己的利益也深深地参与其中了。
   (选自《科技日报》,有改动)
   1.本文说明的主要内容是: 。
   2.本文主要采用的说明顺序是 。
   3.你认为文章使用了哪些说明方法?请写出两种,并各举一例加以说明。
   答:
   4.第⑻中加点的词语“也许”能否删除?为什么?
   答:
   5.本文中,植物吸收或保存二氧化碳有哪些方式?请分列出来。
   答:
   6.作者说:“地球的生物圈是在主动地调节环境,而不是在被动地适应环境”,你认为正确吗?请联系生活实际加以说明。
   答:
  
   (二十五)(2004·河南)
   宇宙飞船的“避火衣”
   ⑴我国“神舟”五号载人宇宙飞船在2003年10月的顺利发射升空与安全返回,是我国载人航天的历史性突破。它从天外“下凡”——返回地球,过程复杂,场面惊心动魄。
   ⑵当宇宙飞船以第一宇宙速度(7.9公里/秒)进入大气层时,飞船的外壁与空气剧烈摩擦,产生极大量的热,在距离地面60公里左右的稠密大气层区域,其表层温度可以达到1000~3000摄氏度,整个机身变成一个炽热耀眼的火球(就像我们常见到的火流星一样)。进入距离地面80~40公里区域时,由于周围空气在高温下电离,宇航员与地面的无线电通讯会暂时中断,进入“黑障区”,就连雷达也无法发现它的踪迹……
   ⑶飞船返回舱中有宇航员和珍贵的仪器资料,必须保证它顺利通过大气层安全返回地面,必须为它研究制造一件“避火衣”。
   ⑷由于宇宙飞船只使用一次,科学家为它(A)精心设计了一件奇妙的“避火衣”,它(B)是用“瞬时耐高温材料”制成的。这(C)是一种由特殊纤维材料或多孔颗粒加上有机物组成的低导热复合材料,这一层材料的厚度经过精心计算设计,在宇宙飞船不同部位的厚度是不同的。
   ⑸当飞船返回舱由太空“下凡”通过大气层,与空气剧烈摩擦产生大量的热量时,这件“避火衣”就会“引火烧身”,自己先燃烧起来,其中大量的有机物会发生化学分解和汽化,带走极大量的热量。更奇妙的是,“避火衣”在燃烧自己的同时,还形成一层厚厚的多孔炭化层,紧紧地附着在返回舱的外壁,这一炭化层具有极好的隔热效能,在它严密的包裹下,能有效地防止外界热量传入舱内,使返回舱中的温度保持在35摄氏度以下,保证了宇航员的安全。
   ⑹这种自我牺牲的一次性“避火衣”设计十分巧妙,结构相对比较简单,重量就比较轻,成本自然也就比较低,具有双重奇妙功能,使宇宙飞船顺利“下凡”,为宇航事业做出了卓越的贡献。
   1.请根据文意,概括宇宙飞船“下凡”时的“惊心动魄”的场面。
   答:
   2.第⑵段中有两处使用了括号,这两处括号中的内容在表达上分别有什么作用?
   第一处:
   第二处:
   3.第⑷段中三个加点的词在文中分别指代什么内容?
   A.它: B.它: C.这:
   4.文中能体现“避火衣”“自我牺牲”特点的词语是 。
   5.从文中看,宇宙飞船的“避火衣”具有的“双重奇妙功能”是什么?
   ①
   ②

(六)(2004·山西)
   争先恐后
   于坚
   ⑴中国是一个争先恐后的国家。常常可以看到这种情况,门一打开,拥挤在门口的人群就像罐头里的沙丁鱼一样喷出来,争先恐后,蜂拥而动。我说的不是公共汽车门口,而是公园门口,展销会门口,飞机的登机口,电影院的出口……大家在斑马线前面等绿灯,没有人愿意站在后面,都下意识地要往前挤,结果绿灯还没有亮,人群就漫到了马路上。印象最深的是下火车,哪怕前面就是终点站,人们也要提前一两站就结束了热火朝天的扑克游戏,纷纷起身,收拾行李到过道上去挤着,好象晚一步,火车就要飞掉。
   ⑵并不是中国人不喜欢秩序,也不是这个国家物质贫乏,空间狭窄。“争先恐后”不过是世界的一种此起彼伏的暂时状态,一个形容词。但在六十年代的词汇中,“先”与“后”却是一种价值判断,“先”,与先进、进步之类有关,“后”却是落后、倒退,“落后就要挨打”。无数运动之后,在这个国家无人不患“恐后”症。由于普遍地“恐后”,争先成为风气,“先”是如此事关生存,到了必须“争”的地步。精神上的“恐后”,最终在潜意识中影响着人们的行为,在日常生活中也就争先恐后了。
   ⑶ “争先恐后”不再是一种日常生活中的暂时状态,而是常态,所以,没有任何必要“恐后”的地方,人们也本能地要争先。
   ⑷争先已经到了这种地步,连小学生考试,95分以下都是后了,所有人都争先,意味着人们对生活的常态和基础的漠视。生活的质量并不表现在世界的先进部分,而在于世界的普遍状况。把“先”作为衡量事物的唯一价值标准,所有人都要争先恐后,只好革命。
   ⑸争分夺秒、力争上游,作为一种可能性,当然是应该鼓励的,但它不是标准。如果也用河流比方,没有下游和中游,上游不是死水一潭么?没有恒定的时间和缓慢的时针的对比,争分夺秒不正是慢么。上游、中游、下游,河流的不同形态,也是世界的不同状况,它们并不存在价值上的优劣。
   ⑹马拉松赛跑永远只有一个人先进,世界注定是普遍落后的,是落后者的乐园。
   ⑺先进的意思在此时代,只意味着价值是否能够立即兑现。不能立即兑现就是落后。我常听人说,不甘落后,眼科医生加入了股票大军,种白菜的姑娘去医院当了护士。
   ⑻我认识一位缓慢的大师,他的一座雕塑花了20年,他是落后的。他开始的时候,受的是先锋贾科梅蒂的影响,但这影响持续了20年之久,没有当即面市,他完成的时候,已经落后了,年轻一代今天决不会从伦勃朗开始,否则永远落后。他们很聪明,从威尼斯双年展开始。
   ⑼一天等于20年,固然是一种时间观。但20年如一日,同样是一种时间观。一天功夫的东西就是一天功夫的东西,20年功夫的东西就是20年功夫的东西,永远不可取代。缓慢的并不是落后的,快也不意味着价值上的优越。这都是常识,但在这个世纪,明白这些却是大智大慧。
   ⑽在古代社会,各文明之间并不争先恐后,因为文明不存在价值上的先进与落后。但在20世纪,“落后就要挨打”,竟然成为风行全球的真理之一,并且已然是,只要落后,就可以打。落后的标准由什么来界定呢?武器的先进。那些文化并不崇尚武力的文明必然总是落后,落后的印度文明,落后的埃及文明,落后的印第安文明,落后的中华文明。这个世界已经昏头了。李白杜甫们的文明怎么会劣于松尾巴焦们的文明呢?日本人假装不知道这一点,居然在中国文明面前以“先进”民族自居,靠的就是武器。此种逻辑已经成了中国人刻骨铭心的经验之一,人们似乎已经忘记此逻辑乃是强盗们创造的。
   ⑾以“争先恐后”的价值观来看历史,不仅会得出缓慢的中国时间劣于格林尼治时间的结论,甚至,把“落后”用于汉语。
   ⑿“争先恐后”,是由于人们丧失了对世界的永恒之基础的安全感。丧失了存在感。
   ⒀ “争先恐后”,乃是逃亡之象。
   ⒁其实在这个世界上,最落后的难道不就是我们相依为命的大地本身么?一成不变的,缓慢的,没有时间的,大地的这些特性是否已经成为这个“先进”世纪“维新”的障碍?
   ⒂长江洪水的泛滥,从根本的方面来说,我以为乃是“先进”的人类不满于大地的落后,所导致。
   1.通读全文,以下选项中不符合作者观点的一项是( )
   A.和谐是世界永恒的基础 B.文明的价值在于落后而不在于先进
   C.我们不应该把争先作为生活的标准 D.我们应该按生命的本来形态生活
   2.造成人们争先恐后的根本原因是什么?请用文中语句回答。
   。
   3.结合上下文,请用自己的话谈谈你对⑹段中画线句子的理解。
   。
   4.从全文看,作者主要是用什么方法阐明观点的?
   。
   5.你同意作者的观点吗?请联系自己的生活实际谈谈你的看法。
  
   (七)(2004·南京)
   读“磨脑子”的书
   书有许多种。有的明白晓畅,使人轻松;有的艰深晦涩,让人费解。而真正值得我们阅读的,往往是那些“磨脑子”的书。
   与一般的书籍比较起来,“磨脑子”的书具有更加密集的信息量和深刻的思想内涵,往往是前人思想的精华、阅历的浓缩。它不是那种读起来驾轻就熟的书,而是不静下心来深入思考就不能理解的书,是要经常在笔记本上记下大量困惑和疑问的书。读这种书的感受就好像啃骨头,费劲,却别有一番滋味在心头。吃现成的肉当然也好,但终究不如啃骨头来得有劲A。
   读“磨脑子”的书,能够提高人的理解力和思维水平。生物学告诉我们,人的智力是用进废退的。能否经常给自己出难题,面对困难超越自我,本身就是一个人素质高下的集中表现。燕雀安于暖巢,只有雄鹰才能在风雨中翱翔。常读一览无余的书,会使原本聪慧的头脑退化。
   人的愉悦程度大抵与其付出的劳动量和劳动时间成正比。读“磨脑子”的书有点像高强度的体育运动,不花上吃奶的力气就不能达到应有的水平。惟其如此,它所带来的快乐才非同一般,它赐给我们的礼物才格外珍贵。试想,上珠穆朗玛峰的快乐岂是随便踏上哪个土包儿可以比拟的?
   俗话说,宁尝鲜桃一口,不吃烂杏一筐B。“磨脑子”的书正像鲜桃,而大量克隆的信息垃圾充其量只配叫做“烂杏”。读一本“磨脑子”的书,还是一次与智者的对话。它使人暂别琐碎与平庸、浮躁与虚妄,而变得神清气爽,心灵宁静。对于这样一种难得的体验,聪明人是不该长久疏远的。
   真正的读书人都有同感,好书一定要精读。一目十行的读法或许适合一般读物,但对于“磨脑子”的书绝不适宜。真正的好书甚至使我们有意放慢阅读速度,就像一个贪婪的孩子舍不得一口吃完手中的糖果一样。
   要真正读一本“磨脑子”的书,还要学会适当地拒绝。著名作家李敖先生夜晚从不看电视,全用来阅读那些经过时间检验和沉淀的“老书”。拒绝,还意味着,既不为某些时髦的写作分心,也不对人为的热点动心,更不为虚假的创造操心。不仅如此,网络的精彩,球赛的热闹,酣睡的香甜, , ,都是应该适当拒绝的。否则的话,什么时候“磨脑子”呢?
   1.阅读全文,说说读“磨脑子”的书有哪些益处。
   答:
   2.文中划线的A、B两句都用了什么论证方法?句中“现成的肉”、“烂杏”各指什么?
   答:
   3.联系实际,你认为文中空格处还可以填入哪些“应该适当拒绝”的内容,请写在下
   面的空格内,句式与上文一致。
   答: , 。
   4.名著往往是“经过时间检验和沉淀”的“磨脑子”的书。请从下列名著中任选一部,简要地写出你熟悉的一个情节,并说说对这一情节的体验和感悟。
   名著:《水浒》《西游记》《钢铁是怎样炼成的》《鲁滨孙漂流记》《童年》
   答:情节:
  
   体验和感悟:
  
   (八)(2004·徐州)
   拒绝句号
   一定会有一些朋友反对我这个标题。他们会说多好的句号啊!句号表示一种完成,一种圆满,一种有志者事竟成,一种成果与收获,或者干脆把这溜圆的句号看成一个个饱满的果实。它们还会问我,当你完成一部几十万字的长篇小说,在那最末一行画上一个句号时,难道你没有如释重负、飘飘欲仙般的感受?没有那种大功告成后该痛快干杯的喜悦吗?
   当然,这样的句号我也喜欢。但人生还有另一种句号。
   打个比方,你在一条路上走,走着走着,忽然有一种“尽头感”时,这句号就隐隐出现,如果你停下来,你足下就清晰地现出一个句号。这条路可不是做一件事时那短短的距离,它是人生追求的路、艺术探索的路和事业奋斗的路。这路原本无止无休,你在任何一处都可以起步,踏上征程;你也可以在任何一处画个句号,退了出来。无论什么都可以成为句号的缘故,那精疲力竭的放弃、自寻清闲的逃逸、江郎才尽的低头认输,乃至收获后的自满自足,甚至在目标达到之后,辉煌的目标也会划为一个句号,尽管这句号闪闪发光。句号,就是停止,就是终结,就是事物最终变为有限的、死去的符号。
   我说的是这种句号。可怕的是,这些句号总是不知不觉地出来。你呢,不知不觉地完结。想想看,你曾经做过了哪些有益的事?究竟是什么时候并怎样弃你而去的?句号往往又是和人的自足、人的彻悟、人的惰性连在一起的。所以句号大多是人心甘情愿给自己画上的。人随时可能舒舒服服给自己画个句号,休止了自己。
   因而,我害怕句号。我对句号保持着近乎神经质的警惕。在与句号的斗争中,我一边感到生命的活力,常常闻到自身肌肉搏斗后散发出热烘烘的清香;一边认识到这原是生命存在所必须进行的奋争,也是与自身惰性与保守的对抗。当然,它何其艰难!跨过每一个句号,都需要付出双倍的力量,其中一半是创造力。然而,只要在人生或艺术的道路上,消灭一个句号,便开始一段崭新的充满诱惑的路。我们还会发现,被我们拒绝和消灭的句号,最终竟然会变成逗号。你是不是也会从中得到启示:
   最积极和充实的人生,是不断努力地把句号变为逗号。
   (《扬子晚报》2004年3月10日,作者:冯骥才)
   1.“拒绝句号”是作者运用比喻手法表达的一种观点。请根据文意,概括其内容。
   答:
   2.用简略的词语或句子,解释文中画线句子中两个“句号”的不同含义。
   当然,这样的句号①我也喜欢。但人生还有另一种句号②。
   答:
   3.文章在阐述观点时,十分强调“自我”这一内因的关键作用。请你认真阅读三至五段,分条加以整理。
   答:
   4.结合自身实际,谈谈对“最积极和充实的人生,是不断努力地把句号变为逗号”这句话的理解。
   答:
   5.本文作者从一个枯燥的标点符号,引发议论话题,阐述人生哲理,给人以启迪。请按照这一思路,自选一种标点符号,展开联想,写一段议论性的文字。(不超过50字)
   答:
   (九)(2004·宿迁)
   带着梦想上路
   章睿齐
   ①许多年前,一个10岁的意大利男孩在那波里的一家工厂做工。他一直想当一个歌星,但是,他的第一位老师却说:“你不能唱歌,五音不全,你的歌简直就像是风在吹百叶窗一样。”回到家里后,他很伤心,并向他的母亲——一位贫穷的农妇哭诉这一切。母亲用手搂着他,轻轻地说:“孩子,其实你很有音乐才能。听一听吧,你今天的歌声比起昨天的乐感好多了,妈妈相信你会成为一个出色的歌唱家的……”听了这些话,孩子的心情好多了。后来,这个孩子成了那个时代著名的歌剧演唱家。他的名字叫恩瑞哥·卡罗素。当他回忆自己的成功之路时这样说:“是母亲那句肯定的话,让我有了今天的成绩。”
   ②也许,卡罗素的母亲从来都没有想到过她的儿子能成为一代名人,也许根本没有指望过靠她那三言两语去改变儿子的一生,然而,事实上,正是她那句善意的肯定成就了那个时代最伟大的歌唱家。
   ③美国篮球巨星乔丹在很小的时候就有了自己的篮球明星梦。一天,他把自己的梦想告诉母亲,母亲大加赞赏。为他有了自己的梦想向他祝贺,鼓励他向篮球明星学习,并且经常抽出时间和小乔丹一起欣赏报纸杂志上篮球队员们驰骋球场、飞身灌篮的矫健身影和飒爽英姿。同时,建议乔丹把那些花花绿绿的图片剪下来,贴到房间的墙上去,便于与偶像们朝夕相伴。
   ④同乔丹、卡罗素一样,几乎每个孩子都有自己的梦想,梦想是孩子对自己未来的美好设计。孩子们在谈到自己的梦想时,往往会神采飞扬、美不胜收。然而,在现实生活中,很多父母却常常对孩子的梦想不屑一顾,甚至大泼冷水。有一个小学三年级的男孩子曾对母亲说,长大了要去当舰长,而母亲却说:“瞧你那糟糕的成绩,打扫军舰都轮不到你的份儿。”孩子的梦想被母亲的讥讽伤害了。如果这位母亲能像乔丹、卡罗素的母亲那样认真对待孩子的那份梦想,孩子日后没准真会成为一位出色的舰长呢。
   ⑤人类需要梦想。梦想是对生活积极进取的态度,对人生深深的企盼。一个人可以失败,可以遭受挫折,可以忍受孤独和不幸,但不可以失去梦想,没有梦想的人生就像鸟失去了双翼,船失去了双浆。对孩子来说,梦想更有着无穷的魅力,具有巨大的牵引和激励作用。梦想能使孩子在学习、工作过程中创造不辍,并获得愉悦的情感体验。
   ⑥面对孩子的梦想,哪怕有些不可思议,父母都应给予肯定和鼓励。鼓励孩子追梦,孩子会产生强劲的内驱力,面对各种困难也会主动想办法去克服。
   ⑦在孩子追梦的过程中,父母还应予以多方面的关注。比如,帮助孩子寻找梦想的偶像,和孩子讨论偶像的成长史、奋斗史、成就史,明确成功必须付出辛劳和汗水,让偶像在孩子心里生根;给孩子的圆梦计划提供建议和支持;经常提醒孩子践诺,在孩子怀疑梦想时给孩子鼓励。
   (选自2004年5月27日《人民日报》)
   1. 卡罗素的母亲与乔丹的母亲教育子女的做法有何异同?
   答:
  
  
   2. 全文的观点是什么?卡罗素、乔丹的故事与文章的观点有什么联系?
   答:
  
   3.第⑤节运用了 的方法从道理方面进行论述,其作用是
   答: 。
   4.“对孩子来说,梦想更有着无穷的魅力,具有巨大的牵引和激励作用。”根据这一观点,为课文补充一个例子。
   答:
  
  
   (十)(2004·温州)
   ⑴我们每读一本书,每见一件事,都应该多动脑筋、多思考。这样才能在人们司空见惯的现象上发现新的东西,由此锻炼出我们敏锐而正确的观察判断能力;这样才能在知识的学习上由浅入深,循序渐进,由此丰富我们的头脑。
   ⑵苹果落地了,这是多么熟悉的事情,大家习以为常,并不觉得这里边还有什么研究的话题。一天,牛顿注意了这一现象,并且提出“它为什么会落地”的疑问,他又思考,又探究,结果发现了其中包含着的“万有引力”定律。
   ⑶壶水开沸,这也是平常得不能再平常的事。瓦特却把它当成了一个大问题来研究,并且根据其原理,发明了蒸汽机,为人类社会做出了划时代的贡献。
   ⑷“缺少知识,就无法思考,缺少思考就不会有知识。”这个格言说出了思考在知识的学习、运用、积累中是十分重要的。人不是一生下来就懂得很多知识的。在人的成长过程中,我们会逐渐接受一些前人留下的知识,我们要充分利用自己懂得的有限知识,多动脑,多思考。思考得多了,自然而然就会发现问题,有了疑问,就会促使我们去探讨、去解决。“观察——疑问——解决问题”是获得新知识的有效途径,在这个过程中,起决定作用的是“思考”,即动脑子。不动脑子也就不会提出什么疑问,没有疑问也就没什么需要探讨、解决的东西,因而也就没有新知识的获得。
   1.作者在文中提出的主要观点是什么?
   答:
   2.文章第⑵、⑶两段证明了动脑的好处是什么?(用原文作答)
   答:
   3.请用简洁的语言概括第⑵段的事实材料。(30个字以内)
   答:
   4.第⑷段画线句子中的“这个过程”是指什么?
   答:
   5.作者在第⑵、第⑶段中用了两个事实论据论证观点,请你再补写一个事实论据。
   答:
   (十一)(2004·安徽)
   我寻求挫败
   ①我一直都在寻求挫败。
   ②有人出发去“征山”,我却从来不是,而且刚好相反,我爬山,是为了被山征服。有人飞舟,是为了“凌驾”水,而我不是,如果我去亲近水,我需要的是涓水归川的感受,是自身的消失,是自我复归拉于零的一次冒险。
   ③记得故事中那个叫“独孤求败”的第一剑侠吗?终其一生,他遇不到一个对手,人间再没有可以挫阻自己的高人,天地间再没有可匹敌可交锋的力量,真要令人忽忽如狂啊!
   ④生来有一块通灵宝玉的贾宝玉是幸福的,但更大的幸福却发生在他掷玉的刹那。那时,他初遇黛玉,一见面,彼此惊为旧识,仿佛已相契了万年。他在惊愕中竟把那块玉胡乱砸在地上,那种自我的降服和破碎是动人的,是真爱情最醇美的倾注。
   ⑤文学史上也不乏这样的例子。陈师道一遇见黄山谷,就烧掉自己的书稿,虚心去向黄山谷学习。一个人能碰见令自己心折首俯的人,并能一把火烧尽自己的旧作,应该算是一种极幸福的际遇。
   ⑥《新约》中的先知约翰,一见耶酥,便屈身降志。他对人们说:“我仅仅以水为你们施洗礼,他却以灵为你们施洗礼,我之于他,只能算一声开道的吆喝声。”《红指传》里的虬髯客一见李靖,便知天下大势已定,于是飘然远引。那使无数人为之倾倒的大唐盛世的李靖,我多么想见他一眼啊!清朝末年的孙中山也有如上风仪,使四方豪杰甘于俯首授命。生的悲剧原不在头断血流,而在于没有大英雄可为之赴命,没有大理想供其追求。
   ⑦我一直在寻求挫败,人生天地间,还有什么比失败更快乐的事?站在千丘万壑的大峡谷前,感到自己渺小得像蝼蚁一样,还有什么时候能如此心甘情愿地卑微下来,享受大化的赫赫天威?曾记得一个夏夜,卧在海滩上,看到满天繁星如雨阵如箭簇,一时几乎惊得昏了过去,感到一种投身在伟大之下的绝望,知道人类永远不能去逼近那百万光年之外的光体。不过,这份绝望使我一想起来就觉兴奋昂扬。试想,宇宙如果像一个窝囊废一样被我们征服了,日子会多么无趣啊!读圣贤书,其理亦然。读一些好书,仿佛看见洞照古今长夜的明灯,听见声彻人世的巨钟,心中自会涌出一份不期然的惊喜,知道我虽遇鲁,天下人间能人正多,这一番心悦诚服,使我几乎要大声宣告说:“多么好!人间竟有这样的人、这样的书!我连死的时候都可以安心了!因为有这样优秀的人,有这样美丽的思想。”此外,在印度见到特瑞沙,在非洲见到史怀哲,或是在美术馆看“八大山人”、石涛,在博物院看周鼎宋瓷,都会兴起一份“我永世不能追摹到这种境界”的激动。这种激动,这种虔诚的服输,是多么难忘的大喜悦啊。
   ⑧如果问我此生还有什么未了的心愿,那便是不断遇到更令我心折的人,不断探得更勾魂摄魄的美景,好让我能更彻底地败溃,从心底承认自己的卑微和渺小。
   1.③④段列举的一反一正两个例子,各说明什么道理?
   答:
   2.填空。
   从第⑥段看,大英雄指___、___、___三人。
   3.第⑦段画线句子中,“这份绝望”是指什么?作者为什么“兴奋昂扬”?
   答:
   4.作者“寻求挫败”的目的是什么?请用文中的句子回答。
   答:
   5.本文标题是“我寻求挫败”,但③-⑥段列举了别人的例子,这是否有必要?请说说你的理由。(可以发表不同的见解)
   答:
   (十二)(2004·福州)
   成功
   季羡林
   什么叫成功?顺手拿过来一本《现代汉语词典》,上面写道:“成功,获得预期的结果。”言简意赅,明白之至。
   但是,谈到“预期”,则错综复杂,纷纭混乱。人人每时每刻每日每月都有大小不同的预期,有的成功,有的失败,总之是无法界定,也无法分类,我们不去谈它。我在这里只谈成功,特别是成功之道。这又是一个极大的题目,我却只是小做。积七八十年之经验,我得到了下面这个公式:
   + + =成功
   “天资”,我本来想用“天才”,但天才是个稀见现象,其中不少是“偏才”,所以我弃而不用,改用“天资”,大家一看就明白。这个公式实在是过分简单化了,但其中的含义是清楚的。搞得太烦琐,反而不容易说清楚。
   谈到天资,首先必须承认,人与人之间天资是不相同的,这是一个事实,谁也否定不掉。十年浩劫中,自命天才的人居然大肆批判天才。葫芦里卖的是什么药,至今不解。到了今天,学术界和文艺界自命天才的人颇不稀见,我除了羡慕这些人“自我感觉过分良好”外,不敢赞一词。对于自己的天资,我看,还是客观一点好,实事求是一点好。
   至于勤奋,一向为古人所赞扬。韩愈的“焚膏油以继晷,恒兀兀以穷年”,更为读书人所向往。如果不勤奋,则天资再高也毫无用处。事理至明,无待饶舌。
   谈到机遇,往往为人所忽视。它其实是存在的,而且有时候影响极大。就以我自己为例,如果清华不派我到德国去留学,则我的一生完全不会像现在这个样子。
   把成功的三个条件拿来分析一下,天资是由“天”来决定的,我们无能为力。机遇是不期而来的,我们也无能为力。只有勤奋一项完全是我们自己决定的,我们必须在这一项上狠下功夫。在这里,古人的教导也多得很。还是先举韩愈。他说:“业精于勤,荒于嬉;行成于思,毁于随。”这两句话是大家都熟悉的。
   王国维在《人间词话》中说:“古今之成大事业、大学问者,必经过三种之境界:‘昨夜西风凋碧树。独上高楼,望尽天涯路。’此第一境也。‘衣带渐宽终不悔,为伊消得人憔悴。’此第二境也。‘众里寻他千百度。蓦然回首,那人却在,灯火阑珊处。’此第三境也。”王国维第一境写的是预期。第二境写的是勤奋。第三境写的是成功。其中没有写天资和机遇。我不敢说这是他的疏漏,因为写的角度不同。但是,我认为,补上天资与机遇,似更为全面。我希望,大家都能拿出“衣带渐宽终不悔”的精神来做学问或干事业,这是成功的必由之路。
   1.根据文章内容,把下面的公式填写完整。
   + + =成功
   2.作者认为成功的关键是 。
   3.本文开头通过 ,自然地引出论题;结尾提醒人们
   ,强化文章的中心。(本题用自己的语言概括)
   4.请为本文补写一个有关勤奋的名人事例。
   答:
   5.文中说“天资是由‘天’来决定的,我们无能为力。机遇是不期而来的,我们也无能为力。”你同意这样的说法吗?请就“天资”或“机遇”任选一点谈谈你的看法。
   答:
   (十三)(2004·恩施)
   心灵的空间
   周国平
   泰戈尔写过一段话,意思是说:一个富翁的富并不表现在他的堆满货物的仓库和一本万利的经营上,而是表现在他能够买下广大空间来布置庭院和花园,能够给自己留下大量时间来休闲。同样,心灵中拥有开阔的空间也是最重要的,如此才会有思想的自由。接着,泰戈尔举例说,穷人和悲惨的人的心灵空间完全被日常生活的忧虑和身体的痛苦占据了,所以不可能有思想的自由。我想补充指出的是,除此之外,还有另一类例证,就是忙人。
   凡心灵空间的被占据,往往是出于逼迫。如果说穷人和悲惨的人是受了贫穷和苦难的逼迫,那么,忙人则是受了名利和责任的逼迫。名利也是一种贫穷,欲壑难填的痛苦同样具有匮乏的特征,而名利场上的角逐同样充满生存斗争式的焦虑。至于说到责任,可分三种情形,一是出自内心的需要,另当别论,二是为了名利而承担的,可以归结为名利,三是既非内心自觉,又非贪图名利,完全是职务或客观情势所强加的,那就与苦难相差无几了。所以,一个忙人很可能是一个心灵上的穷人和悲惨的人。
   这里我还要说一说那种出自内在责任的忙碌,因为我常常认为我的忙碌属于这一种。一个人真正喜欢一种事业,他的身心完全被这种事业占据了,能不能说他也没有了心灵的自由空间呢?这首先要看在从事这种事业的时候,他是否真正感觉到了创造的快乐。譬如说写作,写作诚然是一种艰苦的劳动,但必定伴随着创造的快乐,如果没有,就有理由怀疑它是否蜕变成了一种强迫性的事务,乃至一种功利性的劳作。当一个人以写作为职业的时候,这样的蜕变是很容易发生的。心灵的自由空间是一个快乐的领域,其中包括创造的快乐,阅读的快乐,欣赏大自然和艺术的快乐,情感体验的快乐,无所事事地休闲和遐想和快乐,等等。所有这些快乐都不是孤立的,而是共生互通的。所以,如果一个人永远只是埋头于写作,不再有工夫和心思享受别的快乐,他的创造的快乐和心灵的自由也是大可怀疑的。
   我的这番思考是对我自己的一个警告,同时也是对所有自愿的忙人的一个提醒。我想说的是,无论你多么热爱自己的事业,也无论你的事业是什么,你都要为自己保留一个开阔的心灵空间,一种内在的从容和悠闲。惟有在这个心灵空间中,你才能把你的事业作为你的生命果实来品尝。如果没有这个空间,你永远忙碌,你的心灵永远被与事业相关的各种事业所充塞,那么,不管你在事业上取得了怎样的外在成功,你都只是损耗了你的生命而没有品尝到它的果实。
   (选自《青春潮》2002年第9期)
   1.“心灵的空间”指的是什么?
   答:
   2.“穷人”、“悲惨的人”和“忙人”心灵空间的被占据,各自最主要的原因是什么?哪一个词能揭示这一点?
   答:
   3.“出自内在责任的忙碌”是一种什么样的忙碌?这种忙碌是否同样需要保留一个开阔的心灵空间?为什么?
   答:
   4.你见到过下面三种“忙人”吗?请选择其中一种举一个例子。
   ①发自内心的需要;②为了名利;③为了职务或客观情势。
   答:
   5.作者在文中希望所有的忙人“都要为自己保留一个开阔的心灵空间,一种内在的从容和悠闲”,作者为什么要这样说?
   答:
   (十四)(2004·黄冈)
   错过就会永远失去(青杨)
   ⑴几年前,在北京孔庙的卖纪念品处,反复地放着古乐器埙的音乐磁带,十分地悠远苍凉。从远古飘来的声音,重重地敲击着自己麻木的灵魂。问了一下价,卖15元一盘,只觉得太贵,就放下了。出了孔庙大门,立即就又后悔了,可是再进去还得花15元买门票,便作罢。心想或许别处还能买到。然而,别处竟然没有。那埙的奇音至今揪扯着心中的悔,想起来就隐隐地针扎般地痛。
   ⑵一次,街上来了个卖水仙花根块的,我正骑着车去上班,懒得停下来,心想等一会儿下班时再买也不迟。可是,下班后再找时那人早已不知去向。本来非常喜欢水仙花的翠叶,哪怕就是不开花,只为那一丛亭亭昂首的青葱也值得。然而,又一次与美好的东西失之交臂,那个冬的居处就没有了一簇篷勃的春意在案头挺立,心中也就怎么也生不出厚重的诗情画意来。
   ⑶……
   ⑷一次又一次地错过,一次又一次地失去。失去的都是让自己梦魂牵绕的好东西。失去了,竟然就从此永远无法找寻,无处可觅了。
   ⑸错过就是永远的失去。人们非亲历不能明白这一点。
   ⑹一次失足,将导致一生走不出创伤的阴影;一次堕落,将写下黄河水洗不清的履历;一次抉择的不当,即能造成终生懊悔的遗憾;一次关键时刻的错判,将形成永远不能弥补的过失。
   ⑺命运有时就是一根洪水中的木头,抓不住就失去了生存的机遇;成就有时就是沿着选中的路头也不回地走下去,观望等待得久了也会被淘汰;幸福有时就是吃尽百般苦后品出的甘甜,贪图享受安逸的人很难体味得到;事业有时就是少说多做的人捡到的一枚果实,夸夸其谈都永远嗅不到她的馨香;荣誉有时就是付给孤独的创造者的酬金,不甘寂寞的人不可能获得这份报偿。
   ⑻功业、幸福、成就、荣誉,一个个都曾经从每个人的面前经过。抓住的人就走出了碌碌无为的平庸,一步步踏上胜利的顶峰;错过的人即将陷入无尽的烦恼,一点点跌入失败的深渊。所以,世人永远有高低中下之分。
   ⑼错过就是永远的失去。这一点不容置疑。
   1.全文表达的主要观点是什么?
   答:
   2.在第⑴段中找出与第⑷段“失去的都是让自己梦魂牵绕的好东西”一句相照应的语句。
   答:
   3.请在文中第⑶段省略号处补上一段自己的亲身经历。(50字左右)
   答:
   4.品析文章第⑹段,回答问题。
   ①从修辞运用的角度谈谈本段语言特点及作用。
   答:
   ②用一句名言或俗语概括本段的内容。
   答:
   (十五)(2004·荆州)
   人是一本书
   人是一本书。
   看人如读书。
   人的外表,不就是封面吗?
   人的内心,不就是内容吗?
   有的人,封面与内容同样高雅;有的人,封面与内容同等粗俗。有的人,封面很不起眼,内容却精彩无比;有的人,外表包装得如金似玉,内心却不堪入目。
   好人自然是一本好书,你打开扉页就有一缕鲜花的芬芳迎面而来。
   坏人自然是一本不好的书,你才揭开封面就能嗅到一股难闻的气味。
   ( )的人像一本散文集,( )的人是一本抒情诗,( )的人如同一本幽默辞典,( )的人仿佛一部科学论著。那些祸国殃民最终遗臭万年的民族败类,不就是一套活生生的反面教材吗?而那些为国为民洒尽一腔热血的志士,又多像一卷可歌可泣的英雄史诗啊!
   做人如写书。
   请不要怪母亲没有给你制出漂亮的封面,也不要怪父亲没有给你给出美丽的插图。你自己这本书,主要还得靠你自己来写——用你的勇气和力量,用你的聪明和才智,用你的心血和汗珠……
   然而,总有一些人把不该删除的真诚删除了,结果自己这本书中没有了纯洁的位置。他们都忘记了:在生命的版面上,应该写下高尚,应该填上纯真,应该刻画奉献,应该描绘进取。
   也许,你写了一生也成不了杰作;也许,你写了一世也成不了名著。但决不能因此而粗制滥造,随便应付,哪怕写出的书无人阅读,也要篇篇对得起自己的良心。
   如果有了漂亮的开头,就应该追求完美的结局;如果有了精彩的材料,就应该将它打磨成实在的文章。
   如果人们常常把你亲切地朗读,那就是你无上的光荣;如果人们永世把你深情地背诵,那就是你最大的幸福。
   1.写法体味——贯串全文的一个比喻句是:
   答:
   2.要点概括——这篇文章中能概括地表达全文要点的句子是:
   答:
   3.词句理解——将下面四个短语分别归位于文中括号,其先后顺序应是:
   思维缜密 洒脱飘逸 多愁善感 乐观风趣
   答:
   4.语言品味——从文中摘取一个你认为有格言警句作用的精美句子,写在下面:
   答:
   5.内容探究——本文最后一段话给我们的暗示是:
   答:
   6.情感体验——以“人生”为话题,写一个短小精美有内涵的句子:
   答:

 (二十六)(2004·新疆)
   毕业赠言
   胡适
   诸位毕业同学:
   ⑴你们现在要离开母校了,我没有什么礼物送给你们,只好送你一句话吧。这一句话是:“不要抛弃学问。”
   ⑵以前的功课也许一大部分是为了这张毕业文凭,不得已而做的。从今之后,你们可以依自己的心愿去自由的研究了。趁现在年富力强的时候,努力做一种专门学问。少年是一去不复返的,等到精力衰竭时,要做学问也来不及了。即为吃饭计,学问也绝不会辜负人的。吃饭而不求学问,三年、五年之后,你们都要被后来少年淘汰的。到那时再想做点学问来补救,恐怕已太晚了。
   ⑶有人说:“出去做事之后,生活问题需要解决,哪有工夫去读书?即使要做学问,既没有图书馆,又没有实验室,哪能做学问?”
   ⑷我要对你们说:凡是要等到有了图书馆才能读书的,有了图书馆也不肯读书。凡是要等到有了实验室才做研究的,有了实验室也不肯做研究。你有了决心要研究一个问题,自然会缩衣节食去买书,自然会想出法子来设置仪器。
   ⑸至于时间,更不成问题。达尔文一生多病,不能多做工,每天只能做一点钟的工作。你们看他的成绩!每天花一点钟看10页有用的书,每年可看3600多页,30年可读11万页书。
   ⑹诸位,11万页书可以使你成为一个学者了。可是,每天看3种小报也得费你一点钟的工夫;4圈麻将也得费你一点钟的光阴。看小报呢,还是打麻将呢,还是努力做一个学者呢?全靠你自己的选择!
   ⑺易卜生说:“你的最大责任是把你这块材料铸造成器。”
   ⑻学问便是铸器的工具,抛弃了学问便是毁了你们自己。
   ⑼再会了!你们的母校眼睁睁地要看你们10年之后成什么器。
   1.这篇演讲的中心话题是什么?
   答:
   2.古诗文中,有很多同文段⑵中的划线句子意思相同的名言,请写出两句或两句以上这种名言。(如果所写名言是上下句的,两句算一句;如果名言是单名的,一句算一句。)
   答:
   3.根据上下文说说“凡是要等到有了图书馆才能读书的,有了图书馆也不肯读书”这句话的含义。
   答:
   4.文段⑸使用了什么论证方法,请你再分别补充一个类似的事例和名言。
   答:
   5.请你说说本文的语言特点。
   答:
   6.请给你的同学写一段毕业赠言。
   答:
   7.设想一下十年后你的工作或生活情景。
   答:
   (二十七)(2004·湖州)
   学问与智慧
   学问与智慧有显然的区别。学问是知识的聚集,是一种滋养人生的原料,而智慧却是陶冶原料的熔炉。学问好比是铁,而智慧是炼钢的电火。
   学问是不能离开智慧的;没有智慧的学问,便是死的学问。有许多人从事研究工作,搜集了很多材料,但往往石乞石乞(ku)穷年,找不到问题的中心,得不到任何的结果,纵有结果,亦无关宏旨,这便是由于没有智慧。而有智慧的人则不然,他纵然研究一个极小的问题,也能探骊得珠,找到核心所在;其问题虽小,但映射的范围却往往很大。譬如孟德尔研究豌豆的杂交,居然悟出遗传的定律,奠定了遗传学和优生学的基础。
   学问固然离不开智慧,同时智慧也不能离开学问。有学问的人,虽然不一定就有智慧,正如有智慧的人不一定有很深的学问一样,但是智慧却必须以学问为基础,才靠得住。无学问的智慧,只是浮光掠影,瞬起瞬灭的。它好象肥皂泡一样,尽管可以五光十色,但是一触就破。惟有从学问中产生出来的智慧,才不是浮光掠影,而是像探照灯一样,可以透过云层,照射寥廓天空。惟有从学问中锻炼出来的智慧,才不是瞬息幻灭的肥皂泡;它永远像珍珠泉的泉水一样,一串串不断地从水底往上涌。也惟有这种有根底的智慧,才是靠得住,最为精深,最可宝贵。
   学问与智慧是相辅为用,缺一不可的。我们不但需要学问,而且更需要智慧——需要以智慧去笼罩学问、透视学问、运用学问。
   1.结合全文来看,作者论述的观点是什么?
   答:
   2.仔细阅读第二段,说说该段运用了那些论证方法?
   答:
   3.第三段中划线的词语“肥皂泡”“探照灯”在文中所表达的意思是什么?
   答:
   (二十八)(2004·芜湖)
   鸟儿中的理想主义
   我对笼中继续扑翼的鸟一直怀有敬意。
   几乎每一只不幸被捕获的鸟,刚囚入笼中都是拼命扑翼的,它们不能接受突然转换了的现实场景,它们对于天空的记忆太深,它们的扑翼是惊恐的,焦灼不安的,企图逃离厄运的,拒绝承认现实的。然而一些时日之后,它们大都安静下来,对伸进笼里来的小碗小碟中的水米,渐渐能以一种怡然的姿态享用。它们接受了残酷的现实,并学会把这看成生存的常态。它们的适应能力是很强的。适应能力强,这对人,对鸟,对任何生物,都是一个褒奖的词语。它们无师自通,就懂得了站在主人为他们架在笼中的假树杈上,站在笼子的中心位置,而不是在笼壁上徒劳地乱撞,就像主人期待的那样,优雅地偏头梳理它们的羽毛。如果有同伴,就优雅地交颈而眠。更重要的是,当太阳升起的时候,或者主人逗弄的时候,就适时适度地婉转地歌唱,让人感到生活是如此的自由、祥和、闲适。而天空和扑翼这种与生俱来的事情,也就是多余的了。
   但有一些鸟的适应能力却很差,这大抵是鸟类中的古典主义者或理想主义者。它们对生命的看法很狭隘,根本不会随现实场景的转换而改变。在最初的惊恐与狂躁之后,它们明白了厄运,它们用最荏弱的姿态来抗拒厄运。它们是安静的,眼睛里是极度的冷漠,对小碾小碗里伸过来的水米漠然置之,那种神态,甚至让恩赐者感到尴尬,感到有失自尊。鸟儿的眼睛里一旦现出这样的冷漠,就不可能再期待它们的态度出现转机,无论从小笼子换到大笼子,还是把粗瓷碗换成金边瓷碗,甚至再赏给它一个快乐的伙伴,都没有用了。这一切与它们对生命的认定全不沾边儿。事实上,这时候它们连有关天空的梦也不做了,古典主义者总是悲观的、绝望的,它们只求速死。命运很快就遂了它们的心愿。
   而我一直怀有敬意的,是鸟儿中的另一种理想主义者,这种鸟儿太少,但我侥幸见过一只,因为总是无端想起,次数多了,竟觉得这种鸟儿的数目似乎在我感觉中也多了。
   我见到这只鸟儿的时候,它在笼中已关了很久了,我无从得见它当初的惊恐和焦灼,不知它是不是现出过极度的冷漠,或者徒劳地撞击笼壁,日夜不停地用喙啄笼壁的铁丝。我见到它的时候,它正笼子里练飞。它站在笼子底部,扑翼,以几乎垂直的路线,升到笼子顶部,撞到那里,跌下来,然后仰首,再扑翼……这样的飞,我从来没见过。它在笼中划满风暴的线条,虽然这些线条太短,不能延伸,但的确饱涨着风暴的激情。它还绕着笼壁飞,姿态笨拙地,屈曲着,很不洒脱,很不悦目,但毕竟它是在飞。它知道怎样利用笼内有限的气流,怎样锻炼自己的翅膀,让它们尽可能地张开:尽可能地保持飞翔的能力。
   一般我们很难看见鸟是怎样学飞的,那些幼鸟,那些被风暴击伤了的鸟,那些在岩隙里熬过隆冬的鸟,还有那些被囚的鸟。这是一件隐秘的事。我们只看见过它们在天空中划过,自由地扑翼,桀骜地滑翔,我们只羡慕上帝为它们造就了辽阔的天空。
   但在看到那只在笼中以残酷的方式练飞的鸟之后,我明白,天空的辽阔与否,是由你自己造就的,这种事情上帝根本无能为力。上帝只是说,天空和飞翔是鸟类的生命形式,而灾难和厄运也是世界存在的另一种形式。至于在灾难和厄运中你是否放弃,那完全是你自己的事情。
   (选自《中华活页文选》,作者筱敏)
   1.文中写了哪几种鸟?这几种鸟分别象征社会生活中哪几种人?
   答:
   2.对于笼中鸟来说,既然天空和扑翼是“与生俱来”的事情,为什么此刻成了“多余”的?
   答:
   3.作者对鸟类中的“古典主义者或理想主义者”持什么态度?
   答:
   4.那只鸟儿以残酷的方式练飞的目的是什么?
   答:
   5.你怎样理解“灾难和厄运也是世界存在的另一种形式”?
   答:
   (二十九)(2004·青岛)
   感悟春天
   [阅读提示]
   本文赞美了春天的美好,表达了对春天的热爱,但作者并没有局限于此,而是揭示出一个深刻的道理,从中感悟到人生的真谛。
   ⑴江南的春天,总是在淅淅沥沥的春雨中悄悄来临,“杏花春雨江南”,这是上了名诗之页的。历数我经历的几十个春天,总是乍暖还寒,需要反反复复冲破lǐnliè的西风和反攻的寒潮才降临人间,因此春光很金贵,春光也往往只能雨中窥视,总在不经意中,便进入了初夏的境地,常常令人生出春光难觅的感慨。
   ⑵珍爱春天,是热爱生活的一种执著。这也是对照冬天的岁月,有比较而有鉴别的一种实际行动,虽然说冬天的寒冷对于世界,是净化,是考验,是生命的再生前提和生活的新生要素;这也是一种反衬和折射:没有坚冰,谁去认可红梅的烂漫?没有白雪,谁去判定青松的高洁?但是那似乎是哲学的命题和诗人的想像。只有春天才是大众的,才是为伟人和凡人、诗人和农人共同承认和赞美的。
   ⑶谁不承认春天是美好的?“遇物尽欢欣,爱春非独我”,诗人白居易就是这样说的。翻翻一部唐诗吧,那里面尽有李白、杜牧、韩愈、李贺们对春的感慨。摆着道学面孔的人们,是不是不咏唱春天呢?北宋理学家程颢写下了《春日偶成》,还成了《千家诗》的第一首。朱熹更是有名的“老夫子”,他面对朗朗春光,盈盈春意,也吟唱了“等闲识得东风面,万紫千红总是春”的《春日》诗。和尚是讲究四大皆空的佛门子弟,也不能不被春天打动“凡心”。南宋的志南和尚写下了这样的名句:“沾衣欲湿杏花雨,吹面不寒杨柳风。”在中国的古诗中,可以看出诗人们从各个角度歌唱春天,为春风、春水、春雨、春草、春柳、春鸟、春山、春游,春宵、春眠等尽情歌唱,进而从欣赏春天写到珍惜春光。
   ⑷品味春色,沐浴春风,可以感受生活,感悟人生。这可以说是中外一致,概莫能外的。“闺女中少女不知愁,春日凝装上翠楼。忽见陌头杨柳色,悔教夫婿觅封侯。”如果这位先生在封建时代的女子奈何不了这种生活的话,那么现代的那位美国哲人桑塔耶那则更有甘愿为春天俘虏的意味了。这先生长期在哈佛教书,五十岁那年的春日,有一天在讲台上,偶有知更鸟飞来,立在窗格子上,他注意到了,感受到又一个春天的到来。他回过头来,对学生们说,我与阳春有约!冲出教室,开始了他云游欧洲大陆之行。
   ⑸生命四季,是我们每个人都共有的,而春天更能开启人们的感情之源,心灵之泉,难怪首先在甲骨文里露面的形声字就是“春”,难怪古人殷殷切切地寄语他的友人“此行江南去,千万和春住”。居住在大观园中的史湘云更是大声疾呼:“且住,且住,莫使春光别去!”可是事实上春光难永驻,有来必有去。谁也挡不住“落花流水春去也”的结局。不管是落花有意,还是流水无情,都表现了时间之易逝的自然迹象,但是精神上的青春,意志里的春色,进取中的春光,则当常驻于有志人和奋斗者的岁月年华里!
   1.依据第⑴段中的注音,写出恰当的汉字。
   “需要反反复复冲破lǐnliè 的西风和反攻的寒潮才降临人间。”
   2.通读全文,仔细体会,作者通过对春天的赞美,从中揭示了一个怎样的道理?表达了作者什么样的人生态度?
   答:
   3.从第⑴段来看,作者认为“春光很金贵”的理由是什么?由此引发了作者怎样的感慨?
   答:
   4.第⑵段最后说,“只有春天才是大众的,才是为伟人和凡人、诗人和农人共同承认和赞美的。”请谈谈你对这句话的理解。
   答:
   5.阅读第⑶段,将其中引用的诗句、诗句的作者及其生活的朝代,摘录在下面的读书卡片中,并选取其中你最喜欢的一例诗句,写出喜欢的原因。(要求:最喜欢原因的表述要紧扣诗句要点;语言要准确简明,生动流畅)
   读书卡片编号: 068
   作者 朝代 引用的诗句 最喜欢的诗句及原因
  
  
  
   6.第⑷段开头写道,“品味春色,沐浴春风,可以感受生活,感悟人生。”请联系上下文,反复品味、揣摩这句话,谈谈你的感悟。
   答:
   (三十)(2004·四省区)
   欣赏使人美丽
   ⑴面对同一棵树,有人看到了绿叶,有人看到了毛毛虫。真诚的人能够感知到他人的真诚,而骗子总觉得全世界的人都在说谎。善良的人会被别人的善举感动,在缺乏善心的人眼里,善就是傻……从这个意义上说,会不会欣赏别人是对自身品质的一种检验;学会欣赏别人,则是对自身品质的一种提升;对被欣赏者来说,欣赏是一种引导和激励。
   ⑵19世纪末,美国西部的密苏里有一个坏孩子,他偷偷地向邻居家的窗户扔石头,还把死兔子装进桶里放到学校的火炉里烧烤,弄得臭气熏天。他9岁那年,父亲娶了继母,并对继母说:“亲爱的,你要好好注意他,他是全郡最坏的孩子,他让我头疼死了,说不定会在明天早晨以前就向你扔石头,或者做出别的什么坏事,让你防不胜防。”继母好奇地走近这个孩子,当她对孩子有了了解之后说:“你错了,他不是全郡最坏的孩子,而是最聪明的孩子,只是还没有找到发挥他聪明的地方罢了。”继母很欣赏这个孩子,在她的引导下,这个孩子的聪明找到了发挥的地方,后来成了美国当代著名的企业家和思想家。这个人就是戴尔·卡内基。
   ⑶台湾作家林清玄去一家羊肉馆用餐,老板对他说:“你还记得我吗?”林清玄说:“记不起来了。”老板拿来一张20年前的旧报纸,那里有林清玄的一篇文章,那时他在一家报社当记者。这是一篇关于小偷的报道,小偷手法高超,作案上千次,次次得手,最后栽在一个反扒高手的手上。文章感叹道:“像心思如此细密,手法如此灵巧,风格如此独特的小偷,做任何一件事情都会有成就的吧!”老板告诉他:“我,就是那个小偷,是你的这段话引导我走上了正路。”如今他开了好几家羊肉馆,成了那里颇有名气的大老板。
   ⑷连小偷身上也有可欣赏的地方,连小偷也能在欣赏的引导下走上正路,我们周围还有什么人不能欣赏、不能被引导呢?
   ⑸学会欣赏别人吧!欣赏你的同事,你和同事之间会合作得更加亲密;欣赏你的下属,下属会工作得更加努力;欣赏你的爱人,你们的爱情会更加甜蜜;欣赏你的孩子,说不准他就是下一个卡内基……学会欣赏你周围所有的人,你周围的世界就会变得更加美丽。
   1.从文章第⑴段知道欣赏具有双重作用:一是 ;二是
   。
   2.运用下面格式,依次概括⑵⑶段内容。
   欣赏 ,使他变成 ;
   欣赏 ,使他变成 。
   3.下列句子可以作为本文道理论据的一项是( )
   A.当我们在生活中学会欣赏,我们会惊喜地发现:那些蛰伏的不起眼的种子,原本可以轻易地发芽、开花、结果。
   B.敬人者,人恒敬之;关照别人,就是关照自己。
   C.欣赏别人得到的回报是超值的,就如同通过欣赏本想得到的一片枫叶,而得到的却是一片枫林。
   D.美是生活的阳光,是大地的精华,是艺术的冠冕,是人心中开放的鲜花。没有对美的渴求,人类可能还混迹于一般的动物群中。
   4.比较文中加线句和下面的改句,谈谈原句表达的好处。
   这位小偷心思细密,手法灵巧,风格独特,做任何事情都会有成就。
   原句表达的好处:
   5.你有欣赏别人或被别人欣赏的经历吗?请简要叙述并谈谈当时的心理感受?(不要出现真实的人名、地名和校名)
   经历:
   感受:
   (三十一)(2004·绵阳)
   我对这位传奇英雄的认识,有一个突变。
   为了寻找盖文学馆新馆舍所用国产装饰石材,我和我的同事来到了新疆。
   我知道,鸦片战争之后,林则徐曾被充军新疆,一直发配到伊犁,是真正的受罚。多少年过去了,当地的政府和人民,为了纪念他,特地给他在老的伊犁将军府附近盖了一座纪念馆。我决定抽空去看看。
   在去纪念馆的路上,我看见路旁一条很长很规矩的小河,大概有5米宽,河水流得很冲,岸两旁有高大的树。开车的司机说:它是渠,是人工开凿的,叫“皇渠”,是林则徐大人当年留下来的,为的是把天山上的雪水引下来,灌溉土地,变荒地为良田。这水渠至今已有160年的历史了。
   这个小故事极感人。
   它一下把这位举世闻名的英雄老人拉得很近很近了。我对他,不是一般的崇敬了。复杂了,真的,复杂得多了。该怎么说呢?往大里说,牵扯到人该怎么活,怎样做人,做什么样的人,甚至,整个儿的,人生的价值是什么,都可以由这个故事中派生出一些认真的思考来。往具体里说,想想看,那时怎样的逆境啊:硝烟弥漫之后,戴罪受罚,背井离乡,充军万里,凄凉之至。然而,他却依然一副大将风度,在别人的监控之下,指挥黎民百姓和部分官兵,披星戴月,风尘仆仆,终日挖渠引水不止。何等坦荡,何等乐观,何等大度,何等潇洒!简直就是一个活生生的样板,一个树在你我面前的,伟大的,不说教的,默默的,摸得着看得见的,平凡的,可以效仿的楷模。
   总之,这条小渠给我带来了心灵震撼,令我激动不已。它给我一个巨大的感动。
   林则徐的一生与其说是官运亨通,不如说是充满了大起大落。他多次受罚,比如连降四级,连降五级,竟有许多次。最大的处罚是发配新疆。然而他的态度是:
   苟利国家生死以 岂因祸福避趋之
   一个真正以国家利益为重的人的处世哲学便是如此鲜亮简明,有小渠可以作证。
   新疆土地极肥沃,天生一块大宝地:风和日丽,日照长,有利于植物的光合作用;只要有水,种什么长什么,而且硕大,丰产。林则徐抓住这个要害,挖渠引水,做了表率。而他做这一切的时候,偏偏是个犯人的身份,而且年老体弱,精神上又遭到了极大的打击。
   在最倒霉的时候,在最倒霉的地点,在最倒霉的情况下,顶着最倒霉的屈辱,干着最普通、最费力、最不容易露脸的事。但只要有利于人民,有利于国家,有利于后代,便在所不辞,管他是沉是浮。这就是林则徐的风格。
   这个风格是铮铮硬骨头的作风。
   这个风格是民族脊梁的象征。
   这个风格是真正人生价值的体现。
   我为那遥远的小渠而骄傲,它不是什么“皇”渠,它是地道的“人”渠。
   一个姓林名则徐的神人,用这小渠,在天地之间,龙飞风舞般地书写了一个大大的“人”字。
   什么时候再去看看那小渠,用清澈冰凉的渠水洗洗脸,当做接受一次沁人肺腑的人生洗礼吧。
   1.请在原文中画出与“苟利国家生死以,岂因祸福避趋之”相呼应的一句话。
   2.文中画线的三个句子组合在一起,构成什么修辞手法?有何作用?
   答:
   3.作者为什么称“皇渠”为“人渠”呢?
   答:
   4.作者开篇就说对林则徐的认识“有一个突变”,结合全文,谈谈这个突变的具体内容。
   答:
   5.下面对该文的赏析,正确的两项是( )
   A.林则徐作为一个顶天立地的英雄,不但有为人民鞠躬尽瘁的精神,而且有科学头脑。
   B.“我”原来对林则徐知之不详,到了新疆,看到了这条渠,才对他崇敬起来。
   C.“何等坦荡,何等乐观,何等大度,何等潇洒”中的“潇洒”一词表现了林则徐不拘世俗,风流倜傥的风度。
   D.全文感情真挚,气势酣畅,一唱三叹,缠绵悱恻。
   E.这条水渠不仅造福一方,而且可以成为净化人们心灵世界的镜子。
   (三十二)(2004·南宁)
   生命常常是如此之美
   乔 叶
   ⑴每天下午,接上孩子之后,我都要带着他在街上溜达一圈。闲走的时候,看着闲景,说着闲话,我觉得这是上帝对我劳作一天的最好奖赏。每次我们走到文华路口,我就会停下来,和一个卖小菜的妇人聊上几句,这是我们散步的必有内容。这个妇人脸色黑红,发辫粗长,衣着俗艳,但是十分干净。她的小菜种类繁多,且价廉物美,所以常常是供不应求,我常在她这里买菜,所以彼此都相熟。因此每次路过,无论买不买菜,都要停下和她寒暄几句。客户多的时候,也帮她装装包,收收钱。她会细细地告诉我,今天哪几样菜卖得好,卤肉用了几个时辰,西兰花是从哪个菜市上买的,海带丝和豆腐卷怎样才能切得纤细如发,而香菇又得哪几样料配着才会又好吃又好看。听着她絮絮的温语,我就会感到一波波隐隐的暖流在心底盘旋。仿佛这样对我说话的,是我由来已久的一个亲人。而孩子每次远远地看见她,就会喊:“娘娘!”——这种叫法,是我们地方上对年龄长于自己母亲的女人的昵称。
   ⑵那位妇人的笑容,如深秋的土地,自然而醇厚。
   ⑶一天夜里,我徒步去剧院看戏,散场时天落了小雨,便叫了一辆三轮车。那个车夫是个年近五十的白衣汉子,身材微胖。走到一半路程的时候,我忽然想起附近住着一位朋友,我已经很久没见到她了,很想上去聊聊。便让车夫停车,和他结账。
   ⑷“还没到呢。”他提醒说,大约以为我是个外乡人吧。
   ⑸“我临时想到这里看一位朋友。”我说。
   ⑹“时间长吗?我等你。”他说,“雨天不好叫车。”
   ⑺“不用。”我说。其实雨天三轮车的生意往往比较好,我怎么能耽误他挣钱呢?
   ⑻然而,半个小时后,我从朋友的住处出来,却发现他果真在等我。他的白衣在雨雾中如一团蒙蒙的云朵。
   ⑼那天,我要付给他双倍的车费,他却执意不肯:“反正拉别人也是拉,你这是桩拿稳了的生意,还省得我四处跑呢。”他笑道。
   ⑽负责投送我所在的居民区邮件的邮递员是个很帅气的男孩子,看起来只有二十岁左右。染着头发,戴着项链,时髦得似乎让人不放心,其实他工作得很勤谨。每天下午三点多,他会准时来到这里,把邮件放在各家的邮箱里之后,再响亮地喊一声:“报纸到了!”
   ⑾“干吗还要这么喊一声呢?是单位要求的吗?”我问。
   ⑿他摇摇头,笑了:“喊一声,要是家里有人就可以听到,就能最及时地读到报纸和信件了。”
   ⒀后来,每次他喊过之后,只要我在家,我就会闻声而出,把邮件拿走。其实我并不是急于看,而是不想辜负他的这声喊。要知道,每家每户喊下去,他一天得喊上五六百声呢。
   ⒁生活中还有许多这样的人,都能给我以这种难忘的感受。只是看到他们,一种无缘由的亲切感就会漾遍全身。我不知道他们的姓名和来历,但我真的不觉得他们与我毫不相干。他们的笑容让我 ,他们的忧愁让我 ,他们的宁静让我
   ,他们的匆忙让我 。我明白我的存在对他们是无足轻重的,但是他们对我的意义却截然不同。我知道我就生活在他们日复一日的操劳和奔波之间,生活在他们一行一行的泪水和汗水之间,生活在他们千丝万缕的悲伤和欢颜之间,生活在他们青石一样的足迹和海浪一样的呼吸之间。
   ⒂这些尘土一样卑微的人们,他们的身影出没在我的视线里,他们的精神沉淀在我的心灵里。他们常常让我感觉到这个平凡的世界其实是多么可爱,这个散淡的世界其实是那么默契,而看起来如草芥一样的生命种子,其实是那么坚韧和美丽。
   ⒃我靠他们的滋养而活,他们却对自己的施予一无所知。他们因不知而越加质朴,我因所知而更觉幸福。
   1.将“①沉默 ②愉快 ③挂怀 ④不安”依次填入文中第⒁小段的空白处,正确的顺序应该是(填入序号即可)
   答:
   2.文章描写了三位普普通通的劳动者,他们具有哪些共同的优秀品质?
   答:
   3.认真品味文中画横线的句子,谈谈你对它们的理解。
   ①听着她絮絮的温语,我就会感到一波波隐隐的暖流在心底盘旋。
   答:
   ②我明白我的存在对他们是无足轻重的,但是他们对我的意义却截然不同。
   答:
   ③他们因不知而越加质朴,我因所知而更觉幸福。
   答:
   4.文章可分为记叙和议论两部分,请用“‖”断开,然后说说两部分之间的关系。
   ⑴⑵⑶⑷⑸⑹⑺⑻⑼⑽⑾⑿⒀⒁⒂⒃
   5.请你结合文章内容,谈谈生命“美”在何处?在生活中怎样才能发现生命之美?
   答:
   (三十三)(2004·宁安)
   猎人与狼
   ⑴猎手穿行在灌木丛中,查看前几天挖下的陷阱,可惜一无所获。当他走进一片灌木林时,微微有些心跳,那是最后一处的陷阱了。
   ⑵伪装的树枝不见,陷阱露出阴森森的洞口,猎手狂喜,端着枪走近阱口。猎物是只成年的公狼,金黄色的皮毛闪烁着金属的光泽,在阱底嗥着,焦躁地团团转。
   ⑶靠近点,再靠近点,猎手告诫自己。小心地挪动脚步,边举枪瞄准,他只有将尖锐的弹头射进公狼的前额,才能获得一张完整的狼皮。猎手觉得手心有点湿,全神贯注地往前挪步。突然,他脚下一滑,眼前一黑,耳边响起尘土和落叶的坠落声,猎手意识到,他正掉进自己挖的陷阱里,他忽略了昨晚的那场大雨。
   ⑷现在,猎手和凶恶的公狼,共同被困在狭小的阱底。四周是三米多高垂直光滑的泥壁。猎手的心中滑过一阵苦笑:这大概就是报应吧,打了一辈子猎,结果却要死在亲手挖掘的陷阱和已捕获的猎物口中。
   ⑸在猎手掉进陷阱中的那一刻,公狼被这突如其来的坠物给镇住了,它转过身,目光凶狠地盯着近在咫尺的猎人,龇着白森森的狼牙,鼻孔里发出威慑的低嗥。
   ⑹赤手空拳的猎手知道,无论如何是敌不过眼前凶狠的公狼的。凭他多年的狩猎的经验,他很快镇定下来,也龇着牙,目光狠狠地瞪着公狼不动。
   ⑺人和狼都不敢轻举妄动,互相对峙着,时间似乎静止了,不知过了几分钟或是几小时,公狼也许感到眼前的人对它并不构成威胁,也许无暇顾及,逃命要紧,公狼盯着猎人后退两步,见猎人没有反应,便掉转身,不再顾及背后的人,开始义无反顾地用尖锐的前爪飞快地扒着阱壁。
   ⑻雨后的土很松,老狼很快就在脚下扒下一大堆土,把阱底垫高了一层。公狼用后腿跳上土墩,竖起前爪,继续往高处抓。猎手很快就明白了公狼的意图,不禁在心里骂道:他娘的,狡猾的家伙!旋即跟在公狼的身后,把公狼抓的松土踩实、垫宽,做成台阶状。
   ⑼狼和人,在这一刻成了同舟共济的合作者。人和狼,为了求生,竟配合得如此默契和协调。
   ⑽只用了两个多时辰,狼和人都看见了头顶上的地面。狼一纵身,蹿上地面,向前走了几步,停住,回转身坐下来,目光定定地望着洞口。猎手最后也艰难地攀援上洞口。他看见狼正盯着自己,慌忙拾起掉在地上的枪,端起瞄准狼的前胸。
   ⑾公狼见到猎人上来,全然不觉危险就在眼前,而是偏着头,对猎人眨眨眼,然后缓缓起身,慢吞吞地朝着密林深处走去。在转过山脊时,又回身望了望猎人。
   ⑿猎手端着枪,怔怔地瞄准着渐渐远去的公狼,僵硬的手指,没有扣下扳机,一任猎物从容地隐入林间……
   ⒀从此,猎手不再进山。他告诉别人,他已失去做为猎手应有的那股狠劲了。
   1.通读全篇,补充情节。
   本文的情节由“跌落陷阱、紧张对峙、 、无言分别”四部分组成。
   2.品味关键词语是深入理解文章意蕴的重要手段。请品味第⑽、⑾段中的几个加点词语,体会狼蹿上地面却没有马上逃去,流露出它对猎人怎样的情感?
   答:
   3.文章结尾处猎人说自己已经失去了“做为猎手应有的那股狠劲”。你怎样理解猎手的“狠劲”?猎人失去了“狠劲”,那又多了些什么呢?
   答:
   4.读书贵在质疑,你能从《猎人与狼》这篇文章中提出一个新问题吗?
   答:
   5.综合探究。
   材料一:狼,犬科,耳竖立,尾下垂,性凶残,往往结群伤害禽畜。
   《汉语言大词典》
   材料二:在许多情况下,狼如果不是被逼得走投无路或者饥饿、生病,它是不会主动攻击人类的。
   《科技文萃》
   材料三:1907年美国开巴高原一片葱绿,美丽的鹿在林间嬉戏,可常常被贪婪的狼暗算。当地人恨透了狼,用了25年的时间,消灭了狼和一切鹿的天敌,开巴高原成了鹿的“自由王国”,鹿大量的繁殖。不久,鹿吃光了所有能吃的绿色植物,整个高原,绿色消退,枯黄蔓延……后来美国很多地区开始了“请回狼先生”的工程。
   《作恶的鹿与冤屈的狼》
   材料四:一个猎人说:“一次,我追捕一只带有两只小崽的母狼,它跑得不快,因为小狼脚力不健。我和狼的距离渐渐的缩短,母狼转头向一座巨大的沙丘爬去。我很吃惊。通常狼在危急时,会在草木茂盛处兜圈子,借复杂地形伺机脱逃。如果爬向沙坡,狼虽然爬得快,但人一旦爬上坡顶,就一览无余,狼就再也跑不掉了。我击毙了母狼,又在一堆干骆驼粪中找到了两只小狼。我才恍然大悟,那只母狼是为了保护它的幼崽,爬坡延迟我的速度,赢得了掩藏儿女的时间,并用全力向相反方向奔跑,以一死换回孩子的生。”
   《母狼的智慧》
   结合《猎人与狼》,就链接的四则材料,以及你所知道的有关“狼”的知识或故事进行探究,多角度阐述你对“狼”的认识。
   答:
  
   (三十四)(2004·北碚)
   美好的明天
   刘墉
   ⑴“让我们等待一个美好的明天吧!”每当我听见别人说这句话的时候,就会想,明天真的那么美好吗?我只知道,不论今天是多么令人留恋,明天总会毫不犹豫地来到,我也知道,无论我们抗议或沉默,生存或死亡,明天总是不停地推走。
   ⑵明天是平凡而无情的,它很快地变为今天、化作昨天、成为明天。
   ⑶明天是未可知的,是一连串的问号,用它弯弯的钩子,钩着我们又向前跨进一日,又长大三百六十五分之一岁,又不知觉地添了些许,减少了些许。
   ⑷明天是辛苦的,要工作、要考试、要出操、要买菜、要战斗,只要有一件事没办好,明天就翻脸不认人。
   ⑸明天是脆弱的,如同人生的幸福一般,可能有病痛、有战争,有亲人永远离我们而去,即使是一片瓦默默地滑落,也可能夺走我们的生命。
   ⑹但是,我们不能因此就说明天不再美好,而只能说明天是太纯了,如同一张白纸,雪白得令人发慌。我们可以将它接过来,再随手递出去,成为一张零分的白卷;也可以在上面乱涂几笔,成为糟糕的作品;但更可以赋予它最优美的色彩,最巧妙的情思,成为一幅传世不朽的杰作啊!
   ⑺所以明天又是操之在我的,是等待我们去开创、去塑造的。对于那些恋人,明天可能是他们的佳期;对于那些辛苦耕耘的人,明天可能是收获的日子;对于那些勇士,明天或许面临战斗,却可能胜利;对于那些反抗暴政的人们,明天或许最艰辛,却可能重获自由。即使有一位伟人不幸在明天逝去,也绝不是明天战胜了他,而是他伟大了明天,使明天成为一个值得永远纪念的日子。
   ⑻不要等明天向我们走来,让我们走向明天吧!只有当我们将“等待一个美好的明天”的“等待”改为“开创”时,才能拥有一个真正属于自己的、美好的明天。
   1.本文作者阐述的观点是什么?请简要概括。
   答:
   2.作者是从哪几个方面来论述“明天并不是那么美好”的?
   答:
   3.在文中摘录一句你最喜欢的句子,写在下面的资料卡片上,并在类别栏注明摘句所属类别。
   读书文摘卡
   类别 篇名《美好的明天》作者 刘墉 来源(此处不用作答)内容摘要:
   4.文中第⑶段说“添了些许”是指添了什么?“减少了些许”又是指减少了什么?
   答:

 第十三讲 学作品阅读

   【考查要点】
   在整体上,包括:内容的整体感知,小说要素的辨识,小说情节的归纳概括,塑造人物的手法品析,人物形象与人物性格的分析,小说与散文线索的理解,小说与散文主旨的理解,能评价作品的思想感情倾向,对作品中感人的情境和形象能说出自己的体验,能表达自己获得的对自然、社会、人生的有益启示,对小说的主旨、人物的形象进行创造性的言之成理的理解,等等。
   在局部或细节上,包括:字词的准确运用,字词的含义,字词的情感色彩,字词的表达作用,句子的含义与表达作用,重点段落的品读,文段作用的理解与分析,描写方式的辨识,环境描写的技法及表达作用的理解,抒情方式的品析,文章的构思技巧与表达技巧的品析,自由赏析,等等。
   【知识疏理】
   1.小说的文体知识。2.散文的文体知识。3.童话、寓言、传说、故事的文体知识。4.小说的表现手法。5.散文的表现手法。6.文学作品中的描写。7.文学作品中的抒情。8.文章结构、顺序、线索方面的知识。9.欣赏课文中优美、精辟的语句。10.初步欣赏文学作品中的形象和描写。11.段落品析。12.文学作品的细节性知识如详写与略写、联想与想像、正面与侧面、对比和烘托、伏笔与照应、悬念与释念、实写与虚写、先抑而后扬等等。13.一定的生活知识与生活见闻。
   【试题特点】
   题目角度丰富,题型丰富。如概括题、简答题、品析题、联想题、解说题、赏析题、自由表达题等等都得到充分的运用。
   自由表达题的设计角度多姿多态。如要求谈启示,说看法,写结尾,补例子,作赏析,述感悟及微型写作等等。
   【解题导引】
   例1.
   绝 手
   印白先生4岁握笔,9岁出字。先给左邻右舍写联画贴,后给达官贵人描扇点匾。14岁晋见巡游的皇帝,御前走笔,龙颜赞叹,当场赐银百两,旨令将其抄书立碑留存。
   从此,印白先生名扬大江南北。但名气大了,人也傲了。每天舞笔练字,很少再给别人,就是显贵人家出资丰厚,也遭婉言拒绝。物稀为贵,惜字如金。印白先生的名气更响。据说有人为求几字,竟愿割田百亩。
   野村少佐求字时,印白先生鬓发已白。野村是城防司令,中国通。抓来印白先生,假意训斥下级,扇了绑印白先生小队长的耳光。然后抛手解绳,递烟递茶,恭请印白先生坐下。歉意地说:“先生的,大大的对不起,请你的海涵。”
   印白先生抚摸着红紫的手臂,鼻孔连出几声“哼”。
   “先生,”野村竖拇指夸奖:“大大的了不起。字的,我特别的欣赏……”
   印白先生斜眼野村,架起二郎腿,端茶呷了一口。
   野村斟茶,满脸堆笑,话入正题:“我的,请先生赐字,不知先生的意向?”
   “不写!”印白先生斩钉截铁地回答。
   “……”野村十分尴尬。
   印白先生起身便走,门岗架刺刀拦住他。野村打个手势,岗哨收回刺刀。野村陪印白先生出门,鞠躬道:“好走。”
   印白先生头也没回。
   野村数次登门,软磨硬蹭求字。印白先生始终不买账,对家人说:“此倭贼杀人如麻,即使杀了我,我也不给他写半个字。”
   家人担心野村真下毒手。
   印白先生淡然一笑,说:“倭贼要笼络人心,可能还不至于动我这种有名气的人吧。”
   野村再请印白先生,话不投机,露出杀相,用东洋刀砍下桌角,威胁说:“你的再不写这了,死了死了的!”
   第二天,押印白先生去,见面几句话,就手刃一俘兵。印白先生颤栗。野村抓住他的右臂,指着东洋刀上的血,狞笔说:“快快写,我的好说。”
   印白先生牙咯咯响,犟过头。
   野村用刀支过他的头,狂叫:“写的不写!”
   印白先生牙缝挤出两字:“不写!”
   “咔嚓”,野村下了毒手。印白先生被人扶回,在床上昏睡三天。乡邻好友守候身旁,无不掉泪。印白先生反而安慰他们:“别伤心,倭贼没断我双手,只要有手,我印白还是印白。”
   四年后,野村进山扫荡,被八路军击毙。日本兵在城里祭祀。天亮,贴告示的地方,
   八个斗字:“血刃倭贼,天下称快”,苍劲雄浑,洒脱耀眼。内行者赞曰:“除印白先生,再无可比。”
   1.请概括第⑴⑵段的内容。(各20字以内)
   第⑴段:
   第⑵段:
   2.野村采取了 、 、 的方式求字幅。
   3.“野村手刃一俘兵。印白先生颤栗”。印白先生是因为 而颤栗?
   4.文中最后一段应填入的词语是( )
   A.写着 B.横着 C.张贴 D.耸立
   5.印白先生最终为野村写字了吗?在什么时候,写的是什么字?
   6.内行者赞曰:“除印白先生,再无人可比。”请概括出其中的原因。
   7.文章对印白先生的描写三言两语,却使人感到栩栩如生。请举两例。
   8.请你概括写出印白先生的形象。
   这是一篇小说,读小说要注意故事情节,然后要了解小说中刻画的人物,因此第1题应用一个主谓句来表达:“谁”(或“什么”)“干了什么”(或“怎么样”)。第⑴段:印白先生自幼练字,14岁时就为皇帝赏识。第⑵段:印白先生名声大噪,惜字如金。第2题,野村假意礼贤下士,惺惺作态,然后是数次登门,软磨硬蹭,最后凶相毕露,开东洋刀相逼。第3题,联系上下文,可以看出印白先生有高风傲骨。他不是害怕野村的威逼,而是因为鬼子的凶残使得他气愤得颤栗。第4题,“写着”显着平淡,只是交代情况;“横着”虽有力度,但有杂乱之感;“张贴”一是显得平淡,二是与前面的“贴”语意重复;“耸立”不但可以看出这八个字的力度风骨,还能使人感悟到印白先生崇高的民族气节。因此,答案是D。第5题,在野村被八路军击毙后,印白先生为他写了八个大字:血刃倭贼,天下称快。第6题,原因有两点:一是印白先生书法名扬大江南北,内行者知道他的字的风格,二是印白先生的气节妇孺尽知,也只有他才敢于写出这样的内容。第7题,文中对印白先生的描写凸现了他的鲜明形象。如:印白先生抚摸红紫的手臂,鼻孔连出几声“哼”。印白先生斜眼野村,架起二郎腿,端茶呷了一口。“不写!”印白先生斩钉截铁地回答。印白先生淡然一笑,说:“倭贼要笼络人心,可能还不至于动我这种有名气的人吧。”印白先生牙咯咯响,犟过头。印白先生从牙缝挤出两字:“不写!”,第8题,印白先生是一个不阿谀逢迎,宁死不屈,有崇高民族气节的书法家。
   例2.
   在小说阅读题答题方面,应主要掌握一些突破难点的方法,如:
   1.联系全文的“大”环境,对人物性格进行正确的理解。如果不联系全文主题,单就句子内容来分析人物的思想性格,可能会出不少的问题。2.着眼于文中的情节,在具体的语境中对人物的言行进行分析。3.根据文中具体的语境,来品味文中的语言。4.调动学习积累,应对试题中出现的疑难。
   在散文阅读题答题方面,应主要掌握一些品味细节的方法,如:
   第一步,整体式阅读。即从整体的角度对材料反复扫读、领会、理解,这一步的目的是为了熟悉材料。
   第二步,细节式品味。即对材料从各个不同的角度进行分析,将阅读的全过程细节化,同时,又将每一细节化的阅读与“全文”这个整体联系起来进行品味。
   有了这样的阅读,就为阅读答题打下了基础。
   【常见失误】
   文学作品常见的答题误区有:不能判定选文在构思上的独特之处,不知道如何找到选文的线索,概括不出对人或物的描写、表现的角度,对人物的性格特点或事物的特点概括不全面,不能准确地对文句进行品析,阐释不清句子的含义,看不出选文在细节上的表现手法,阐释不了词、句段的表达作用和表达效果,难以讲清楚答题的理由,概括文章的主旨有困难,等等。
   【考点精练】
   (一)(2004·连云港)
   高等教育
   司玉笙
   (1)强高考落榜后就随本家哥去沿海的一个港口城市打工。
   (2)那城市很美,强的眼睛就不够用了。本家哥说,不赖吧?强说,不赖。本家哥说,不赖是不赖,可总归不是自个儿的家,人家瞧不起咱。强说,自个儿瞧得起自个儿就行。
   (3)强和本家哥在码头的一个仓库给人家缝补篷布。强很能干,做的活儿精细,看到丢弃的线头碎布也拾起来,留作备用。
   (4)那夜暴风雨骤起,强从床上爬起来,冲到雨帘中。本家哥劝不住他,骂他是个傻蛋。
   (5)在露天仓垛里,强查看了一垛又一垛,加固被掀起的篷布。待老板驾车过来,他已成了个水人儿。老板见所储物资丝毫未损,当场要给他加薪,他就说不啦,我只是看看我修补的篷布牢不牢。老板见他如此诚实,就想把另一个公司交给他,让他当经理。强说,我不行,让文化高的人干吧。老板说,我看你行——比文化高的是人身上的那种东西!
   (6)强就当了经理。
   (7)公司需要招聘几个大专以上文化程度的年轻人当业务员,就在报纸上做了广告。本家哥闻讯跑来,说给我弄个美差干干。强说,你不行。本家哥说,看大门也不行吗?强说,不行,你不会把这里当成自个儿的家。本家哥脸涨得紫红,骂道,你真没良心。强说,把自个儿的事干好才算有良心。
   (8)公司进了几个有文凭的年轻人,业务红红火火地开展起来。过了些日子,那几个受过高等教育的年轻人知道了强的底细,心里就起毛说,凭我们的学历,怎能窝在他手下?强知道了并不恼,说,我们既然在一起共事,就把事办好吧,这个经理的帽儿谁都可以戴,可有价值的并不在这顶帽上……
   (9)那几个大学生面面相觑,就不吭声了。
   (10)一外商听说这个公司很有发展前途,想洽谈一项合作项目。强的助手说,这可是条大鱼呀,咱得好好接待。强说,对头。
   (11)外商来了,是位外籍华人,还带着翻译、秘书一行。
   (12)强用英语问,先生会汉语吗?
   (13)那外商一愣,说,会的。强就说我们用母语谈好吗?
   (14)外商就道了一声“OK”。谈完了,强说,我们共进晚餐怎么样?外商迟疑地点了点头。
   (15)晚餐很简单,但有特色。所有的盘子都尽了,只剩下两个小笼包子,强对服务小姐说,请把这两个包子装进食品袋里,我带走。
   (16)虽说这话很自然,他的助手却紧张起来,不住地看那外商。那外商站起来,抓住强的手紧握着,说,OK,明天我们就签合同!
   (17)事成之后,老板设宴款待外商,强和他的助手都去了。
   (18)席间,外商轻声问强,你受过什么教育?为什么能做得这么好?
   (19)强说,我家很穷,父母不识字,可他们对我的教育是从一粒米、一根线开始的。后来我父亲去世,母亲辛辛苦苦地供我上学。她说俺不指望你高人一等,你能做好你自个儿的事就中……
   (选自《微型小说选刊》,有改动)
   1.第(5)节中,老板让强管理一个公司,他认为“比文化高的是人身上的那种东西”。请写出“那种东西”所指的具体内容。
   答:
   2.第(7)节中,本家哥所说的“良心”与强所说的“良心”意思有所不同,请分别写出两人所说的“良心”各自的具体含义。
   答:①本家哥所说的“良心”:
   ②强所说的“良心”:
   3.请举例说明小说主人公强身上体现出了哪些性格特征。(至少答出3点)
   答:①
   ②
   ③
   4.根据小说内容,具体说说小说标题“高等教育”的内涵。
   答:
   5.在你的成长过程中,对你影响最大的教育是什么?你如何看待这种教育?(不超过100字)
   答:
  
  
  
  
  
  
  
  
  
  
  
   (二)(2004·大连)
   丹麦首都哥本哈根。凌晨2时15分。
   当班的见习消防队员拉斯马森拿起突然铃声大作的电话,一阵微弱的喘息声传来:“我摔倒了,救命啊……”声音像一位老夫人,在确定不是恶作剧后,拉斯马森询问道:“请把您公寓的地址告诉我!”
   “我……想不起来了。”
   “那请把您的名字告诉我!”
   “我记不得了。”
   “请您不要把电话挂断,我们好通过电信局找到您的住处!”拉斯马森想到了电信局,他马上与电信局取得了联系。可值班人员却表示,现在人员不齐,要查清电话是从哪里打来的,根本做不到……无计可施的拉斯马森叫醒了中尉,中尉拿起了电话:“夫人!……夫人!能告诉我床前地毯旁边是什么吗?”“是镶木地板,老式的打蜡地板。”“天花板高吗?”“高……我觉得很高……”“这么说,您住的是老式房子!百叶窗关着没有?”“没关。”中尉兴奋地对拉斯马森说道:“寻找一幢窗户狭长的老式房子,房子的窗口有灯光,大约在二三层……”中尉又对着话筒问道:“夫人,能告诉我,您住的街道的名称吗?”
   然而,话洞里再没有人搭腔了。中尉知道,老太太大概昏过去了,没有把电话挂上。
   时间在一分一秒地流逝着,每一秒钟每一分钟都变得漫长起来,室内挂钟的滴答声越来越响,每一次的滴答声都像小锤子般敲在几个人的心上。
   一个小时过去了,老夫人仍无反应。焦灼和冷汗悄悄地爬上了几个人的面孔、心头……拉斯马森对中尉说道:“我有个主意……”他的主意立即被采纳。
   清晨3时30分,14辆轻便消防车同时出动,开往依然沉睡着的城市的各个街区,警笛的尖啸声撕碎了夜的宁静。指挥部里拉斯马森把电话听筒紧紧地贴在一只耳朵上,把耳机扣在另一只耳朵上。整个哥本哈根城都被惊动了,家家户户灯火通明……拉斯马森突然叫道:“中尉,我听见了!我听见警笛声了!消防车大概就在离老夫人不远的街上!”中尉用手中的对讲机命令道:“1号车,停止鸣笛!”拉斯马森对中尉说道:“我还听得见。”“2号车,停止鸣笛!3号车,停止鸣笛!……6号车,停止鸣笛!……12号车,停止鸣笛!”拉斯马森惊叫道:“就是这里!”
   “12号车,我是指挥部,就在你们那个地段,用扩音器说明一下理由,让这条街上所有的灯光都熄灭,最后亮着灯的一定是老夫人的家!”
   5分钟后,12号车所在街区的窗口都黑了下来,只剩下一扇窗口还亮着。稍后,拉斯马森在电话里听到了撞破房门的声响,继而是一位消防队员的声音:“喂!指挥部吗?我们已到现场,老夫人昏迷了,颅骨有伤,大概是床头磕的,脉搏有轻微跳动……”
   太阳升起来的时候,消防队不断有电话打进来,询问老夫人的情况。当获悉这位72岁的老人因为救助及时,已经在医院苏醒过来时,人们纷纷表示,那个凌晨,响彻哥本哈根的消防车的尖啸声,是他们听过的所有声音中最动听的声音。
   1.本文所写的故事发生于凌晨2时15分的哥本哈根,起因是□□□□□□□□□□□□□□□□□□□□,经过是□□□□□□□□□□□□□□□□,结果是老夫人顺利获救。(每处不得超过16字)
   2.请为本文拟一个恰当的题目。 (8个字以内)
   3.从拉斯马森、中尉与老夫人的通话中,可以看出消防队员 的品质。
   4.“指挥部里拉斯马森把电话听筒紧紧地贴在一只耳朵上”一句与上文的哪句话呼应?
   答:
   5.文中画线句子为什么不直接引用中尉下达的“12号车,停止鸣笛”的命令,而是不厌其烦地从1号车说起?
   答:
   6.下面两句话,写得很好。任选一句,说说好在哪里。[如两题都答,按第①题计分]
   ①每一次的滴答声都像小锤子般敲在几个人的心人。
   答:
   ②警笛的尖啸声撕碎了夜的宁静。
   答:
   7.对于本文叙述的故事,你有什么感受?请写在下面。(含标点60字以内)
   答:
  
   (三)(2004·盐城)
   爬上窗台的绿色
   窗台在二楼,那晌午里散着泥土糙香、夜晚嚼着寒露沁凉的绿色就顺着我家的门铃线义无返顾地爬了上来。不知不觉间,一线的绿色和点缀其间的串串白花便搭在了小院上空,并在窗台前探头探脑,使劲摇曳着深秋的阳光,映照出一户庭院的温馨与恬静。
   望着这线积极蓬勃的生命,真让人心情爽快并急于俯向贴面的绿色。也许,这正是绿色爬上窗台的唯一心愿吧。
   确切地说,这束绿色来自一株黄瓜和一株莓豆。早在夏天它们就兄弟样并肩挽手攀援于小院南侧,在这个无风无雨的夏季艰辛地生长着,暑气和干旱丝毫没有抵消它们坚强向上的意志,它们的信念,永远是忠诚于脚下这片临时堆砌的菜园和我那朝夕呵护的妻子。除了寒冬,已经没有什么能让这群绿色在小院中消失,尤其在秋天,远离了燥夏的绿色反而越加显现出遮掩不住的郁葱,在院落、窗台无声地抖动着朗朗秋波。
   这时爬上窗台的绿色犹如举向天空的思想和灵感,带着大地和秋天独有的体温,干脆利落地扑向你,让人觉得这绿色从一开始就是为了在这里等着你,让你怦然心动,让你热血奔涌。
   仰望天空,仰望这线执著的绿色,总令我心旷神怡,思绪万端,无以言说的沟通感拓展着我也分解着我。它与我们一样,也向往自由,追逐光明,真心热爱脚下的一方水土和苦乐生活呀。有谁能说自然是冷漠的、无情的?江水奔流不息,倾诉的是自己澎湃的波涛;树木傲雪参天,挺拔的是自己无边的苍翠; , 。而我一直相信在宇宙规律的作用下,世间万物都有生命有情感有思维,你无法评说谁更伟大谁更渺小谁更补益于对方。就像今晚,秋空如洗,夜凉如水,银河横波,一盘银月垂至天幕中央,说不出是欲升欲沉还是欲远欲近,最主要的是我无法忽略窗台前的这线绿色,它笼在迷人的夜色里,梯次排开的豆荚和瓜纽如同披上了一身轻纱,轮廓分明地在秋风下回忆曾经汗流浃背的夏天的火热,算计着丰硕的收成,说不定一声秋虫的低吟浅唱就成全了豆荚们瓜纽们酝酿了整夏的爱情,刚刚坠入甜蜜的小生命于是急不可待的地探出绿色的小手敲打窗子,要把一肚子的幸福与快乐说给你听。
   若是在晌午呢,前提的阳光并不火辣,只是响脆脆地跌在这线绿色上,那些叶呀花呀瓜呀豆呀的心情和爱情虽被一夜寒露湿透,却能在顷刻间还你会飞的欢笑与娇媚。就在不远处,建筑工地上不知歇息的轰鸣,也因了这绿色的宁静抚慰而富有节奏感。绿色,就这样以生的迫切、力量和宁静,同时给了我们视觉和听觉上的冲动愉悦。
   终于爬上窗台了,为实现这个简洁纯真的心愿,黄瓜和莓豆共同托举着绿色,克服了来自韭菜、芹菜、油菜等园内其它蔬菜的嫉妒嘲讽,选择惟一向上的道路来到我的窗前,让绿阴、生机、果实、清新悄然挤进房间的每一个角落,它相信有爱的地方就应该有绿色,有家的地方就该布满生机。这时候,绿色的内心便会感到不虚此行,并以芬芳的花蕾和沉实的种子重新构思着盎然的激情。
   (选自2003年第11期《中华散文》,有改动)
   1.根据文意的理解,第2段和第7段划线的两句中“惟一心愿”和“简洁纯真的心愿”分别什么?
   (1)“惟一心愿”
   (2)“简洁纯真的心愿”
   2.细读第5段,按要求回答下列问题。
   请结合对文意的理解,仿造横线前面句子的形式,再写一个句子。
   江水奔流不息,倾诉的是自己澎湃的波涛;树木傲雪参天,挺拔的是自己无边的苍翠;
   , 。
   作者写“江水”、“树木”等是为了说明什么?请用文中相关的语句回答。
   答: 。
   3.文章赞美的绿色来自黄瓜和莓豆,在最后一段有写到“韭菜、芹菜、油菜”,这样写有什么作用?
   答: 。
   4.绿色给人以愉悦,绿色给人以想象,绿色给人以启迪。请联系本文回答问题。
   ⑴从全文看,“爬上窗台的绿色”中”的“绿色”具有哪些方面的精神品质?
   ⑵“绿色”一词在生活中还被赋予许多新的内涵,如“绿色通道”中的“绿色”主要指安全、畅通而不受任何阻碍。请你再举一例说说它的另一种内涵。
   答: 。
   5.下列对这篇散文的分赏析,不正确的一项是[ ]
   A.文章先写爬上窗台的“绿色”给人以惊喜,然后交代“绿色”的由来,随后以叙述、描写、抒情、议论相结合的手法来热情礼赞“绿色”。
   B.本文以爬上窗台的瓜藤豆蔓为素材,讴歌了蓬勃向上的绿色生命,揭示了人与自然的和谐关系,意蕴深长。
   C.作者在文末说,“绿色”要“以芬芳的花蕾和沉实的种子重新构思着盎然的激情”。这样的结尾升华了文章的主题。
   D.文章先采用了倒叙的手法,然后转入顺序,以“我”的心情变化为线索来结构全文,显得灵活而严谨。
   6.这篇散文的语言生动,描写细腻,多处运用拟人、比喻来丰富“绿色”的内涵,来揭示“绿色”的精神。请选择你最喜欢的语句(上面题目涉及的语句除外),作一点具体分析。
   答:
   (四)(2004·宿迁)
   家是地球的中心
   谭延桐
   “回家多好啊!”
   这是美国哲学家威廉·詹姆斯临终时说的最后一句话。1907年,威廉辞去了哈佛大学的教职,开始了他计划中的欧洲之旅。所到之处,都会有“欢迎伟大的威廉·詹姆斯教授”的欢呼声暴雨似的淋到他智慧的头上。可是,再了不起的荣誉也替代不了家,替代不了,他太想家了。当他三年后终于乘上回美国的轮船时,他已是疲惫不堪了。就在轮船快要接近终点时,他一下子瘫倒在了坐椅中,瘫倒在了一个越来越明亮的梦想中,从此就再也没有起来。他终于算是把自己给领到接近家的地方了。家,终于定格在他最后的时光里了。
   家,这个不是天堂却胜似天堂的地方,谁又不魂牵梦绕呢?
   我似乎又一次听到那位阔人的哭泣了。他在野外徘来徊去,泪水涟涟,风怎么也抹不去他脸上的斑斑泪痕……有一位打那里走过的人问他:你这位先生啊,你怎么就不知道回家呢?他感慨万端地说:家?我有家吗?我的家在哪里呢?我虽然有漂亮的房子,可房子是家吗?……那位路人怔住了。
   同样,我也怔住了。是啊,荣誉再多,财富再多,可又有什么用呢?当家是“1”的时候,荣誉越多,财富越多,后面的零无疑也就越多了;可是,当家是“0”的时候呢?……真是无法想象,一个人一旦没有了家,他的幸福又将存放在哪里。没有家的灵魂,是孤独的,凄凉的,甚至是悲哀的,无望的……难道不是吗?
   我曾不止一次地目睹过春节期间火车站和汽车站的热闹场面:长长的从售票大厅一直蜿蜒到车站广场的一眼望不到头的购票队伍……他们等啊等啊,等了一整天甚至几天终于算是轮到他们了……票终于拿到手了!似乎一个家终于攥在了他们的手里一样!他们笑了……他们怀着无比迫切的心情,急匆匆踏上了回家的路──车上再挤,也挤不掉他们结实的渴望。哪怕是单腿站着……对,只要车上能容得下一只脚,就能容下他们对于家的拥抱……在外打工,钱是赚了一些,可钱再多,也不能作为足够的邮费,把整个的家从远方邮到自己身边来的啊!钱终究买不来家所给予的一切啊!回到家里去!只有一种声音在他们的心中反复地播放着:回到家里去!回到家里去!回到家里去!……我理解了他们!
   我理解了……家。啊,家,究竟是什么呢?
   1995年,台湾举办了一项“公车欣赏,快乐上路”活动,即把一些精短诗作制成海报,张贴在公共汽车车厢内,让诗歌伴随着人们或长或短的旅途,其中就有这么一首题为“家”的小诗──
   我的家就像铅笔盒。
   爸爸是铅笔,出外帮人写字赚钱。
   妈妈是橡皮擦,功课不对就帮我们擦掉。
   爷爷是2B铅笔,画个小鸭给我看。
   奶奶每天买菜花好多钱,就像自动铅笔每天都要吃笔芯。
   我呢?是铅笔爸爸、橡皮妈妈的小铅笔……
   这是一位11岁的叫做苏幸的女孩子写的。在这位女孩的眼中,家就像一个铅笔盒。哦,铅笔盒,里面装着她的童话,她的一切。多美好的事情啊。如果拿走了这样一个“铅笔盒”,也便拿走了她心中的诗了。由她心中的诗外化成的这首小诗,我们也就再也读不到了……
   在你的眼中,他的眼中,家又是什么呢?
   也许,家是你最大的伤疤;也许,家是你最重的行囊;也许,家是你梦想中的梦想,牵挂中的牵挂;也许……你又会以怎样的态度对待它呢?
   (选自《散文百家》2003年)
   1.这篇散文开头引用美国哲学家詹姆斯临终时说的“回家多好啊”这句话,这样写有什么作用?
  
  
   2.请体会文中画线句子的含义。
   ①票终于拿到手了!似乎一个家终于攥在了他们的手里一样!
  
  
   ②也许,家是你最大的伤疤;也许,家是你最重的行囊。
  
  
   3.结合本文内容简析小女孩苏幸的《家》这首小诗妙在何处。
  
  
   4.请你仔细阅读全文,概括出作者对“家”的认识。
   ①
   ②                                               
   ③                                                
   5.下面是对这篇文章的理解和赏析,错误的两项是 【 】 【 】
   A.本文举詹姆斯的例子,意在说明思念家乡热爱祖国的思想感情,不仅中国人有其他国家其他民族的人也有。
   B.正是这充满温情的令不同职业、不同年龄的人都魂牵梦绕的一个个家,才构成社会这个整体,才形成这个“地球村”。
   C.本文用例似乎是信手拈来,但实际上每个例子都经过了严格筛选,从不同角度阐明主旨。
   D.本文语言生动活泼,运用反复、夸张、象征等多种修辞手法,读来耐人寻味。
   E.这是一篇以“家”为话题的散文,所选事例范围广泛,恰到好处的运用了过渡语段,结构上显得自然严谨。
  
   (五)(2004·杭州)
   洁白的木槿花
   一到夏天,院子里的木槿就开花。母亲一见洁白的木槿花,就会眼含泪水,嘴唇哆嗦着说:“我那时好糊涂,好糊涂啊。”这里面有个故事。
   我五岁那年,右腿忽然青肿了一块,痛得我成天哭喊。母亲见我熬不过,就背着我去廖医生家。廖医生是个老中医,退休后在家里开了个小诊所。他医术好,心也善。
   廖医生看了我的腿,责怪我母亲:“再晚来两天,右腿就废了。这是骨髓炎,严重了,就得截肢!”母亲不敢出声,眼眶却红了。其实不能怪母亲,父亲去世后,母亲终日为一日三餐发愁,哪有闲钱给我治腿?母亲担心地问能否治好,得到廖医生的肯定回答后,脸色晴朗了许多,可片刻后,又一脸的阴云。
   廖医生给我敷了草药,又交给母亲两帖中药,说两天后再来换药。母亲颤声问要付多少钱,廖医生说五块钱,母亲的脸红了,局促不安地说:“我,我只有两块多钱,能不能下回补上?”廖医生爽快地说:“行,不急,啥时有钱啥时给。”
   母亲借不到钱,但我的腿不能不治。母亲只好揣着八个鸡蛋去廖医生家。那时鸡蛋很便宜,几分钱一个。廖医生不肯收,母亲鼻子一酸,眼泪就掉下来了。廖医生的眼睛也湿了,安慰母亲:“好,我收下。钱的事,不要说起。”看了几次腿,母亲已欠廖医生二十八块钱了,这在当时不是一个小数目。
   再去廖医生家,母亲带上了家里惟一的母鸡。廖医生死活不肯收,母亲哽咽着再三哀求。廖医生拗不过,说:“好,就算我买你的,十块钱,好吗?”说着,硬塞给母亲十块钱。那时一只鸡最多卖四五块钱。母亲说啥也不肯接钱。回到家里,母亲发现中药纸包里有十块钱,当晚就去了廖医生家,把钱从门缝里塞进去了。
   两天后,又该去换药,可母亲不敢去,只是默默地流泪。不料廖医生找上门来了。谈话间,廖医生惊喜地发现了院子里的木槿花,说:“木槿花是上好的中药,一两值两块钱呢!摘了卖给我,好吗?”母亲惊喜万分,连忙摘木槿花。廖医生拿秤一称,竟有一斤。他说:“这下好了,你再也不用欠我钱了。”此后,去廖医生家治腿,母亲就带点木槿花去。母亲不用欠廖医生钱,廖医生反而每次付给母亲一点钱。不久,我的腿治好了。
   第二年,廖医生竟去世了。母亲十分伤心,去廖医生家吊唁。母亲从他家属口里知道了一个天大的秘密:木槿花是不能做中药的。母亲哇的一声嚎哭起来:“我的恩人呀……”
   几年后,母亲也去世了。临终拉着我的手说:“儿啊,我死后,在我的坟墓周围栽一圈木槿。”如今,每到夏天,母亲坟墓周围就开满木槿花——圣洁无比的木槿花。
   1.第1段中说,母亲一见木槿花,就会含着泪嘴唇哆嗦着责备自己:“我那时好糊涂,好糊涂啊。”母亲为什么要反复责备自己“好糊涂”?根据文意,作简要回答。
   答:
   2.以下是有关母亲的描写,根据文意,分别指出其表达作用。
   ①脸色晴朗了许多,可片刻后,又一脸的阴云。
   ②可母亲不敢去,只是默默地流泪。
   答:
   3.文章结尾说,母亲坟墓周围开满了“圣洁无比的木槿花”。这“圣洁无比的木槿花”有什么象征意义?请作简要回答。
   答:
   4.母亲和廖医生是文中的主要人物,请你对其中的一个人物作出评析。注意从文中选择典型材料,进行分析和归纳。不超过80个字。
   答:
  
   (六)(2004·安徽)
   地震中的父与子
   1989年发生在美国洛杉矶一带的大地震,在不到4分钟的时间里,使30万人受到伤害。
   在混乱和废墟中,一个年轻的父亲安顿好受伤的妻子,便冲向他7岁儿子上学的学校。他眼前,那个昔日充满孩子们欢声笑语的漂亮的二层教学楼,已变成一片废墟。
   他顿时感到眼前一片漆黑,大喊:“阿曼达,我的儿子!”跪在地上大哭一阵之后,他猛地想起自己常对儿子就的一句话:“不论发生什么,我总会跟你在一起!”他坚定的站起身,向那片废墟走去。
   他知道儿子的教室在楼的一层左后角处,他疾步走向那里开始动手。
   在清理挖掘时,不断有孩子父母急匆匆地赶来,看到这片废墟,他们痛哭并大喊:“我的儿子!”“我的女儿!”哭喊过后,他们绝望地离开了。有些人上来拦住这位父亲说:“太晚了,他们已经死了。”这位父亲双眼直直地看着这些好心人,问道:“谁愿意帮助我?”没人给他肯定的回答,他便埋着头挖着挖着。
   救火队长挡住他:“太危险了,随时可能起火爆炸,请你离开。”
   这位父亲问:“你是不是来帮助我?”
   警察走过来:“你很难过,难以控制自己,可这样不但不利于你自己,对他人也有危险,马上回家去吧。”
   “你是不是来帮助我?”
   人们都摇头叹息地走开了,都认为这位父亲因失去孩子而精神失常了。
   这位父亲心中只有一个念头:“儿子在等着我。”
   他挖了8小时、12小时、24小时、36小时。他满脸灰尘、双眼布满血丝,浑身上下破烂不堪,到处是血迹。在第38小时,他忽然听见底下传出孩子的声音:“爸爸,是你吗?”
   是儿子的声音!父亲大喊:“阿曼达,我的儿子!”
   “爸爸,真的是你吗?”
   “是我,是爸爸!我的儿子!”
   “我告诉同学们不要害怕,说只要我爸爸活着就一定来救我,也就能救出大家,因为你说过不论发生什么,你总会跟我在一起!”
   “你现在怎么样?有几个孩子活着?”
   “我们这里有14个同学,都活着,我们都在教室的墙角,房顶塌下来架了个大三角形,我们没被砸着。”
   父亲大声向四周呼喊:“这里有14个孩子,都活着!快来人!”
   过路的几个人赶紧上前来帮忙。
   50分钟后,一个安全的小出口开辟出来了。
   父亲声音颤抖地说:“出来吧,阿曼达。”
   “不!爸爸,先让别的同学出去吧!我知道你会跟我在一起的,我不怕。不论发生什么,我知道你总会跟我在一起。”
   这一对了不起的父子在经历了巨大灾难的磨难后,无比幸福地紧紧拥抱在一起。
   1.父亲赶到学校时,“顿时感到眼前一片漆黑”的原因是什么?
   答:
   2.父亲请求帮助时,为什么人们都“摇头叹息”,没人帮他挖?
   答:
   3.填空。
   在地震灾难中,父子俩都想起了“不论发生什么,我(你)总会跟你(我)在一起”,这句话除了体现父子间的深挚的爱,还体现了父亲对儿子的__,儿子对父亲的__。
   4.文中画线句子对描写父亲起什么作用?
   答:
   5.文中除了表现父子之爱,还表现了那些人之间的关爱?由此,你对人世间的爱有什么深切的认识(用一两句话概括)?
   答:
   (七)(2004·福州)
   榕树,生命进行曲(节选)
   刘再复
   我爱恋的榕树,不知道使多少陌生人为它兴叹过,倾倒过。
   真是太壮阔了。只要你接近它,就会感到它的全身,都充满着一种最动人的东西,这就是生命。
   善于思辩的哲学家说,美就是充满生命的人和物。我相信,因为榕树,我才相信。
   几乎是整个童年时代与少年时代,我都在观赏这种洋溢着生命的大树。
   我喜欢这种绿色世界在无风中的平静、雍容、丰盛、满足,像沉默的大山一样岿然而立。
   我更喜欢它在风中的时刻。榕树的每一片绿叶,都像风帆那样善于捕捉最弱的微风。因此,当轻风吹拂的时候,它的叶子就会颤动起来,刹那间,树上好像千百万绿色的蝴蝶,在一开一翕地扇着翅膀,共同编织着生命的织绵。
   更使我陶醉的是雄风吹动的时候。此时的榕树,瞬息间从沉默的大山变成汹涌的大海,波浪在树梢上澎湃着,时时发出拍打蓝天的沙沙响声。
   有一位很重感情的北方朋友告诉我,他第一次见到南国土地上的高大榕树时,几乎吓呆了。榕树那企图笼罩大地的浓阴,那企图吞没白云的树冠,那企图饮尽地下全部水分的根群,那陡立而又弯曲多节的巨枝所构筑的殿廊、山脉、峡谷和道路,一起在放射着生命的光波与音波。这种柔和而强大的波浪,把他的心灵摇撼得很久很久。
   在撼动中,他感到自己的生命被另一种强大的生命所照明,所溶解,所征服。觉得自己完全被这种强大的生命所俘虏,并且被剥夺了身上的渺小、卑琐、颓唐与消沉。在树下,澄清的空气中,他觉得自己的灵魂升腾起来了,仿佛也变成了一只扇动着翅膀的绿蝶,也在这个充满生命的葱茏世界中快乐地翔舞。
   我比这位北国的友人更了解榕树,生命里积淀着更多的榕树的碧叶。
   小时候我迷恋过一棵倔强的小榕树。它就在几乎没有泥土的地方发展它的生命。它那生的征程,就在我家屋后的一块浑圆形的岩石上进行。大约三年时光,我一直追随着它的足迹,注视着它那平衡而坚实的脚步。
   我不知道它是在岩缝的哪一处破芽而出,只看着它从缝穴里长出来的最初的嫩枝。这株嫩枝在岩石的悬崖上,沉着地、缓慢地跋涉,攀登,开拓着本没有路的路,本没有前方的前方。当它发现岩石身上的小坑洼处,有一点薄薄的尘土,就果断地在那里扎下了根,扎下一个营寨,然后又向前伸延,迈进,不倦地继续寻找着前方险峻的路,险峻的希望。
   更使我惊讶的是,它在找不到任何营寨的时候,竟从生命深处撒出一束根须,像蚕儿抛出的银丝。柔韧的丝朝下生长,直至亲吻到地面上的小草。后来,我才知道,这就是所谓气根。在没有泥土的时候,气根凭借它奋发的天性,吸收空气中的水分,然后把自己养育成榕树另一翼的生命线。
   突破、挣扎、发展、挺进,这是一支青绿色的生命进行曲,这是一支铁流似的生命凯旋曲。
   正是这支无声、无畏的歌,把巍峨的韧性,第一次灌进了我的贫穷而干旱的童年,灌进了我的还在襁褓中的人生。
   1.从上文看,作者对故乡榕树的情感有:爱恋、 、 、 、 、赞叹。(用文中词语填空)
   2.“(我)生命里积淀着更多的榕树的碧叶”一句中“碧叶”的意思是
   。
   3.联系上下文,谈谈你对文中画线句子的理解。
   答:
   4.作者说“我比这位北国的友人更了解榕树”,作者还了解到榕树的哪些特点?
   答:
   5.本文是一篇语言优美的散文。请你以读书笔记的形式,摘录两处富有表现力的语句,并作简要的点评。(友情提醒:点评,可谈语句的表达效果,可说自己喜欢的理由。)
   摘录 1.2.
   点评
  
   (八)(2004·龙岩)
   改变一生的闪念
   那是一个老师告诉我的故事,至今仍珍藏在心里,让自己明白在人世间,其实不应该放过每一个能够帮助别人的机会。
   多年前的一天,这位老师正在家里睡午觉,突然,电话铃响了,她接过来一听,里面却传来一个陌生粗暴的声音:“你家的小孩偷书,现在被我们抓住了,你快来啊!”在话筒里传来一个小女孩的哭闹声和旁边人的呵斥声。她回头看着正在看电视的惟一的女儿,心中立即就明白过来是怎么回事了。
   她当然可以放下电话不理,甚至也可以斥责对方,因为这件事和她没任何关系。
   但自己是老师,说不定她就是自己的学生呢?通过电话,她隐约可以设想出,那个一念之差的小女孩,一定非常惊慌害怕,正面临着尴尬的境地。犹豫了片刻之后,她问清了书店的地址匆匆忙忙赶了过去。正如她预料的那样,在书店里站着一位满脸泪痕的小女孩,而旁边的大人们,正恶狠狠地大声斥责着。她一下子冲上去,将那个可怜的小女孩搂在怀里,转身对旁边的售货员说:“有什么事就跟我说吧,我是她妈妈,不要吓着孩子。”在售货员不情愿的嘀咕声中,她交清了28元的罚款,才领着这个小女孩走出了书店,并看清了那张被泪水和恐惧弄得一塌糊涂的脸。
   她笑了起来,将小女孩领到家中,好好清理了一下,什么都没有问,就让小女孩离开了,临走时,她还特意叮嘱道,如果你要看书,就到阿姨这里,阿姨有好多书呢。
   惊魂未定的小女孩,深深地看了她一眼,便飞一般地跑掉了,从此便再也没有出现。
   时间如流水匆匆而过,不知不觉间,多少年的光阴一晃而过,她早已忘了这件事,依旧住在这里,过着平稳安详的生活。
   有一天中午,门外响起了一阵敲门声。当她打开房门后,看到了一位年轻漂亮的陌生女孩,露着满脸的笑容,手里还拎着一大堆礼物。“你找谁?”她疑惑地问着,但女孩却激动地一句话也说不出来。好不容易,她才从那陌生的女孩的叙述中,恍然明白,原来她就是当年的那个偷书的小女孩,刚从某名牌大学毕业,已找了份令人羡慕的工作,现在特意来看望自己。女孩眼睛泛着泪光,轻声说道:“当年情急之下的那个电话,幸亏打到您的家里。虽然我至今都不明白,你为什么愿意充当我的妈妈,解脱了我,但这么多年来,我一直想了一桩心愿:喊您一声‘妈妈’。” 话音刚落,女孩已泪流满面。老师的眼睛也开始模糊起来,她有些好奇地问道:“如果我不帮你,会发生怎样的结果呢?”女孩的脸上立即变得忧郁起来,轻轻摇着头说:“我说不清楚,也许就会去做傻事,甚至去死。”
   老师的心猛地一颤。
   望着女孩脸上幸福的笑容,她也笑了。
   1.“心中立刻明白过来是怎么回事了”中的“怎么回事”是指什么事情?
   答:
   2.小女孩临走时,老师“特意叮嘱”她再来看书的用意是什么?
   答:
   3.“喊您一声‘妈妈”’,句中“妈妈”一词表达了女孩怎样的情感?
   答:
   4.“望着女孩脸上幸福的笑容,她也笑了。”句中教师的“笑”(加点)有何含义?
   答:
   5.标题“改变一生的闪念”中,“闪念”指的是谁的“闪念”?文中“老师”的行为又告诉我们一个做人的什么道理?
   答:
   6.当“教师”把小女孩带回家后,她“什么都没有问,就让小女孩离开了”,如果当时“教师”对其批评教育后再让她走,其教育效果有无更好?请申述理由。
   答:

(九)(2004·恩施)
   写荷
   朱群英
   ⑴因一湾碧水,我便开始写荷。
   ⑵在水之湄,荷婷婷如盖。一叶碧绿平静地展开,从水面缓缓地升起,水在荷叶的上面,亦在荷叶的下面,荷涌动,水便涌动。自池边向河心里漫去。
   ⑶于是想起伞,对天撑着,一束束的阳光收集在伞下,没有风,没有雨,荷便显得格外地幽远、明丽。目光踏着一片荷叶,一寸寸地远去,小小荷塘,竟无边的开阔。
   ⑷荷是水中的君子,出污泥而不染纤尘。画荷是一种意境;唱荷也是一种意境;写荷仍是一种意境。平静的地面,荷叶慢慢升起像一只张开翅膀的水鸟,又像是一只小小的船,横于水面。阳光只是一片金黄色的落叶,与荷轻轻的摩擦,继而又被一阵风吹走,消失在茫茫的黑夜之中。
   ⑸雨中看荷,像是把自己平放在水上,细雨轻吻着嫩绿的荷叶,一种迷人的音乐把你融入荷叶之内,你摇动着,荷亦摇动着,便会有一种韵味,轻轻地咬你,细口细口地啄你。雨水积在荷心,又一叶一叶地倒入水中,宁静的荷捕捉着一种清新的动感,一滴晶莹的水珠,悬在荷心,挂在你的眼角。雨过以后,一弯彩虹横于荷塘,那聚在荷心的水滴,亮闪闪地滚动着,颤抖着,像是从荷叶上亮起的一盏小小油灯。
   ⑹夏日的荷变得无边的茂密,整个荷塘像一块绿色的陆地,一层层地远去,把一个一个荷池连成一个整体,间或有一只红红的荷花,倏地跃出水面,大摇大摆地把花瓣放开,露出黄黄的花蕊,含苞欲放的荷躲在叶下,很不容易被人看到,像是怕羞,在往上长,一直高进荷叶,花蕾慢慢地养大,像要胀开似的。有一线蜻蜓,一直飞进来,落在花蕾上,停住后一动不动。夏日的荷由浅入深,像是一种境界,放眼望去,竟有一种无限的空间。
   ⑺夜幕降临,清新的荷塘便掠起一绺微风,轻轻地袭来,有一种沁凉,沿着荷叶一圈圈地荡开,月光很轻,从荷上移动。月下的荷塘萌生一种使人茅塞顿开的朦胧,一朵莲花,仿佛在梦里。一叶一叶的花瓣,挤开浓浓的黑夜,仿佛从污泥里抽出。配上蛙鼓,荷塘便喧闹起来,探荷的人仿佛一片荷叶轻飘飘的浮在水上,借着月色,荷叶依然地清脆,碧透。像水一样青嫩的荷绿仿佛一池易碎的翡翠,没有月光的晚上仿佛有一缕清香自荷中掠起,沾在袖上,掸也掸不去。荷在宁静的夜色中,浓浓地抒情,于恬静的田园岁月是一处精致的点缀。天亮了,荷也一一地亮起来。
   ⑻莲子熟时,一条窄窄的小船划破荷塘,向深处驶去,采莲女藕节一样的手像一尾鱼,在碧翠的荷叶中游动着,碧的水、绿的叶、红的花、白的手,无一不成为夏日荷塘的主题,便有一种斑斓,借荷为一种寄托,寓荷为一种归宿。于是伏案写荷,纸上的荷渐渐地醒了过来。
   ⑼便有轻轻的摇曳。
   (选自《散文天地》2003年第3期)
   1.文题中的“写”字在文章中的意思是:
   2.第⑶段中的“目光踏着荷叶”运用的是一种什么修辞手法?为什么说“小小荷塘,竟无边的开阔?”
   答:
   3.请根据第⑷段中划线句子意思写出《爱莲说》中相应句子。
   答:
   4.李清照在《如梦令》中写道:“兴尽晚回舟,误入藕花深处”,联系文中第⑼段,当你进入荷花深处时,荷花给你是一种什么样的感觉?
   答:
   5.请各用四个字概括文中下面几个语段的内容。
   第⑸段:
   第⑹段:
   第⑺段:
   6.你认为下面四幅画哪一幅切合第⑹段文字的内容?为什么?
   答:
   7.根据你对荷花的理解,请你依据文章内容自拟一道题目并且作答。
   题目:
   答案:
   (十)(2004·四地)
   风骨
   马德
   一天,我正要去上课。
   突然,有人在背后喊我,声音远远的。我扭过头看去,是一个农民模样的人,但我却不认识他。
   他说,马老师,马上就要上课了,我给闺女捎了些钱,麻烦你转交给她。噢,原来他是我们班一个女生的家长。他随即从上衣口袋里掏出一沓钱,当时我并没有太多的在意,只是想着家长尽快把钱交给我,因为上课铃已经响了。
   但他迟迟不肯给我,不断地数着他手中的钱。我这才注意到了,那一沓钱最外面的一张是100元,里边有两张20元,还有一张10元,剩下便是厚厚的一沓两元一元的零钞了。他又翻来覆去地数了几遍,嘴里念叨,怎么会少了一张呢。
   看着这些零钞,我当时突然有一种哽咽的感觉。十几年前我上高中时,父亲在一个大雪纷飞的冬天给我送钱来,冻得红裂的手心里攥紧的便是类似这样的一堆零钱,甚至里边夹杂着旧版的分纸币。而今天的这一堆零钱当中,可能也有省下的柴米油盐的钱,可能也有父母得病了舍不得吃药的钱,也许有几块钱是刚刚卖了鸡蛋得来的,甚至有的还是借别人的,上面尚留有别人的余温。可现在,他都给他的女儿拿来了。
   我问,少了多少呢?
   5元。家长有些捶胸顿足。嘴里不停地说,走的时候,我明明凑够了的,怎么会少了呢?这要我怎么办?这位父亲显然有些着急了。
   我说不要紧,就这样先给我吧。家长有些迟疑,但最终还是给了我。后来,家长走了,一边走,一边还不断地上上下下摸自己的衣兜,寻找他那不知遗失在何处的5元钱。
   那节课,我上得很不好,脑海中总是浮现着家长找钱的着急样子,鼻子酸酸的。下课后,我也没有把钱给我的学生,而是直接回到了办公室。
   在搭上自己的5块钱后,我把所有的零钱都换成了整钞。给我的学生的时候,我也只是轻描淡写,简单地告诉这是她父亲捎来的。学生点了点头便走了。
   我深知那一堆零钞的重量。我不想把它压在我的学生稚嫩双肩上。我知道,我这样做实际上也并没有改变什么,但我似乎只能做到这一点。
   我以为这件事就这样过去了。不料一天上午,这位家长又找到我,有些局促不安地从兜里掏出了5元钱给我,并说:闺女前些日子写信给我,说这次给她捎来的钱有些不一样,因为她从来没有收到过家里这么齐整的钱。读完信后,我便猜出了事情的原委,并且感觉到你肯定垫进去了几元钱,所以我今天给你送来了。
   我百般推辞。我说5元钱的,就算了吧,但家长却极认真的样子。半天推搡过后家长突然好像生气了,一把把那5元钱塞到了我的手里,简单地几句客气话之后,便一扭头走进深秋的风里。
   我那位可爱的学生,作为贫穷人家的子女,她竟然知道贫穷人家的钱是什么样子的;我更喜欢这样的父亲,因为他知道贫穷的风骨是什么。
   这个世界穷人不少,但能够高擎自己的灵魂活着的人不多。更多的人常常因为很可怜的一点利益而丢失自己最可宝贵的东西,从而使缺少精神之钙的虚弱身体在这个世界猝然跌倒。
   (选自《文学故事报》)
   1.全文通过哪两件事刻画了家长的形象?
   答:
   2.品读文中划横线的句子,揣摩它们分别表现了人物怎样的心理?
   ①我也只是轻描淡写,简单地告诉这是她父亲捎来的。
   答:
   ②(家长)简单地几句客气话之后,便一扭头走进深秋的风里。
   答:
   3.联系上下文,并发挥想象,概括文中的闺女给父亲写信的要点。(至少三点)
   答:
   4.说说文中倒数第二段表达了作者什么样的思想感情?
   答:
   5.联系全文,简要谈谈你对最后一段中“最可宝贵的东西”的理解。
   答:
   (十一)(2004·肇庆)
   借你一个微笑
   杨保中
   ①李俊是个性格内向的学生,阅完的试卷一发下,我发现他眉头又锁到一起了,他只得了58分。
   ②一个从来不及格的学生,自信心有多差就不用说了。
   ③我合上教案而无表情地走出了教室,李俊跟了上来,他喉头动了一下,然后眼泪就要掉下来了。我站住,等他说话。同学们也围了上来,他的脸涨得通红。我静静地站着,希望他能开口,但他的嘴唇好像紧紧锁住了似的。
   ④他递过一张纸条:老师,我的物理太差,您能不能每天放学后为我补一个小时的课?
   ⑤我可以马上答应他,但面对这样的一个学生我决定“迂回”一下。我牵着他的手到僻静处说:“老师答应你的要求,可这两天我太忙,你等等好不好?”他有些失望,但还是点点头。我知道他中计了,接着说,你必须先借一样东西给我!他着急起来,可还是说不出一句话。
   ⑥“你每天借给我一个微笑,好不好?”
   ⑦这个要求太出乎他的意料,他很困惑地看着我。我耐心地等待着,他终于眼噙泪花艰难地咧开嘴笑了,尽管有些情不由衷。
   ⑧第二天上课,我注意到李俊抬头注视我,我微笑着,但他把脸避开了,显然他还不习惯对我回应。我让全班一起朗读例题,然后再让他重读一遍。他没有感觉我为难他,大大方方地站起来读了。也许想起了昨天对我的承诺,读完后,很困难地对我笑了笑。见他这样,我心生一计,又给他设置了一道障碍。我说,你复述一下题目的要求,这回他为难得快要哭了。不少同学对他的无能表现得很不耐烦,七嘴八舌地争着说起来,我制止住了大家。他终于张口了,语无伦次。我笑着让他坐下。
   ⑨他开始和同学来往了,一起上厕所,回教室……这样过了好长一段时间,我都没提为他补习的事。一天下课李俊又拦住我,我知道他要干什么,很幽默地向他摊开手。他一愣,老师您要什么?我说,你写给我的条子呀。他笑了:我不写条子了,您给我补补课吧。我面带笑容:“功课你不必着急,到时我会主动找你的,但我向你借的你还没给够我。”
   ⑩“好的,我一定给足您。”等他高高兴兴又蹦又跳地走出好一段路后,我才像想起来什么似的把他叫回来,递给他一张纸条,那里有我为他准备的一道题。我告诉他,一天之内把它做出来,可以和同学讨论也可以独立完成。我知道,他宁可“独吞”,也决不会和同学讨论的。这正是性格内向学生的最大弱点。下午他说还没做出来,我有点不高兴,说晚自习你还没做好,我可要收回承诺了。 自习时我见他站在一个男生边上,忸忸怩怩很不自然的样子,我得意地笑了。就这样我先后为他写了4张纸条,题目一次比一次难。后来,纸条一到手他就迫不及待地和同学们争论开来。
   ⑾期末考试李俊成绩尚可,科科及格——看来我为他补的都差不多了。新学期刚开学,李俊休学了,因为他爸遇车祸瘫痪了,而他自小就被妈妈遗弃了——这也是他忧郁的一个原因。我有些担心,一个连话都不大愿说的少年,能担负起养护父亲的责任吗?
   ⑿星期天,我和几位朋友到茶室聊天。刚坐下就被一群小孩子围上了,硬要为我们擦皮鞋。只有一个小孩没冲进来,在外面吆喝着:擦皮鞋擦皮鞋!……离开茶室,我从那个小孩子面前走过时,发现那孩子竟是李俊!
   ⒀“老师,让我为您擦一次皮鞋吧。”他说,脸上没有腼腆也没有沮丧。我答应了,伸过鞋子让他很用心地擦着。他一边擦一边说,他虽然不缠人,生意也不错。顾客告诉他,他的笑容很好看。
   ⒁我说是吗?他又笑着告诉我,不久他还会复学的。他学会了笑,他的笑让他挣半天钱也能养活他和爸爸了。
   ⒂我也高兴起来,我说我一定等你回来。可转过身,我的泪水就出来了。李俊大声地在后面喊,老师您要笑呀,您不要哭!我点点头,反而呜咽有声了。
   ⒃我终于没有给他补课,是他为我补了一堂人生课。
   (选自《2002年中国微型小说精选》)
   1.通读这篇小说,简述题目“借你一个微笑”的深层含义。
   答:
   2.这篇小说的神态描绘逼真传神,第③自然段中画线语句写出了李俊什么样的心理?请简要回答。
   答:
   3.第⑦自然段写李俊听说“我”要向他每天借一个微笑后,始而“很困惑地看着我”,终而“眼噙泪花艰难地咧开嘴笑了,尽管有些情不由衷”。由于受叙事方式的限制,这当中作者没有写李俊复杂的心理活动,假如你是当时的李俊,请将你的心理活动描写出来。 (不超过80字)
   答:
   4.在这篇小说中,为了让李俊每天能够借给“我”一个微笑,作为老师的“我”具体做了哪几件事?请概括作答。
   答:
   5.作者在第⑩自然段写道:“期末考试李俊成绩尚可,科科及格——看来我为他补 的都差不多了”,而在小说结尾却又说“我终于没有给他补课”,这是否矛盾?为什么?请简要作答。
   答:
   6.在小说结尾,作者饱含深情地写道:“是他(李俊)为我补了一堂人生课”,请简要说说李俊为“我”补的这堂人生课的主要内容。
   答:
   (十二)(2004·内江)
   今天上午发生的事情使我认识到卡罗纳是怎么样的一个人了。
   我去上学的途中,碰到二年级的女老师,她说准备到我家去一次,问我什么时候在家。这样,我耽误了些时间,晚到学校一会儿。到了学校,我们班的老师还没来。三四个同学正拿可怜巴巴的科罗西开心取笑。科罗西不是别人,正是那个长着红头发、一只胳膊因残废而垂在胸前、妈妈以卖菜为生的孩子。他们用尺子戳他,朝他脸上扔栗子壳儿,模仿他的残臂挂在脖颈上,把他比作畸零人、残废人和妖魔鬼怪。他一个人孤苦伶仃地坐在课桌的尽头,脸色苍白,极力忍受着他们的污言秽语。为了得到片刻的安宁,他用祈求的目光一会儿看看这个,一会儿又望望那个。但是他们变本加厉地嘲笑他,挑逗他。他气得面红耳赤,浑身直打哆嗦。突然间,那个厚颜无耻、爱搞恶作剧的弗朗蒂跳到课桌上,学着科罗西的母亲胳膊上挎着菜篮子的样子,逗得在场的所有同学捧腹大笑。科罗西的母亲一般是这个样子在校门口等儿子的,只因她最近生病没来接儿子。
   这时候的科罗西已失去了理智,抄起一个墨水瓶狠狠朝弗朗蒂的脑袋上砸过去。弗朗蒂敏捷地闪到一旁,而墨水瓶正好打在刚进教室的老师的胸脯上。
   同学们争先恐后地跑回各自的座位,个个惊吓得默不作声。脸拉得长长的老师走到讲台前,气呼呼地大声问道:
   “是谁?”没人吱声。
   老师再一次提高嗓门,吼叫道:
   “到底是谁?”
   见没人答话,卡罗纳向可怜巴巴的科罗西瞥了一眼,猛然站起来,语气果断地说:
   “是我!”
   老师上下打量他一番,又望了望呆若木鸡的其他同学,然后语气平和地说:
   “不是你。”
   过了片刻,老师接着说:
   “肇事者今天将不会受到处分,快站起来说吧!”科罗西站起来,伤心地哭着说:
   “他们打我,欺辱我,我气疯了,拿起……”
   “坐下。”老师打着手势对科罗西说。老师接着以命令的口吻大声说:
   “那些肇事者赶快站起来!”
   四个人耷拉着脑袋站起来。
   老师用严肃而有力的声音说道:
   “你们肆无忌惮地欺负一个从不打扰你们的同学,嘲笑一个可怜巴巴的孩子,打了一个毫无自卫能力的弱者,你们的行为是最卑鄙无耻的,实在是玷污了‘人’这个美丽而又神圣的字眼!一群胆小鬼!”
   老师一口气说完,走到课桌跟前,一只手抚摸着低着头的卡罗纳的下巴颏儿,顺手托起他的脑袋,深情地说:
   “你的心灵真美!”
   卡罗纳趁机跟老师交头接耳,咕哝了几句,谁也听不清楚说的是什么。老师接着转过身来,用生硬的语气对几个肇事者说:
   “这次我宽恕你们!”
   1.文中第一句“今天上午发生的事情使我认识到卡罗纳是怎么样的一个人了”中“认识”是什么意思?“我”是通过哪两上情节“认识”卡罗纳的?
   ①(解释)认识:
   ②两个情节:
   2.老师称赞卡罗纳“你的心灵真美”,你觉得卡罗纳的“心灵美”包含哪些品质?(4分)
   答:
   3.卡罗纳趁机跟老师交头接耳,咕哝了几句,你知道咕哝的是什么吗?请结合人物性格和上下文,用对话的形式写出他们咕哝的内容。
   答:
  
   4.这篇文章对人物的描写很有特色,请任选一点,具体说说你的体会。
   答:
  
   (十三)(2004·兰州)
   天职
   ⑴海尔曼博士是位医术高超、医德高尚的大夫。他开的诊所已远近闻名,在布拉沙市里没有人不知道海尔曼和他的诊所的。
   ⑵海尔曼这个倔老头子,像他那把用最好的钢材做成的手术刀一样坚硬锋利。
   ⑶一天夜里他的诊所被一个小偷撬开了,一点现金和几样珍贵的药物,都被小偷放在提兜里准备带走。不巧,小偷慌忙中撞倒吊瓶支架,又被氧气罐绊倒,摔折了小腿,要跑也爬不起来了。这时,海尔曼和助手从楼上下来,助手说:“打电话让警察把他带走吧!”
   ⑷“不,在我诊所的病人不能这样出去。”海尔曼把小偷抬上手术台,连夜给他做了连肢手术,并打上了石膏绷带。一直在诊所里把他彻底治好,才把他交给警察。
   ⑸助手说:“他偷了您的财物,您怎么还如此给他治疗呢?”
   ⑹“救死扶伤是医生的天职。”
   ⑺小偷自然感激得五体投地,惟在交警察前,他恳求把他放了。他说:“海尔曼博士,您不愧是上帝的儿子。我愿再次得到你的拯救,不到那阴森的牢里去领面包……”海尔曼博士两手一摊说:“先生,对您这个要求,我这把手术刀就无能为力了。”
   ⑻又一天,一个女人护送一位车祸中受重伤的人来诊所。海尔曼一愣:啊,是她?她早已徐娘半老,怎么仍这般漂亮?这是他被人夺去的爱妻。至今她在他的眼里,仍然具有不可代替的魅力。
   ⑼女人泪流满面地说:“海尔曼,亲爱的海尔曼,你还恨我吗……为了拯救他的生命,我不得不来求你,你是全市惟一能给他做手术的人。”
   ⑽受重伤的人是他原来爱妻的后夫,就是这个人把她夺去了。当时就差点同他进行古老的决斗。
   ⑾“亲爱的海尔曼,我和他都对不起你,可是我们遇了难……但愿您的手术刀不带着往日的仇恨。”
   ⑿海尔曼曾经受过他们的侮辱。现在在这种场合重逢,使他不由地心潮起伏,思绪万千,他始终一言未发。
   ⒀列夫斯基一直处于昏迷状态,待进了手术室才清醒过来,见拿着手术刀的是海尔曼,不由大吃一惊,连忙挣扎着要起来。
   ⒁“老实躺好,这是上帝的安排。你是我永远难以宽恕的情敌,你又是我必须抢救的患者。”
   ⒂一个修补颅骨的手术,让海尔曼站了十多个小时,最后晕倒在手术台旁。
   ⒃列夫斯基伤愈后,夫妻俩在海尔曼面前惭愧地说:“如果您不嫌弃,我们愿意为服侍您而献出余生。”
   ⒄海尔曼说:“医生在手术室里记住的只是他的天职,忘记的是个人恩怨。”
   ⒅这年,德国发动第二次世界大战,占领了布拉沙。一个盖世太保头目,被波兰地下战士一枪打中了胸部。随军医生没人能给他做这样的大手术,便把他化了装送到海尔曼的诊所。海尔曼一眼就认出这是个最凶残的德国刑警队警官,在这个城市里不知有多少波兰人丧生在这个人的枪口下。他心中猛然一震,暗自喟叹,这也是上帝的旨意啊!
   ⒆海尔曼支走了所有助手和医护人员,他洗手、刮脸,重新穿好上教堂才穿的那套西服,罩上一件最新的白外套。然后拿起他最大的那把手术刀,一下子剖开那个盖世太保的胸膛。他没有去找子弹,而是用那把手术刀结束了这个法西斯刽子手的生命……
   ⒇在受审时,德国人说:“你玷污了你的手术刀。”
   (21)“没有,它用得其所。”
   (22)“你忘记了医生的天职。”
   (23)“没有,此时此刻,反法西斯就是最高的天职!”他一字一顿,字字千钧,全市人都听到了。
   (24)海尔曼牺牲了。可城里到处都张贴着“天职”两个大字,不用再加其他文字,它就成了一条具有巨大号召力量的反法西斯标语。
   (选自《读者》2002年第8期)
   1.本文共写了3个小故事,每个故事的前面,都有一个词语表示故事的开始,请分别找出这3个词语;再分别给这3个故事拟一个小标题。
   ①第一个故事:词语
   小标题
   ②第二个故事:词语
   小标题
   ③第三个故事:词语
   小标题
   2.第⑴段在全文中的作用是 。
   3.第⑵段运用了比喻的修辞方法,联系上下文看这儿运用比喻手法的好处是
   ;句中的“坚硬锋利”表现了海尔曼的 。
   4.第⑿段说海尔曼“不由地心潮起伏,思绪万千”,请结合上下文,在30字内写出他此时的心理活动。
   答:
   5.文中第20至第23段中,德国人和海尔曼对“天职”的理解分别是:
   ①德国人
   ②海尔曼
   6.阅读完全文后,谈谈你对海尔曼的认识。
   答:
   (十四)(2004·昆明)
   天使
   ⑴小时候,我是一个捣蛋、不爱学习又极爱报复的孩子。无论在家里还是在学校,父母和老师、兄弟和同学都极其厌恶我,然而,在心里我渴望着大家的关爱,就像人们渴望上帝的福泽一样。我一个人独处的时候常常默默祈祷:上帝啊!给我善良、给我宽厚、给我聪明吧,我也想如卡尔列一样成为同学们的榜样。可是,上帝正患耳疾,我的祈祷没有一句应验。我依然是个令人生厌的坏孩子,甚至因为我,没有老师愿意带我们这个班。
   ⑵三年级的第一个学期,学校里来了一位新老师,她就是年轻的玛丽娅小姐。玛丽娅小姐刚一站到讲台上,整个班里都沸腾了,她太漂亮啦!我带头吹口哨、飞吻、往空中扔书本,好多男生跟我学,我们的吵闹声几乎要把房顶掀开。
   ⑶玛丽亚小姐没有像其他老师那样大声叫嚷:“安静!安静!”她始终面带微笑在望着我们。奇怪,这样我反而感到很无聊,于是,我打一个手势,大家立即停止了胡闹。玛丽娅小姐开始自我介绍,当她转身想把自己的名字写到黑板上时,才发现讲桌上没有粉笔,我注意到她的眉头皱了一下,很快又舒展了。心想;糟了,她肯定识破了我们的把戏。但是,玛丽娅小姐却转过身来问:“谁愿意替老师去拿盒粉笔?”刚刚平静下来的沸腾又开始了,怪声怪气的笑声再次淹没了整个教室,好多男生争着去干这件事。
   ⑷玛丽娅小姐请大家不要争,她会挑一个最合适的人选。玛丽娅走下讲台,仔细查看了每一个人,最后她说:“基恩,你去吧。”我说:“为什么是我?”“因为我看得出你热情、机灵又具号召力,我相信你会把事情做得很好。”
   ⑸我热情?我机灵?我具有号召力?我竟然有这么多优点?玛丽娅一眼就看出了我的优点!要知道,在此之前从未有人说过我哪怕一点点的好处,甚至我自己也认为我是一个被上帝抛弃的孩子。
   ⑹我很快取回一盒粉笔,因为它就藏在教室后面的草丛里。当我正要把粉笔递给玛丽娅小姐时,我发现我的手指甲缝里存满了污垢,衬衣袖口开了线,裤腿上溅满了泥点,更糟糕的是我五个脚趾全从破了口的鞋子里露出了头。我很不好意思,可玛丽娅小姐一点也不在意这些,她接粉笔的时候给了我一个天使般的微笑。
   ⑺从此,我决定做一个上进、体面的人,因为我知道天使正在注视着我。
   1.给下列加点的字注音。
   祈( )祷 污垢( ) 淹没( )
   2.第⑴段的描述对下文进一步展示“我”是一个铺垫。请你用一句话评价一下这时的“我”是个什么样的孩子?
   答:
   3.玛丽娅小姐根据什么对基恩作出“热情,机灵,具有号召力”的评价?请用文中的语句回答。
   答:
   4.第⑹段对“我”的外貌描写的作用是什么?
   答:
   5.本文人物描写最主要的方法是什么,请作简要分析。
   答:
   6.本文为何以“天使”为题,谈谈你的理解。
   答:
   (十五)(2004·青海)
   母爱
   这是一个真实的故事。故事发生在我们青海省一个极度缺水的沙漠地区。这里,每人每天用水量严格地限定为三斤,这还得靠驻军从很远的地方运来。日常的饮用、洗漱、洗菜、洗衣,包括喂牲口,全都依赖这三斤珍贵的水。
   人缺水不行,牲畜也一样,渴啊!一天,一头一直被人们认为憨厚、忠实的老牛渴极了,挣脱了缰绳,强行闯入沙漠里的也是运水车必经的公路。
   终于,运水的军车来了,老牛以不可思议的识别力,迅速地冲上公路,军车一个急刹车戛然而止。老牛沉默地立在车前,任凭驾驶员呵斥驱赶,它都不肯挪动半步。五分钟过去了,双方依然僵持着。运水的战士以前也碰到过牲口拦路索水的情形,但它们都不像这头老牛这样倔强。人和牛就这样耗着,最后造成了堵车,后面的司机开始骂骂咧咧,性急的甚至试图点火驱赶,可老牛不为所动。
   后来,牛的主人寻来了,恼羞成怒的主人扬起长鞭狠狠地抽打在瘦骨嶙峋的牛背上,牛被打得皮开肉绽,哀哀叫唤,但还是不肯让开。鲜血沁了出来,染红了鞭子,老牛的凄厉哞叫,和着沙漠中阴冷的酷风,显得分外的悲壮。一旁的运水战士哭了,骂骂咧咧的司机也哭了。最后,运水的战士说:“就让我违反一次规定吧!我愿意接受一次处分。”他从水车上取出半盆水——正好三斤,放在牛面前。
   出人意料的是,老牛没有喝以死抗争得来的水,而是对着夕阳,仰天长哞,似乎在呼唤什么。不远处的沙堆背后跑来一头小牛,受伤的老牛慈爱地看着小牛贪婪地喝完水,伸出舌头舔舔小牛的眼睛,小牛也舔舔老牛的眼睛。静默中,人们看到了母子眼中的泪水。没等主人吆喝,在一片寂静无语中,它们掉转头,慢慢往回走。
   20世纪末的一个晚上,当我从电视里看到这让人揪心的一幕时,我想起了幼时家里的贫穷困窘,想起了劳作的苦难的母亲,我和电视机前的许多观众一样,流下了滚滚热泪。
   1.联系上下文语境,说说加点词语的意思。
   ①戛然而止:
   ②贪婪:
   2.文中依次写了三个感人的情节,分别是:老牛 ,强忍鞭打执意索水,护犊饮水母子情深。
   3.请从文中摘抄最使你感动并且最能表现老牛母子情深的句子,写在下面横线上。
   答:
   4.文中画线句运用的主要表达方式是什么?它有什么好处?
   答:
   5.本文对你有什么启迪?请你谈一点体会。
   答:
   6.假如你是运水战士,你会甘受处分而给老牛半盆水吗?为什么?
   答:
   (十六)(2004·咸宁)
   父亲的信
   胡智慧
   ⑴父亲来信了,内容非常简单:“慧:你已经离家归队多日,也没有写信说你平安到达,你妈非常担心。每次归队后都应先写信。父。”
   ⑵于是,一种深深的愧疚慢慢弥散在心头。
   ⑶到远离家乡的辽北当兵,有了写家书的必要。但现在老了点,当军官了,潜意识中总认为做农民的父亲无话可说,而所谓家书其实只是例行公事地报平安而已。父亲却认真得很,不但对我每次报平安的信仔细答复,隔一段时间,还会主动给我写信。父亲的信很短一般不超过一页纸,有时只有三五句话,无非是告诉我家中一切都好,要学会照顾自己等等。
   ⑷父亲的信是很随意和马虎的,有时写信的纸好像是随手捡来的,正面还有无关的字迹,反面才有几嘱托。而且家中好像也没有一支真正能用的笔了,父亲能找到什么样的就用什么样的,所以给我的信中常常有各色笔迹。我曾提醒父亲不要用红色笔写信,但父亲“顽固”依旧。战友们读到家书的时候,往往都是激动异常,心情久久不能平静,而读父亲的信却很少有这种感觉。父亲像在无意中写了这些信,从信中我似乎读不到家的温馨,也读不出家里人是否对我有想念之意。我总是平静地读父亲的来信,像读一些与我无关的语句,然后收起来,留意下这是父亲的第几封信。
   ⑸探家的时候,我开玩笑般对母亲提起父亲的信,想让母亲告诉父亲,如果没事就不必经常写信了。在我看来,父亲那些没有多少意义的信是不值那几角邮资的。而母亲的回答却出乎我的预料:“你爸写这些都成习惯了,到时候如果不给你写信,他就觉得像把你丢了一样。”像把我丢了一样?我不禁一怔,一种不可名状的东西在心中升起。
   ⑹偶然翻看父亲专用的那个抽屉,意外的发现一沓信封,每个信封都贴好了邮票,而且都写上了我的名字。我随手把信封一字排开,形成了一条长长的线。此时此刻,我不由泪流满面:我已是高空中的风筝,或许永远没有回归的那一天;而父亲却在用信拉起一根长长的线,以便知道他的孩子在何处飞翔啊!
   ⑺到部队后, 。
   1.结合文意理解,作者为什么说“父亲的信是随意和马虎的”?
   答:
   2.第⑸自然段中划横线的句子的意思,第⑷自然段有一句话对此埋下伏笔,请写出。
   答:
   3.文章第⑺自然段“到部队后,‘我’会怎样做”?请发挥你的想象,用叙述性或者议论性的文字,写一两句话,将横线处的内容补充完整(不超过50个字)。
   答:
   4.文章的题目是《父亲的信》,能否用《父亲的爱》作标题?为什么?
   答:
   5.综观全文,随着情节的发展变化,作者的感情也随之发生了变化,请写出文中作者情感变化的三个阶段的相关句子。
   答:① 。
   ② 。
   ③ 。

第十四讲 命(半命)题作文
  
   【试题特点】
   命题作文是传统的作文方式,命题作文的题型特点是:出题者给出了完整的、确定的作文题。考生必须用试卷中这完整的、确定的题目进行作文。
   从近几年中考命题作文的“命题”来看,有这样几种方式:
   1.直接式命题。此类题单纯明快,或直接点明叙说的人和事,或明确提示议论的中心,或清晰地显现说明的对象。考生做此类题,在审题、取材上一般不会发生什么困难。
   2.导语式命题。此类题或在文题出现之前写有一段文字,或在文题出现之后写有一段文字,它们有的起着撩拨学生情感的作用,有的起着引导学生思考方向的作用,有的起着提示考生不要失误的作用,有的起着建议考生如何取材的作用。对这样的题,考生要读好“导语”与“提示”,获取其中对自己有益的信息。
   3.中心语式命题。此类题给出了一个“题”,但考生也可以对这个题附加上其它的词语,从而构成对自己有利的题,这时,命题人给出的题实际上就成了考生自由命题的中心语。其实这样的题已带有话题作文和半命题作文的味道。考生做这样的题,要看清写作的要求,不要失去一次好机会。
   半命题作文也叫“补题作文”,即将一个完全命题省去一部分,变为一个不完全命题,省去的部分由考生去补充完整,然后按照命题作文的要求作文。
   1.命前半题,补后半题。如:
   (1)我关心
   (2)爱使我
   2.命后半题,补前半题。如:
   (1) 的喜悦
   (2) 牵动我的情思
   3.命首尾,补中间。如:
   (1)在充满 的日子里
   (2)一份 的答卷
   4.命中间,补首尾。如:
   (1) ,别再让我
   (2) 变
   【解题导引】
   例1.(2003·重庆)
   [题目]:责任
   提示:做一个有责任心的人,是时代的需要。有人把“天下兴亡,匹夫有责”改为“天下兴亡,我有责”。意思是说,“我”应当义不容辞地担负起对国家、对社会的责任。的确,我们每一个人都应当做一个有责任心的人,都应当承担起自己的责任。国家遇到了困难、学校出现了问题、家庭发生了矛盾、同学违反了纪律、自己成绩不理想……都应当问一问:“我应当承担什么责任?”
   要求:①可以写自己的经历,也可以写别人的事可以从正面写,也可以从反面写;可以抒发情感,也可以发表议论。②除诗歌外,文体自选。③不少于600字。④文中不得出现真实的人名、校名。
   [导写] 我国即将实施的课程改革,将责任感列为培养人才的重要目标之一。可见,“责任”这个题目,具有较强的现实意义。
   本题写起来并不难,难的是写活,写生动,写深刻。很多同学面对这个题目会选写议论文。这就需要特别注意一个问题,那就是千万别落入空喊口号的误区。要诠释好“责任”,光说大话是不行的,应该紧密结合当今社会的实际,让发生在我们身边的极富责任感的事件现身说法(比如,中国“入世”谈判首席代表龙永图先生讲过一个故事,他在国外某地入厕时,发现一个小男孩在不停地拉水箱冲便,因拉不下水而着急。龙先生感慨道,由此事可以看出这样的小孩已有了可贵的责任意识),并加以情真意切的抒情议论。这样才能给读者以心灵的撞击,使读者产生情感的共鸣。在写作形式上要力求创新,可以采用会议纪要、电视现场谈话、剧本、日记、书信等形式,写出独具特点的好文章。
   具体的写作思路参考:一、提倡每个人都应加强责任感,搞好本职工作。二、谈青少年要培养责任意识,这种责任是对自己的,也是对家庭的,更是对民族的,国家的,社会的,人类的。三、揭露互相推诿、不负责任的世相,诠释领导干部、公务人员应承担起肩头责任的主题。四、谈责任是双向的,我们十分强调个人对社会的责任,也不能忽视了同样重要的另一方面——社会对个人的责任。比如,政府应为下岗工人建立必要的社会生活保障体系,等等。
   [例文]
   责 任
   英国王子查尔斯曾经说过:“这个世界上有许多你不得不去做的事,这就是责任。”
   责任不是一个甜美的字眼,它仅有的是岩石般的冷峻。一个人真正地成为社会一分子的时候,责任作为一份成年的礼物已不知不觉地卸落在他的背上。它是一个你时时不得不付出一切去呵护的孩子,而它给予你的,往往只是灵魂与肉体上感到的痛苦,这样的一个十字架,我们为什么要背负呢?因为它最终带给你的是人类的珍宝——人格的伟大。
   20世纪初的一位美国意大利移民曾为人类精神历史写下灿烂光辉的一笔。他叫弗兰克,经过艰苦的积蓄开办了一家小银行。但一次银行遭抢劫导致了他不平凡的经历。他破了产,储户失去了存款。当他带着一个妻子和四个儿女从头开始的时候,他决定偿还那笔天文数字般的存款。所有的人都劝他:“你为什么要这样做呢?这件事你是没有责任的。”但他回答:“是的,在法律上也许我没有责任,但在道义上,我有责任,我应该还钱。”
   偿还的代价是三十年的艰苦生活,寄出最后一笔“债务”时,他轻叹:“现在我终于无
   债一身轻了。”他用一生的辛酸和汗水完成了他的责任,而给世界留下了一笔真正的财富。
   责任的存在,是上天留给世人的一种考验,许多人通不过这场考验,逃匿了。许多人承受了,自己戴上了荆冠。逃匿的人随着时间消逝了,没有在世界上留下一点痕迹。承受的人也会消逝,但他们仍然活着,死了也仍然活着,精神使他们不朽。
   我们都知道那个天黑了还不肯回家,站在路边哭泣的孩子。因为他要站岗,别的孩子早散了,可他为了坚守岗位宁愿站着哭泣,因为这是他的责任。
   愿我们所有的孩子都有这样的心灵,责任从小就在那里成长。
   愿我们所有的人都把责任之心携带在人生的道路上,让人生散发出淡淡的,金子般的光辉。
   [点评] 这篇文章先引英国王子查尔斯的话,对“责任”一词作了解释;再用喻证法、例证法和正反对比论证,揭示出承担责任带来的收获——显示人格精神,深入地阐明了恪守自己应尽的责任的意义。
   文中不少语句凝练而富于意蕴,耐人咀嚼,发人深思。
   例2.(2004·安徽)
   [题目] 作为中学生,我们生活在幸福的时代,我们拥有知识,拥有青春,拥有激情。有时我们也会埋怨命运不公,感叹生活太累。其实,丰富多彩的生活值得我们珍惜的很多很多。
   请以“珍惜所拥有 ”为题,写一篇文章。
   提示和要求:
   (1)首先在横线上把题目补充完整。横线上可选填的“青春、幸福、知识、激情、健康、荣誉”等词语。(2)选择你最能驾驭的文体,抒发你真挚的感情,写你最熟悉的事情。(3)文中不得出现真实的校名、地名、人名,否则会扣分。(4)字数不少于500字。(5)不得抄袭。
   [导写] 这个世界上值得我们珍惜的事物太多太多,亲情、友情、爱情值得我们珍惜;生命、人生、生活值得我们珍惜;山川、湖泊、大海值得我们珍惜……但透过材料我们可以看出“珍惜”自己的一切才是我们行文的关键。我们每一个人都是世间的一个奇迹,我们应该看重自己的存在,看重自己的价值。
   [例文]
   珍惜所拥有的青春
   刚刚抖落一身的稚气,刚刚放飞完美的天真,青春的微笑还很矜持。
   ——题记
   我喜欢清澈见底的漓江水,也喜欢平波展镜的西湖水。然而,生命之长河中那湾青春水更是我爱的至点。
   定立于青春水边,沉思着如何渡河,水中嬉戏的鱼儿或许会扰乱你的思绪,在这样的扰动下,或许你会放弃原有的那份真挚与纯洁,悄悄地走上“完美的殿堂”。因为青春的心总是溢满幻想,然而,朋友,你是否想过青春水很急,不允许我们过于追求完美,过于追求完美,总是以不完美收场。
   喜欢大雾弥漫的天气,因为它呈现给我的是一片朦胧。然而,青春月光所呈现的那一片柔和的朦胧亦是我爱的屋脊。
   笼罩在月光下,一颗幼稚的心升华至频频的跳动,那股柔情的袭击,你或许按捺不住心中的悸动,在无声无息中,你迈进了“朦胧的宫殿”。因为青春的心充满惊奇,然而,朋友,你是否知道,青春月光所散布的那一片朦胧需要擦拭你的双眼,在平静中观赏美,不要在朦胧的美丽中迷失了方向。
   喜欢温柔多情的春天的生机勃勃,因为它给我带来的是希望与宁静。然而,夏天无情的狂风暴雨更是我爱的心极,因为它象征的是青春的火热炽情。
   躺在青春中春天的怀抱,因为“春天是希望的象征”,你忘记了追求。沐浴于阳光下,给你带来的是一份安逸。然而,生活的过度安逸却是通往灭亡的起点,或许,你的结果便由“死于安乐”来印证。青春是充满激情的,多情的夏天给它注入新的活力,让它有了搏击人生的资本。春天的幼芽还很羞涩,丰硕的果实还有待于夏天狂风暴雨的考验。
   漫步于青春中的四季,不要对春天的景致流连忘返,紧紧地拥抱夏天吧!
   珍惜所拥有的青春,因为青春的微笑还很矜持。
   ——后记
   [点评] 这是一篇难得的考场精美散文。首先,立意正确,主题鲜明,观点明确,内容具体充实;其次,文章结构严谨,构思新颖独特,开头有点明主旨的题记,为全文定下了感情基调;结尾有后记,呼应文章开头,收束全文,升华文章主题;再次,文章感情充沛而真挚,展示了当代中学生的青春风采;第四,文章语言流畅,优美生动,形象可感,尤其是一些排比句、比拟句用得很贴切。第五,这篇文章的作者书写态度非常端正,字迹工整清楚,美观大方,卷面整洁。当然,文章也还有一些不足之处。但是,瑕不掩瑜,文章的优点是很明显的,更何况它是在限定的考试时间和紧张心态下完成的。鉴于以上这些特点,阅卷老师评分时给了满分。
   写作中考命(半命)题作文,要做好下面几个方面的工作。
   1.审清题意。
   (1)注意文题含义。有些文题的含义很丰富,是需要细细琢磨一番的。如“长大的感觉”,如果能寓理于事,从不同的角度写出正处于花季年龄的初中生成长中的追求、向往、烦恼和困惑,以及对人生的初步认识,写出人生中的各种各样的责任在心中出现,那么这样的思考就是准确地把握了文题的含义。
   (2)注意文题题眼。文题中一般都有关键词,这些词就是所谓的题眼。把握了题眼,也就抓住了文章要突出的重点,了解了写作要避免的误区。
   (3)注意文体类别。中考文题对文体要求的表述主要有二种形式。一是明确规定只能写一种文体;二是明确规定可以写多种文体。
   (4)注意立意信息。有些文题还有立意方面的暗示信息,明察到这一点,对于写出高质量的考场习作是大有帮助的。
   (5)注意选材指向。有些中考作文题对写作材料还有着或明或暗的指向。体会到这一点,才会避免选材失误,提高选材质量。
   2.选好材料。
   (1)选精当的材料。材料不能游离主题,主题是一篇文章的灵魂,你所组织的材料一定要契合文章主题;材料不能胡编滥造,这就是说,组织材料应力求真实、准确,那些“拾到东西一等几小时”之类胡编滥造的材料只会弄巧成拙,影响文章的质量;材料不能过于狭窄。不要一写“奉献爱心”,就是给“希望工程捐款”;一写“家乡变化”,就是写家里添了电视机,买了收音机。
   (2)选新颖的材料。“新”在独具慧眼,材料要新,选材者需有一双慧眼;“新”在独运匠心,有时,你可能对组织的材料举棋不定,这时你最好以你的“匠心”度一度他人之“腹”,再决定材料的取舍;“新”在独辟蹊径这“蹊径”往往表现为一种与众不同的“视角”。
   3.确定主旨。
   确立主旨就是确立一篇文章的中心,也就是我们通常说的“立意”。我们无论是写哪一种文体,都应在立意上下功夫。其要求是健康、新颖、深刻。健康就是要保证文章的思想观点和感情正确,要体现出一定的思想意义,能给读者以鼓舞和启迪;新颖,就是立意要跳出陈旧的框框,“言人之所欲言”,“言人之所未能言”;深刻,就是要透过现象看本质,挖掘出文中深广的意蕴,以增加文章的穿透力和厚重感。
   4.重视首尾。
   开头结尾在文章中的作用十分重要。它能提炼中心,完整结构,吸引读者。它能使文章锦上添花,令阅卷者倦处生神。开头或直奔主题,也就是开门见山;或设置悬念,增添文章波澜,展现文章的布局之美;或引用材料;或描写人物。考场作文的结尾应尽量做到简洁、紧凑、恰到好处。或顺其自然,把内容表达完之后,自然而然的收束全文;或呼应篇首,使文章首尾圆合,结构完整;或揭示主旨。
   5.对半命题作文还要注意:
   首先要在弄清题目所提供的词语意思的基础上,弄清未受限制的内容(即横线上可填哪些词语)。半命题作文一般要提供词语。如果题目要求只能用提供的词语作文,那么你可从中选择一个词语;如果在提供的词语后面打上省略号或者出现“等等”字样,则暗示你可用提供的词语,也可用未提供的词语。其次,所选用的词语填入文题后构成一个完整的文题,力求新颖别致而且自己能写得出采。文题新颖别致,能使阅卷老师眼睛为之一亮;自己能写得出采,才能写出佳作;另外,填好词语以后,要从语意、语法、语感的角度检查一下,力求准确、简明、生动。
   【新题导写】
   1.跨向明天的一步
   青春年少的你正在一步步走向成熟,在你成长的过程中,一定有许多令人心动的故事。请用上面的文题写一篇记叙文。
   [点拨]从文题类型看,这是一篇命题作文,从文体类别看,这是一篇记叙文,从文题含义看,这是一个含 比喻意义的文题。“跨向明天的一步”比喻青少年在成长过程中思想的逐步成熟,性格的不断完善,学习的点滴进步等等。从选材方面看,应写与自己有关的真实故事,应写成长途中令人怦然心动的故事。比如:认识了知识的重要,懂得了友谊的珍贵,领悟了深奥的道理;由孤僻变得合群,由懦弱变得勇敢,由依赖变得独立;电脑考试达标了,书法作品获奖了,得意之作发表了……文题中的“一步”还暗示应从方方面面中选一个侧面展开,不可面面俱到。
   2.①我家的常客 ②我家的稀客
   请从上面两个文题中任选一题作文,文体不限。
   [点拨] 论文题含义,“常客”指经常到我家的客人,“稀客”指很少到我家来的客人。二题都可以写成写人为主的文章。当然将“客”理解为某种“物”也未尝不可。论写作重心,“常客”,可写其方方面面;“稀客”,只宜于写一、二次特殊的印象,无论是写“常客”还是写“稀客”,都应把客人放在“我家”这个环境中来写。论文题类型,这是一道选题作文,只能二中选一,不可两个都写。
   3.为 而陶醉
   生活是美好的。美好的生活中,无论人、事、物、景等,总有令你陶醉的时候,请抓住印象最深、感受最深的某一“陶醉”点,补充文题,写一篇文章,体裁不限。
   [点拨] 这是一道半命题作文。半命题作文往往题材宽泛,有话可写。就本文而言,选材面十分广阔。可以说是琴棋书画、天文地理、风土人情、名胜古迹……样样皆可入文中。究竟怎样选好材料呢?一是要新颖独特。有些材料很熟悉,但正因为太熟太俗可能被很多人看好,为了避免雷同,你最好另辟蹊径。二是要宜于抒发自己的情感,特别是能表现“ 陶醉”,因为写好“陶醉”是本文成功的关键。
   4.感受
   生活中的每一个人都在“感受”,感受时间、感受关爱、感受友谊……你对什么东西“感受”得最深呢?请将上面的文题填充完整,写一篇文章,文体不限。
   [点拨] 因为是半命题作文,写作当然有广阔的空间。不管准备写哪一种文体,你在选择写作对象时,都要注意这样几点:①宜于立意,这种立意固然要新要深,但必须是一个你容易驾驭的主题,不然,你就会因把握不准而出现立意失误。 ②宜于铺展。也就是说能围绕你确立的主旨铺展出丰厚的内容。③宜于点示。有时候,文中一两句精妙的点示往往能使文章熠熠生辉,写本文时若点示得法,肯定不乏这样的佳句。
   【考点精练】
   1.(2004·苏州)以“心事”为题,写一篇不少于600字的文章。文中不要出现(或暗示)本人的姓名、所在学校。
   2.(2004·泉州)题目:和父母交朋友
   要求:①不少于600字;②文体自选;③不得出现真实的地名、校名、人名。
   3.(2004·桂林)请以“读书真好”为题,写一篇作文。
   要求:写一篇600字以上的文章,文体不限,有真情实感,书写整洁、美观,文中一律不得出现真实的校名、人名。
   4.(2004·铜仁)我们这个世界既需要参天的大树,也需要无名的小草。你也许不是最美丽,但你可以最可爱;你也许不是最聪明,但你可以最勤奋,你也许不会最富有,但你可以最充实;你也许不会最顺利,但你可以最乐观……在未来的人生道路上,你也许不能成名成家,不能名垂千古,但你可以成为同行业千千万万普通劳动者里最好的一个。是的,每个人都能成为最好的自己。请结合自己的特长、兴趣、爱好、理想,发挥自己的联想和想象,以“我是最好的一个”为题,写一篇不少于 500字的文章,文章可以写今天的你,也可以面向未来写理想中的你。亲爱的同学,请你敞开心扉,大胆地写出你心目中“最好的自己”。
   5.(2004·北京)
   题目: 需要我
   要求:1.在横线上填入恰当的词语,使题目完整。2.写一篇不少于600字的文章。3.文中不能出现所在学校的校名和师生姓名。
   6.(2004·莱西)教育部颁布的《语文课程标准》中,为初中生的课外阅读指定了必读的书目。这些书目是下列10部名著:《西游记》、《水浒传》、《朝花夕拾》、《骆驼祥子》、《繁星•春水》、《芙蓉镇》、《鲁滨逊漂流记》、《格列佛游记》、《童年》、《钢铁是怎样炼成的》。
   请从上面书目中任意选择一部,填入“我读《 》”中的横线上,使之成为一个完整的作文题目,然后以之为题写一篇文章。
   要求:1.可以写成任何文体,比如:可以用记叙文的形式记叙自己跟读这部书有关的故事,可以用读后感的形式谈自己读这部书后的感想,可以用评论的形式对这部书的主题或人物或情节进行品评,等等。2.字数不少于500字。3.如果上面的10部书你都没有读过,选择你读过的其它一部课外书也可以。
   7.(2004·宿迁)
   题目:我渴望
   要求:①先将题目补充完整,写一篇不少于600字的文章;②思想健康,写出真情实感;③文中不得出现真实的地名、校名、人名;④不得抄袭。
   8.(2004·盐城)不管是生活在喧闹的都市,还是在宁静的乡村,每个人对大自然都会怀有浓浓的依恋之情。某一次走进自然的行动,会给你留下难忘的记忆:也许是观赏万顷碧波,也许是留意田间小路,也许是 仰望当空皓月,也许是谛听枝头蝉鸣……自然的美妙让你流连忘返,心醉神迷。
   请以“ 让我陶醉”为题,写一篇文章。
   要求:(1)根据上述提示,在横线上填上相应内容,补全题目。(2)文体不限,不少于600字。(3)文中不得出现真实的地名、校名、人名。(4)不要使用试卷中的阅读材料。
   9.(2004·绍兴)亲爱的同学,当你还在牙牙学语时,你的小脑瓜里已装满了小问号,它们会一个个冒出来:天为什么这么蓝?草为什么这么绿?冬天里小狗没有穿衣服会冷吗?……你也就在这一个个的问号中渐渐地长大。
   问号里有困惑,也有发现;问号里有泪水,也有笑声。
   问号把人类从荒蛮带向文明,问号把人类从陆地带向蓝天,带进太空……
   请以“? ”为题写一篇文章,文体不限。
   要求:①先将题目补写完整。如“?,我成长的伙伴”、“?,让我有了一个新发现”、“?的启示”等等;②文中不要出现真实的地名、校名和人名;③字数不少于500个(诗歌不少于16行)。
   10.(2004·安徽)作为中学生,我们生活在一个幸福的时代,我们拥有知识,拥有青春,拥有激情,然而有时我们又会埋怨命运不公,感叹生活太累。其实,丰富多彩的生活值得我们珍惜的有许多许多……
   请以“珍惜所拥有的___”为题,写一篇文章。
   提示和要求:(1)先将题目补充完整,然后写作。题目的横线上可填“青春、健康、亲情、幸福、荣誉”等词语。(2)可以大胆选择你最能驾驭的文体,写你最熟悉的内容,表达你的真情实感。(3)文中不要出现真实的地名、校名、人名,否则扣分。(4)不得抄袭。(5)不少于500字。
   11.(2004·随州)在我们成长的过程中,有许多值得感谢的人和事、情和景。如养育我们的父母,谆谆教诲的老师,团结友爱的同学,祥和安宁的生活、自然、学习环境,有时候,意外、磨难、不幸因激发了我们向上的动力也让我们心存感激……可以说,懂得感谢是一种生活态度,一种善于发现美并欣赏美的道德情操,也是现代中学生应具备的一项素质。请你以《 感谢》或者《感谢 》为题,写一篇文章(除诗歌外,文体不限)。
   12.(2004·江西)芊芊芦苇,触动了文人的心灵;巍巍“神舟”,触动了国人的心灵;眷眷亲情,触动了游子的心灵;殷殷师恩,触动了学子的心灵……同学们,相信五彩斑斓的大千世界里,一定也会有许多触动你心灵的人或事。
   请以“      ,触动了我的心灵”为题,写一篇文章。
   要求:①根据自己所写文章内容,将题目补充完整;②选择你最能驾驭的文体抒写真情实感;③文中不得出现真实的地名、校名、人名;④不少于600字。
   13.(2004·湛江)一个人在成长的历程中,需要的有时很多,有时很少。有的人需要无限的鼓励、关怀、理解和空间,有的人则只需要一张书桌,一个上学的机会;有的人渴望成长,无惧挫折,有的人则害怕成长所要付出的代价……
   亲爱的同学,你认为成长最需要什么?请结合自己的经历、认识和感受,在下面题目的横线上填入一个你自己认为合适的词语(如宽容、空间、挫折、成功、快乐、付出代价等),然后作文。
   题目:成长,需要___________________
   注意:①除诗歌外,文体不限;②字数不少于600;③文中不要出现真实的校名、人名;④不能抄袭。

第十五讲 材料作文
  
   【试题特点】
   这里的材料作文,指的是文字材料作文。就是出题者给出一份文字材料,要求根据材料或针对材料进行作文。
   在话题作文没有出现之前,材料作文是最为时兴的作文形式之一。
   材料作文的题型特点是:
   ①题目有文字材料。
   ②要求考生依材料作文,或改写材料,或续写材料,或扩写材料,或根据材料写读后感,或针对材料中的“现象”写短评,考查的角度多种多样。
   ③“题”的变化形式多样,可以是命题,可以是自由命题;可以是半命题,也可以是无命题(如要求给给材料的人写信),其中以给一段材料、一个命题的为最多。也有以中考卷中的阅读材料为考场作文材料进行命题的。
   ④文体比较单一,或记叙,或议论,或说明,“不限文体”的写作要求很少出现。
   材料作文考查读写结合能力,考生要阅读、分析、提炼、联想、表达,才能完成写作任务。从考试角度看,由于它能极好地避开师生的猜题押题,又能让所有考生有据而述,有的而议,有感而发,所以是一种优秀的题型,但现在开始受到冷落了。
   【写作指导】
   写材料作文,首先要“读”:
   ①读文题。要研读文题,明确考试范围,了解写作要求。
   ②读材料。粗知材料内容,初步理解含义,以做到心中有底。
   写材料作文,其次要“析”:
   析读材料是必不可少的一步,这个过程需要认真地咀嚼、品味、联想、提炼。
   由于“析材料”是材料作文的关键性的一环,所以可运用如下的读法:
   ①发现法。主要用于意义显豁、中心明确的材料。发现的目标,一是材料的中心句,二是文题中关于中心的提示,三是材料中关于中心的提示。发现了这些内容,就找到了据以立意的突破口。
   ②设想法。主要用于有头无尾或有尾无头的材料,如续写的材料。设想的内容,或是与材料合理衔接的情节,或是与材料合理衔接的论述。设想的过程就是思维的发散,所以要注意发散的合理性。
   ③归纳法。主要用于意义明晰,但既无中心句,也无提示句的一则或多则材料,归纳出其中心意思,立意便有了立足之处。
   ④提炼法。主要用于含义对立的对照型材料。从正反对比中、矛盾对立中、正误相对中提炼出材料所要表达出的意图,以作立意的依据。
   ⑤撷取法。主要用于多段型的材料,从一系列彼此并存的材料中撷取文题所需要的部分内容,并据此清理写作的线索。
   ⑥揣摩法。主要用于喻义型、寓义型、象征型的材料,在反复体味、比较之中品评出材料的喻义、哲理、观点等,然后据此形成自己的观点。
   析读材料之后,就可以根据题目或自己的命题进行作文了,这时就进入了“命题作文”的程序,但要注意,要特别防止如下6个方面的失误:
   ①扩写后的记叙文原貌依旧,手法单调,叙事枯燥,仅仅只是增加了若干字节而已。
   ②改写后的记叙文或由于套用而与原文意思相去甚远,或添加的细节违背生活常理。
   ③议论文在引述材料之后就抛开材料“另起炉灶”,材料没有得到利用或运用。
   ④由于重视了材料而淡漠了文题中的关键词,写出来的文章偏离了所要求表达的中心。
   ⑤由于对材料的理解不深而提炼不出精粹的要点,文章立意过偏、过浅。
   ⑥文体有误、格式有误。
   【解题导引】
   例1.(2004·海淀)
   [题目] “学做人”是青少年成长中至关重要的课程。下面列出的几条人生准则,哪一条引起了你的共鸣,唤起了你的回忆和思考?请你据此写一篇作文,或写出你初中生活中与这一准则相关的经历和感受;或就你对这一准则的理解谈谈自己的看法。
   要接受自己——世间万物都有自己独特的价值, 即使是流星也能划破夜空的沉寂, 即使是一滴水也能折射太阳的光辉。无论怎样,先接受你自己,试着去发现自己的优点,挖掘自身的潜力。
   要欣赏别人——欣赏别人,是善待他人的一种方式,是以人之长补己之短的明智之举。在欣赏别人的同时,这个世界在你眼中也会变得更加美丽。
   要为自己的行为负责——负责任,是一个人最基本的品质。只有当你懂得为自己的所作所为负责,而不是逃避责任或在别人的目光中才勉强承担责任时,你才真正地长大。
   要求:1、自拟题目,除诗歌、戏剧外,文体不限。
   2.将自拟的题目写在答题卡的相应位置
   3.文中不得出现所在学校的名称和人物的真实姓名。
   4.字数600—800。
   [导写] 文题的核心词是“学做人”,也就是说所给的三条人生准则:要接受自己,要欣赏别人,要为自己的行为负责,是从三个不同的角度引发考生对“学做人”的共鸣、回忆和思考,没有明确这点,行文就会产生偏差。
   提示的前三行中特别应该引起重视的是三个“一”字,即“哪一条引起了你的共鸣”,“或写出你初中生活中与这一准则相关的经历和感受”,“或就你对这一准则的理解谈谈自己的看法”,它所要求的是考生须只就一条人生准则写出其经历和感受,或谈谈看法,如果把三条准则一一罗列写出,就不符合文题的要求。此外要求写出“初中生活中”的经历和感受,也明示了选材的范围。
   读懂了提示前三行文字,就把握住了写作的大方向,而后要把分析的目光锁定在对人生准则进行阐释的语句,正确理解“准则”的含义,由此确定自己写作的思路。
   如何理解“要接受自己”呢?流星的稍纵即逝是它的不足,但它以自己璀璨的光芒瞬间照亮了夜空,留下了闪耀而优美的“星迹”;一滴水渺小不足道,但却能折射太阳的光辉,这就是流星、水滴自身拥有的宝贵价值,由此我们拓展写作思路,就是先接受平凡而有不足的自己,再试着去发现自己身上的闪光点,而后发掘自身的潜力,这就是“要接受自己”的过程。选择这一准则入笔,就要写出这一过程,而把这一准则简单地理解为欣赏自己、相信自己、为自己喝彩等,都属于理解上有偏颇,没把握住“接受”二字。
   下面来看看“要欣赏别人”。首先,这“欣赏”要从“我”这个立足点出发,是通过“我”的眼睛,“我”的心,写出自己欣赏别人的经历。而这种欣赏包括三个不同层面的意思:一是善待他人,二是以人之长补己之短,三是欣赏别人的同时,你就拥有了发现美的眼睛,你就会发现世界的美丽。如果在你的文章中只是大谈别人有何优点,别人是如何表现其优点的,就等于踏入了写作偏题的雷区。
   最后来谈谈“要为自己的行为负责”,就是要懂得为自己的所作所为负责;或对自己曾想逃避责任,只是在别人的目光中才勉强承担责任的行为有所认识,明白了勇于负责的重要等等,要写出自己在承担责任的过程中的心路历程,写出自己的体会、感悟,而不能以旁观者的角度着重去写别人要对其自己负责。
   [例文]
   欣赏夏日的清凉
   我欣赏夏日的百花齐放,我欣赏夏日的酷热,我更欣赏那酷热中偶尔飘过的一丝清凉。
   初二时学习成绩上的飞跃使我变成了一个内心骄傲的人,我知道骄傲是种病,是种顽固的病,需要良药来治,世上真的有这种药吗?
   初二后的暑假,我没有像其他人恶补功课,而是轻松地安排着时间。漫步在弥漫着酷热的大街上,心里幻想着吹空调的清爽。突然,我注意到了自己“满面尘灰烟火色”的皮鞋,皱了皱眉头,径直走进了家擦鞋的小店。
   怎么是她!我有些惊讶,本来是可以走开的,然而此时她也看见了我。四目相对,她的眼神中也写着惊讶,但显然比我的小得多。
   她是采莲,我的同班同学,我绝没想到我俩会在这里相遇,我只得一边走过去坐下,一边思考着如何调节这种尴尬的气氛。如果此时是我坐在对面,我甚至没有拿起鞋刷的勇气。采莲已经准备好了,她先给鞋洗尘,接着擦掉水渍……空气里弥漫着燥热,大滴大滴的汗珠从我身上涌出。采莲倒是很平静,她一边擦着,一边讲着自己为什么来擦鞋,她说母亲下岗了,父亲查出是乙肝,家里一下拮据起来……
   我甚至到最后也没想出如何调节尴尬的气氛,只得默默地听着。采莲讲得很平静,没有抱怨,没有伤心,甚至没有一丝预想的窘态。汗水不再涌了,我第一次发现,原来苦难是可以如此平静地对待。突然,发现自己对采莲有种不敢直视的感觉,从她那平静的诉说中,我欣赏到了什么是大气,什么是不卑不亢,什么是处变不惊,什么是直面困难。顿时,一股清凉的感觉传遍了全身,我似乎找到了治疗骄傲的良药。那个暑假过后,许多认识我的人都说我变了,具体变在哪却也说不出。只有我知道,每当我取得了好的成绩,那颗虚荣的心飘起的时候,我就会想起采莲,想起她边擦皮鞋边诉说时的平静,那是真正的平静,在她那令人欣赏的平静与大气面前,我这点成绩简直不值一提!
   我终于找到了治疗骄傲的良药,那就是欣赏他人身上自己不具备的优点,找到自己达不到的高度,那颗虚荣的心自然会落地,也就骄傲不起来了。
   那个夏天,飘过一丝令人神往的清凉。
   [点评] 伴着“欣赏夏日的清凉”这一新颖、优美的文题,我们被小作者带入了那家擦鞋的小店,邂逅了因家庭经济拮据而出来擦鞋的采莲,采莲对自身境遇平静的诉说,给了“我”极大的触动,“原来苦难是可以如此平静地对待”。“我”欣赏采莲直面困难的平静与大气,而正是这种欣赏,成了医治“我”骄傲之心的良药,并令“我”有了极大的转变,从中学会了做人。
   本文以“一丝清凉”之感贯穿全篇,增添了文章动人的抒情韵味,也使阅卷老师的心间掠过一袭清凉。
   【新题导写】
   1.根据提供的材料,进行合理的想象,写一篇文章。
   晚上,一家人又为看电视发生了矛盾。爸爸要看运动会,奶奶要看《还珠格格》,妈妈要看《牵手》,我要看《笑傲江湖》。于是,家里出现了这样一幕……
   [点拨] 这是根据提供的材料写想象作文,它展示的是学生的创造思维。下面提供一种创新的构思。晚上,爸妈走亲戚未回,奶奶在隔壁聊天,“我”可独享《笑傲江湖》了。哪知奶奶回来要看小燕子,“我”怎么也不同意。正闹着,爸妈推门而入,爸一进门就嚷着看足球赛,妈一坐下就喊着夏小雪,频道争来抢去,屏幕闪来晃去,谁都在看,谁也没看清什么。“我”不满了,决定耍蛮锁定令狐冲,突然,停电了!于是,一家人都在等电来,耐心或不耐心地等,可电就是不来。
   2.读下面的材料,自拟题目,写一篇文章,文体不限。
   报载,某地有一美食城,设置了“小皇帝宝座”。每个包厢一桌三椅。孩子高坐在“小皇帝宝座”上,父母则坐于下首,充当左右“丞相”,小心侍候。该店表示,这种特色经营项目,今后还将加大广告力度,隆重推出。
   [点拨] 这则材料可以写记叙文,也可以写议论文。如果写记叙文,要在描写和叙述的字里行间,“渗透”作者的情感,还要通过精妙的议论文字,点示出文章的主旨。如果写议论文,立意的角度很多,比如,可从“小皇帝”身上着手,可从“小皇帝”父母身上落笔,还可以从美食城老板身上着墨等。无论从哪一角度入手,都要在主题的深刻上多动脑筋,力争写出文章的穿透力。
   3.阅读下面的材料,自拟题目,写一篇文章,文体不限。
   据报载,美国有个富翁规定还在读小学五年级的儿子,每周必须谋生一次。每当那天早晨,富翁亲自驾车几十公里,把儿子送到闹市区,赶儿子下车,令其自想办法谋生,傍晚他再驾车接回儿子。尽管往返所耗油费远远超过儿子一天的生活费,但富翁却乐此不疲。
   [点拨] 题目中有关文体的要求是“文体不限”,但只要认真审读上述材料,就会发现其实“文体有限”,此题较适于写记叙文或议论文,绝不可能写成说明文。无论写记叙文还是议论文,都应认真读懂材料,充分利用材料。若写记叙文,就应以材料提供的内容为骨架,补充具体的时间、地点、次要人物,进行合情合理的想象,写成一个有头有尾、有声有色、可触可感的故事。若写成议论文,则可从不同角度提炼出不同的论点。
   【考点精练】
   1.(2004·海淀)“学做人”是青少年成长中至关重要的课程。下面列出的几条人生准则,哪一条引起了你的共鸣,唤起了你的回忆和思考?请你据此写一篇作文,或写出你初中生活中与这一准则相关的经历和感受;或就你对这一准则的理解谈谈自己的看法。
   要接受自己——世间万物都有自己独特的价值,即使是流星也能划破夜空的沉寂,即使是一滴水也能折射太阳的光辉。无论怎样,先接受你自己,试着去发挥自己的优点,挖掘自己的潜力。
   要欣赏别人——欣赏别人,是善待他人的一种方式,是以人之长补己之短的明智之举。在欣赏别人的同时,这个世界在你眼中也会变得更加美丽。
   要为自己的行为负责——负责任,是一个人最基本的品质。只有当你懂得为自己的所作所为负责,而不是逃避责任或在别人的目光中才勉强承担责任时,你才真正地长大。
   要求:1.自拟题目,除诗歌、戏剧外,文体不限。2.将自拟的题目写在答题卡的相应位置上。3.文中不得出现所在学校的名称和人物的真实姓名。4.字数600~800。
   2.(2004·烟台)度过了人生的十五六个春秋,回顾过去,你有后悔的事吗?你从中悟出了什么道理?请围绕着“悔与悟”写一篇文章。题目自拟,文体不限(诗歌除外),600字左右。文中不得出现真实的人名、校名。
   3.(2004·台州)以下面作文格中开头处和结尾处提供的基本情况为依据进行联想和想象,然后按要求作文。
   要求:①以“赵君到李云家发现了什么”、“后来赵君为此做了些什么”为基本思路进行联想和想像。②将所联想想象的内容,包括基本事件(情节)和有关的环境、细节、心理等,写在开头和结尾两处之间的空格上(不一定正好写满空格),使其与开头、结尾两段文字衔接、贯通,融为一体。③自拟题目,写在文前的横线上。④600字左右。
     题目:____________
     (开头)赵君和李云是初三·一班的尖子学生。离期终考试还有一个来月,赵君觉察到李云经常迟到、请假,整天没精打采的,学习成绩明显下滑。
      一天,李云又请假了,赵君放了学就到李云家去探访。
     (结尾)不迟到、不请假,活泼灵秀的李云又回来了。期终考试的前两名还是她(他)俩。
   4.(2004·宜昌)
   学校文学社将开展以“夜之韵”为主题的语文学习活动,上边方框内的文字是这次活动的宣传语。在这个活动中,同学们可以展示语文积累,可以开展阅读欣赏,可以进行文学创作,可以组织艺术交流,……那一定会是一段快乐而又充实的语文学习经历。请你设想一个或一组你和同学们参加“夜之韵”主题活动时的精彩片段,把它记录下来。题目自己拟,字数在500个左右,文中不要出现真实的人名、校名和文学社名。
   提示:可以从某个角度也可以从多个角度来品味夜的韵味。注意引用资料要适量。
   5.(2004·遂宁)阅读下面这段文字,按要求作文。
   (一)一群蛤蟆在进行竞赛,看谁先到达一座高塔的顶端。周围有一大群围观的蛤蟆在看热闹。竞赛开始了,只听到围观者一片嘘声:“你们无法到达塔顶的!”
   其它的蛤蟆都被说服停下来了,只有一只蛤蟆一如既往继续向前,并且更加努力。
   比赛结束,其它蛤蟆都半途而废,只有那只蛤蟆以令人不解的毅力,一直坚持了下来,竭尽全力到达了塔顶。
   其它蛤蟆都很好奇,纷纷向他讨教成功的秘诀,结果大家才发现—它是一只聋蛤蟆!
   (二)丘吉尔在牛津大学的演讲:“我的成功秘诀有三个:第一是决不放弃。第二是决不放弃,决不放弃。第三是决不,决不,决不放弃。“成功的秘诀很多,请以“成功的秘诀”为话题,联系自己或社会生活实际写一篇文章。
   要求:1、自选角度,自拟标题,自定文体(诗歌除外)。2、书写规范工整,字数不少于600字。3、文中不得出现真实的人名、校名、地名(可用××表示)。
   6.(2004·兰州)
   上课时,小雨向小欣借橡皮,王老师误认为两人搞小动作,不专心听讲,当众点名批评了两人。下面是两位同学受到批评后的两种处理方式?
   方式一:小欣受到批评后,怒火中烧,他认为老师冤枉好人,使自己当众受辱。因而,很不服气地斜瞥了老师一眼。老师看到后也很生气,冲突趋向白热化……
   方式二:小雨受到批评后,心中非常委屈,但他并未辩驳,而是等到下课后跟随老师到办公室,为惹老师生气道歉并诚恳地向老师说明情况。老师听后,真诚的承认自己也有错误,并在下一节课上公开向小雨和小欣表示道歉。
   读了以上文段后,或许能勾起你与老师相处中的回忆,或许引发你对此想说而没有说出的心里话……请根据自己的感悟,自选角度作文。
   要求:①题目自拟。②要有真情实感,思想要健康,努力做到有新意、有创见。③除诗歌外,文体不限。④不少于600字。⑤文中不得出现真实的地名、校名、人名。
   7.(2004·河北实验区)阅读下面的材料,根据要求写一篇文章。
   初三时,第一天上生理卫生课,老师就在教室后面的墙上挂了一幅人体解剖图,图上标明了重要的骨骼、肌肉的名称和部位。整整一个学期,那幅图都挂在那里。
   临近期末的一次考试,我们一走进教室,那幅解剖图便被老师收起来了,而整张试卷也只有一道试题:列举人体主要骨骼的名称和部位。
   我们几乎异口同声地提出抗议:“老师没有讲过,我们也从来没有学过。”
   “这不是理由。”老师微笑着说,“那些知识已经挂在墙上好几个月了。”我们勉强答了一会儿,便草草交了卷。
   “请同学们永远记住,”老师忠告我们,“学习不仅仅是学别人告诉你的东西。”
   要求:①根据你对材料的理解,自选角度,自拟题目,写一篇不少于600字的文章。②可以记叙经历,抒发感情,发表见解……但不要对材料扩写、续写改写。③除诗歌、戏剧外,文体不限。④文章中不要出现真实的地名、校名、人名。

 选题作文
  
  
   命题者出二至三个作文题,由考生任选一个进行作文,这样的作文形式叫选题作文
   选题作文如同文摘类期刊一样,不能独立生存,因此,必须借助其他命题方式“合成”。具体来说,命题作文、半命题作文、材料作文和话题作文等非选题作文已经成为选题作文的备选项。
     一是“全命题+全命题”作文组合,如广东省广州市(“我也是富翁”和“假如再有一次机会”)、山西省太原市(“第一次亲密接触”、“痛并快乐着”和“梦里花落知多少”)等。
     二是“全命题+话题”作文组合,如湖北省恩施市(“风雨过后是彩虹”和话题“我们的生活需要绿”)等。
     三是“半命题+半命题”作文组合,如湖北省随州市(“____感谢”和“感谢____”)等。
     四是“半命题+话题”作文组合,如河南省(“我找回了____”和话题“信念与成功”)、湖南省长沙市(“我美丽,因为我____”和话题“我们年少,我们需要”)、福建省宁德市(“夸我____”和话题“夸奖”)、浙江省绍兴市(“,我成长的伙伴”和话题“?”)、新疆乌鲁木齐(“  ,原来是这样的”和话题“诺言”)、青海省西宁市(“这里有____”和话题“动物也是我们的好朋友”)等。
     五是“半命题+材料”作文组合,如吉林省(“我的____”和根据重模仿与重创造的两个书法家的故事自选角度写作)等。
     六是“话题+话题”作文组合,如湖北省襄樊市(“真诚”和“卸下负担”)、山东省淄博市(“我就是一道风景”或“你就是一道风景”)、浙江省宁波市(“小”或“小和大”)、北京市海淀区(“接受自己”、“欣赏别人”和“负责任”)、北京市东城区(“成功”、“失败”、“诱惑”和“机遇”)等。
     七是“话题+材料”作文组合,如黑龙江省(话题“如何做人”和根据关于理想、美德、自信的材料写作)、黑龙江省哈尔滨市(话题“超越”和根据一则写科幻小说家凡尔纳成功原因的材料写作)等。
     写法
     考生怎样才能提高选题作文的写作能力?笔者认为以下几点值得注意:
     广泛涉猎各种形式的作文题目。话题作文、全命题作文、半命题作文、材料作文等形式是选题作文的基本组成部分,如果平时练习到位,熟悉它们的不同写法,考场上就能从容应对选题作文。
     迅速确定写作题目。一般来说,选题作文的文字量较非选题作文更大,对考生的干扰也更大一游离于备选题中举棋不定就会浪费时间。因此,在较短的时间里确定写作试题,能为自己的考试赢得主动。挑选的原则应该是选熟不选生、选易不选难。在几种类型的作文中,首先是话题作文,其次是半命题作文、全命题作文和材料作文。
  
     找出展示才情的切入点。选题作文的一大优点,就是能够提供多维切入点来考查考生的写作才能。找出恰当的切入点,在记叙、议论或抒情中层示自己编故事、善描写、会剖析和抒真情等多方面的优势,就能写出个性十足、亮点频现的优秀作文。
   典型试题
   2004年黑龙江题
   题目:
   (1)理想篇 生活的理想,是为了理想的生活,理想是指路明灯。朋友,你的理想是什么?你想怎样去实现?
   (2)美德篇 美是体魄之德,德是灵魂之美。一个人可以没有金钱、地位和名利,但是绝对不可以没有品德、修养和情操,美德是永不凋谢的花朵。
   (3)自信篇 自信是成功的第一要诀,自信和希望是青年的特权。不是因为有些事情难以做到,才失去自信,而是因为失去自信,有些事情才难以做到,自信的价值将无法计算。
   文题一:初中毕业是人生的新起点,联系上述材料,请以“如何做人”为话题,写篇作文。
   文题二:可选用上述一篇为材料,也可用上述几篇为材料,写篇作文。
   从以上两个文题中任选其一,题目自拟,文体不限(诗歌、戏剧除外),文中不要有真实校名、姓名,字数600字以上。
   快拿起笔来吧,只要把真情实感流露,那将是一篇优秀宣言。
   写作导引
   今年黑龙江省的中考作文试题,继续采用的是选题作文这种形式,所提供的文题由去年的三个减为今年的两个。写好第一个文题的难度是可想而知的,因为初中生的认识水平还很有限,当面对“如何做人”这样一个大而无边的话题时,许多人会觉得无话可说,即使勉强去写,也往往谈不深、论不透,或者写不出什么新意来。相对而言,第二个文题就容易得多,尤其是只“选用上述一篇为材料”时,更容易下手,因为“理想”、“美德”、“自信”都是我们非常熟悉的词汇,它们几乎就挂在我们的嘴边,我们不仅对它们的含义有着清楚的了解,我们还对历史上一些人的“理想”、“美德”、“自信”有着清楚的了解,对身边人的“理想”、“美德”、“自信”有着清楚的了解,而这些,正是我们写好作文的基础。因此,从“写什么”的角度看,建议大家选择文题二来写,而且只“选用上述一篇为材料”来写。当然,如果你有实力,你觉得自己有把握写好文题一,或者觉得自己能够综合运用好所给的三段材料,你完全可以做出自己的选择。从文章的体裁上说,写成议论文可以,写成记叙文也可以;写一个虚拟的戏剧场景可以,写一个辩论会、“美德拍卖会”等也可以。在处理“怎样写”的问题时要特别考虑到“出新”,人云亦云的文章是不可能得高分的。
   优秀作文一
   我的未来不是梦
   “我的未来不是梦,我认真地过着每一分钟……”每当听到张雨生唱这首《我的未来不是梦》时,心中的理想之花就会绽放开来。
   当一名人民教师,是我的远大理想。
   很小的时候,由于爸爸经常出差,妈妈整天在外上班,家中只剩下我孤孤单单的一个人。再大一点的时候,妈妈总是把我带到学校,每当看着妈妈和那些同学谈心说笑,互相沟通、了解的时候,我心中油然而生一种敬意。于是,当一名优秀的人民教师这一远大的理想在我心里埋下了深深的根。
   后来,我上学了,在妈妈的督促下,我努力而认真地学习着。我从不因遇到挫折而退缩,而是勇往直前,继续拼搏;我从不因遇到一点困难而倒下,而是克服困难,战胜自我,重新面对新的世界,新的一天;我从不因遇到难题而把笔一摔,放弃做题,而是深思苦解,直到最后得出一个满意的结果,让自己尝到了成功的喜悦与欢乐的滋味。
   我知道,理想的实现是伴随着辛勤和汗水的,为了实现自己的远大理想,五年苦苦煎熬的小学生活过去了,我的成绩名列前茅,我得到了老师的喜爱、同学的拥护。
   时间如水,生命如歌。步入中学大门之后,我仍旧努力学习,刻苦读书,因为我知道;我要为实现自己的理想而奋斗。
   那是初三时的一节外语课,外语老师别出心裁地提出了让同学来讲课的主意,我非常高兴,而且跃跃欲试,因为锻炼的机会终于来了!
   我迈着轻快的步伐走上讲台,望着那一张张熟悉的面孔,开始我还有些紧张,当我的脑海里浮现出妈妈与她的学生亲切交谈、沟通的情景时,我的心情变得轻松起来。我也像妈妈那样与三年来朝夕相处的同学们进行了心与心的沟通。45分钟很快就过去了,当我走下讲台的时候,我赢得了同学们一片热烈的掌声。望着同学们那羡慕、崇拜的眼神,听着老师那表扬的话语,我感到自己特别幸福。
   与此同时,我对自己理想的实现更有了自信,充满了希望.我对自己说:加油!努力!因为我知道,我的未来不是梦。
   点评:
   这是一篇谈理想的考场佳作.作者的理想是像“妈妈”那样当一名光荣的人民教师。文章的优点很多:一是感情真挚自然。在实现理想的过程中.作者有过许多的体验,而这种种体验又都在文章中有所体现,它们或热烈奔放,或含蓄内敛,但所有情感都是真实的、自然的,没有半点的矫揉造作。二是内容丰富。作者不仅交代了自己理想的产生,还具体叙述了自己为了实现“远大的理想”而付出的辛勤和汗水,时间跨度达十几年。三是重点突出,详略得当。初三时那次“走上讲台”是作者刻意描写的重点。四是语言的驾驭能力强。无论是从成语、俗语的使用方面看,还是从修辞方法的使用、句式的选择等方面看,小作者都具有很强的驾驭语言的能力。
   优秀作文二
   宽容是美德
   宽容是我们中华民族的传统美德之一,一个人要想在社会生活中有一席之地,你首先必须学会宽容。
   宽容就像一盏明灯,在黑暗中放射着万丈光芒,照亮了善意的心扉;宽容又像立足社会的通行证,让人懂得什么是真诚。翻开人类历史的长卷,你就能清楚地看到,许多的伟人都是持着宽容这张通行证而立足于社会的。且不去讲那个大家早就熟悉了的“将相和”的故事,也不去说美国的第一位总统是如何宽容他的敌人,让国家走向大治的,就说说我们敬爱的周总理吧,他可是我们所有中华儿女的榜样呀。在周总理的身上,值得我们学习的
   美德极多,而宽容就是其中之一。
   一次,周总理去理发店理发,在理发的过程中,他突然咳嗽了一下,而不巧的是理发师刮坏了总理的耳朵,理发师很着急,也很害怕,连说对不起,一时竟不知道该怎么做才好了。周总理见此情景后说:“这不能怪你,因为在我咳嗽前并没有告诉你呀。”总理的话,没有半点埋怨和责怪,让理发师听了如释重负。试想,如果当时两个人都互不相让,一个摆开总理的架子,另一个想方设法把责任推向对方,那结果一定会很糟糕,对彼此的影响都不会好。
   两个人之间能够做到彼此宽容,带来的是两个人之间关系的和谐;所有社会成员之间能够做到彼此宽容,带来的是整个社会的和平与稳定。也许你要说,这也宽容,那也宽容,岂不是没有原则的一团和气了吗?这话是有道理的,在原则问题上,在大是大非面前,我们不仅不能随便宽容,而且还要针锋相对、寸土必争。
   人人都渴望生活的稳定、社会的和平.那么,就让我们多一份理解,多一份宽容吧!对别人无意中的过错和冒犯不要太计较、太在意,因为谁都有犯错误的时候,人无完人嘛。“忍一忍风平浪静,退一步海阔天空”,宽容是一种美德,谁能把宽容记在心里,谁就能把浩瀚的海洋装进胸膛,即使再小的帆也能远航,不是吗?
   点评:
   这是一篇谈美德的考场佳作,作者所谈的美德是“宽容”。这篇文章具有这样一些特点:一是论题选择得合适。大家知道,关德有许多种,宽容不仅是其中之一,而且是一个“大众化”的论题,很适合中学生谈论,很对中学生的“口味”。二是论点突出、鲜明。作者不仅开门见山提出了论点,还在结尾的时候进行了重申。三是论据典型、充分,且与论点有着密切的联系。以第二自然段中的三个事实论据为例,虽然在叙述时有详略之分,但却来自古今中外,有着很强的代表性。四是论证过程严密,论证方法灵活多样。第四自然段后半部分的对比论证就写得很好。五是语言极其个性化色彩。对考生来说,选择汉论文体来写是一种勇气,能够把议论文写得这样好是一种水平,看得出,小作者具有很强的论辩能力。
  
   1.(2004·上海)
   从下列两题中任选一题,写一篇600字左右的文章。
   (1)我们是初升的太阳;我们是风华下正茂的中学生.
   校园里我们生机勃勃;校园外,我们热情奔放……
   以”我们是初升的太阳”为题作文.
   (2)以”我的视线”为题作文.
   要求:(1)文中不能出现考生本人有关的校各和姓名,若不可避免,用代号表示,如A中学、B老师、小C。(2)不得使用试卷中的阅读材料。
   2.(2004·吉林)
   以下两个文题任选其一。
   (1)题目:我的
   在题目的横线上填上适当的词(如心愿、乐园、最爱、财富、老师、未来、座右铭、
   课余生活等等),使题目完整。
   (2)根据你对以下材料的理解,自选角度,自拟题目,写一篇作文。
   有这样一个故事:清代乾隆年间,有两个书法家,一个极认真地模仿古人,讲究每一
   笔每一划都酷似某某,如某一横要像苏东坡的,某一捺要像李太白的。另一个则正好相反,不仅苦苦地练,还要求每一笔每一划都不同于古人,讲究自然。有一天,第一个书法家嘲讽第二个书法家,说:“请问仁兄,您的字有哪一笔是古人的?”后一个并不生气,而是笑眯眯地反问一句:“也请问仁兄一句,您的字,究竟哪一笔是您自己的?”第一个听了,顿时张口结舌。
   齐白石先生说:“学我者生,似我者死。”走不出前人的框架,自然也就不会有自己的
   天地。
   作文要求:①选择你最擅长的文体抒写真情实感。②不少于500字。③文中出现的地
   名、校名、人名,请用“××”代替。
   3.(2004·山西)
   以下两题任选一道
   (1)常言道:播种一种心态,收获一种思想;播种一种心习惯,收获一种命运。请你以“播种”为话题,写一篇作文。
   要求:①题目自拟;②文体不限;③不少于600字(如写诗歌不得少于12行);④文中不得出现真实的人名、校名、地名。
   (2)题目:你准备好了吗
   要求:①文体不限;②不少于600字(如写诗歌不得少于12行);③文中不得出现真实的人名、校名、地名。
   4.(2004·青岛)
   请从下面两题中任选一题作文
   (1)题目:老师,你______________
   初中三年,你与老师朝夕相处,对老师或心存感激,由衷敬佩,或由于误解,产生隔阂……请依据上述提示(不限于此范围),结合自己的真切感受,将题目补充完整,写一篇600字左右的文章。文中不能出现真实的校名和人名。
   (2)题目:这事真让我______________
   在日常学习和生活中难免会发生一些事情,这些事情或让你高兴、欣慰,或让你伤心、沮丧,或让你失望、后悔……请依据上述提示(不限于此范围),结合自己的切身体验,将题目补充完整,写一篇600字左右的文章。文中不能出现真实的校名和人名。
   5.(2004·广州)
   从下面两个题目中,任选一题写作。
   ①请以“假如再有一次机会”为题目,写一篇文章。
   ②国外一位年老的亿万富翁望着朝气蓬勃的青年人说:“真是羡慕你们,如果可能,我愿意用所有的财富买回青春!”是啊,年轻是世界上最令人羡慕、最有价值的财富。其实,除年龄外,你还拥有许多许多,如亲情、友谊、健康、毅力、知识、活力等等,只要细心体味,你就会发现自己是很富有的。
   请以“我也是富翁”为题目,写一篇文章。
   要求:①文体自选。②600字以上。③文中不能出现考生姓名和所在学校名称。
   6.(2004·青海)
   请在以下二个题中任选一题作文。
   ①藏在我心中的秘密
   ②社会是一所学校,生活是我们的良师。在受到挫折时,我们学会了坚强;在不利的环境中,我们学会了生存;在怒火胸中燃烧时,我们学会了忍耐;在受到猜忌时,我们学会了理解;在别人伤害了自己时,我们学会了宽容;在别人遇到困难时,我们学会了关心……
   请以“学会 ”为话题,写一篇文章。
   要求:①有真情实感、内容具体、中心明确、语言通顺。②除诗歌外,文体不限。③字数不少于600字。④凡涉及人名、校名、地名一律用×××代替。
   7.(2004·西宁)
   请从下面两个题中任选一题作文。
   (1)这里有 (如亲情、信任、希望、欢乐、竞争、春天、阳光、丰富的宝藏等)
   提示:括号内所提供的词语可选用,也可不用;务先将题目补充完整,然后作文。
   (2)蝴蝶、猫、斑羚、威风凛凛的狮虎……它们或小或大,但也有着与我们一样的快乐和痛苦,一样平等而又美丽的生命。当我们人类在无辜地伤害它们的生命的同时,实际上也在伤害我们自己。请以“动物也是我们的好朋友”为话题,写一篇作文。
   提示:题目另定,立意角度自选。
   要求:①情感真实,符合题意,中心明确,内容充实,语言通畅,书写工整。②除诗歌外,文体不限。③不少于600字。
   8.(2004·甘肃)
   下列两题任选一题。
   (1)“头顶一片天,脚踏一方土。”有谁离得开大地母亲,离得开自己脚下的这片土地?女娲造人的神话说明了祖先对土地的崇拜;开疆拓土是古代许多统治者成就霸业的目标;而保卫土地的故事更是不计其数。如今,土地又被赋予了环境保护的意义。土地,哺育了人类,续写着辉煌。请以“土地”为话题,写一篇700字左右的文章。
   (2)成长的路上,总有许许多多的人、事、物给了你生活的启迪、感情的滋润……这些人、事、物深深地烙在了我们的记忆深处。请以“永远的 ”为题,写一篇700字左右的文章。(注意:根据你作文的内容在横线上填写恰当的词或短语,补全题目;文章应有一些抒情色彩。)
   9.(2004·宁安实验区)
   从下面的二个文题中任选一个,写一篇作文。
   文题一:上网热、追星热、开车热、旅游热、读书热、考研热……“热”作为一种社会现象,也许你目睹过、参与过,也许你迷失过、追寻过……无论怎样,“热”都体现了你对生活的关注和热情,请以“热”为话题,抒写你的经历或思考。
   文题二:一段旅程的结束意味着新的开始,一次小小的转折也同样意味着新的开始。心情转变是开始,误会消除也是开始……在迈向开始的瞬间,审视已结束的失意或成功,畅想即将到来的精彩或挑战,都是幸福的,愉快的。请以“开始”为话题作文。[提示:标题可以出现“开始”,如:“他开始唱歌了”,“美丽的开始”;也可以不出现“开始”,如:“起点”、“出发”。如果你喜欢,这几个题目可以选用,有自己的创意更好!]
   要求:①题目自拟,文体不限。②诗歌不少于15行,其他文体不少于500字。③作文中请不要出现真实的姓名、校名。
   10.(2004·海口实验区)
   下面两道作文题,任选一题,按要求作文。(文中不得出现真实的校名、地名、人名)(1)小草渴望雨露,花儿渴望绽放,雄鹰渴望蓝天,骏马渴望驰骋,溪流渴望江海……
   请以“我渴望”为话题,自选角度,自定文体(诗歌亦可,不少于20行),自拟题目,写一篇600字左右的文章。
   (2)面对失败和挫折,一笑而过是一种乐观自信,然后重整旗鼓,这是一种勇气。
   面对误解和仇恨,一笑而过是一种坦然宽容,然后保持本色,这是一种达观。
   面对赞扬和激励,一笑而过是一种谦虚清醒,然后不断进取,这是一种力量。
   面对烦恼和忧愁,一笑而过是一种平和释然,然后努力化解,这是一种境界。
   这段话对你有什么启发?能引发你怎样的联想?请依据材料中的一个关键词、一句话或几句话,结合你的生活经历和感受,写一篇600字左右的文章。文体自定(诗歌亦可,不少于20行),题目自拟。
   11.(2004·四省区实验区)
   从下面两个题目中,任选一题作文。
   要求:①文体不限;②字数在500~800之间;③不要出现真实的人名、地名和校名。
   (1)在岁月的河流中,每个人都有过数不清的难忘的经历:欢乐的、痛苦的;悲壮的、欣喜的;成功的、失败的……然而,所有的经历都如一级级向上的阶梯,使我们一步步地从稚嫩走向成熟。从这个意义上说,经历是人生的巨大财富,是十分宝贵并值得珍惜的。
   请以“经历”为话题,自拟题目,写一篇文章。
   (2)善待 (亲人、生命、自己……)
   请根据括号中的提示补全题目,写一篇文章。
   12.(2004·郫县实验区)
   从下面两题中任选一题作文。
   ⑴三年来,同学们愉快地走过了人生的一段美好时光,留下了美好的回忆。大家都说,伴随着课标教材,我们成长起来了。是的,几年来,同学们在体验中成长,在合作中成长,在成长过程中,都有自己的经历和体会。
   请以“在 中成长”为题,写一篇作文。
   要求:①在题目的横线上填上恰当的词语(如沟通、鼓励、合作……),使题目完整。②文体不限。③文章应写出真情实感。④文中不能出现真实的人名、地名。⑤不少于600字。⑥不得抄袭。
   ⑵有一个小男孩不小心踩了邻居新铺的小泥路面,将来整光滑的水泥地印上了几个清晰的脚印。小男孩十分害怕邻居伯伯知道,赶紧回家把新鞋子冲洗干净。但他心里一直不安,终于决定告诉邻居伯伯。邻居伯伯看了现场后,拍拍他的肩膀说:“好孩子,很高兴你告诉我这件事,现在我还能够修补它。假如你不马上告诉我,等水泥干掉以后,脚印就无法修补了。”
   生活中类似的情况还很多。请认真阅读上面这段文字,以“修补”为话题,写一篇作文。
   要求:①题目自拟。②文体不限。③可以用这则材料,也可以不用这则材料;可以写自己的亲身经历体会,也可以写别人。总之,要围绕“修补”这一话题作文。④文中不能出现真实的人名、地名。⑤不少于600字。⑥不得抄袭。
   13.(2004·湟中实验区)
   请在下面两个题中任选一题作文。
   (1)以“我好想 ”为题,写一篇文章。
   要求:先把题目补充完整,如“有个知心朋友”“有个好老师”“有自己的小天地”“快快长大”“再看一眼”或“美梦成真”“飞上月球”等。
   (2)美国作家威尔逊说:“理解无疑是培育一切友情之果的土壤。”在生活中,理解是沟通心灵的桥梁,是相互信任的基石,是消除隔阂的良药……
   请以“理解”为话题,自拟题目,写一篇文章
   要求:①有真情实感,内容具体,中心明确,语言通顺。②文体不限,不少于600字。若写诗歌不少于20行。
   14.(2004·大连实验区)
   任选一题作文。
   (1)从前乡下有些地方设有凉亭,供路人休息。有些好心人常常煮桶茶放在那儿,让过路人解渴。有一位老人怕赶路的人喝水急而呛着,于是在茶水里撒了一点点谷皮,这样,喝水的人就必须把谷皮轻轻吹开,然后慢慢地喝。
   每个人都会从这则材料中获得启示,也会引起一些回忆,请你以自己的启示、回忆为内容,写一篇600字左右的文章。所给材料可用可不用,但所写内容必须是从这则材料中获得的启示或回忆。
   要求:①立意自定,文体自选,题目自拟;②文中不得出现生活中的人名、校名,如需要,可用A、B、C等字母代替。
   (2)题目:讲给的事儿(爸爸、妈妈、老师、同学……)
   要求:①根据所写文章的内容,将题目补充完整;②要写出真情实感;③字数在600字左右;④文中不得出现生活中的人名、校名,如需要,可用A、B、C等字母代替。
   15.(2004·北碚实验区)
   从下面两个题目中任选一题。
   (1)愧疚,《现代汉语词典》的解释是:惭愧不安。愧疚,可以让我们“三省吾身”,净化自己的灵魂;愧疚,可以让我们对朋友多一份真诚,少一份虚伪;愧疚,可以让社会多一分良知,多一分责任,多一分美好。冯骥才说:“纯洁的人生从忏悔开始。”在我们的经历中,总有那么一些为之愧疚的人,为之愧疚的事。请以“愧疚”为话题,自拟题目作文。
   (2)体验
   在横线上填上适当的词语(如亲情、综合性学习、合作、探究、成功……),将题目补充完整,然后作文。
   要求:①文体自定。②600字左右。③文中不得出现真实的师生姓名和学校名。

05中考模拟题(2)
   一、积累和运用(共20分)
   1.根据拼音写汉字。(2分)
   楚辞汉赋唐诗宋词……语感错落有致,语调起伏跌dàng( ),或壮丽豪迈气势磅bó( ),或清丽wǎn( )约一咏三叹。真草隶篆行,异彩纷呈,这些灿烂的篇章和飞扬的文采,zhàn( )放出中国文化独特的艺术魅力和韵味。
   (摘自《人民日报》2004.4.20《请尊重我们的母语》)
   2.按要求填写。(5分)
   ①《送董邵南游河北序》中体现作者政治思想的一句是 , 。
   ②《白雪歌送武判官归京》中将诗人因朋友离去而产生的无限惆怅之情抒写到了极致的诗句是 , 。
   ③《天净沙·秋思》中作者直抒胸臆,道出天涯游子之悲的句子是 。
   ④《醉花阴·薄雾浓云》中的千古名句是 , 。
   ⑤《别云间》中表达坚强不屈,宁死不改抗清志的诗句是 ,
   。
   3.对下面这首元曲文句的解说,错误的一项是(  )(2分)
   峰峦如聚,波涛如怒,山河表里潼关路。望西都,意踌蹰。伤心秦汉经行处,宫阙万间都做了土。兴,百姓苦;亡,百姓苦!
   A “山河表里”,指潼关外有黄河,内有华山,地势险要。表里,外内之意。
   B “望西都”即遥望长安;“意踌蹰”指步履沉重,不忍前行。
   C “伤心秦汉经行处”说秦皇汉武当年游幸的处所,如今是这样的凄冷,使人感到伤心。“经行处”即行经处。
   D “宫阙万间都做了土”,说诗人经过秦汉故地,看见原来成群的宫殿都化为尘土,成了一片废墟。
   4.用一句话提炼出下面语段所透露出来的主要信息。(2分)
   绿色,代表生命,代表健康和活力,是充满希望的颜色。国际上对“绿色”的理解通常包括生命、节能、环保三个方面。为了使人们较系统的了解,一些环保专家把绿色消费概括为5R,即节约资源,减少污染;绿色生活,环保选购;重复使用,多次利用;分类回收,循环再生;保护自然,万物生存这五个方面。具体的说,“绿色消费”有三层含义,一是倡导消费者在消费时选择未被污染和有助于公众健康的绿色产品。二是在消费过程中注重对垃圾的处理,不造成环境污染;三是引导消费者转变消费观念,崇尚自然,追求健康,注重环保,节约资源和能源,实现可持续消费。
   答:
   6.请研究下图,写出你的探究结果。(3分)
  
  
   答:
  
   7.名著阅读。(2分)
   “黑旋风”是我国古代文学名著《水浒传 》中 的绰号;“醉打 ”、“血溅鸳鸯楼”等情节说的是这部名著中另一位英雄人物武松的故事。
   8.仿写。(3分)
   仔细阅读下面三段文字,按照其内容和形式仿写一段,不超过40字。
   操劳了一辈子的母亲,衣着俭朴,辛劳不辍,仍笑呵呵地说:“全家平平安安,比什么都让我快乐!”
   重新找到工作的下岗女工说:“有一份工作,真快乐!”
   背着沉重书包的中学生说:“星期天早上让我睡够了,最快乐!”
  
  
   二、阅读理解(50分)
   (一)阅读《太空医生》,完成7—9题。(10分)
   太空医生
   1980年,美国发射了一颗观测太阳的卫星。不久前,这颗卫星“生病”了——它的几根保险丝烧断了,不能正常工作。
   有人曾这么说过:“宇宙火箭是燃烧着美钞上天的!”这话不假。美国的“阿波罗”登月飞行,飞了5次,耗资达200多亿美元。当然,这颗太阳观测卫星,也是靠“燃烧着美钞上天的”,它“告病假”,自然是莫大的损失。好在只消把保险丝换一下,它就能继续正常工作。能否派“医生”到太空给卫星看“病”呢?
   1984年,美国科学家派出了“太空医生”——航天飞机修理这颗卫星。当航天飞机靠近这颗卫星时,把宇航员弹到卫星上去。宇航员“抓”住了卫星,再借助航天飞机上的机械手,把卫星搬到货舱里的维修平台上,细细修理。修好后,再把卫星放回原先的轨道,让它继续工作。这样,“太空医生”略施小技,便可救活一颗人造卫星。
   如今,在2000多颗运行的人造卫星中,“病号”不少。
   人造卫星的“病”,有好多种。
   一种是“不治之症”。一般地说,人造卫星的运行轨道越高,寿命越长。我国在1970年发射的第一颗卫星,轨道高,已经十几“岁”了,仍在太空中运行。一些低轨道的人造卫星(特别是用于军事的侦察的卫星,轨道低,“看”得清楚),寿命短,有的只有几个月,甚至只有几天。这种陨落“重症”,是很难治好的。
   另一种是“小毛病”。比如,前面提到的那颗美国太阳观测卫星,便是患“小毛病”。1966年,苏联向月球发射了一颗卫星——“月亮10号”。这颗卫星绕月飞行不久,就报废了。因为这颗卫星上用的是化学电池,电力用光了,也就失效了。其实,当时卫星上的仪器都完好无损。倘能派出“太空医生”,换上新电池,就不会报废。
   也有的生的是“急性病”。1970年美国发射了“阿波罗13号”,起飞后46小时,飞船里的一个燃烧箱爆炸了,使飞船无法完成登月使命。三名宇航员历尽艰辛,才使生了“急性病”的飞船溅落在太平洋上。卫星或飞船生“急性病”,需要“太空医生”及时赶来抢救。
   谁是最合适的“太空医生”呢?
   1981年,美国“哥伦比亚”号航天飞机飞上太空,成功地进行了人类历史上第一次航天飞行。从此,航天飞机成为“太空医生”的极其合适的“候选人”。
   为什么人们会看中航天飞机呢?
   这是因为航天飞机可以来回于地球与太空之间,一般可以重复使用100多次,使花钱如流水的空间科学活动大大降低了费用。据美国科学家统计,航天飞机飞行一次,大约花费1000万美元,这比重新发射一颗卫星要合算得多。随着航天飞机技术的改进,它的飞行成本还将进一步降低。
   另外,航天飞机返回之后,一般经过两星期检修,又可上天,使用方便。科学家建议,今后在发射宇宙飞船或人造卫星时,应同时让一架航天飞机进入待命状态,一旦飞船或卫星在发射后生了“急性病”,“太空医生”立即出发,及时进行抢救。
   为了能够在太空中“抓”住卫星,航天飞机装有机械手。它的机械手有两只,其中—只备用。机械手用电脑控制。美国“哥伦比亚”号航天飞机在首次试飞成功之后,再度上天,着重试验机械手的性能。宇航员穿好充压宇航服,可以在航天飞机舱外活动,便于修理卫星。宇航员可以在舱外连续工作6——7个小时。
   (选自《中华活页文选》2003年15、16期,有改动)
   7、为什么说“航天飞机是最合适的太空医生”?(3分)
  
   8、本文语言很有特点,把“卫星出了故障”说成是“生病”,把“不同程度的故障”说成是“不治之症”“小毛病”“急性病”。请从文中再找出一处类似的词句,并说说其作用。(3分)
  
  
   9、“太空医生”的诞生很有创意。通读全文,说说催生“太空医生”这一角色的因素有哪些?依你看,随着人类探索太空活动的日益频繁和深入,未来的太空还会诞生什么角色?(4分)
  
  
   (二)阅读《读书以养心》,完成10—12题。(10分)
   读书以养心
   念 轩
   古人云:流水之声可以养耳,青禾绿草可以养心。没有书的日子会使人心灵蒙尘,渐渐消蚀的读书热情会使心灵的肌肤起皱。太忙碌于现实,太驰骛于外界,总有疲倦的时候,此时书便是心灵最好的栖息之所。
   如今媒体众多,比如电影、电视、广播、报刊、书籍、互联网等等。相比之下,书籍的那一份独有的沉静、深远和幽雅,最让人难以割舍。繁忙的工作之余,捧一本书,或坐于书房,或行于户外,无声的书籍均能给你一份闲情逸致,你可深入,也可浅出,在无声中聆听美好的旋律,在无画中观赏美妙的图景。
   苏轼曰:“书富如入海,百货皆有”。书籍是人类漫长生活的积淀,它能给予我们的不仅仅是知识的增多,精神的享受,还在于感情的升华和情操的陶冶。书籍又是一个磁场,从过去到现在,再到未来,心灵的罗盘都会在这里发生强烈的感应。我们从《论语》中学到智慧的思考,从《史记》中感受历史的严肃,从鲁迅那里学得醒世哲理,从毛泽东、邓小平等老一辈革命家那里学得观察问题、解决问题的方法。
   在阅读中,我们会发现,文字也是有个性的。譬如周作人的清淡平和,许地山的睿智自然,丰子恺的细腻亲切,冰心的清雅飘逸,钟敬文的清朗绝俗;又譬如海明威的冷峻,伏尔泰的咏叹,川端康成的精美……读书不仅要择书而读,更要择人而读。作品中的思想,映射着创作者本人的人生态度、思想方式、创作意图。真正会读书的人,会着力挖掘作者深层次的创作意图,以求获得深层次的美感。
   一部好书就是尘世的一盏明灯,它照亮了人们的心灵,也照亮了人类历史的路径。读书是高尚而神圣的,它可以给人带来心灵的丰富和精神的振奋,可以让人活出人生的一大境界。试想,在万籁俱静的夜晚,排除杂念,或半倚床侧,或微伏案边,捧书入静,不也是一种最美的享受吗? (选自《读写月报》2004年第4期)
   10、找出文中具体说明“养心”含义的句子。(2分)
  
  
   11、请根据你所学过的知识及上下文的意思,说说第1段中两个加点词语的含义。(4分)
   蒙尘:
   驰骛:
   12、请仿照文中画线句式,再举一个你所了解的作家的例子,并补充其相关的材料证明“作品中的思想,映射着创作者本人的人生态度、思想方式、创作意图”。(4分)
  
  
   (三)阅读《诚实的回报》,完成13—16题。(14分)
   诚实的回报
   司玉笙
   强高考落榜后就随本家哥去沿海的一个港口城市打工。
   那城市很美,强的眼睛就不够用了。本家哥说,不赖吧?强说,不赖。本家哥说,不赖是不赖,可总归不是咱自个儿的家,人家瞧不起咱。强说,自个瞧得起咱自个儿就行。
   强和本家哥在码头的一个仓库给人家缝补篷布。强很能干,做的活儿精细,看到丢弃的线头碎布也拾起来,留作备用。
   那夜暴风雨骤起,强从床上爬起来,冲到雨帘中。本家哥劝不住他,骂他是个憨蛋。
   在露天仓库里,强查看了一垛又一垛,加固被掀动的篷布。待老板驾车过来,他已成了水人。老板见所储物资丝毫不损,当场要给他加薪,他就说不啦,我只是看看我修补的篷布牢不牢。
   老板见他如此诚实,就想把另外一个公司交给他,让他当经理。强说,我不行,让文化高的人干吧。老板说我看你行!
   强就当了经理。
   公司刚开张,需要招聘几个大专以上文化程度的年轻人当业务员,就在报纸上做了广告,本家哥闻讯跑来,说给弄个美差干干。强说,你不行。本家哥说,看大门也不行吗?强说,不行,你不会把这儿当成自己的家。本家哥脸涨得紫红,骂道,你真没良心。强说,把自个儿的事儿干好才算有良心。
   公司进了几个有文凭的年轻人,业务红红火火地开展起来。过了些日子,那几个受过高等教育的年轻人知道了他的底细,心里就起毛说,就凭我们的学历,怎能窝在他手下?强知道了并不恼,说,我们既然在一块儿共事,就把事办好吧。我这个经理的帽谁都可以戴,可有价值的并不在这顶帽上……
   那几个大学生面面相觑,就不吭声了。
   外商听说这个公司很有发展前途,想洽谈一项合作项目。强的助手说,这可是条大鱼哪,咱得好好接待。强说,对头。
   外商来了,是位外籍华人,还带着翻译、秘书一行。
   强用英语问,先生,会汉语吗?
   那外商一愣,说,会的。强就说,我们用母语谈好吗?
   外商就道了一声“OK”。谈完了,强说,我们共进晚餐怎么样?外商迟疑地点了点头。
   晚餐很简单,但有特色,所有的盘子都吃尽了,只剩下两个小笼包子。强对服务小姐说,请把这两个包子装进食品袋里,我带走。
   虽说这话很自然,他的助手却紧张起来,不住地看那外商。那外商站起,抓住强的手紧紧握着,说,OK,明天我们就签合同!
   事成之后,老板设宴款待外商,强和他的助手都去了。席间,外商轻声问强,你受过什么教育?为什么能做这么好?
   强说,我家很穷,父母不识字。可他们对我的教育是从一粒米、一根线开始的。后来我父亲去世,母亲辛辛苦苦地供我上学。她说,俺不指望你高人一等,你能做好你自个儿的事儿就中……
   在一旁的老板眼里渗出亮亮的液体。他端起一杯酒,说,我提议敬她老人家一杯——你受过人生最好的教育——把母亲接来吧!
   (选自武进教育网站“中学语文”2004年2月19日“每日推荐”)
   13、用简洁的语言概括文章的主要内容。(2分)
  
   14、早期教育对人的影响是深远的。强的父母从“一粒米、一根线”开始教育他,这对强的行为产生了哪些直接的影响?请根据文中有关内容,作简要说明。(4分)
  
  
   15、现在,许多家长以将孩子送到“贵族”学校、重点学校就读,甚至留洋出国视为最好的教育,而文中强的老板却把强从小受到的“一粒米、一根线”的教育称为最好的教育。对此,你是怎么看的?(4分)
  
  
   16、文中强有许多优秀品质,其中你最欣赏的是什么?请结合自身实际,说说你的理由。(4分)
  
  
   (四)阅读下面《送东阳马生序》片段,回答17-20题。(共12分)
   余幼时即嗜学。家贫,无从致书以观,每假借于藏书之家,手自笔录,计日以还。天大寒,砚冰坚,手指不可屈伸,弗之怠。录毕,走送之,不敢稍逾约。以是人多以书假余,余因得遍观群书。既加冠,益慕圣贤之道。又患无硕师名人与游,尝趋百里外从乡之先达执经叩问。先达德隆望尊,门人弟子填其室,未尝稍降辞色。余立侍左右,援疑质理,俯身倾耳以请;或遇其叱咄,色愈恭,礼愈至,不敢出一言以复;俟其欣悦,则又请焉。故余虽愚,卒获有所闻。当余之从师也,负箧曳屣,行深山巨谷中,穷冬烈风,大雪深数尺,足肤皲裂而不知。至舍,四支僵劲不能动,媵人持汤沃灌,以衾拥覆,久而乃和。寓逆旅主人,日再食,无鲜肥滋味之享。同舍生皆被绮绣,戴朱缨宝饰之帽,腰白玉之环,左佩刀,右备容臭,烨然若神人;余则缊袍敝衣处其间,略无慕艳意,以中有足乐者,不知口体之奉不若人也。盖余之勤且艰若此。
   17、解释下列句子中加点的词语(4分)
   (1)走送之  (    )     (2)既加冠  (    )
   (3)尝趋百里外从乡之先达执经叩问(           )
   (4)左佩刀,右备容臭,烨然若神人(           )
   18、用现代汉语写出下列句子的意思。(2分)
   以中有足乐者,不知口体之奉不若人也。
  
   19、用自己的话归纳作者是从哪几个方面写出求学的“勤且艰”的。(3分)
  
  
   20、选文中哪些语句与下面材料中借书者“必虑人逼取”、“惴惴焉”的心理相类似?请把有关语句摘录在后面的横线上。(2分)
   非夫人(指借书者)之物而强假焉,必虑人逼取,而惴惴焉摩玩(摩挲,抚弄)之不已,曰“今日存,明日去,吾不得而见之矣。” (清代袁枚《黄生借书说》)
  
   (五)阅读下面的文言文段,完成21-22题。(4分)
   与善人居,如入芝兰之室,久而不闻其香,即与之化矣。与不善人居,如入鲍鱼之肆(店铺),久而不闻其臭,亦与之化矣。丹之所藏者赤,漆之所藏者黑,是以君子必慎其所处者焉。
   21、根据文意,请用一个成语概括君子要“慎处”的原因。(2分)
  
   22、对这段文字所包含的观点你有什么看法?请作简要回答。(3分)
  
  
   三、作文(50分)
   23、阅读下面的文字,根据要求作文。
   把目光放在底处,看一看街上那些生活在底层的人们,比如工地上的劳工,比如沿街的小贩……
   把目光放在底处,关注一下身边默默服务的人们,比如校园内的清洁工、比如收发室的老大爷……
   把目光放在底处,你会看到,是他们,让你我的生活变得如此温馨而美丽。
   请以“把目光放在底处”为话题,写一篇文章。
   注意:①所写内容必须在话题范围之内。②立意自定。③除诗歌外,文体不限。④题目自拟。⑤不得出现真实的地名、校名和人名。⑥不少于600字。
  
   题目:_____________________________________
   参考答案及评分标准
   一、(共24分)
   (一)1、宕、 礴、 婉、 绽(2分,对两个得1分) 2. ①明天子在上,可以出而仕矣 ②山回路转不见君,雪上空留马行处 ③断肠人在天涯 ④莫道不消魂,帘卷西风,人比黄花瘦 ⑤毅魄归来日,灵旗空际看 3. C、B 4.什么是绿色消费 5.①大多数高校毕业生能找到工作; ②本科文凭就业形势较为乐观; ③高校毕业生找不到工作是正常现象。 7、李逵 蒋门神(2分) 8、要点:一是要写出某种境况下的人物,说话者话语前应有反映人物境况的修饰语(1分);二是写出对快乐的理解。(2分。对快乐的理解符合人物心境得1分,有一定内涵、语言通顺得1分) 例:日晒雨淋了一天的建筑民工说:“收工后一包花生米、三两白酒的日子,就快乐!”
   (三) 6、参考: 由于人们乱挖乱采,过度放牧,草原连骆驼、羊群生存所需的草的数量减少,造成骆驼哭泣、羊群撕咬。(4分。结果和原因各2分。原因一点1分。如提出牧人一方面为防止羊群撕咬为其穿上衣服,一方面又在纵容孩子破坏生态环境,建议加1-2分。)
   二、(共46分)
   (一)7、降低了费用,使用方便,便于修理卫星。(3分。答到一点得1分) 8、例:这颗太阳观测卫星,也是靠“燃烧着美钞上天的”,它“告病假”,自然是莫大的损失。运用了拟人手法,把卫星发生故障停止工作说成是“告病假”,生动形象且通俗易懂。意对即可。(3分。找出类似句子得1分,写出作用得2分) 9、因素有:一是运行的人造卫星中有许多出现了不同程度的故障,影响使用;二是经济因素;三是空间技术的发展提供了技术支持,如航天飞机的研制成功、人类能在太空行走等。未来可能诞生“太空清洁师”、“太空警察”等。因素答到一点得1分,角色想象合理得1分。
   (二)10、(书籍)在于感情的升华和情操的陶冶,心灵的罗盘会发生强烈的感应。或(书籍)可以给人带来心灵的丰富和精神的振奋,可以让人活出人生的一大境界。(2分。任选一处,意对即可) 11、蒙尘:指不能拥有高尚、圣洁的精神境界;驰骛:指一心向往、追求外在的东西,忙碌、奔波于琐碎的事务中。(4分。每个词语2分。意对即可,如只解释字面意思不得分。) 12、如:鲁迅的犀利(尖锐),例如:《孔乙己》一文通过塑造孔乙己这一形象,尖锐地讽刺了科举制度的罪恶和当时社会的人情世态。再如:朱自清的真挚朴实;欧·亨利的诙谐幽默等。(4分。句式一致,能概括文字个性得1分,所概括文字个性与作家、作品相符合1分;补充材料与所概括的文字个性一致1分,能涉及材料所表现的创作者本人的人生态度或创作意图等1分)
   (三)13、高考落榜的强在打工期间因办事诚实而成就了一番事业(或因他的人格力量得到了人们的尊重信任)。(3分。应紧扣“诚实”和“回报”答题,意对即可。应包含人物、特点、结果,各1分。) 14、强干活时,看到丢弃的线头碎布都拾起来留作备用;强在接待外商时,准备的晚餐很简单,最后把剩下的两个小笼包子装进食品袋里带走。(3分。找出两个方面各得1分,能作简要概括得1分) 15、应围绕“什么是最好的教育”来谈。如赞同第二种看法,可从家庭教育、个人行为习惯、人格修养的培养、早期教育等角度择一而谈,说明其重要性;如赞同第一种看法,则应写出择校就读对孩子身心发展方面的优越性。此外,也可认为最好的教育就是适合自己发展的教育等。(4分。答题能涉及对以上两种看法的评述,侧重评述其中之一,得1分。有自己独特的见解(指能不囿于以上两种看法),理由充分,语言通顺,得3分;有独特见解,但理由不充分或语言欠通顺,或见解一般,但语言通顺,得2分;见解肤浅,语言欠通顺,得1分。) 16、评析人物的优秀品质可从自尊、诚实、勤俭、公私分明、有责任心等方面考虑(1分)。结合自身实际(1分)。理由充分,有真情实感,语言通顺(2分)。
   (四)17、(1)跑 (2)已经 (3)有名望的前辈(或有道德有学问的前辈)(4)光彩照耀的样子(2分。答对2个得1分) 18、因为内心有足以快乐的事,也就不觉得吃的穿的不如人。(2分。意思对即可。) 19、四个方面:(1)得书之难;(2)求师不易;(3)奔波之苦;(4)生活清贫。(2分。若有其他答案,意思对即可给分。) 20、计日以还;弗之怠;不敢稍逾约。(答到两点得1分,三句全找出得2分)
   (五)21、近朱者赤,近墨者黑(2分) 22、应从环境对人的影响的角度来回答。能围绕观点(环境的作用)得1分,有独到的看法得1分。
   三、(50分)
   略





























中考语文总复习参考答案
第一讲 语音和汉字
1.C 2.B 3.D 4.D 5.A 6.C 7.B 8.A 9.B 10.B 11.D 12.C 13.C 14.B 15.C 16. D 17.C 18.D 19.①揆 ②chù ③霾 ④jī 20.峻 qǐ bó 遐 21.①闪烁 ②匀称 ③xù ④hè 22.zhēngróng chù qìn 23.(1)xiào (2)sǒng (3)偃 (4)畸 24.撒 缚 绳 罩 25.①筹 ②恬 ③洒 26.拓 滞 腾 搏 27. ①苟 ②伦 ③匠 ④挠 28.①hè ②yàng ③寞 ④恍 29.栋 娴 拥 30.①瞰 ②枢 ③烁 ④畸 31.①怯 ②隘 ③垠 ④诣 32.溃 赏 xùn wù 33.①xiè ②yǎo ③qiè ④砥 ⑤夙 ⑥礴 34.B 35.A 36.C 37.A 38.B 39.D 40.A 41.B 42.C 43.C 44.B 45.C 46.B 47.D 48.B 49.B 50.A
51.疏 宵 妄 源 52.绌 炙 鉴 猾 53.愁—筹 部—步 辩—辨 判—叛 54.词 戛 55.成 辨 事 闻 蛋 56.①建—键 ②竞—竟 ③象—向 57.奇—其;消—销;众—重;燥—躁 58.欲 刻 释 闻 59.①曦 嬉 ②遗 怡 60.“赢”改为“营”,“咳”改为“刻”。

第二讲 词语和句子
1.C 2.C 3.C 4.B 5.D 6.B 7.D 8.C 9.B 10.A 11.A 12.③⑦⑩⑨ 13.A 14.B 15.A 16.D 17.B 18.D 19.D 20.C 21.C 22.书写正确,必须是成语。 23.“敏捷”改为“敏锐” 24.①刮目相看(待)(吴下阿蒙) ②三顾茅庐(鞠躬尽瘁) ③四面楚歌(破釜沉舟) ④示例:曹刿——一鼓作气 陈胜——揭竿而起(鸿鹄之志) 25.击败(打败) 大胜(轻取) 险胜(小胜) 力挫(力克) 26.D 27.C 28.B 29.D 30.D 31.D 32.B 33.B 34.D 35.B 36.C 37. 第①句与上下文语意不连贯,应改为“对待挫折的态度不同,得到的结果也就不一样”。②句中“成就”和“理想”搭配不当,把“理想”改为“事业”或“大业”。③句中“毫无作为”和“一事无成”重复另哆嗦,应删去其中一个。 第④句语序不当,把“无法承受任何挫折的打击”和“把挫折看成可怕的陷阱”对调。第⑤句句子结构不完整,应补上“的意志”。 38.(1)将“喜闻乐见”改为“喜爱”。(2)将“把它”改为“被”。(其他改法符合要求也可。) 39.①“然而做每件事都要细心瑾慎,小心翼翼。”句中,“然而”改为“所以(因此)”;“瑾”应改为“谨”;“细心瑾慎”与“小心翼翼”重复,删去其一。 ②“有时也许就是因为一个不留神的错误,造成了无法挽回的一个遗憾”,欠通顺,改通即可;“有时”后可加“,”;“一个”调到“无法”前。 ③“朋友:珍重地走好每一步,不要对小事就吊以轻心”句中,“:”改为“,”;“珍重”改为“慎重”;“吊”改为“掉”。④“希望你稳健地走完自己的旅程”欠通顺,改通即可。(若有其他改法,只要正确合理亦可。) 40.第③句,修改意见:删去“阻止孩子不上网”中的“不”。 第④句,修改意见:将“有效的”调至“寻找”后面。 41.①中国是赞同成立联合国的首批国家之一。 ②晋文公当上了晋国国君后,发展生产,整顿政治,训练军队,使晋国成为北方一大强国。 ③因此,为了您的人身安全,请勿用左手启动家电,以防万一。 ④他们坐在一张桌子的两端,面对着面。 42.①删去“不但” ②“而且”改为“而”,或删去“而且” ③删去“也” 43.①删去“处于”,或在“发掘”后加“状态” ②改为“而有些基建单位却擅自动工,导致破坏性挖掘,这种状况令人担忧”,或“而令人担忧的是有些基建单位却擅自动工,导致破坏性挖掘” ③删去“已经”或“正” 44.①将“展示”改为“暴露”  ②在“终于成为”前加上“我国”或“中国”等。 45.(1)去掉“地笑”或去掉“忍俊不禁地” (2)去掉“否”或在“是”后填写“能不能” 46.字词:“拆”改为“折”;“竟”改为“竞”。 标点:“……会当击水三千里。”引号中的句号放在引号外,或者将“豁达是一种自信,”中的逗号改为句号 句子:“不是……就是”改为“不是……而是”。

第三讲 语言运用
1.要求仿写的句子与原文画线句子的结构基本相同;内容方面既要写出课外阅读的作品名称,还要写出作品给自己的启示。所写作品可不限于《大纲》规定的课外阅读推荐篇目。示例:读《西游记》,我们能学到孙悟空的嫉恶如仇;读《红楼梦》,我们能体会到封建社会大家庭衰落的必然性;读《格列佛游记》,我们了解了当时英国统治阶级的腐朽与罪恶;读《童年》,我们体会到当时俄罗斯下层人民生活的苦难。 2.句式正确、内容合理即可。 3.可以从健康、知识、网络、快乐、亲情、友情等角度仿写。示例①:拥有知识,就拥有了无限的光明和希望;拥有知识,就拥有了无限的力量和财富。②:拥有友情,就拥有了一份理解和支持;拥有友情,就拥有了一份快乐和温馨。③:拥有网络,就拥有了时间和空间;拥有网络,就拥有了世界和梦想。 4.示例:(1)鲜花,如果害怕凋谢,那它永远不能开放。(2)矿石,如果害怕焚烧(熔炉),那它永远不能成钢(炼成金子)。 (3)蜡烛, 如果害怕熄灭(燃烧),那它永远不能发光。 (4)航船,如果害怕风浪,那它永远不能到达彼岸。 5.只要能写出体现夏、秋两季节特征的词语即可。如:绿叶、绿树、绿荫;稻子、枫叶、果实…… 6.能达到“写物寓意”的写作要求,语言精美、内容健康即可。 7.如:用崇高的理想布局,用坚定的信念写作,用奋斗的精神修改等等。 8.我愿是一片青翠的树叶,为春天点缀一丝新绿。(答案不是唯一,符合要求即可。) 9.略 10.示例:当一个人悲观失望的时候,他忘记了建立自信是一种习惯;当一个人志得意满的时候,他忘记了谦卑为怀是一种习惯等。(内容要求:前句为面临的反面的不良的处境或表现,后句应从正面提出应对策略;句式要求:应与例句一致。) 11.略 12.要求:①内容表达及表达方式符合要求即可②每段话中所规定的三个词语必须全用。 13.示例:①生活是一部大百科全书,包罗万象;生活是一把六弦琴,能弹出多重美妙的旋律;生活是一条河,奔腾不息,淘尽人间善恶。②幸福是禾苗,晃动稚嫩的身躯向天际发出无声的诉说;幸福是蚕宝宝,咀嚼着香脆的桑叶奏出“沙沙沙”的乐曲;幸福是雏燕,展翅穿过鹅黄的柳梢剪出一片春色。③爱心是一片冬日的阳光,使饥寒交迫的人感到人间的温暖;爱心是沙漠中的一泓清泉,使濒临绝境的人看到生活的希望;爱心是洒落在久旱土地上的甘霖,使心灵枯萎的人感到情感的滋润。 14.略 15.略 16.略 17.②④⑥③⑤⑦①或②④⑥①③⑤⑦ 18.⑤②④①③ 19.示例:(1)(张书记)您认为举办这次活动,对盐城经济文化的发展有什么重要意义?(2)(郑先生)您觉得我们盐城的投资环境怎么样? 20.理由①你们不是执法人员,你们没有权利对我搜身,你们这样随意侵犯人权是犯法的。②如果你们要强行对我搜身,我将诉诸法律。③商场不是装了监视器吗?你们能拿出我偷东西的证据吗? 21.示例:人家的孩子学习成绩有多好(多聪明、多体谅人、多懂事); 人家的孩子比我强(好)。 言外之意:你不如人家的孩子;你比人家的孩子差。 22.示例:郝老师,李蓉生病住院,不能按时交作文,她很抱歉(很对不起)。 23.①围绕“请不要生气”的意思作答即可。例如:请别生气;请你用微笑来对待这件不愉快的事情;生气(发火)会损坏你美好的形象;生气(发火)会破坏同学间友好的关系。②从顺意化解方面回答。如:谢谢你的提醒,我重写一幅好了;为了人类更好看,我接受你的微笑。 24.行动重于语言 25.①批评那位同学松懈拖沓,上课了不按时进教室。②你说的话要体现出亲切平和,讲清事理,避免讽刺挖苦。 26.①略②示例:尊师、爱生、勤奋、笃学。③略 27.妈妈说参考:孩子,这次考得不好,你的心情我理解,你的基础好,不应灰心,关键是要找准原因,集中精力学习,妈妈相信你下次会考好的。(同学说略) 28.(1)如:老大爷:您好!我是向阳中学的学生,想问一下,有人把垃圾倒入河里,将工业废水直接排到河中,对这种做法您怎么看? (2)如:局长:您好!我是向阳中学的学生,想了解现在环保部门对护城河水污染处理的措施,您能谈谈吗? 29.A:自己的事情应该自己解决了。 B:每个人都要遵守公共秩序,我们也不能搞特例啊。 30.(只要言之有理即)开卷未必有益 (1)社会上各类书刊鱼龙混杂,青少年对好书坏书缺乏必要的鉴别能力。 (2)社会上因读黄色书刊而犯罪的事例也不少。 是的,从某种角度上说,你们讲的也有一定的道理,但是,我们还应该看到它的另一面,即消极的一面,我们认为开卷未必有益。 31.略 32.示例:遮风挡雨固然是一种爱护,而过分溺爱反而会适得其反。(符合要求即可) 33.此题能合情合理、简明地说出图标的包含的某一种意思即可。如:由中国电信英文首字母CTC组成。 上半部分的C字是一个牛角,代表中国电信上市的美好前景。等等 34.(1)中国传统的印章;一个向前奔跑、舞动着迎接胜利的运动人形;现代“京”字;奥运会举办的时间与地点:“2008”“北京”;表示五大洲的奥运环。 (2)共有四个涵义,答出其中的一个即可。①中国文化。印章是中国传统文化艺术的一种表现形式,同时也是诚信的一种表现形式。 ②红色。选用中国传统喜庆颜色——红色,具有代表国家、代表喜庆、代表传统文化的特点。 ③中国北京欢迎世界各地的朋友。传递出奥林匹克的理念和精神。 ④冲刺极限,创造辉煌。 35.人口问题(或资源问题) 36.①预计2030年以后,世界能源将以核电和太阳能为主要能源。②示例:①要节约能源;②使用无污染(或绿色清洁)的能源;③要保护环境;④要珍惜资源。…… 37.略 38.“儿童远离泡沫饮料可防肥胖”或“儿童饮泡沫饮料过多会发胖(超重)” 39.教育消费已成为(部分)家庭的重要开支,教育质量还存在(一定)问题(或不尽如人意等)。 40.高玉宝两大理由支持美猴王申吉(语意完整且符合要求即可) 41.中国少年科学院小院士表彰大会(在京)召开(举行) 42.第四届全国暨四川省科技活动周今天(或“15日”)在绵阳开幕 43.“农夫山泉”追加投资丹江口市;或“农夫山泉”在丹江口追加投资。 44.示例:在任何情况下(不管发生什么情况),哪怕牺牲自己的利益甚至付出沉重的代价,也要讲诚实,守信用。 45.公共文化设施免费向未成年人开放。(“公共文化设施”、“免费”或“免票”、“未成年人”) 46.“奥运圣火到北京”或“奥运圣火8日(或首次)抵京”等。只要意思接近,不超过10个字即可。 47.本来已经退化了的器官又重新出现在某个生物体上的现象叫返祖现象。 或:返祖现象是本来已经退化了的器官又重新出现在某个生物体上的现象。(大意对即可) 48.沙尘暴席卷北疆,首府沙尘浓度超上限。 49.流言传播得次数多了,也会让人相信。 50.略 51.略 52.例:“金星凌日”是指金星运行到太阳和地球之间所形成的一种天文现象,就和月亮处在太阳和地球间所形成的日食相似。 53.黑马:比喻实力难测的竞争者或出人意料的优胜者。 54.略 55.①遥远创作的世界和平女神像在法国揭幕。 ②中国人民热爱和平;世界和平终将实现。 56.内容表达符合要求,语言表达新颖、优美即可。 57.扣住“自拟”“努力奋进”作答即可 58.A破产和截肢的打击; B.人生的挫折和失败。 59.示例:①教孩子学会做人关系到孩子一生的健康成长。 ②家长言传身教是孩子健康成长的保障。 ③我国政府重视未成年人的健康成长。 ④“父母亦师”或“和孩子交朋友”。 60.示例:梅花 你绽放在北风凛冽、大雪纷风的数九严冬,把缕缕暗香送给人间。 启示:艰苦环境更能磨练人的意志。 61.(1)小学课堂教学忽视学生求异思维的培养。课堂教学缺乏民主或课堂教学民主意识不够。(只要符合材料内容,其他不同的文字表述亦可。) (2)师生回答均有道理,只是看问题角度不同。(意思相同的表述即可) 62.略

第四讲 文学常识
1.D 2.B 3.C 4.A 5.B 6.A 7.A 8.B 9.C 10.B 11.C 12.A 13.B 14.A 15.B 16.B 17.A 18.莎士比亚,英国;塞万提斯,西班牙。《威尼斯商人》或《莎士比亚全集》、或《唐吉·诃德》 19.关羽(答关云长、关公、关帝亦可);只要是有关关羽的故事即可,如走麦城、水淹七军、温酒斩华雄、桃园三结义等。 20.王熙凤、林黛玉。 21.拉车 刘四(刘四爷) 22.被解救的人:林冲(豹子头林冲、林教头);故事情节的名称:误入白虎节堂、风雪山神庙、守护草料场、被逼上梁山、火并王伦、活捉扈三娘。等等。(任写一个即可。) 23.略 24.略。(只要是这三篇小说中写到的人或事,分析合理即可。) 25.能简要写出一个情节,写出与情节有关的体验和感悟,并言之成理。 26.①《聊斋志异》 ②《三国演义》 27(1)、奥斯特洛夫斯基 《钢铁是怎样炼成的》 (2)、沙悟净(沙僧或沙和尚) 贾宝玉 28.①关羽(关云长、关公)  ②藤野先生  ③鲁滨孙(鲁滨逊) 29.华威先生——虚伪、庸俗;不做实事,只尚空谈;媚上欺下,投机钻营 蔡芸芝先生——魏巍 藤野先生——黑瘦、八字须、戴眼睛、穿衣服模糊 韩麦尔先生——《最后一课》 30.①鲁智深 吴用 ②《三国演义》 31.《钢铁是怎样炼成的》 奥斯特洛夫斯基 32.①孙悟空(或孙行者、悟空) ②林教头 33.荆轲 屈原 诸葛亮 苏武 34.A.③ B.④ C.② D.① E.⑥ F.⑤ 35.①写孙悟空的出身和大闹天宫等故事。 ②汤姆 蓓姬 36.①如武松醉打蒋门神、鲁提辖拳打镇关西、鲁智深大闹野猪林等 ②《钢铁是怎样炼成的》 37.示例:写下一页页辉煌的篇章 38.略。(只要是这三篇小说中写到的人或事,分析合理即可。) 39.如《三国演义》,罗贯中;诸葛亮;一个人不仅要有智慧,还要善于用人。
第五讲 背诵默写
1.(1)江流天地外 (2)惟吾德馨 (3)枯藤老树昏鸦 (4)留取丹心照汗清 (5)择其善者而从之 (6)似曾相识燕归来 2.①长河落日圆 ②志在千里 ③鬓微霜 ④晴空一鹤排云上 ⑤波涛如怒 ⑥寻向所志 ⑦如使人之所欲莫甚于生 ⑧其文疏荡 3.(1)悠然见南山 (2)化作春泥更护花 (3)山重水复疑无路 (4)下笔如有神 4.①得道多助 ②随风潜入夜 ③悠然见南山  ④千里共婵娟 ⑤直挂云帆济沧海 ⑥忽如一夜春风来 ⑦一览众山小 ⑧江山代有人才出 5.①却话巴山夜雨时 ②似曾相识燕归来 ③人则无法家拂土 ④黑云压城城欲摧 6.①千树万树梨花开 ②出淤泥而不染 ③江入大荒流 ④沉舟侧畔千帆过 ⑤佳本秀而繁阴 ⑥夜半钟声到客船 ⑦后天下之乐而乐 ⑧春色满园关不住 ⑨听取蛙声一片 ⑩各领风骚数百年 7.老骥伏枥,志在千里(烈士暮年,壮心不已) 8.忽如一夜春风来 千树万树梨花开 9.老骥伏枥,志在千里。 10.感时花溅泪,恨别鸟惊心 11.海内存知己,天涯若比邻 12.力尽不知热,但惜夏日长 13.山回路转不见君,雪上空留马行处 14.气蒸云梦泽,波撼岳阳城 15.江山代有才人出,各领风骚数百年16.夜阑卧听风吹雨 铁马冰河入梦来 17.几处早莺争暖树,谁家新燕啄春泥 18.海内存知己 天涯若比邻 19.海内存知已,天涯若比邻 20.几处早莺争暖树,谁家新燕啄春泥 21.山重水复疑无路,柳暗花明又一村 22.海日生残夜 江春入旧年 23.日暮乡关何处是 烟波江上使人愁 24.莫愁前路无知己 天下谁人不识君 25.春蚕到死丝方尽,蜡炬成灰泪始干 26.僵卧孤村不自哀,尚思为国戍轮台 27.会当凌绝顶,一览众山小 28.映日荷花别样红。 29.大漠孤烟直 长河落日圆 30.山重水复疑无路 柳暗花明又一村 31.山光悦鸟性  潭影空人心 32.江流天地外,山色有无中 33.落红不是无情物,化作春泥更护花34.曲径通幽处 万籁此都寂 35.海内存知己,天涯若比邻 何当共剪西窗烛,却话巴山夜雨时 36.不畏浮云遮望眼 只缘身在最高层 37.春蚕到死丝方尽 蜡炬成灰泪始干 38.我寄愁心与明月,随君直到夜郎西。 39.长风破浪会有时,直挂云帆济沧海 40.春蚕到死丝方尽,蜡炬成灰泪始干。 41.山重水复疑无路,柳暗花明又一村。 42.潮平两岸阔,风正一帆悬 43.大漠孤烟直,长河落日圆 44.会当凌绝顶,一览众山小 45.人生自古淮无死,留取丹心照汗青 46.山回路转不见君,雪上空留马行处 47.海日生残夜,江春人旧年 48. 忽如一夜春风来,千树万树梨花开。 49.沉舟侧畔千帆过,病树前头万木春 50.长风破浪会有时,直挂云帆济沧海 51.落红不是无情物化作春泥更护花 52.例:不畏浮云遮望眼,只缘身在最高层 问渠那得清如许,为有源头活水来 山穷水复疑无路,柳暗花明又一村。 53.持节云中,何日遣冯唐? 54.兴,百姓苦,亡,百姓苦 55.莫道不消魂,帘卷西风,人比黄花瘦 56.枯藤老树昏鸦 古道西风瘦马 57.无可奈何花落去,似曾相识燕归来。 58.日暮乡关何处是?烟波江上使人愁 59.马作的卢飞快 弓如霹雳弦惊 60.晴空一鹤排云上 枯藤老树昏鸦 61.过尽千帆皆不是 斜晖脉脉水悠悠 62.但愿人长久,千里共婵娟。 63.人比黄花瘦 64.受任于败军之际 奉命于危难之间 65.先天下之忧而忧,后天下之乐而乐 人生自古谁无死,留取丹心照汗青 落红不是无情物,化作春泥更护花 66.黄发垂髫并怡然自乐 67.谈笑有鸿儒,往来无白丁 68.陟罚臧否 不宜异同 69.舍生而取义者也 70.三人行,必有我师焉 71.天时不如地利,地利不如人和 72.学而不思则罔,思而不学则殆。 73.得道多助,失道寡助(或:得道者多助,失道者寡助)或者:失道寡助、失道者寡助、寡助之至,亲戚畔之 74.虽乘奔御风不以疾也 75.受任于败军之际,奉命于危难之间 76.不求甚解 77.翼然临于泉上 78.先天下之忧而忧,后天下之乐而乐 79.择其善者而从之,其不善者而改之。 80.好读书,不求甚解。 81.政通人和,百废具兴(百废俱兴) 82.好读书,不求甚解 83.急湍甚箭,猛浪若奔。 84.人不知而不愠,不亦君子乎? 85.政通人和,百废具兴 86.巴东三峡巫峡长,猿鸣三声泪沾裳 87.舍生而取义者也 88.叫嚣乎东西,隳突乎南北,哗然而骇者,虽鸡狗不得宁焉 89.出淤泥而不染,濯清涟而不妖 90.心凝形释,与万化冥合 91.亲贤臣,远小人(或亲贤远佞) 92.出淤泥而不染,濯清涟而不妖

第六讲 诗词赏析
1.B 2.A 3.B 4.D 5.D 6.A 7.C 8.D 9.(1)D (2)清晨,破山寺后禅院清幽脱俗的景致 颇为浓厚的禅隐 10.(1)浩荡 (2)D 11.(1)C 理由是:其余三项可看作“豪放派”风格,而C属于婉约派。 (2)A (3)示例:①夜来风雨声,花落知多少。 ②风劲角弓鸣,将军猎渭城。 ③林暗草惊风,将军夜如弓。 ④野火烧不尽,春风吹又生。 ⑤随风潜入夜,润物细无声。‖①忽如一夜春风来,千树万树梨花开。 ②欲将轻骑逐,大雪满弓刀。 ③孤舟蓑笠翁,独钓寒江雪。 ④千里黄云白日曛,北风吹雁雪纷纷。 ⑤窗含西岭千秋雪,门泊东吴万里船。 12.寄托了为培育人才以及维护自己崇高理想而献身的情怀。 13.豪放、热情 凄愁 借景抒情 14.(1)知时节(或:当春发生)。 (2)第二联从听觉来写,第三联从视觉来写。(3)示例:潜;这个词生动形象地写出了春雨悄无声息的特点。 15.示例:[内容]这是一首哲理诗,含蓄地说明了生活中许多看似容易的事,其实做起来并不容易的道理。或:启迪人们面对困难应有足够的认识和准备。 [写法]运用拟人的修辞手法,如“拦”字形象地写出了山的动态感。或:用形象的比喻,把“山”比作生活中的困难,寓含着深刻的哲理。 16.(1)选取了莺歌、红花绿柳(树)、水乡、酒店小旗、寺庙、春雨等。 (只要答出四点即可) (2)动、静结合、视觉听觉多角度描写(或声色结合)、点面结合或局部景致与概括描写结合等多种方法。 (3)①错在不懂得“千里”在这里是虚写而非实数或误解了艺术创作虚实结合的方法; ②既然是写江南春色,以“千里”概之亦扣题目。 17.⑴表面上写的是山河依旧、草木蔓生的情景,实际上是写草茂人稀,荒凉萧条,物是人非的社会状况(2)抒发了感时忧国、思家念亲的情感。 18.(1)“风怒号”从听觉上突出了秋风之狂,“卷”从视觉上形象地写出了秋风之肆虐。“飞”“渡”“洒”“挂罥”“飘转”等词写出了秋风对茅屋的破坏。这样的描绘为后文写屋漏遇雨、自己的境遇之惨蓄势。 (2)以小见大,推己及人,表现诗人忧国忧民的济世情怀。 19.示例:“新”与“旧”的对比;或“去”与“来”的对比。好处:“新”是唱新词的环境,“旧”是饮酒唱词时的环境——旧亭台,新词旧景对比,抒发了今是昨非的怅惘情思;或“去”是花落去,“来”是燕归来,来去对比,增加了对时光逝去的惋惜之情。 20.①早莺争树(莺争暖树) 草没马蹄 ②可以让读者根据前面的描写,充分发挥联想和想象,从而取得“含不尽之意见于方外”的效果。或:诗贵含蓄,这样含蓄地结尾,常取得言有尽而意无穷的效果。 21.(1)鸣鞭、挥鞭 (2)落花 自己 国家(或民族) 22.(1)A (2)“采菊”一方面说明作者爱菊,追求菊那样高尚的品格,另一方面“采菊”这种自得其乐的生活正体现了作者的清高自洁、不慕荣利。作者认为,具有菊花般品格的人和拥有“采菊”这种闲适生活的人生才是真正有意义的。 23.(1)卷土重来未可知。肯为君王卷土来? (2)①项羽 ②《史记》 ③司马迁 ④史家之绝唱,无韵之《离骚》 24.(1)E (2)改“望”不好。“见”表明诗人看到山不是有意为之,而是采菊之间无意中山的形象忽然映入眼帘。而“望”即有意远看,改“望”此篇神气索然。 (3)鸟儿在空中经受了风雨严寒的折磨,倦而知还,作者以飞鸟自喻,以飞鸟倦而知还来表达他弃官归隐的惬意情怀。 25.这首诗表达了作者对当权者不理国事,沉溺享乐的批判、忧虑,开篇两个“笼”字写出了水边夜色的迷蒙、冷寂,奠定了全诗感伤、忧愁的基调。 26雪花给树木披上银妆,仿佛突然刮起强劲的春风,一夜之间催开了千树万树的梨花 27.(1)对自己无罪遭贬的怨愤之情。(答出“怨愤”之意,即可。) (2)第六句既是实写也是虚写,实写大雪阻路,马难前行,虚写诗人留恋长安,不忍离去的感情。 28.(1)偏知。 (2)春风一吹,万物从冬眠中苏醒过来,昆虫在篱笆、墙角发出声声鸣叫,透过绿色窗纱传了过来,这不正是春天来了吗? 29.①低沉愤懑 高昂乐观 ②放眼望去,展现在眼前的是一幅这样的景象:大江之上,沉舟之侧仍有千帆竞发;大自然中,虽有病树但万木依旧争春。新事物必将取代旧事物。 30.①春天黄昏 ②无可奈何花落去 似曾相识燕归来 31.①江、鸟、山、花 鲜明的色彩 ②抒发了诗人的思乡之情。(意思对即可)

第七讲 综合考查
1.(1)“大约”或“左右” (2)安家落户(答“安家立业”“安居乐业”“安营扎寨”等也可) (3)⑥ (4)示例:北极——鸟类的天堂(乐园) 2.(1)①C ②D ③B ④A (2)通过阅读,人们不仅能增长知识,丰富自己的经历,而且能够获得精神的陶冶,从读书学做人。(意思对即可) 3.(1) ① 青蛙 ② 动辄 (2) 刹 yú (3)“逆”改为“匿” (4)A (5)例①一朵鲜花凋谢了。 例②一个女孩在病魔的摧残下凋谢了。 例③一个充满着美好憧憬、正含苞欲放的少女在病魔的摧残下过早地凋谢了。 例④它凋谢了,—个被誉为“民族之花”的国有企业,倒在了一群蛀虫的巧取豪夺之下。 4.①弥 屹 ②C ③听惯了赞誉的华丽之辞,觉得自己“完美无缺”,因而我们对批评总抱有敌意。 ④例如:从而甘耐寂寞、默默无闻,甘愿用生命装点绿色大地。 5.(1)xuàn (2)邃╱憩 (3)A. (4)例如:平凡是泥土,孕育着收获,只要你肯耕耘。 6.(1)sè 绽 (2)示例:诗意指自然带给人的美好的感受。(3)示例:也可以面对秋风中飘舞的金色落叶发出动情的咏叹。 7.(1)② ① (2)略 (3)亭台楼阁 或:亭台轩榭 或:楼阁亭台 多 (4)示例:①范仲淹的一篇《岳阳楼记》,使得一座平凡的楼阁成了中华民族的精神楼。 ②欧阳修的一篇《醉翁亭记》,终让滁州山中一座普普通通的亭子名扬四海。 ③崔颢的一首《黄鹤楼》,竟让大江岸边的这只千年黄鹤久留于世人的心中而不飞。 ④鲁迅先生的一篇《孔乙己》,使得浙江绍兴的“咸亨酒店”至今生意兴隆。 ⑤徐志摩的一首《再别康桥》,让英伦三岛的这座桥长久地留在人们心中。 ⑥张艺谋导演的电影《大红灯笼高高挂》,让山西的乔家大院从此蜚声省内外。 8.①“来稿”(或“赐稿”)。 ②“都以文章质量高低决定取舍”、“我刊将一视同仁”。 ③“必须”改为“请”。

第八讲 其他训练
1.D 2.D 3.B 4.①我说服妈妈,和你一起走。 ②我说服妈妈和你,一起走 5.D 6.D 7.C 8.C 9.C 10.A 11.C 12.C 13.D 14.上联:……堤/皆因/苏子/……或下联:……观/尽是/刘郎/…… 15.D 16.B 17.B 18.D 19.D 20.C 21.B 22.C 23.C 24.B 25.D 26.D 27.D

第九讲 文言文阅读
(一)1.D 2.不足为外人道也。 3.(他们)问起现在是什么朝代,竟然不知道有过汉朝,更不必说魏晋了。
(二)1.(1)苦于 (2)顽固 2.意思表达符合原意,能准确表达言者的语气即可。3.嘲讽、怀疑 语言、动作 4.示例:夸父逐日 精卫填海等 5.充分尊重学生独特的理解认识,只要符合要求即可。
(三)1.B、E 2.①(他)去喝酒,总是把酒喝完,希望一醉方休。(造:到……去;期:期望、希望。)②不因贫贱而忧愁,不为追求富贵到处奔走钻营。(戚戚:愁苦的样子;汲汲:急切追求的样子。) 3.民风淳朴,人人生活安乐,恬淡自足。(意思对即可) 4.结合自己读书习惯;赞成或不赞成且言之成理。参考答案:赞成:爱好读书,重在领悟文章精神实质。不赞成:根据文章内容精读与粗读结合。
(四)1.C 2.C 3.译文:两岸都是连绵的高山,几乎没有中断的地方。4.(1)描写对象和写法相同:同写三峡、同咏猿鸣,都采用了寓情于景的写法;(2)表达的情感不同:诗句流露出作者重获自由的喜悦、欢快之情;渔歌则表达了三峡渔民对人生艰辛的悲凉感慨。或语言风格不同:渔歌更口语化、更通俗。(同异各答出一方面即可)
(五)1.号求救于人/甚哀 2.①相互间隔一里左右 ②头胀痛得厉害,走路都不知道走到哪里了。 3.越巫心理变化过程:先是强作镇静,再就是心慌意乱,最后是丧魂落魄(或六神无主)。 越巫是一个既无真才实学,又胆小如鼠的江湖骗子。
(六)1.(1)易:改变。(2)仕:做官。 2.(1)苟/慕义强仁者/皆爱惜焉 (2)③ 3.B 4.姑且凭你这次的前往测定一下吧。 5.①借乐毅的经历宽慰董邵南,说明政治上失意的人古已有之,对于“不得志”不要太在意。②借乐毅在赵受封一事暗示董邵南:有才能的人终会有用武之地的。
(七)1.①适逢,恰巧遇到。②揣度,推测,估计。③带领,率领。④爱。2.C 3.①天下百姓被秦王朝统治,受苦受难已经很大长时间了。(天下百姓长期受秦王朝统治,痛苦不堪。)②如今果真把我们的人冒充公子扶苏和项燕的军队,向天下发出倡导,应当有很多响应的人。 4.有卓越的领导才干和超人的才略胆识
(八)1.有时(或许)  喂养(或通“饲”)  等同(或“—样”)    2.怎么能要求它能够日行千里呢?    3.策之不以其道,食之不能尽其材,鸣之而不能通其意。   意思接近即可。
(九)1.①安求/其能千里也 ②食之/不能尽其材 2.①这②同“饲”,喂养 ③鞭打(驱使) ④有时 3.C 4.能识别人才的人 食马者 5.略。
(十)1.①“仓”同“苍”,青色。②道歉。 2.我明白了:韩国、魏国灭亡,可安陵国却凭借五十里的土地幸存下来只是因为有先生啊。 3.①你最好是将你们的土地奉送给我,不然的话我将发怒,那后果将不堪设想。 ②我将效法他们三人,与你同归于尽。 4.首先,唐睢针锋相对,寸步不让,用“布衣之怒”来对抗秦王的“天子之怒”。其次,唐睢列举了三个布衣之士的例子来反驳秦王,并表示要效法这些有胆识的勇士,不畏强暴,刺杀秦王。最后,唐睢“挺剑而起”,以死相拼,舍生取义,挫败秦王。 5.示例:①墨子 阻止公输盘和楚王攻打宋国。 ②晏子 出使楚国,令楚王自取其辱。 ③诸葛亮 舌战群儒,联吴抗曹。……
(十一)1.①凭依(或:凭借、依仗)②美丽 2.A 3.句子:风烟俱净,天山共色。翻译:烟雾都消散尽净,天空和群山呈现出相同的颜色。(或:没有一丝儿风,烟雾也完全消失,天空和群山是同样的颜色。) 4.空一:从容出游,怡然自得(或:舒畅自如,游兴极浓等) 空二:千丈见底 空三:幽静(或:寂静、静等) 空四:鄙弃名利(或:淡泊功名、淡泊人生等) 空五:皆生寒树
(十二)1.消散干净 争着,竞争 伸向更远的地方 看,看到 2.C 3.清澈 湍急 4.负势竞上,互相轩邈;争高直指,千百成峰 5.寒 远上寒山石经斜或寒蝉凄切 6.D
(十三)1.(1)希望;(2)同“悦”,高兴;(3)明智,聪明;(4)引见 2.D 两次(A中两个“为”一是“替”.一是“造”;B中两个“以”一是介词“用来”,一是动词“认为”;C中两个“之”一是指“这件事”,一指“他”) 3.(1)先生有什么见教? (2)我已经对大王说过这事了。或:我已经向大王禀报过这件事了。 4.(1)智、仁、忠、强、义。 (2)反对不义战争、能吃苦耐劳(不畏艰辛)、坚守正义、善于说理(能言善辩)、机智等。
(十四)1.永州八记 2.①通“戮”,这里是遭到贬谪的意思。②穷尽。 3.空隙时,就漫步而行,漫漫而行…… 4.故意言过其实。既表明对贬谪流放的愤慨,又引起下文的“出游”。
(十五)1.①感到奇怪。 ②从前。 2.是:这。示例:是亦不可以已乎?/是马也,虽有千里之能,食不饱,力不足,才美不外见。 3.启发或感悟可从以下角度回答:①只有志存高远,胸襟开阔,才能有所作为。 ②人与自然应该和谐相处,甚至达到天人合一的境界。 ③只有躬身实践,善于探索发现,才能正确认识事物。
(十六)1.时:季节 陈:陈列,摆开 2.野花开了,有一股清幽的香味,好的树木枝叶繁茂,形成一片浓郁的绿阴 3.朝暮四时游玩之乐 4.美丽的山间景色 百姓安定富足的生活
(十七)1.欧阳修 2.C 3.云归而岩穴暝 风霜高洁4.①一座四角翘起像鸟展翅欲飞似的亭子,高踞在泉水上边。②(欣赏)山水的乐趣,领会在心里,寄寓在酒上。5.本意不在此而在别的方面。
(十八)1.①B ②C ③B ④A 2.湖面上金光闪烁,月影犹如一块璧,静静地沉浸在水底。3.壮阔、浩渺 4.阴晦萧索 晴朗明丽 悲凉低沉 喜悦昂扬 5.答案示例:①范仲淹为天下先忧后乐。②欧阳修与吏民同醉同乐。
(十九)1.①却、可是②这③护城河④放弃 2.①震慑天下不能单靠武器装备的精良。(或“震慑天下不能靠武力的强大。”) ②帮助他的人少到了极点,亲属朋友都背叛他。(把“亲戚”译成“内亲外戚”亦可) 3.D 4.天时不如地利,地利不如人和。 5.“人和” 得道者多助,失道者寡助
(二十)1.①所以 ②这种人 2.这样人们才会明白忧患使人生存发展,而安逸享乐使人萎靡死亡。3.略(所举事例要实事求是,并具有较强的说服力。) 4.略(不论考生就一点来谈或两点结合起来谈,不论考生持什么观点,只要所言有积极意义,符合情理即可)
(二十一)1.陈元方、友人;陈元方:不卑不亢、倔强任性;友人:误时无礼、错而能改。 2.起因:陈太丘与友期行;发展:友人日中不至乃去;高潮:元方据理驳斥友人;结局:友人惭而致歉。 3.对:①耐心说明;②据理反驳;错:入门不顾。 4.略
(二十二)1.A 2.①生命,是我所喜爱的,大义,也是我所喜爱的,如果这两样东西不能同时都具有的话,(那么我宁愿)牺牲生命而选取大义了。 ②不仅仅贤人有这种本性,人人都有这种本性,(只是)贤人能够不丧失罢了。 3.舍生而取义;用“舍鱼而取熊掌”作比喻提出的。(答成用“鱼”和“熊掌”作比喻提出的也可) 4.参考:文天祥宁死不降元。朱自清宁愿饿死也不吃美国的救济粮。
(二十三)1.①放弃 ②祸患、灾难 ③践踏或踩 ④停止 2.D 3.①这两种东西不能同时得到,我就放弃生命而选择正义。 ②吆喝(或:喝斥)着给他,过路的人都不会接受。 4.赞扬舍生取义;批评苟且偷生(见利忘义)。看法:(言之成理即可)。
(二十四)1.①苟且取得,这里是“苟且偷生”的意思 ②祸患,灾难;“辟”通“避”,躲避。 ③“乡”通“向”,从前。 ④停止,放弃。 ⑤天性,天良。 2.用现代汉语翻译下列句子:①鱼,是我所喜爱的,熊掌,也是我所喜爱的。 ②不仅贤人有这种本性,人人都有,不过贤人能够不丧失罢了。 ③用脚踢着(或踩过)给别人吃,乞丐也不愿意接受。 ④(有人见了)“万钟”的优厚俸禄却不辨是否合乎礼义就接受了。(大意正确即可) 3.①道理 举例答案示例:南宋末年,文天祥组织力量坚决抵抗外来入侵,失败被俘后,面对元朝的威逼利诱毫不动摇,视死如归,最终被杀,他这种高尚的民族气节和为正义献身的精神是永远值得后人学习的(大意正确即可得分)。 ②答案示例:a.生于忧患,死于安乐。b.得道多助,失道寡助。(只要是孟子有哲理的名言均可)
(二十五)1.①亲自,自身 ②地位低下,身份低微;低微鄙陋或身份低微,见识短浅 ③感动,感激 ④奔走效力(或奔走效劳) 2.愿陛下亲之信之/则汉室之隆/可计日而待也 3.正反论证 划线句子为“亲贤臣,远小人,……此后汉所以倾颓也。” 4.北定中原 攘除奸凶 兴复汉室 还于旧都 5.心情:夙夜忧叹 行动:五月渡泸,深入不毛 6.在兵败的时候接受重任,在危难的关头奉命出使。(只要能表达出原句的大意,语句通顺即可。) 7.在当时对(用人等)内政建设方面有进步意义;在今天仍然有借鉴意义。(只要结合文段,言之成理即可。)
(二十六)1.A希望 B勉励 C景象 D从前 2.且夫人之学也,不志其大,虽多而何为? 3.译:太尉(您)如果认为我可以教育就请(您)屈尊教导我,那就太幸运了。 4.能。“书”中言辞恳切,感情真挚,表现出苏辙虽然年轻,但才华不凡,勇气超群,颇有见解。
(二十七)1.D 2.(1)大大小小的案件,即使不能一一细察,也一定要依据实情来处理。(2)(这是)尽了本职的一类(事情),可凭借(这个条件)打一仗,(如果)作战,就请允许我跟着去。 3.夫大国,难测也,惧有伏焉 4.鲁国获胜的主要启示有,获胜的根本是取信于民,获得人民的支持;战争中要善于分析敌情,把握战机;统治者应善于听取并采纳群众的意见……(学生结合课文任选一个角度,联系实际谈出看法,言之成理即可)
(二十八)1.⑴热水 ⑵通“披” ⑶因为 2.因为内心有足以快乐的事,不觉得吃的穿的不如别人。3.计日以还 不敢稍逾约 4.用自己的切身体会勉励马生勤奋学习。(意思对即可)
(二十九)1.高祖/因之/以成帝业。 2.A 3.①此诚不可与争锋。②西和诸戎,南抚夷越。
(三十)1.C 2.A 3.①认为 ②指责 ③满一年 ④朝见 4.①在公共场所批评议论(我的过失),传到我耳朵里的可得下等奖赏。 ②这就是所说的在朝廷上战胜(别国)。 5.表明了齐王对邹忌劝谏的肯定(或“赞同”)(或表明了齐王接受邹忌劝谏的态度)。 6.第一问原因是:一是有邹忌这样的敢于直谏且善于劝谏的忠臣,二是齐威王善于采纳群言。(意思相近即可)第二问是一道开放性试题,能结合自身生活体验谈看法且言之有理。
(三十一)1.把肉晾干 等待,含有“希望”之意 比 2.异 赋敛之毒甚是蛇 3.D 4.略
(三十二)1.③ 2.D 3.对于贫弱的人,用仁义抚尉他们(用仁义安抚贫弱的人)4.①强者绥之以德,弱者抚之以仁,恣其所安 ②家君没有效法袁公,袁公也没有效法家君
(三十三)1.①极:穷尽;②易:改变。 2.D 3.临川城东,临着溪水有一个缓缓隆起的高坡,称做新城,新城之上有一个很深的长方形水池,就是荀伯子在《临川记》中所说的王羲之的墨池。 4.①莫非他们所下的学习功夫不如王羲之吧?②只要多读书多写作,自然会精湛的。 5.勤学苦练的观点。还需要的条件如:明确的学习目的,科学的学习方法,理论联系实际,用心专一等。
(三十四)1.本文写吕蒙在孙权劝说下开始学习,其才略很快就有惊人长进的故事。(意对即可)2.告诫儿子要注重修身养性,生活节俭,以此来培养自己的品德。(意对即可) 3.①研究儒家经典。经,指《易》《诗》《书》《礼》《春秋》。 ②另眼相看,用新的眼光看待。刮目,擦眼。 ③ 用简朴来培养自己的品德。 ④ 振奋精神。励,激励,振奋。 4.①你说事务多,谁比得上我事务多呢?我经常读书,自以为大有益处。② 学习必须身心宁静,才智必须经过学习,不学习就不能使自己的才学广博起来,没有志向就不能使自己的学识有所成就。(意对即可) 5.用自己读书的体会,现身说法。(意对即可) 6.提示:本题可就学习的重要性,以及志向、目标、修身、养德等其中一个方面谈自己受到的启发或教育。(只要言之成理即可)
(三十五)1.①告诉 ②离开 ③用 ④于是,就 2.C 3.例如:荀巨伯不忍丢下有病的朋友独自避难,而且愿“以我身代友人命”,这种把情义看得比生命还重的精神实在难能可贵。
(三十六)1.喝得痛快——腰间插着——已经——从袖子里 2.⑴客人中有姓郭、姓尤的两个人,很懂音乐。 ⑵在座的客人都举杯痛饮而纷纷醉倒。 3.抛砖引玉或醉翁之意不在酒。
(三十七)1.①坏、毁坏;②本来;③模仿、仿效;④理睬。2.①用船运回家并向别人夸耀。②等到敌人侵入他们的国境。3.①不要不懂装懂,胡乱吹嘘②不要轻信,不能盲目模仿③不能凭主观判断事物,应认识事物的本质 ④不要自以为是⑤要听从劝告,接受正确的建议
(三十八)1.D(“下”是“使……下车”之意) 2. 君王(大王、您)将要去哪里(到哪里去)? 3.贤明,敬重人才,嫉恶如仇。
(三十九)1.①告诉 ②地方或处所。 ③全。 2.一位富家子弟熟背驾船方法,夸夸其谈,却没有实践经验。一次采宝途中,船师猝死,由他驾船,结果船沉人亡。 3.①理论联系实际,学以致用。或:既要重知识,也要重能力。 ②不盲从,不轻信。或听其言,观其行。
(四十)1.这样尝试着和别人换一下位置,平心静气地揣度它,我果真没有一点过失吗?或:如果设身处地,平心静气地揣度它,我难道没有一点过失吗? 2.严以律己,宽以待人。或:治学要严谨,不要自以为是。 3.指出现象→讲述道理→得出结论 或 提出问题→分析问题→得出结论 4.示例:①这种行为不妥当。对古书没有认真分析研究,不要妄加评论和否定。②对这种行为不能简单肯定或否定。读古书要取其精华,去其糟粕。③这种行为有可取之处。“尽信书不如无书”,要敢于质疑,大胆质疑。
(四十一)1.B D 2.渡者根据自己的生活经验,在仔细观察小书童(小书童人小书重,书捆得又不结实)后,作出了这样的应答。(如用文中的句子回答也可,例如:小奚以木简束书从;时西日沉山;望城二里许;渡者熟视小奚等。) 3.做事应有条不紊,不急不躁,如盲目求快,则可能事与愿违或欲速则不达。
(四十二)1.A.转移  B.对……说。 2.B 3.A.“坐”同“座” B.“亡”同“无” C.“寤”同“悟” D.“畔”同“叛”。 4.以前如果你能听取那个人的建议,就不需要演样摆酒席请客,也就不会发生火灾。 5.①人要善于听从别人的建议,知错就改。 ②做任何事都要有预见性,才能有备无患。

第十讲 记叙文阅读
(一)1.“它可以联结亲情,联结友情,联结爱情。”或“拥有了牵挂,便拥有了感情的寄托。”或“常能给人以无穷的力量和勇气。”或“牵挂,让我们在生活中品味酸甜苦辣,更为生活增添了一道亮丽的风景”或“让人生变得更加丰富多彩,让世界变得更加绚丽多姿”。 2.从“牵挂别人”和“被别人牵挂”两方面作答即可
(二)1.它的飞行时间之长、速度之快、动作之敏捷,堪称鹰中之最。 2.①雕鹰出生不久,就要经受千百次残酷的训练,否则,就不能获得食物。②母鹰把幼鹰从高处摔下,胆怯的幼鹰便被摔死。③母鹰折断幼鹰翅膀中的大部分骨骼,并将幼鹰再次从高处摔下,有很多幼鹰成为飞翔悲壮的祭品。 3.①折翅训练(最后一次残忍的训练)②激情、意志、勇气和希望 4.①写出了母鹰内心的矛盾和最终的选择。母鹰用近乎残忍的训练为孩子未来的生存打下坚实的基础。②本句充满了哲理。不管外界条件如何,成功最终靠自己。 5.①为了孩子的生存,动物对子女的训练是残酷的。②磨难成就了一批成功之士。③溺爱令人担忧。④“残忍”的爱比溺爱更有价值。……
(三)1.yuān piāo yān háng 2.架、震惊、心甘情愿、衔接 3., , , , 4.不畏强敌、团队战斗 为了族群,牺牲自我的献身 5.通过场面描写 直抒胸臆的方式(直接抒情) 6.虚写 、实写 ;映衬烘托 7.运用比喻,形象写出战斗之激烈(“暴风中飞转”、“漩涡状”),鹤群阵容之庞大(“云堆”),形态之优美(“上下左右飞舞”,“光莹莹”),场面之壮观 8.抓住眼神、咩声、步伐等方面,想到处境的危急,诀择的痛苦,飞渡的残酷,结局的悲惨。 9.主仆的关爱之情、鲁提辖(鲁达)和金氏父女、父子亲情,真挚爱情。
(四)1.不叫 叫 不停地叫(或不知疲倦地叫) 2.时间 蝈蝈 3.拳头大小的 不安地 可怜巴巴地 仿佛在呼唤、祈求 4.①把它关在笼子里,它生气呢! ②不!它是在哭呢!关在笼子里多难受,它在哭呢! 5.蝈蝈是白发老人放走的 因为他听到孩子们动情的议论之后“叹息了一声” 6.作者意在启发我们要与动物和谐共处,要与自然和谐共存(与之相近的表达也可)(与之不同的有创意的合理看法也可)
(五)1.书 讨书(或乞书、要书) 买书(购书) 2.拍、扔 财大气粗,没有修养(只填“粗野”、“粗俗”亦可) “我才懒得选” “只要是书,只要有名气,都要” 不爱读书(填“不学无术”、“虚荣心极强”、“精神贫乏”等亦可) 3.例:一组:方先生与乞丐的对比。方先生有别墅,可没有一本书,说明他不爱读书,这与乞丐想读书形成了鲜明的对比。一个是精神乞丐,一个是物质乞丐,这一鲜明对比,更加突出地揭示了文章的主题——物质的匮乏并不可怕,可怕的是心灵的荒芜、理想的氓灭。(或:这一鲜明的对比,更加突出地揭示了不读书者的可悲。)二组:申女士的言行与方先生的言行形成了鲜明的对比。方先生见到乞丐以调侃的语气说话,粗俗傲慢;而申女士对乞丐是“和颜悦色”地说,礼貌诚恳。这一鲜明对比,突出了申女士“腹有诗书气自华”的彬彬有礼、知书达理的形象,衬托出方先生是一个不学无术、粗俗鄙薄的人。三组:方先生整个别墅没有一本书与申女士随身携带书形成了鲜明对比,这一鲜明对比,衬托出方先生是一个不学无术、不爱读书的人。 4.书不是用来装点门面的,用书装点门面,是对书的糟蹋。书应该属于那些热爱书、热爱知识,把书视为“有灵魂的生命”的人。 5.例:我觉得书是有灵魂的生命。我曾读过《钢铁是怎样炼成的》,保尔·柯察金面对一次次挫折,仍执著地追求理想的精神,让我震动。每当我遇到困难或挫折时,保尔·柯察金的形象便浮现在眼前,使我增强了克服困难的信心。 6.略。
(六)1.①轻轻地,小心呵护。 ②太阳光线不强烈,热度不够。 2.蓦然看见,一个课桌的缝隙里,有一撮小草芽。 3.不好。因为是“小野花”,极其娇小,用“染”非常恰当,用“插”不能准确地表情达意。 4.①交代了时令、地点;②描绘了早春的景象;③以野外的缺乏生机、暖意的春景引出教室内孩子们对春的热望、生机和活力,以此形成对比,起了反衬作用。 5.因为孩子们以及他们身上所表现出来的活力和精神原本就是春天,他们就是春天的象征。或答第⑸段“春天是从孩子们身上产生……染了山川”的一段话。
(七)1.A.偏僻 B.垂涎 2.总是 永远 (或:似乎) 3.答案要求:至少表现两方面内容,一方面,担心秦霜看见自己,造成秦霜的尴尬难堪;另一方面担心别人知晓秘密,会给秦霜带来伤害。 示例:秦霜,我就在窗外,你可不要向外看啊!此地不宜久留,赶紧走。我一定要装着像没有看到一样。绝不能让别人知道! 4.“我”的行为对秦霜产生了巨大的影响,它所表现出来的真诚、善良、宽容的品质是无价的。 5.糖罐(或:糖) 偷糖(或:吃糖) 补糖(或:倒糖) 念糖(或:谢糖、想糖、感糖) 6.可从不同角度作答,言之成理即可。 角度示例:①能宽容同学的一时过失并能真心帮助其改过自新。②知错能改。
(八) 1.字面上是指女儿拉着母亲的手,深一层是女儿在母亲的爱和引导下成长       (也可说母女相依相爱,永不离弃)。 2.(1)很小时拉(攥)着母亲的食指; (2)稍大时用胳膊勾住母亲的小臂;(3)十三四岁还把母亲的手掌攥在自己的手心。作用:写出了女儿 在母亲的牵手中长大了但依然爱恋信任母亲。 3.(1)爱女儿,扶助女儿是做母亲的天职 ⑵而女儿对母亲的信任和依赖 (或说“爱恋”)是永远不变的。 4.一位母亲对女儿无尽的爱,愿为女儿付出一切并希望女儿永远依恋和信任母亲。 5.答“可是妈妈老了之后,你还能这样紧紧地牵住妈妈的手吗,我的孩子?”或“到那时候,你会牵我的这只手吗,我的孩子,我的女儿?”(两句中任意一句均可)
(九)1.生机;气势;活力(文字不求一致,意思对即可) 2.形状;态势;花色(文字不求一致。) 3.为了衬托油菜花的黄。或:为了更加突出油菜花的黄。(意思对即可) 4.油菜花唤醒了“我”生命中沉睡的信念,使“我”认识到生活是美的,生命是可恋的。(意思对即可) 5.油菜花遍地都是,呈现一片黄色,犹如遍地铺满黄金;油菜花在“我”生命中具有重要的影响,像黄金一样宝贵,值得珍视。(意思对即可)
(十)1.只有1%的希望,就用99%的努力去争取 2.为下文爹不为说情所动作铺垫,同后文爹的态度改变形成对比,使文章富有感染力 3.分别写出文因高考落榜的懊丧和复读无望的辛酸之情 4.这样层层蓄势,来表现 “文”的锲而不舍的精神 5.详写“文”争取复读的过程,表现“文”的坚定信心,而复读的努力,考上大学的结局可想而知。 6.“文”是一个认定目标不言放弃的有志青年
(十一)1.(言之成理即可)作者在成长岁月中感晤到的至真至诚的亲情。 2.(言之有理即可)表现父亲对远方儿子的牵挂和企盼。 3.(无错别字且书写工整即可)报得三春晖。 4.(结构基本相似,能用比喻的修辞手法,意思基本正确即可)校园是一畦充满希望的苗圃,让无数学子一批又一批地成为参天的大树;校园是一艘乘风破浪的船,船里总装着老师对学生的情和爱。 5.(理解言之有理即可)例:做个赢家吧,赢家有能力随遇而安。 要做生活的强者,必须有平常的心态,有主动适应生存环境的能力。
(十二)1.①甲:B 乙:C ②语言描写。 2.如:“左手揽住母亲的脖子,右手揽住她的腿弯,使劲一抱……” 3.①嘲笑“我”不了解、不理解母亲生活的状况。 ②母亲轻得出乎意料之外,表现出“我”的惊讶。(意思接近即可) 4.母亲为自己一生的辛劳终于被儿子所理解,看到儿子终于“长大了”,自己终于得到了些许的回报、安慰、感动幸福等。(答出一点即可。答案只要符合文意,言之成理即可) 5.第一次抱母亲(若有其他答案,只要合理,亦可) 6.略。
(十三)1.她认为自己长大后,生活的性质不会有什么大的变化,只不过生活的形式有变化而已,生活的沉重使她不敢想,也不愿想。 2.孩子们为自己没有对父母表达过自己的爱和感激之情而羞愧、自责和思考。 3.①示例:比喻:“这个想法就明明亮亮冒了出来,像一片初生的青草,娇娇嫩嫩,又微微酥麻,不停地在她心中温柔地擦来擦去”形象地说明了“她”的这个想法形成时的情形,将抽象的想法具体化、形象化了。②比喻:“生活没有一丝波澜”用水的波澜比喻生活中的曲折。③拟人:“石竹心思多,石竹护孩子”用拟人的手法,说明石竹的善良和慈爱。④反问:“在他们这个家,即使是一小块肥皂都要精打细算地用,怎么可能花钱去买一束花,一束花店里的花呢?”用反问的手法,强调了女孩家中生活的窘迫,买花不易。 4.A
(十四)1.①臃 ②懵 2.①“我”对年轻女子的信任、热情。 ②对自己的行为感到后悔并怀疑年轻女子是骗子。 ③对误会年轻女子感到愧疚并被她的行为深深感动。 3.参考示例:①我曾经收到一个读者送给我的礼物,是两个青木瓜。 ②我怎么也忘不了两个青木瓜,因为这两个木瓜的清香,现在还在我心中萦绕。 ③我怎么也忘不了两个青木瓜,这两个青木瓜寄寓了一位真诚的读者对我的一片深情和厚望。 4.参考示例:①“意外相逢”指相互认识的人意外相见。因为年轻女子读过“我”的书,书上有“我”的照片,她和“我”仿佛是熟人,对年轻女子来说是“意外相逢”。②“萍水相逢”指从来不认识的人的偶然会面。因为“我”与年轻女子在此之前从未谋面,对“我”而言,不能是“萍水相逢”。 5.①模样 身材 打扮 ②质朴(或:朴质、朴实、纯朴) 6.参考示例:①有道理。因为社会上确实有不少这样的骗子存在,中国人有一种传统观念是“防人之心不可无”。 ②家人的话没有道理。因为人与人之间应该真诚相待,多一份理解,多一些信任。 7.①写出青木瓜本身特有的清香味道 ②写出年轻女子身上体现出来的真诚、热情 ③写出人与人之间应当有的真诚与信任 8.参考示例:我是在邮局前和您相遇的那个人,过年回家给您带了两个自己家种的木瓜,表示一点心意。祝您写出更多更好的作品。 9.参考示例:①本文以小见大,借一件小事表达了一个值得重视的社会问题。 ②文章最后一段的句子,表意含蓄丰富,有余味。 ③运用了悬念,使文章波澜起伏。 ④文章略写了年轻女子的生活经历,详写了“我”与年轻女子交往的前前后后,突出了中心。 ⑤文章细腻地写了“我”的心理活动,以“我”的感情变化贯穿全文,不仅使人物的形象更加丰满,而且使文章的线索更加清晰。 ⑥文章第⑶段中的省略号省略了年轻女子与本文主题无关的其他话,使文章的文字更加简约。
(十五)1.(季羡林先生)为一个年轻学子看行李。 (冰心老人)病魔缠身仍关心年老病人的状况。(以上回答意思对即可。) 2.冰心虽然离我们远去了,但留给我们的是高尚的人格和博大的爱心。(意思对即可) 3.季先生的人格魅力在学子心中留下的投影。 4.先成为—个优秀的人,然后成为—个优秀的新闻人,最后自然地成为—个优秀的节目主持人。 5.关爱一切生命,尊重每一个人。(意思对即可) 6.不设统一答案。要求:写出自己阅读后的人生感悟,观点明确,语言通顺。
(十六)1.答案要点:突出了小乞丐劳苦的样子。 2.“这样”指匆匆走过。 想帮助他。 3.涂着。 “我”想消除他内心的孤寂。 4.答案要点:①内心受到触动 ②恼怒 ③悲伤 ④委屈 ⑤想躲避 5.答案要点:相同之处:担忧(惦念) 不同之处:“我的心很沉重”,还含有自责的意思。 6.答案示例:“我”不再只是简单地把他看成是乞丐,而是逐渐意识到他是有心灵、应该给予关爱的孩子。 7.略
(十七)1.示例:特蕾莎修女是一位把毕生精力投入到慈善事业中的人。2.示例1:第②段的事例描写了特蕾莎修女去世时人们悼念她的隆重场面,借雨和泪突出了人们对她的真挚情感,从侧面烘托出了她在人们心目中的崇高地位。 示例2:第⑥段中特蕾莎修女面对诺贝尔奖巨大的荣誉,她只是用朴实的语言提醒大家关注贫穷世界。这个典型事例突出了她对穷人的关心,表现了她善良美好的心灵。 3.示例:美不仅是外在的容貌、漂亮的衣着,美更应该是爱的代名词。简爱容貌普通,却赢得了最美好的爱情;德拉剪掉了美丽的金发,在杰姆的眼中却更加动人,这都是因为她们心中有爱。美,只有与爱结合,才能散发永恒的魅力。 4.示例:人物——杨利伟。他只是一个普通的军人,但他却是实现中华民族数千年飞天梦想的第一人。当成功的花环将他簇拥的时候,我们没有忘记他为这一天流下的汗水,做出的牺牲;没有忘记他超乎常人的坚毅、执著和镇定。不管过去了多少年,我们都能回忆起他自信的目光和灿烂的笑容。
(十八)1.①“父亲”把《送别》中的“长亭外”误听为“长城外”。 ②“我”把“乌秋”误认为“燕子”。 2.第一件事与第二件事性质相同,都属于“美丽的错误”;作者由第一件事自然地引出了第二件事。 3.可惜那首歌唱的不是自己的家乡,可惜自己感动了这么多年竟然是听错了。 4.表现了作者突然发现“燕子”的惊喜之情;同时也表现了作者心中对故乡深深的思念之情。 5.虽然有的时候,在人生的道路上,我们是应该面对所有的真相,可是,有的时候,我们实在也可以保有一些小小的美丽的错误,与人无害,与世无争,却能带给我们非常深沉的安慰的那一种错误。 6.作者从小就远离故乡,“燕子”早已成为故乡的一种象征,寄托着作者浓郁的思乡之情,所以作者无论如何也舍不得自己心中那一只小小的燕子。 7.示例:燕子啊,你可是从我遥远的故乡飞到了我的身旁?你的声声呢喃,可是故乡人托你带给我的问候?燕子啊,你可以自由自在地飞来飞去,什么时候我能够像你一样自由地飞回自己的家乡?
(十九)1. (父亲打造的)斧头;“我” 2.善于教育孩子,手艺高超,做事从容不追,干自己爱干的事 3.①运用拟人手法,形象生动地写出了麦子等待收割的情形。 ②抓住句中重要词语(如“一声不吭”“狠狠”“啃”等),分析父亲的形象或教育孩子的方式,言之成理即可。 4.第一问:没干成一件事;做事没有耐心;总喜欢想入非非。 第二问:踏踏实实做事;耐心地采草药;帮父亲打造斧头;专心学习等。(根据文章内容合理想像,答出一点,言之成理即可)
(二十)1.“我”听到了下岗夫妇的艰辛与收获,听到了小姑娘的勤奋与进步。应该多为他人着想,不能只为自己着想。(意思对即可。) 2.(1)指楼上楼下的吵闹声弄得我精神不振,心情烦躁。(2)形象生动的写出了楼下摩托车声的难以阻挡及对我的影响,突出了“我”心里的烦躁。(意思对即可。) 3.不设统一答案。答题要点:句中的观点是基于一定的情境的,并没有绝对化(文中有“许多”一词的限制,即可说明),不能断章取义。如对句中观点持否定意见,没从“正确理解”的角度作答,只要言之有理(类似于楼下楼上值得同情、可以理解的声音用爱倾听是适用的,反之则不适用)也可。 4.原因:希望孩子将来能成才或望女成凤等。(意思对即可。) 看法:没有标准答案。 5.不设统一答案。如持肯定意见,可围绕女邻居和莎利文老师、露茜修女一样都有爱心和毅力,且她的努力已初见成效等内容作答;如持否定意见,可根据女邻居虽经多年努力,小男孩也没有学会简单的语句,海伦和丹只是一个特例等内容作答。(观点明确,理由充分有说服力,语言通顺即可)

第十一讲 说明文阅读

(一)1.“勇气”号探测器是人类遣往其他行星的第一个可移动的、自动化的、用来对火星进行科学探测的大型实验室。 2.⑷ ⑺ 3.答案要点:①了解火星及其演化过程;②了解地球自身的过去和未来;③探测火星上是否有生命;④探测火星上是否有水;⑤为人类进一步探索银河系打开一扇希望之门;⑥探索太空奥秘是人类的千年梦想。 4.(甲)句:“以类似人的视角环视四周”“这儿瞧瞧,那儿看看”形象地写出了“勇气”号寻找探测目标时视角广阔、动作灵活的特点,写得饶有趣味。(乙)句:用“敲敲打打”“搜集”“前进”等词语形象地写出了“勇气”号工作时像地质学家一样科学严谨、有条不紊;语言生动有趣。5.第一问答案要点:①孩子代表人类的未来和希望;②相信孩子具有丰富的想像力;③激发孩子探索宇宙奥秘的兴趣;④从小培养孩子的科学精神。 第二问:因为人类探索宇宙奥秘需要能力,需要有不畏惧任何艰难险阻的精神。
(二)1. shèn 2.“控制和消除土壤污染的办法(措施)”或“如何控制和消除(治理)土壤污染” 3.“强调(突出)了经济损失之大”或“强调(突出)了土壤污染导致严重的直接经济损失”。 4.吸收有害物质;集中焚烧处理 5.①土壤污染会危害人(畜)健康 ②因为原文是按照由主要到次要的顺序来说明的 6.示例:还我青山碧水,还我土地芬芳。 保护环境,人人有责(从我做起)
(三)1.领起下文 或:承上启下 2.示意图上方从左到右依次为A、C 图右侧上为B 3.泥沙淤积河床抬高(或:水少沙多 地上悬河)的老问题,又添“生产堤”的新问题 4.综合目标调控作用巨大或:四库联合调度和水沙联合调度作用巨大 5.A类:不是常规调度使用(在正常情况下属违规调度)B类:小浪底水库过量拦洪不属于常规调度 6.①处理好局部救灾和整体防洪的矛盾 ②依靠科技 或:加快建设“数字黄河”工程
(四)1.①纽     ②栖 2.森林能产生直接的经济效益,同时又有巨大的生态环境效应。(或森林资源是人类赖以生存的基础资源。同时,森林还具有维护地球生命、改善人类生存环境的生态价值。) 3.①动物资源    ②非生物资源    4.具体而准确地说明了森林在涵养水源方面的重要作用。 5.不可以。森林只是吸收部分二氧化碳,降低温室气体浓度,减轻温室效应;不能吸收所有温室气体,使温室效应消失。(意思对即可) 6.森林能够有效地净化空气和降低噪声。 7.这是一道开放性试题。
(五)1.①动物世界最明显的特点是躯干部分两侧的对称性;②艺术家利用对称创造出美;③对称的现象引发了科学家对未知领域的探索。2.示例:文中列举生活中极常见的例子,生动具体(深入浅出)的揭示了对称美的原因。3.示例1:对称创造出的是整齐、端庄的美,故宫因此显得气势恢宏;不对称创造出的是自然的美,苏州园林因此给人以“如在画图中”之感。示例2:静立的马是对称的,但它只有自然之美;“马踏飞燕”中的马打破了对称,呈现出了艺术之美。后者灵动的姿态,张扬的个性,体现出了艺术家对美的独特感受。示例3:如果说对称之美是一座庄严、凝重的高山,那么不对称之美就是一条欢快、灵动的小溪。对称能给人以整齐之感,如故宫建筑的恢宏、肃穆;不对称则给人以飘逸之感,如苏州园林的自然、闲适。不同的美感,源于不同的艺术风格。4.八卦炉中逃大圣
(六)1.空间育种(航天育种) 2.①在太空,物体处于微重力状态;②太阳光辐射比地面强得多,宇宙高能重粒子极其丰富;③太空环境近似真空,没有污染。 3.我国空间育种研究的成果。 4.加点部分去掉后,就不能形象生动地说明空间育种后的农作物的神奇,不能激发读者的兴趣。
(七)1.①引用杨万里的诗句增添了文章的文学色彩,以激起读者的阅读兴趣。(言之成理即可) ②花青素的形成是枫叶变红的主要原因。(意思正确即可) 2.花青素既具有碱的性质,能与酸性物质作用生成盐;又具有酸的性质,能与碱性物质作用生成盐。(意思正确即可) 3.由于深秋气温骤降,光照减少,叶片内光合作用制造的淀粉造成叶细胞内糖分的积累,促进了花青素的形成。(意思正确即可) 4.大自然中的植物之所以呈现出万紫千红的景象,主要取决于植物叶片所含的色素物质。(意思正确即可)
(八)1.作比较;时间 2.①以小电动机为动力扇动两只涤纶薄膜翅膀或“扑翼飞行” ②科学技术高度发展 3.①形象地表明了航空爱好者对扑翼飞行的研究历经艰难。 ②准确说明了各国航空界对“机械鸟”重视程度之深。) 4. ①幻想是发明创造的基础和动力或要大胆想象善于观察思考;②发明创造要靠不懈的探索。
(九)1.①说明病毒没有完整的生命结构。或“说明病毒没有赖以生存的配套设施”。 ②说明病毒不会因反复变异而使杀伤力越来越大。 2.四方面作比较:①体积大小②生命结构。或“生存的配套设施”③生存能力④对抗菌素的反应 3.病毒是没有完整生命结构的毫无独立生存能力的可以致病的微生物。 4.说明病毒并不可怕。或:病毒具有致命的弱点,对人类的威胁是很小的。
(十)1.(1)归罪 (2)比喻没有办法。  2.要点:(1)因为在人类文明的早期,时间和空间跨度都非常有限,人为干预很少,大规模的物种迁移难以形成,因此不会造成生态系统的严重失衡。(答到大意即可) (2)造成经济损失威胁生物物种 (3)“生物入侵者”是指原来生活在某地区的、通过非自然途径转移到新的生态环境中的生物物种。 (4)说明语言特点:①生动形象 ②准确; 举例略。  3.能联系材料和选文,谈出自己对外来生物物种迁移的认识,言之有理,语言通顺即可。
(十一)1.产生(形成)作用 2.负氧离子对人的健康长寿非常有益。 3.栽花种树、修建喷泉、添置负氧离子发生器等
(十二)1.洞穴巢 屋宇巢 编织巢 2.编织巢 3.“主要”表明“这种巢”绝大多数由大型鸟类所为,但不排除少数玲球小鸟“也在地面营巢”。用“主要”一词体现了语言表述的准确和严密(起到了准确严密的修饰限制作用)。 4.鸣禽是长子鸣啭,巧于营巢的雀形目鸟类。(或:鸣禽是雀形目鸟类,它们长子鸣啭,巧于营巢。) 5.在寒冷的冬天,鸟巢像人类的“家”(或:星罗棋布的鸟巢,或:鸟巢)给人们的生活带来了一道温暖而平安的风景。6.开放性答案,就“保护自然环境”、“维护生态平衡”、“爱护鸟类动物”诸方面之一谈感想均可。
(十三)1.a 2.E 3.ABCD
(十四)1.①火星的直径大约是地球的一半。②火星上有相当于地球上三分之一的重力。③火星上有相当于地球百分之一的大气浓度。④火星上有水。 2.祖布林:地球之外是否有生命。弗雷德曼:了解生命的缘起和归宿。 3.放在A处,“我们将不得不再次审视人类和地球的关系”才有依据,正因为不能往火星上移民,地球才是我们唯一的家园,进一步强调保护地球的重要性。 4.略
(十五)1.② 2.⑤ 3.弹射 飘流 4.种子远离母体,找到适于自己生存的地方(与之相近的表达也可) 5.第一,增加文章的趣味性,第二,引出对椰子等植物种子传播途径的介绍(与之相近的表达即可) 6.利用风力到处飘洒 附着在其他生物的身上带到远方
(十六)1.因为啄木鸟专门捕食危害树木的昆虫,保护了树木的健康成长。 2.引出说明对象,形象生动,增强可读性。 3.①围绕着树干螺旋式地向上攀登。②因为这样可避免重复劳动,又不会有所遗漏。 4.①作比较;②将啄木鸟足趾的排列与一般鸟类进行比较,从生理特征上更清楚地说明了啄木鸟的足趾刚劲有力,适合攀援的特点。 5.本文未曾提到的有关啄木鸟的内容均可。如“啄木鸟捕食森林害虫的相关数据。”
(十七)1.噪音对人的危害。 2.按噪音等级由低到高的顺序(或:噪音对人的影响程度的大小顺序;逻辑顺序)安排的。 更清晰地表明强度不同的噪音对人造成不同程度的危害(或:使文章条理更清楚)。 3.变得越来越差 列数字 举例子 作比较 4.A
(十八)1.因为它栖息着无数的海洋生物和供养着大量的鱼类(犹如“热带雨林”,生长着大量的动植物)。 2.(1)写徐闻珊瑚保护区的位置、面积及生成(发育)史。(2)徐闻西海岸珊瑚区形成的原因(环境)和珊瑚的种类繁多。 3.本题是开放性的,答题可从三个角度中任意一个切人:(1)写作上,用形色结合或动静结合的描写手法写得生动传神。前者如“那密密麻麻的鹿角、牛角、羊角几乎探出水面”,“‘鲜花’,橙黄蓝白红”;后者如“那密密麻麻的鹿角、牛角、羊角几乎探出水面”,“散落在丛林中的翡翠’、‘玛瑙’,形态各异”。(2)表达方式上用了具体描写的手法把说明对象写得鲜明、生动、形象,例如写珊瑚美如“鲜花”、“翡翠、玛瑙”。(3)语言运用上是不同于前两段,显得形象鲜明生动;多用比喻写出了珊瑚礁的美丽多姿,例如“翡翠、玛瑙”、“鲜花”。 4.(1)珊瑚礁形成不易而要破坏它却很容易,要避免重蹈覆辙。大意对即可。 (2)要避免过度开发,才能长久保留美好景观,使人类和自然界和谐相处,也是为子孙后代造福。
(十九)1.传统意义上的粮食指的是大米、小麦、玉米,海洋食物指的是“其它的能够满足人类营养需要的食物”。 2.举例子、列数据、作比较 3.其中仅海带中,目前产量比早先的野生状态提高了2000多倍。(用自己语言概括也可。) 4.不能删掉。因为它是捕捞海洋浮游生物的必要条件,如果删掉,就让人们误以为可以任意捕捞,不顾生态环境5.不超过10个字,题目设计语言精练,紧扣文章内容,突出海洋食物对未来的意义即可
(二十)1.①有效繁殖高附加值的牲畜②扩大繁殖优良品种动物③挽救珍稀动物④对于研究癌生物学……等有不可低估的作用。⑤研究为人类自身提供的“配件” 2.数字说明,比喻说明,举例说明,例子略。 3.议论,“科技越发展……问题。”
(二十一)1.太阳风。 2.因为来自太阳的物质流动时所产生的效应与空气流动十分相似。 3.①干扰地球磁场,使地球磁场的强度发生明显变化。②影响地球的高层大气,破坏地球电离层的结构。③影响大气臭氧层的化学变化,使地球气候发生反常变化。 ④影响地壳,引起火山爆发和地震。 4.不能。一般情况是指通常情况,不包含特殊情况,这样,语言更加准确。 5.作诠释、作比较、列数字(据)。说到二种即可。
(二十二)1.光脑 2.略 3.作比较、分类别、列数字 4.光子传播速度块,通讯耗时短 光的频带大,具有极大的信息存储量 5.“基本”说明大部分工作原理相同,也还存在一定的差异,表明说话留有一定的余地。 6.如数码电视、数码相机、电子计算机、神舟五号宇宙飞船等。
(二十三)1.总领全文,点明题旨。 2.应加入顿号。不加标点,可以一气呵成,突出作者所见水之多,衬托下文两处水(都江堰和九寨沟之水)之经典、耐看。 3.A 4.因为都江堰的水能服务于人类,服务于社会(意对即可)。 5.⑸⑹段,或⑹段 6.最富性格,最值得一看,最典型(有代表性),最有特点。 7.喜欢都江堰之水的,要抓住它服务于人类的精神或柔顺、可塑,长流不息的特点来阐发;喜欢九寨沟之水的,要抓住它启迪人类、净化心灵、包容一切等特点来阐发。
(二十四)1.生命(动植物)对气候的影响(调节) 2.时间顺序 3.①举例子 例子主要在第⑵~⑸段 ②列数字 例:1)“它们使空气中的二氧化碳含量高达今天的20倍。”2)“它们粗重的呼吸及对植物的破坏使空气中的二氧化碳高达今天的3倍。”(表时间的词语不属于列数字)③打比方 例:“地球的气候是由地球的两大生命王国——动物界与植物界的斗争所决定的。” 4.不能删除。“也许”表估计、推测,表明下文的结论是作者的一种推断,还未经科学证实,这样更能体现说明语言的准确性。 5.①通过光合作用从空气中吸收。 ②通过草地形成软土来保存。 6.略
(二十五)1.①飞船外壁与空气剧烈摩擦,整个机身变成一个炽热耀眼的火球。 ②宇航员与地面的无线电通讯暂时中断,连雷达也无法发现它的踪迹。(只要答出“整个机身变成一个炽热耀眼的火球”,“连雷达也无法发现它的踪迹”即可) 2.第一处:用具体数字让读者对第一宇宙速度有具体可感的认识。第二处:使读者对飞船当时的情况有直观、形象的认识。3.A.宇宙飞船返回舱(或宇宙飞船,或返回舱) B.避火衣 C.瞬时耐高温材料 4.引火烧身 5.①自己先燃烧起来,散去极大量的热量。 ②燃烧自己的同时形成一层厚厚的炭化层,有效防止外界热量传入舱内。
(二十六)1.南极鱼不怕冻的原因。2.作比较、列数字,说明南极鱼的血液具有抗冻特点。3.因为科学家的发现不是主观臆断猜出来的,而是经过实践探究(实验论证)才得到准确的结论;同时这也体现了说明语言的准确性。 4.逻辑。 抗冻糖蛋白分子能挤入互相靠近的两个细胞冰晶体的间隙中,从而阻止了冰晶体的生长。 5.海水含有盐等其它成分,凝固点自然自会降低,传统的“溶液的凝固点降低理论”。
(二十七)1.①雨水多了,森林能贮水;雨水少时,人们可利用森林涵养水源。②森林吸收大气中的二氧化碳,吐出新鲜氧气。(意思接近即可) 2.①地球上的森林有三分之一以上被采伐和毁掉; ②地球上的燃烧物增多,二氧化碳的排放量增加。 3.打比方(或作比喻) 形象、具体地说明森林“能吞能吐”的特征和在维护生态环境中的作用。 4.时间之短 破坏范围之大或数量之多 5.示例:①水土流失②动物家园遭到破坏等。 6.略。只要围绕“保持森林”提建议或拟标语即可。
(二十八)1.一是建筑物下的土质及地面的移动量有多少能够转移到建筑物上来;二是支柱的数量;三是支柱的牢固性;四是采用何种方式减少横摆度。 2.科学家找到了决定抗震能力大小的四个因素和解决问题的方法。有的国家在抗震建筑方面取得了一定的经验。 3.成本高昂。 4.略
(二十九)1.荒漠(荒无人烟)、干旱、高温。 2.罗布泊原来并不是一片沙漠;史书记载 斯文的描述 3.①植物枯死 ②胡扬林成片死亡 ③沙漠向湖中心推进。 4.提醒人们不要为了眼前利益,而破坏自然界的生态平衡。或提醒人们要保护生态平衡。或提醒人们要合理开发利用资源,否则,会遭到大自然的报复。 5.罗布泊的悲剧;罗布泊之死(标题能紧扣文意即可) 6.人们对塔里木河水的不合理开发利用,导致了罗布泊消失和胡杨林成片死亡,最终导致了沙漠化的扩大。(答出几者之间的因果关系即可) 7.前一问:严格控制塔河两岸开荒;退耕还林还草;推广节水农业;由博斯腾湖向塔河下游输水。
(三十)1.比喻从观察到的部分,可以推测出全部或全貌。 2.主要说明纳米技术的发展及其广泛应用的价值。 3.纳米技术也称为超微科技,是研究制造极其微小物体并加以应用的一门新兴的科学技术。 4.①纳米技术使计算机信息存储芯片体积越来越小,存储量越来越大。 ②使用纳米技术可以选定原子来构成分子,制造新物质。 ③纳米技术可以制造使人无法想像的新材料。 5.使文章生动而富有趣味;使纳米技术这一抽象复杂的科学事理变得通俗易懂,易于理解。 6.答案示例:名称:纳米课本 特点:可读、可视、可听 用途:广泛应用于教学和阅读之中。
(三十一)1.B 2.例:“一把钥匙开一把锁”运用了打比方的说明方法,形象地说明了一种酶一般只对一种生物化学反应起作用。(文中两个“例如”用来说明酶的专一性) 3.正常情况下,一种酶只对一种生物化学反应起作用:不排除有的酶在特殊情况下对多种生物化学反应起作用,因此,加上“一般”能体现说明文语言的准确性。 4.特性:专一性(或“一种酶一般只对一种生物化学反应起作用”) 形状特征:与它们所催化的物质的形状恰好咬合 作用:催化作用(催促反应的快速进行) 5.因为洗衣粉中的酶能加速汗液中蛋白质的分解,因此,衣服就洗得干净。
(三十二) 1.C 2.日本索尼公司推出的超薄型电子书优势更明显。从质量上看,它大大小于日本松下公司的“西格玛书”电子书;从功能上看,“西格玛书”电子书只有单纯的读书功能,而索尼公司的超薄型电子书不仅具有读书功能,而且还在电子书内装置了4本电子词典,附带声音播放和检索功能,可用于朗读文章和学习外语。 3.功能的完善体现在阅读时可以观看彩色图片,还可以观看带插图的小说以及彩色漫画等。解决的问题是如何减轻产品质量,因其质量远远超过了松下公司和索尼公司所推出的电子书。4.不能删。因为“又”字表明这一计划成功之前,我国教育产业电子信息化的进程已经迈出了重要一步。如果删去“又”字,这一意思就无法表达出来,与实际情况不吻合,语言表达就不准确。 5.这是一道开放性题,允许学生有自己的理解,认同或不认同别人的观点都可以,只要明确观点且言之有理,表述清楚即可。
(三十三)1.时间顺序。1957年→20世纪70年代→目前→眼下→几十年、上百年后 2.运用了打比方的说明方法,生动形象地说明了太阳系在宇宙中很小很小。 3.原因:①太阳系的边界十分广阔; ②目前人类对宇宙的了解认识和所具有的航天技术,对宇航来讲还远远不够,还远远不能适应飞出太阳系的要求。 4.必须开发质量小、作用时间长和高能的空间动力能源,必须大大提高宇宙飞船的速度。运用找出相关段落,筛选重点信息,提炼归纳,有机组合的方法。 5.①承上启下。 ②答案多元。想象要新颖奇特,合乎情理,描述要生动形象,语言要简洁流畅。


第十二讲 议论文阅读

(一)1.答案要点:从身体到灵魂 2.答案示例:想成为别人 真正成为自己可不是一件容易的事 3.总是为外在的事务忙碌,没有自己的内心活动 4.C 5.这一标准是对积极人生的独特领悟和坚守。 强调坚持积极的人生态度对于真正成为你自己的重要意义。
(二)1.(1)蒙娜丽莎微笑时隐时现(意到即可)(2)蒙娜丽莎含蓄的笑容,给人以丰富的联想。 2.
视网膜区域名称 功能 论断
小凹区 对阴影不敏感,善于分辨彩色和细节。 蒙娜丽莎微笑的忽隐忽现,是由于眼睛注视的部位不同所造成的。
外围区 对彩色和细节不敏感,善于辨别运动和阴影。
3.蒙娜丽莎微笑之谜终于解开了 糟蹋艺术,做煞风景的事 4.(1)对事物的兴趣和好奇心(2)丰厚的知识储备(3)锲而不舍的精神(4)勇于实践的精神(5)严谨的科学态度
5.得分要点:(1)能从科学或艺术相结合等角度驳斥对方,确立自己的观点。(2)言之有理,能自圆其说。(3)表述完整。
[附原文]“达·芬奇乃文艺复兴之旷世奇才,既是艺术家,又是科学家。他特别对物理学、生理学和医学感兴趣,研究人体颇有心得,作出过许多贡献。如今蒙娜丽莎微笑之谜揭开了,达·芬奇在天之灵当拊掌赞之日:后生可畏,深得吾心!”
(三)1.(1)因为一切历史都是当代史,学习历史能从过去了解现在成功和失败的原因。(或学习历史看懂过去,才能看懂现在) (2)因为历史是一种智慧,学习历史可以总结和吸取前人经历中的经验和教训,为现在的发展提供借鉴。 2.“环境的封闭”是指与外界隔绝,远离竞争。“心态的封闭”是指身处竞争的环境中而又无视竞争。 3.答案要点:现在是在过去的基础上发展起来的,它们有前后联系的因果关系。(或现在这棵树,是长在过去这个“本”上) 4.聆听历史的开放心态是一种才智;能够听见历史在说什么也是一种才智;能够听懂历史在说什么就是伟大的智慧。或“能够以开放的心态从历史中总结出人类社会发展的经验和教训作为现在发展的借鉴,这就是一种伟大的智慧”。 5.评分要点:能够运用文中“开放心态”的观念来分析现在的人或事,即要看人或事的现在,更要从过去找出现在成功或失败的原因,或展望未来。
(四)1.此题为开放性试题,答案不惟一,例:我们喜爱荷花,因为它总是亭亭玉立于炎热的夏季(只要句子结构和内容与例句语境相符即可) 2.世界上所有的美好,都有有效期限,所以我们应该学会珍惜。 3.激发读者的阅读兴趣,引出论题。 4.美好事物 5.举例论证 道理论证 比喻论证 6.(名言、警句与珍惜时间有关即可。)如:莫等闲,白了少年头;空悲切——岳飞《满江红》;少壮不努力,老大徒伤悲。——乐府诗 《长歌行》等。 7.此题为开放性试题,答案不惟一,就语言特点或思想内容某一方面谈即可。 例如:“香九龄,能温席”,孝顺是中华民族的传统美德。一个九岁的孩子尚且知道以行动孝顺父母,我们更应该关心父母,尊重父母。
(五)1.文章一方面论述了谅解在人际交往中的重要作用,另一方面论述了我们怎样才能学会谅解。 2.文章第④段先列举三个典型事例从正面论证,有力地证明了谅解的重要作用。接着,又从反面进行论证,形成对比,进一步论证了论点,使论证更充分。(意思对即可) 3.文章第⑤段在结构上起承上启下的过渡作用。(意思对即可) 4.这一段强调了谅解是有原则的,如果删去,将会使文章的论证不严密。(意思对即可) 5.这是一道开放性试题,
(六)1.B. 2.是由于人们丧失了对世界的永恒之基础的安全感。丧失了存在感。(或:是由于人们丧失了对世界的永恒之基础的安全感。) 3.例如:与少数“先进”相比,普遍落后肯定是生活的常态,落后者不把争先作为生活的标准,能够按生命的本来形态生活,所以更能用平和的心态体验和领略到生活的美好,世界也就成为他们的乐园。(意思对即可。) 4.举例论证、道理论证、对比论证、归谬法。(答对三个即可。) 5.(本小题属于开放性试题,无论同意与否,只要能够联系自己的生活实际,言之成理,并且观点的价值取向正确合理即可。)
(七)1.能够提高人的理解力和思维水平得到非同一般的快乐;是与智者的对话。(意思对即可) 2.第一问:比喻论证。 第二问:“现成的肉”指不磨脑子的书;“烂杏”指大量克隆的信息垃圾。 3.例:游戏的畅快 电脑的奇妙 影视的缤纷 旅游的轻松 散步的悠闲 4.略
(八)1.在人生的道路上不能中止自己奋斗的步伐。 2.①指“成功”,“完成”,“收获”等;②指“停止、终结”。 3.从两方面回答:①句号大多数是自己画上的。②拒绝句号要靠自己与句号作斗争,要与自己的惰性和保守作斗争。 4.要点:不满足取得的成绩,不停止前进的步伐,把每一点成绩作为继续前进的起点。5.略
(九)1.同:对孩子的梦想给予肯定、鼓励 异:乔丹母亲多方面帮助 2.①观点:教育孩子要多赏识、鼓励,不要轻易否定,更不要讥讽打击。 ②联系: 自然引出观点,同时也起到证明观点的作用。 3.对比(或比喻), 增强了观点的说服力(或增强了说理的形象性)。4.所举事例能证明观点。
(十)1.我们每读一本书,每见一件事,都应该多动脑筋、多思考。 2.这样才能在人们司空见惯的现象上发现新的东西,由此锻炼出我们敏锐而正确的观察判断能力。 3.牛顿从苹果落地现象研究发现了“万有引力”规律。 4.观察——疑问——解决问题。 5.例1:阿基米德洗澡时从浴盆水溢出的现象中发现了著名的浮力定律。例2:鲁班上山爬坡时,手被茅草划破,受到启发,发明了锯子。
(十一)1.前例说明没有挫败使人孤独、痛苦;后一例说明挫败可以带来惊喜、幸福。2.耶酥 李靖 孙中山 3.(1)“这份绝望”指的是面对宇宙的伟大,感到人类永远无法逼近那百万光年之外的光体。(2)“兴奋昂扬”是因为意识到人类尽管无法征服自然,达到人生的崇高境界,但也永远不会失去追求和理想。(第二问,答“意识到人类尽管无法达到人生的理想境界,但绝不会放弃对理想的追求”,“意识到人类虽然永远不能逼近那百万光年之外的光体,但绝不会放弃对宇宙的探索和认识”,“意识到人们有时无法追摹到某种境界,但会在追求和发现中心悦诚服”也可) 4.不断遇到更令我心折的人,不断探得更勾魂摄魄的美景,好让我能更彻底的败溃,从心底承认自己的卑微和渺小。 5.①有必要。作者广泛举例,可以证明“寻求挫败”不仅是自己的个人体验,而且具有普遍意义。②无必要。既然写“我”寻求挫败,就应该只写自己的经历和体验。③有必要,但略嫌多。可以删去一点,以突出“我”寻求挫败这一重点。
(十二)1.天资 勤奋 机遇 2.勤奋(自身努力) 3.借助词典,解释“成功”(设问) 必须以勤奋的精神去做学问干事业(要获得成功必须勤奋) 4.示例:①苏秦以锥刺股,勤奋学习,终于成为战国时期著名的纵横家; ②闻一多为了著书立说,足不下楼,目不窥园; ③爱迪生为了发明创造,勤奋工作,每晚只睡三四个小时。 5.示例:①天资方面:同意的理由是,人不可能决定自己的天赋。不同意的理由是,经过后天的努力,人可以不同程度地提高自己的素质。 ②机遇方面:同意的理由是,人不可能随意地创造机遇。不同意的理由是,机遇虽然总是“不期而遇”,但有了机遇还有一个能否“抓住”、“把握”的问题,这就要靠主观努力。
(十三)1.是一个快乐的领域,其中包括创造的快乐,阅读的快乐,欣赏大自然和艺术的快乐,情感体验的快乐,无所事事地休闲和遐想的快乐,等等。 2.“穷人”:日常生活的忧虑;“悲惨的人”:苦难的遭遇和身体的痛苦;“忙人”:名利和责任;逼迫。 3.全身心地投入一种事业,在忙碌中真正感受到创造的快乐。需要;因为只有在这个心灵的空间中,才能把事业作为生命的果实来品尝。 4.答案略。 5.一个终日忙碌的人,惟有在心灵的空间中,才能把事业作为自己生命的果实来品尝;如果没有这个空间,一味地永远地忙碌,让自己的心灵永远被与事业相关的各种事务所充塞,不管这个人在事业上取得了怎样的外在成功,都只能算是损耗了自己的生命而没有品尝到它的果实。
(十四)1.错过就是永远的失去。 2.那埙的奇音至今揪扯着心中的悔,想起来就隐隐地针扎般地痛。 3.略。 4.①运用排比的修辞手法,使论据充分,说服力增强。②示例:一失足成千古恨;一着不慎,满盘皆输。……
(十五)1.人是一本书 2.看人如读书做人如写书 3.洒脱飘逸 多愁善感 乐观风趣 思维缜密 4.在生命的版面上,应该写下高尚,应该填上纯真,应该刻画奉献,应该描绘进取(或:你自己这本书,主要还得靠你自己来写——用你的勇气和力量,用你的聪明和才智,用你的心血和汗珠……) 5.我们每个人都要写好自己这本书,争取做一个非常优秀的人(与之相近的表达也可) 6.(答案不惟一,要符合题目要求)
(十六)1.具体形象地描写了小凌所说的“我活得挺自在。我为自己高兴”,使文章更有说服力。 2. 对比论证(或:正反论证) 不要为自己立下高不可及的标杆,更不要被别人往往确实是出于好意的刺激而陷入自卑自怨自责自苦的泥潭! 3.诚然(或:虽然、固然) 然而(或:但是) 4.示例:①“为自己高兴”并不是满足现状,不思进取,而是要为自己的平凡充实的存在而高兴,而不要总是为那些自己高不可攀,不切实际的目标而徒生烦恼。②人还是要有一个远大目标,并要为此作出最大努力。完全满足现状不思进取不应该是我们中学生所具有的人生观。少壮不努力,老大徒伤悲。
(十七)1.外压,迷乱,急迫等特定的客观情形;达观、自信、胸有成竹的主体因素 2.从境界、进取、自由、修炼、自守等五方面来阐述的。 3.①收束全文或总结全文(或点题)②“从容”内涵十分丰富,面对浮躁的社会现象,即使不能完全做到,只要能够坚守自我,点滴做起,努力过,付出过就行。(意合即可) 4.B、D,
(十八)1.生活中需要思考。 2.举例论证。爱因斯坦因思索发现“狭义相对论” 爱迪生因思索制成电灯泡,造福人类。 3.因为青春是短暂的,是人生的一个重要阶段。走好这一步在人生中具有十分重要的意义。青年人任重道远,所以说“青年更需要思索”。 4.略
(十九)1.文题中的“冬天”侧重于人生要经历的挫折、磨难,“跑过冬天”即战胜人生路上的苦难。 2.以特写的画面唤起女儿对幼年生活的亲切回忆,点题,为下文说理作铺垫。 3.①强身健体、节俭助人、坚强尽力、守住自我 ②注重内在气质的培养、衣着清洁协调、珍惜时间热爱生命、尽人生之责 4.①人生短暂,要尽力使自己的人生发出光芒。 ②从来没有“救世主”,人生只能靠自己。 5.[示例]标题:形象、醒目,耐人寻味。内容:这封父女间的交流书信,凝聚着父亲的人生经验,饱含着真挚的人间真情。写法:用女儿五岁时晨跑的经历开篇,亲切自然;用排比表达父亲的告诫和期望,条理井然。
(二十)1.文中的“登山”既指登临自然界的高峰,也指翻越人生的高峰(或翻越内心屏障,超越自我) 2.生活中有许多山峰等待你去攀登,如果你不去努力,就可能在平庸中度过一生。 3.登高望远:欲穷千里目,更上一层楼。俯视一切:会当凌绝顶,一览众山小。 4. 瓦特的事例是为了说明从身边小事做起,即是登山的开始,生活中的山峰就在你的身边(或答读书可以增长知识、开阔视野也可)。 5.排比句式,既增强了语言气势,又淋漓尽致地写出“登山”过程的美好感受。 6.例:学习中有许多困难需要我们战胜它,每克服一个困难就是翻过一座山,就是一次心灵的陶冶、自我的超越。
(二十一)1.谜:是指河东岸和河西岸的羚羊大不一样,前者繁殖能力比后者更强,而且奔跑速度每分钟要快13米。 底:是指因为东岸的羚羊群附近居住着一个狼群,这使羚羊天天处在一个“竞争氛围”中。 2.优裕的生活条件。 3.举例略。要求能证明本文观点,而且是人类生活中的例子。 4.提炼论点略。要求与本文秃鹰的遭遇这一材料吻合,而且用论点的形式表述出来。 5.不能放在本文的任何位置。因为本文论证的是“感谢对手”这一观点,而选段旨在谈“环保”问题,谈动物濒临灭绝的现实,与本文无关。
(二十二)1.针对工作忙,“头绪多,没有时间读书的现象。这样的写法有针对性,有说服力。 2.读书有三益。 3.答出“忘我”方面的内容即可。 4.充当事实论据论证读书可以疗疾。5.A 6.言之有理即可。
(二十三)1.巧是人们经过长期的社会实践,勤学苦练得来的。 2.勤能补拙、熟能生巧等俗话、成语。 3.①牛顿幼时为大猫小猫各开一洞。 ②梅兰芳幼时学戏多时不能上口,老师嫌他笨,不教他。 ③达·芬奇长年累月练习画蛋。 4.笨拙 5.略
(二十四)1.学贵质疑 引出全文的中心论点 2.知识也是无止境的(大意正确即可) 3.“疑是建立在丰富的知识和认真思考之上的,绝不是无端的猜疑或随便的怀疑”“任何有效的怀疑,都依赖于对事实的仔细分析和对理论的深入研究” 4.答案示例:如果哥白尼没有对“地心说”的怀疑,就不会有“日心说”的创立。(列举中国老一代地质学家怀疑西方提出的“中国贫油论”从而开发了一个个大油田以及其他因怀疑前人而创立新学说的典型事例均可。) 5.答案示例:学贵质疑。质疑是获取新知和创新发展的基础,刘昱同学正是因为敢于质疑,善于探疑,才最终恢复了“燧人氏钻木取火”的历史地位。因此我们不能盲目迷信权威和书本,要敢于质疑,这样才能成为一个知识渊博的人。
(二十五)1.天才就是非凡的傻劲 阅读议论文找论点应先关注题目。或:可根据论据反推出论点。 2.参考示例:示例一:①徐霞客写《徐霞客游记》34年; ②哥白尼写《天体运行》36年。 示例二:①司马迁从20岁开始漫游,足迹遍及黄河、长江流域,兀兀穷年,终于写出《史记》; ②居里夫人在1350多个日夜里,重复着脏重的体力劳动,才从8吨沥青铀矿残渣中提炼出1克镭。 示例三:①曹雪芹写《红楼梦》批阅十载,增删五次; ②爱迪生发明蓄电池,试验了一万多次才告成功。 3.举例 道理 对比 4.第⑷段 5.参考示例:合作能力 意志 毅力 勤奋 刻苦 6.聪明(聪明才智) “傻劲” 名言示例:天才出自勤奋
(二十六)1.不要抛弃学问。 2.白发方悔读书迟;少壮不努力,老大徒伤悲;一寸光阴一寸金,寸金难买寸光阴;一年之际在于春,一日之际在于晨……。 3.只要愿意读书,没有条件也会创造条件读书,不愿读书条件再好也没用。 4.举例(事例)论证和事理(道理)论证 事例:高尔基(华罗庚)一边打工,一边挤时间读书;英国著名物理学家霍金身残志不残,最终成为世界著名科学家;张海迪(史铁生)身残志不残,自学成才,为社会做出很大贡献…… 名言:积土成山,风雨兴焉;积水成渊,蛟龙生焉;不积跬步,无以致千里(不积细流,无以成江海);鲁迅说,我把别人喝咖啡的时间都用来读书了;方丈高楼平地起;千里之行始于足下…… 5.①亲切自然(娓娓道来、以理服人) ②通俗易懂(深入浅出) 6.略 7.略
(二十七)1.学问与智慧是相辅为用,缺一不可的。(意思对即可)  2.对比论证和举例论证 (讲道理和摆事实)   3.肥皂泡:形容没有学问的智慧肤浅,瞬起瞬灭,一触即破。(意思对即可) 探照灯:形容从学问中产生的智慧有深度,有广度。(意思对即可)
(二十八)1.第一种,“适应能力很强”的鸟;第二种,适应能力很差的鸟;第三种,以残酷的方式练飞的鸟。这几种鸟分别象征:第一种,随遇而安,放弃理想和自由的人;第二种,面对厄运悲观、绝望的人;第三种,在灾难和厄运中,为实现理想而英勇搏击的人。 2.因为它们已经接受了残酷的现实,并学会把这看成生存的常态,理想和自由不再成为它们的追求,抗争和拼搏已变得毫无意义。 3.肯定它们抗拒厄运、漠视恩赐的精神;批判其狭隘、守旧,以及到理想破灭后的悲观、绝望。 4.尽可能地保持飞翔的能力,以实现在天空飞翔的梦。 5.灾难和厄运也是一种客观存在,对于永不放弃的勇者来说,灾难是一种机遇,一种磨练,只有迎难而上,在厄运中拼搏,才能生存和发展,实现理想。
(二十九)1.凛冽 2.①春天是美好的,但春天短暂而易逝,我们应该倍加珍惜。 ②人应该保持青春的活力,以积极向上的人生态度不断奋斗和进取。 3.①因为春“总是乍暖还寒,需要反反复复冲破凛冽的西风和反攻的寒潮才降临人间”;“总在不经意中,便进入了初夏的境地”。 ②引发了作者“春光难觅”、“春光易逝”的感慨。 4.在春天的怀抱里,不论什么样的身份和地位,人人都能尽享春天带来的欢乐,这正是春天被所有热爱生活的人所承认和赞美的原因。 5.要求:最喜欢原因的表述要紧扣诗句要点,语言要准确简明,生动流畅。(可以从诗句的思想内容、表达技巧、语言特点以及得到的启示和感悟等方面作答) 6.美丽的春色,习习的春风,给人带来美的享受和精神的愉悦,促使人们从心里感受到生活的美好,进而引发人们更加热爱春天,热爱生活,珍视人生。
(三十)1.提升自身品质 引导和激励被欣赏者 2.“坏孩子”的聪明 著名的企业家和思想家 “小偷”的心思、手法和风格 很有名气的大老板 3.A 4.语气委婉,含有欣赏和鼓励,便于听取接受。 5.略
(三十一)1.“但只要有利于……便在所不辞”或到“……管他是沉是浮” 2.排比,加强语势,增强感染力,充分赞美了人格力量。 3.因为这是林则徐在被贬的情况下还心系百姓,为百姓修的渠;它体现了大写的“人”的风格和精神。 4.由对英雄的一般性崇敬,引发了对人生价值的思考(或:找到了人生效仿的楷模)。 5.AE
(三十二)1.②③①④ 2.心地善良,乐于助人,工作认真负责。 3.①卖菜妇人和我拉家常,使我对她产生了亲近的感觉,从而感受到人与人之间互相关心,互相帮助的温暖。(意思接近即可)②我是一个普通的人,不可能对他们的生活产生任何影响,但是他们使我感受到了人性的美好,对我的生活具有重要意义。(意思对即可)③他们从来也没有意识到自己对别人、对社会的奉献,因而显得更加质朴;我因从他们身上感受到生活中的美而更加觉得幸福。(意思对即可) 4.⒀‖⒁ 记叙部分是为下文的议论作铺垫,议论部分点明了文章的主题,使记叙的内容得到了升华。 5.美在平凡的生活中,美在人与人之间互相关爱和勤勤恳恳的工作中;只要怀着一颗善良之心,学会感激别人,你就能发现生命的美。(意思对即可)
(三十三)1.合作求生。 2.关注(关心)友好(信任、友善)依恋(留恋、惜别) 3.①冷酷、残忍、敌对、杀机 ②对动物的仁慈、友善、尊重、温情 4.略 5.略
(三十四)1.我们不要等待,而要去开创一个美好的明天。 2.作者从明天是平凡而无情的,明天是未可知的,明天是辛苦的,明天是脆弱的四个方面来论述的。 3.类别须与引用的句子相符。 4.开放性答案,言之成理即可。添了些许:如知识、经验,或阅历,或由幼稚走向成熟,或情感更丰富等。减少了些许:如生命的时光,或童年的欢乐,或童心等。

第十三讲 学作品阅读
(一)1.“那种东西”是指强身上体现出来的敬业、诚实的品格。(意思正确即可) 2.①本家哥所说的“良心”是指人应知恩图报。②强所说的“良心”是指人应敬业(或“做好自己应做的事”)。(意思正确即可) 3.①强对本家哥说要瞧得起自己,体现了他的自尊。(与外商洽谈合作项目时,用汉语而不用英语,体现其自尊,或民族自尊,或爱国心均可)②看到丢弃的线头碎布也拾起来,留作备用,体现了他的勤俭。③暴风雨中主动加固被掀起的篷布,体现了他的敬业。④老板要为他加新,他说他只是看看修补的篷布牢不牢,体现了他的诚实。⑤他拒绝进城的本家哥求职的要求,体现了他不徇私情。⑥面对几个受过高等教育的年轻人的不服,没有打击他们,而是说服他们,体现了他胸怀宽广。(此例也反映了他善于团结同志,善于合作的精神。) 4.“高等教育”是指包括诚实、勤俭、敬业等在内的人的品格教育。(意思正确即可) 5.言之有理。
(二)1.起因:拉斯马森接到老夫人的求救电话。经过:消防队员全力寻找老夫人住址(消防队员全力救助老夫人。) 2.最动听的声音(爱心救助、爱心大营救、救命电话) 3.“有责任感”或“有爱心”;“聪明”或“机智” 4.“中尉知道,老太太大概昏过去了,没有把电话挂上。” 5.这样写真实地表现出寻找的过程,突出了寻找过程的紧张(有序)。 6.①(比喻),生动、形象地写出了人们焦灼(紧张、沉重、沉痛)的心情。 ②(拟人),形象地写出(渲染)了当时紧张(焦急)的气氛。 7.示例:人的生命是最宝贵的,真情救助是最美丽的。如果我们人人都能尽职尽责奉献爱心,世界一定会变成美好的人间。
(三)1.(1)(绿色)用自己积极蓬勃的生命,让人心情爽快。 (2)(绿色)爬上窗台。 2.(1)示例:山岭巍峨起伏,显示的是自己坚强的体魄。 (2)以此说明自然不是冷漠无情的。 或说明“世间万物都有生命有情感有思维” 3.反衬了(来自黄瓜和莓豆的 )绿的积极向上的精神。 4.(1)①积极蓬勃 ②坚强向上 ③ 奉献 ④执著追求 ⑤ 生的迫切、力量和宁静 答对其中的两点即可(2)略 5.D 6.能就某一点举例分析,且言之成理即可。
(四)1.①点题;②抒发感受,为全文定下感情基调;③名人的话更有代表性。 2. ①写出费尽周折才买到票的欣慰,形象地写出回家心情迫切。 ②比喻家给你精神上带来的极大创伤,家给你带来沉重的负担。(意思对即可) 3.①运用比喻;②形象地表明了家是和谐、温馨的,充满浓浓亲情或家是小女孩的童话、梦想和美好的一切(意思对即可)。 4.①家是旅人魂牵梦绕的地方或家是人们灵魂的归宿;②家是人的精神支柱,支撑人们奋斗并使人的奋斗富有价值;③家是孩子的一切。(答对两点即可)5.A 、D A项“热爱祖国”理解牵强,拔高了文章的主题。 D项文章中没有运用“象征”手法。
(五)1.痛悔自己竟不明白廖医生的良苦用心,不知道廖医生谎称木槿花可作中药,并收购木槿花,目的是使自己能心安理得地接受帮助,坚持为儿子治好病。(意思基本符合即可) 2.①得知儿子的腿能治好,心里感到宽慰;但又为贫穷无钱治病而犯愁。 ②因无力支付医药费而失却了继续医治的勇气,为无力治好儿子的腿而伤感。 (意思基本符合即可) 3.象征人世间纯洁善良、扶弱济困的美好情感。 4.母亲:贫困而自尊。缺钱为儿子治病,定要以鸡蛋、母鸡相赠表心意;当晚就把廖医生给的钱送回去。 廖医生:心地善良,尊重弱者。不计较医药费,还用巧妙的方式接济他人,维护他人的自尊。
(六)1.因为他看到儿子上学的小楼已变成废墟,认为儿子一定死了,感到绝望。 2.人们认为这位父亲因失去孩子而精神失常,对他深表同情;但断定孩子绝无生还的可能,继续挖下去还会有危险。 3.责任感  信任 4.突出父亲挖的时间长;表现父亲的执著(坚毅、坚韧、顽强),对孩子的深挚的爱。 5.(1)父爱:①父亲安顿好受伤的妻子。 ②救火队长关心父亲的生命安全。③警察劝父亲控制情绪,并关心群众的安全。 ④好心人劝慰父亲。⑤过路人帮助父亲救出14个孩子。⑥儿子鼓舞同学。遇救时先让同学出去。(2)认识(示例):①爱能够创造奇迹。 ②爱能够激发人的潜能。③爱使人高尚。 ④爱是精神的支柱,能够使人绝处逢生。⑤爱使人坚毅。⑥爱是驱散黑暗的阳光。
(七)1.喜欢 陶醉 迷恋 惊讶 2.榕树的精神品格 3.北方朋友为榕树的强大生命力所震撼,所鼓舞,从而使心灵得到净化,灵魂得以升华。 4.顽强而默默无闻 沉稳而奋发进取 5.示例:①“此时的榕树……沙沙的响声。”运用比喻、夸张手法,形象表现榕树沉稳的特点,突出其强大旺盛的生命力。 ②“它在找不到任何营寨的时候……像蚕儿抛出的银丝。”“撒”“抛”两个动词,用得生动传神,表现了榕树顽强的生命力。
(八)1.指一个小女孩因偷书被售货员抓住,而她又不肯让家人知道,所以就胡乱说了一个电话号码,不巧就打到“这位教师”的家里。(言之成理即可) 2.用意:①不想让小女孩再做类似的“傻事”。 ②宽慰小女孩:一个爱看书的小女孩犯了错误是应该可以得到原谅的,小女孩不必让“偷书”的罪名永远背在身上。 3. ①表达了女孩无限感激之情。②表达了女孩希望像报答母亲一样报答“教师妈妈”的教育和保护之恩。(答案应含“感激”、“报答”两层意思) 4. ①为女孩走出“偷书”的阴影,健康成长而感到欣慰。②为自己正确妥当的教育方式获得成功而感到欣慰。5.指老师的闪念。道理:不应该放过每一个能够帮助别人的机会。 6.例①不好的理由:在当时特定的情境下,不问更好。因为她已经知道错了,并且她幼小的心灵已受到极大的刺激和打击,如果这时候再对她进行批评教育,只会加剧她心灵的创伤,使她产生更重的负罪感,更难以从惊恐、内疚中解脱出来。例②好的理由:对犯错误的孩子应该进行批评教育,批评是一种帮助、一种警醒、一剂良药,只有这样,才能使她更深刻地认识到自己的错误,进而避免再犯类似的或其他的错误;如果不对她进行批评教育,会使她产生一种侥幸心理,这样反而害了她。
(九)1.观、赏、咏 2.拟人;荷给人带来的是一种境界,给人的心灵创造的是一种无限的空间。 3.出淤泥而不染,濯清涟而不妖。 4.荷花亭亭玉立,荷叶飘曳多姿,荷香沁人心脾。 5.第⑸段:雨中看荷;第⑹段:夏日赏荷;第⑺段:月夜探荷。 6.D图:因为:①整个荷塘像一块绿色的陆地,红红的荷花跃出水面,大摇大摆地把花瓣放开;②荷池中由于有了人的活动,整个荷池给人一种无限空间的感觉。 7.略。
(十)1.通过捎(送)钱和还钱两件事刻画了家长的形象。 2.①表现“我”不想增加学生的心理负担。(或答:不想张扬自己搭钱、换钱的事。不想详叙父亲送钱的细节。不想将零钞的重量压在学生稚嫩的肩上。)②表现家长还钱态度坚决。(或答:对还钱极认真) 3.要点:①向父母问好,收到钱很感动;②钱由马老师转交给了自己;③这次收到的钱有些不一样,从来没有这么齐整;④要努力学习,报答父母等。 4.对这位学生的喜爱(或同情),对这位家长的赞美(或钦佩)。 5.指有风骨(或:像文中家长那样,虽然家境贫穷,但人穷志不穷,不贪便宜,不计小利,灵魂高尚。)
(十一)1.“借你一个微笑”,就是作为老师的“我”,想方设法通过多种途径让李俊能够逐步学会微笑,向“我”微笑,以便培养他的自信心,使他的性格由内向忧郁变得乐观开朗,由自卑变得自信,进而培育他健全的心理和人格。(意思对即可。) 2.李俊很想得到老师的帮助,但又不敢出声的那种期待、难过、窘迫、紧张的心理。 (意思对即可。) 3.老师向我借微笑?微笑也能借吗?是不是老师不喜欢我平时愁眉苦脸的样子,要我笑?是老师不愿意为我这差生补课而找借口?还是……不过,只要老师愿意为我补习物理,那我就笑笑吧。4.做了两件事:①课堂上“我”想方设法让李俊开口说话; (答“课堂上‘我’有意让李俊朗读例题和复述题目要求”也算对):②“我”“别有用心”地递题目纸条给李俊,“迫使”他和同学进行讨论。(答“我特意递题目纸条给李俊,促使他和同学讨论完成”也算对。) 5.不矛盾。①“我为他补的都差不多了”是指“我”为他弥补了性格上的缺陷。②“我终于没给他补课”是指“我”没有给他补物理知识的课。 6.①李俊因为父亲的瘫痪面临生活的困境而休学,但他凭着“我”帮助他树立起来的自信和乐观,勇敢坚强地挑起了生活的重担,对人生充满希望。②这让我领悟到,一个人只要有了自信和乐观,就能克服生活的种种困难,就能经受住人生的任何考验。(意思对即可。)
(十二)1.①了解,②为了保护弱者挺身而出,请求老师宽恕犯了错误的同学,(意合即可) 2.勇敢 正直 善良 宽容 富有同情心 3.内容要切合“请求老师宽恕那些犯了错误的同学”,形式上是对话,要求简明得体。 4.人物的动作、语言、肖像切合人物身份,作品语言极富感情色彩和趋向性
(十三)1.①一天夜里 连夜为小偷做连肢手术 ②又一天 为情敌做修补颅骨手术 ③这年 用手术刀结束法西斯刽子手的性命 2.总领全文 3.形象生动地表现了海尔曼医生的性格特征 倔强、爱憎分明、医术高超、医德高尚。 4.如:“此刻海尔曼想起那些被列夫斯基夺去爱妻痛苦不堪的日子,曾被他们侮辱的难堪,但内心更多的是自己作为医生的天职,于是,他毫不犹豫地拿起了手术刀……” 5.①救死扶伤是医生的天职。 ②反法西斯、维护和平、反对战争是每一个有良知的公民的天职。 6.如:海尔曼是一个珍爱生命,不为个人私情玷污医德,为人类和平敢于献身的医务工作者。
(十四)1.祈(qí)垢(gòu)没(mò) 2.“我”是一个调皮、捣蛋,但内心深处非常渴望上进的孩子。 3.①“我带头吹口哨、飞吻、往空中扔书本,好多男生跟我学,我们的吵闹声几乎要把房顶掀开。”②“我打一个手势,大家立即停止了胡闹。” 4.“我”开始注意自己的外表,意识到自己的不足(或“自惭形秽”)。暗示“我”已开始转变。 5.心理描写。表现了“我”善良,渴求关爱,希望成为同学的榜样,得到了玛丽娅小姐的热情鼓励和积极引导后,决定做一个上进、体面的人。 6.因为玛丽娅小姐具有天使般的美德——宽厚,仁爱,善于发现学生优点;点明文章中心。
(十五)1.突然停止 喝不够的样子 2.挣脱僵绳阻拦运水车 3.不远处沙堆背后跑来一头小牛,受伤的老牛慈爱地看着小牛贪婪地喝完水,伸出舌头舔舔小牛的眼睛,小牛也舔舔老牛的眼睛。静默中,人们看到了母子眼中的泪水。 4.描写 它形象生动地描绘出老牛以死抗争,执意索水的感人场面。 5.答案不惟一。可以从“神圣的母爱”、“改变生态环境”、“解放军爱民为民”、“解放军战士舍己救难的仁爱之心”等方面谈。(能就某一点展开,言之有理即可。) 6.会。因为场面悲壮感人,因为人性的善良,等等。不会。因为军人的纪律高于一切,因为救人急于救牛,等等。(言之成理即可)
(十六)1.说明父亲对“我”的那种朴素、真挚的爱是那样的自然,那样的不加任何雕饰(父爱无痕)(只要答出“朴素”、“真挚”、“自然”等意思即可) 2.父亲像在无意中写了这些信 3.示例:(叙):我也买了许多信封,写上父亲的名字,并经常给父亲写信,报告平安和问候(议):我深深地自责:我竟然对父亲那种朴实、深厚的爱全不知晓,甚至反感,简直无知之极。我要用百倍的努力去回报(只要意思相同即可) 4.可以 前者是以文章的线索作标题,后者是以文章的主题作标题 5.①一种深深……在心头 ②我不禁一怔……升起 ③此时此刻……泪流满面

 

Copy Right 飞跃家教中心 2021-2026 网站许可证号: 鄂ICP备20000519号